Download as pdf or txt
Download as pdf or txt
You are on page 1of 201

BEST SELLER For 2025 Exam

CBSE
QUESTION
CBSE

BANK
QUESTION

Chapterwise & Topicwise

CLASS 9
SAMPLE
BANK

MATHEMATICS
QUESTION PAPERS

Get the #OswaalEdge


CLASS 9

1 100% Updated
with Latest Syllabus & Questions
Typologies

2 Timed Revision
with Topic wise Revision Notes,
Smart Mind Maps & Mnemonics
MATHEMATICS

3 Extensive Practice
with 1000+ Questions & SAS Questions
(Sri Aurobindo Society)

4 Concept Clarity
with 500+ Concepts
& Concept Videos

5 100% Exam Readiness


with Competency Based Questions
& Artificial Intelligence

(1)
20th EDITION YEAR 2024-25
I SB N “9789359589589”

CENTRAL BOARD OF
SYLLABUS SECONDARY EDUCATION
COVERED DELHI

PUBLISHED BY
OSWAAL BOOKS &
LEARNING PVT. LTD.
C OPY RIGHT
RESERVED 1/11, Sahitya Kunj, M.G. Road,
Agra - 282002, (UP) India
BY THE PUBLISHERS
All rights reserved. No part of 1010, Cambourne Business Centre
this book may be reproduced, Cambridge, Cambridgeshire
stored in a retrieval system, or
CB 236DP, United kingdom
transmitted, in any form or by any
means, without written permission
from the publishers. The author
and publisher will gladly receive 0562-2857671
information enabling them to
rectify any error or omission in
subsequent editions.
contact@oswaalbooks.com

www.OswaalBooks.com

DISC L AIM ER
This book is published by Oswaal Books and Learning Pvt Ltd (“Publisher”) and is intended solely for educational use,
to enable students to practice for examinations/tests and reference. The contents of this book primarily comprise a
collection of questions that have been sourced from previous examination papers. Any practice questions and/or
notes included by the Publisher are formulated by placing reliance on previous question papers and are in keeping
with the format/pattern/guidelines applicable to such papers.
The Publisher expressly disclaims any liability for the use of, or references to, any terms or terminology in the book,
which may not be considered appropriate or may be considered offensive, in light of societal changes. Further, the
contents of this book, including references to any persons, corporations, brands, political parties, incidents, historical
events and/or terminology within the book, if any, are not intended to be offensive, and/or to hurt, insult or defame
any person (whether living or dead), entity, gender, caste, religion, race, etc. and any interpretation to this effect
is unintended and purely incidental. While we try to keep our publications as updated and accurate as possible,
human error may creep in. We expressly disclaim liability for errors and/or omissions in the content, if any, and further
disclaim any liability for any loss or damages in connection with the use of the book and reference to its contents”.

Kindle Edition

(2)
Contents
l Latest CBSE Syllabus 6 - 11

In each chapter, for better comprehension, questions have been categorized according to the typology issued by CBSE
as follows :
R - Remembering, U - Understanding, A - Analysing, AP - Applying, C - Creating, E - Evaluating.

Unit I : Number Systems


1. Real Numbers 1 - 25
l Self Assessment Paper-1 26 - 27
Unit II : Algebra
2. Polynomials 28 - 43
3. Linear Equations in Two Variables 44 - 51
l Artificial Intelligence 52 - 52
l Self Assessment Paper-2 53 - 54
Unit III : Coordinate Geometry
4. Coordinate Geometry 55 - 63
l Artificial Intelligence 64 - 65
l Self Assessment Paper-3 66 - 68
Unit IV : Geometry
5. Introduction to Euclid's Geometry 69 - 79
6. Lines and Angles 80 - 93
7. Triangles 94 - 108
8. Quadrilaterals 109 - 121
l Artificial Intelligence 121 - 122
9. Circles 123 - 138
l Artificial Intelligence 139 - 139
l Self Assessment Paper-4 140 - 141
Unit V : Mensuration
10. Areas 142 - 154
11. Surface Areas and Volumes 155 - 166
l Artificial Intelligence 167 - 168
l Self Assessment Paper-5 169 - 170
Unit VI : Statistics
12. Statistics 171 - 180
l Artificial Intelligence 180 - 182
l Self Assessment Paper-6 183 - 184
qq

(3)
How to use this Book

Chapter Navigation Tools

Syllabus Topicwise & Revision


Conceptwise Notes
Segregation

Prescribed by For Focused Chapter Summary


CBSE & Systematic Developed by
Study Oswaal Experts

Latest Mind Maps QR Codes


Typologies of &
Questions Mnemonics

Objective/SA/LA/ For better For Concept


Case-based retention of Videos
Questions concepts

Previous Academically Practice


Years’ Board Important Questions
Papers Questions
To decode To Look out for For Better
the paper pattern Highly Expected Practice
Questions for the
upcoming exams

Self Commonly
Competency Made Errors
based Assessment
Papers & Answering
Questions Tips
To demonstrate
For Unitwise To write perfect
application based
Assessment Answers
knowledge of
Concepts

(4)
Preface
Elevate Your Performance, Surpassing the Past
Elevate Your Performance, Surpassing the Past Get ready for another epic journey through the academic
wonders of the academic year 2024-2025 with your trustworthy companion—Oswaal Books! Remember
last year’s triumphs? Well, buckle up because we are about to make this year even more awesome!

As the legendary dancer Martha Graham once said, “Practice means to perform, repeatedly in the face
of all obstacles, some act of vision, of faith, of desire.” We have taken this wisdom to heart and packed
it into our brand-new Question Banks for 2024-2025. They are a magical mix of CBSE Board Updates,
cool questions from the past, and specially crafted ones tailored to the Latest Typologies. Oh, and did we
mention the fantastic Learning Resources that come with them?

What makes these Question Banks truly exceptional?


• 100% Updated Syllabus & With Latest Questions Typologies: We have got you covered with
the latest and 100% updated curriculum
• Timed Revision with Topic-wise Revision Notes, Smart Mind Maps & Mnemonics: Study smart,
not hard!
• Extensive Practice with 1000+ Questions & SAS Questions (Sri Aurobindo Society): To give you
1000+ chances to become a champ!
• Concept Clarity with 500+ Concepts & Concept Videos: For you to learn the cool way—with
videos and mind-blowing concepts
• NEP 2020 Compliance with Competency-Based Questions & Artificial Intelligence: For you to be
on the cutting edge of the coolest educational trends.
If you are looking to conquer every study challenge, these Question Banks are your secret weapon. It is like
having a superhero ally for your exams! So, let’s kick off this exciting journey, fill those learning gaps, and
rock the year with ease and confidence.
Big shoutout to our superhero team—the Oswaal Editorial Board! They’re the brains behind this incredible
resource, working day and night just for you. And a massive thank you to you, our fellow Students, Parents
& Teachers for your awesome inputs that make this book one-of-a-kind.
Wishing you all the best, superheroes-in-the-making! Strive for greatness! Team Oswaal Books

(5)
Syllabus

MATHEMATICS
COURSE STRUCTURE Class - IX (Code No. 041)
Units Unit Name Marks

I NUMBER SYSTEMS 10

II ALGEBRA 20

III COORDINATE GEOMETRY 04

IV GEOMETRY 27

V MENSURATION 13

VI STATISTICS 06

Total 80

UNIT I : NUMBER SYSTEMS


1. REAL NUMBERS (18) Periods

1. Review of representation of natural numbers, integers, and rational numbers on the number line.

Rational numbers as recurring/ terminating decimals. Operations on real numbers.

2. Examples of non-recurring/non-terminating decimals. Existence of non–rational numbers (irrational

numbers) such as 2 , 3 and their representation on the number line. Explaining that every real
number is represented by a unique point on the number line and conversely, viz. every point on the

number line represents a unique real number.

3. Definition of nth root of a real number.


1 1
4. Rationalization (with precise meaning) of real numbers of the type a +b x and x + y (and their

combinations) where x and y are natural number and a and b are integers.

(6)
Syllabus

5. Recall of laws of exponents with integral powers. Rational exponents with positive real bases (to be

done by particular cases, allowing learner to arrive at the general laws.)

UNIT II : ALGEBRA
1. POLYNOMIALS (26) Periods

Definition of a polynomial in one variable, with examples and counter examples. Coefficients of a

polynomial, terms of a polynomial and zero polynomial. Degree of a polynomial. Constant, linear,

quadratic and cubic polynomials. Monomials, binomials, trinomials. Factors and multiples. Zeros of a

polynomial. Motivate and State the Remainder Theorem with examples. Statement and proof of the

Factor Theorem. Factorization of ax2 + bx + c, a ≠ 0 where a, b and c are real numbers, and of cubic

polynomials using the Factor Theorem.

Recall of algebraic expressions and identities. Verification of identities :

(x + y + z)2 = x2 + y2 + z2+2xy + 2yz + 2zx

(x ± y)3 = x3 ± y3 ± 3xy (x ± y)

x3 ± y3 = (x ± y) (x2 xy + y2)

x3 + y3 + z3 – 3xyz = (x + y + z) (x2 + y2 + z2 – xy – yz – zx)

and their use in factorization of polynomials

2. LINEAR EQUATIONS IN TWO VARIABLES (16) Periods

Recall of linear equations in one variable. Introduction to the equation in two variables.

Focus on linear equations of the type ax + by + c = 0. Explain that a linear equation in two variables has

infinitely many solutions and justify their being written as ordered pairs of real numbers, plotting them

and showing that they lie on a line.

UNIT III : COORDINATE GEOMETRY


COORDINATE GEOMETRY (7) Periods

The Cartesian plane, coordinates of a point, names and terms associated with the coordinate plane,

notations.

(7)
Syllabus

UNIT IV : GEOMETRY
1. INTRODUCTION TO EUCLID'S GEOMETRY (7) Periods

History - Geometry in India and Euclid's geometry. Euclid's method of formalizing observed phenomenon

into rigorous Mathematics with definitions, common/obvious notions, axioms/postulates and theorems.

The five postulates of Euclid. Showing the relationship between axiom and theorem, for example:

(Axiom) 1. Given two distinct points, there exists one and only one line through them.

(Theorem) 2. (Prove) Two distinct lines cannot have more than one point in common.

2. LINES AND ANGLES (15) Periods

1. (Motivate) If a ray stands on a line, then the sum of the two adjacent angles so formed is 180° and

the converse.

2. (Prove) If two lines intersect, vertically opposite angles are equal.

3. (Motivate) Lines which are parallel to a given line are parallel.

3. TRIANGLES (22) Periods

1. (Motivate) Two triangles are congruent if any two sides and the included angle of one triangle is

equal to any two sides and the included angle of the other triangle (SAS Congruence).

2. (Prove) Two triangles are congruent if any two angles and the included side of one triangle is equal

to any two angles and the included side of the other triangle (ASA Congruence).

3. (Motivate) Two triangles are congruent if the three sides of one triangle are equal to three sides of

the other triangle (SSS Congruence).

4. (Motivate) Two right triangles are congruent if the hypotenuse and a side of one triangle are equal

(respectively) to the hypotenuse and a side of the other triangle. (RHS Congruence)

5. (Prove) The angles opposite to equal sides of a triangle are equal.

6. (Motivate) The sides opposite to equal angles of a triangle are equal.

(8)
Syllabus

4. QUADRILATERALS (13) Periods

1. (Prove) The diagonal divides a parallelogram into two congruent triangles.

2. (Motivate) In a parallelogram opposite sides are equal, and conversely.

3. (Motivate) In a parallelogram opposite angles are equal, and conversely.

4. (Motivate) A quadrilateral is a parallelogram if a pair of its opposite sides is parallel and equal.

5. (Motivate) In a parallelogram, the diagonals bisect each other and conversely.

6. (Motivate) In a triangle, the line segment joining the mid points of any two sides is parallel to the

third side and in half of it and (motivate) its converse.

5. CIRCLES (17) Periods

1. (Prove) Equal chords of a circle subtend equal angles at the center and (motivate) its converse.

2. (Motivate) The perpendicular from the center of a circle to a chord bisects the chord and conversely,

the line drawn through the center of a circle to bisect a chord is perpendicular to the chord.

3. (Motivate) Equal chords of a circle (or of congruent circles) are equidistant from the center (or their

respective centers) and conversely.

4. (Prove) The angle subtended by an arc at the center is double the angle subtended by it at any

point on the remaining part of the circle.

5. (Motivate) Angles in the same segment of a circle are equal.

6. (Motivate) If a line segment joining two points subtends equal angle at two other points lying on

the same side of the line containing the segment, the four points lie on a circle.

7. (Motivate) The sum of either of the pair of the opposite angles of a cyclic quadrilateral is 180° and

its converse.

(9)
Syllabus

UNIT V : MENSURATION
1. AREAS (5) Periods

Area of a triangle using Heron's formula (without proof)

2. SURFACE AREAS AND VOLUMES (17) Periods

Surface areas and volumes of spheres (including hemispheres) and right circular cones.

UNIT VI : STATISTICS
STATISTICS (15) Periods

Bar graphs, histograms (with varying base lengths), and frequency polygons.

( 10 )
Syllabus

MATHEMATICS
QUESTION PAPER DESIGN

Time : 3 hrs Max. Marks : 80

%
Total
S. No. Typology of Questions Weightage
Marks
(approx)
1. Remembering : Exhibit memory of previously learned
material by recalling facts, terms, basic concepts, and
answers.
43 54
Understanding : Demonstrate understanding of
facts and ideas by organizing, comparing, translating,
interpreting, giving descriptions, and stating main ideas
2. Applying : Solve problems to new situations by applying
acquired knowledge, facts, techniques and rules in a 19 24
different way.

3. Analysing : Examine and break information into parts


by identifying motives or causes. Make inferences and
find evidence to support generalizations
Evaluating : Present and defend opinions by making
judgments about information, validity of ideas, or 18 22
quality of work based on a set of criteria.
Creating : Compile information together in a different
way by combining elements in a new pattern or
proposing alternative solutions
Total 80 100

INTERNAL ASSESSMENT 20 MARKS


Pen Paper Test and Multiple Assessment (5+5) 10 Marks
Portfolio 05 Marks
Lab Practical (Lab activities to be done from the prescribed books) 05 Marks
qqq

( 11 )
Order Form

ONE FOR ALL OLYMPIADS (Multicolour) NCERT TEXTBOOK+EXEMPLAR


Classes 6 - 12
Classes 1 to 5
Science, Mathematics, Physics, Chemistry, Biology
Mathematics, English, Reasoning, General Knowledge,
Science & Cyber CBSE 10 PREVIOUS YEARS’ SOLVED PAPERS
ONE FOR ALL WORKBOOKS CONCEPT-WISE Class 10
(Multicolour) English Lang. & Lit, Science, Social Science, Sanskrit
Mathematics Standard & Computer Applications
Classes 1 to 5
Class 12
Mathematics, English, General Knowledge, Science Science Stream (PCM)-(Chemistry, Physics, Mathematics
& English Core), Science Stream (PCB)-(Chemistry, Physics,
NCERT WORKBOOKS (Multicolour / B&W) English Core & Biology) Commerce Stream- (Economics,
Accountancy, Business Studies, Mathematics & English
Classes 1 - 5 Core), Humanities Stream- (Geography, History, Political
English, Hindi, Mathematics & EVS Science & English Core)

ONE FOR ALL OLYMPIADS CBSE RMT FLASH CARDS


Class 10
Classes 6 to 8 Mathematics, English Lang. & Lit., Social Science &
Mathematics, English, Reasoning, General Knowledge, Science
Science & Cyber
CBSE ONE FOR ALL
NCERT & CBSE One For All Workbook Classes 6 - 8
Classes 6 to 8 English, Hindi, Mathematics, Science, Social Science
& Sanskrit
Mathematics, English, Science, Social Science, Hindi
& Sanskrit Classes 9 & 10
CBSE WorkBook Classes 9 & 10 Mathematics, English Lang. & Lit., Social Science,
Science, Hindi-A, Hindi-B, Mathematics (Stand.) &
English Lang. & Literature, Science, Social Science Mathematics Basic
& Mathematics
Class 12
CBSE & NCERT Laboratory Manual English Core, Physics, Chemistry, Mathematics, Biology,
Classes 6 - 12 Accoutancy, Business Studies & Economics

Mathematics, Science, Physics, Chemistry & Biology CBSE LMP-Last Minute Preparation System
Class 10
CBSE QUESTION BANKS (CHAPTER-WISE Combined (English Lang. & Lit., Science, Mathematics
& TOPIC-WISE SOLVED PAPERS) Standard & Social Science)
Classes 9 & 10 Class 12
Hindi-A, Hindi-B, English Lang. & Lit. (With NCERT Science (Physics, Chemistry, Mathematics, Biology &
Workbook), Science, Mathematics, Social Science, English Core) Commerce (Accountancy, Business
Computer Applications, Sanskrit & English Studies, Economics, Mathematics & English Core)
Communicative (With CBSE Textbook), Humanities (History, Geography, Political Science &
English Core)
Classes 11 & 12
Hindi Core, English Core, Mathematics, Chemistry,
CBSE SAMPLE QUESTION PAPERS
Applied Mathematics, Biology, Physics, Accountancy, Classes 9 & 10
Economics, Business Studies, Geography, History, Hindi-A, Hindi-B, English Lang. & Lit., Computer Appl.,
Political Science, Computer Science, Entrepreneurship, English Communicative, Mathematics, Science, Social
Informatics Practices, Physical Education, Sociology Science, Sanskrit & Samajik Vigyan (For Class 10th Only)
(For Class 12th Only), Psychology (For Class 12th Only)
Classes 11 & 12
NCERT EXEMPLAR PROBLEMS- Hindi Core, English Core, Mathematics, Applied
SOLUTIONS (Hindi & English Medium) Mathematics, Physics, Chemistry, Biology, Accountancy,
Economics, Business Studies, Geography, History,
Classes 9 - 12 Political Science, Computer Science, Informatics
Practices, Physical Education, Entrepreneurship (For
Science, Mathematics, Physics, Chemistry, Biology,
Class 12th Only), Psychology (For Class 12th Only) &
Mathematics, xf.kr] HkkSfrd foKku] jlk;u foKku] tho foKku Sociology (For Class 12th Only)
NCERT Textbook Solutions CBSE 20 Combined Sample Question Papers
Classes 6 - 12 Classes 10 & 12
Science, Mathematics, Physics, Chemistry, Biology Science, Commerce & Humanities

Exam Preparation Books for Class 1 to 12 : CBSE, CISCE Boards & Karnataka Board | OLYMPIADS |
JEE | NEET | CUET | NTSE | NDA | CDS | CAT | CTET | GATE | UPSC| RRB | BPSC | CLAT & AILET | UGC NET | NRA CET & more

We are available in all leading e-commerce platforms and all good bookstores.

( 12 )
PPositive
ositive A ffirmations
Affirmations

An affirmation is a positive statement or phrase that individuals repeat to


themselves with the intention of fostering a positive mindset, self-confidence, or
personal development. The practice involves affirming desired beliefs or qualities,
often in the present tense, to reinforce a positive self-image and encourage a
more optimistic outlook.
“ EMBRACE YOUR UNIQUENESS, NOURISH SELF-CARE, AND PRESS
FORWARD RELENTLESSLY. TO ACHIEVE FLIGHT, UNLOAD THE
BAGGAGE THAT SLOWS YOUR ASCENT."

I strongly trust in my vision and


work deligently towards
making it a reality. I am capable of learning and
growing everyday.

I am not defined by grades &


test scores. My true worth lies in
my character, compassion and
positive impact I have on others.

I celebrate my quirks, for they


I embrace my mistakes, learn make me beautifully unique.
from them and use them to
fuel my growth & resilience.

E
ANG
C H

I am not the product of


My voice matters and I circumstances, I am the
believe in the power of my product of my own choices.
words to shape the world
around me.
I face challenges with courage
and determination.

I believe that the twists &


I celebrate the achievements
turns of my life lead me to an
of others without diminishning
extraordinary destinations.
my journey.

( 13 )
Be mindful. Be grateful.
Be positive. Be true. Be kind

Three things Three people Three simple A challenging Three ways to


that make you are things you experience inject gratitude
you special grateful for are grateful that made into a current
and why for you stronger challenge

Describe the A fear you Three What made Three things


last time you did have activities you you smile you love about
something nice overcome enjoy most today? your family
for someone and why

What is your Three things The last time A risk you are Three everyday
favorite place, you love most you were grateful you items you are
and why? about overcome took and why grateful for
yourself with joy

Three songs What skill do you One luxury Describe a Three things
that bring you have that you you are rejection you about your
joy are grateful for thankful for are grateful body you are
and why? for grateful for

What are you Three things you Three items in Say thank you Something in
most grateful are grateful for your home to someone nature you are
for in your daily about where you are grateful for
life? you live grateful for

A person in your Something at Describe the What is your Three things


past you are school you’re last time you proudest you want to
grateful for grateful for laughed so accomplish- manifest
hard you cried ment?

( 14 )
Exclusive School Books Suppliers
ANDHRA PRADESH MAHARASHTRA
VIJAYAWADA Sri Vikas Book Centre, 9848571114, 9440715700, PUNE Madhusheela Books & Stationery, 7875899892
ASSAM JALNA Anil Paper Mart, 9422722522, (02482) 230733
WEST KAMENG Dutta Book Stall, 8729948473 TAMIL NADU
KARNATAKA CHENNAI Bookmark-IT, 7305151653
BANGLORE Satish Agencies, 8861630123 TELANGANA
GUJRAT HYDERABAD Sri Balaji Book Depot , 9676996199, (040) 27613300
RAJKOT Royal Stationers, 9824207514 WEST BENGAL
KOLKATA United Book House, 9831344622

Our Distributors
ANDHRA PRADESH INDORE Bhaiya Industries, 9893326853, Sushil Prakashan,(0731) 2503333, 2535892,
VISAKHAPATHAM JBD Educational, 9246632691, 9246633693, Sri Rajeshwari Book Link, 9848036014 9425322330, Bhaiya Store, 9425318103, Arun Prakashan, 9424890785,
VIJAYAWADA Akshaya Books Corner, 9666155555 Bhaiya Book Centre, 9424081874, Seva Suppliers, 9826451052
ANDAMAN & NICOBAR ISLAND JABALPUR Vinay Pustak Sadan, 8962362667, Anand Books and Stationers, 9425323508
PORTBLAIR Krishna Book Centre, 9474205570, Kumar Book Depot, 9932082455, Kumar Book SAGAR Princi Book Depot, Sagar, 9977277011
Depot, 9932082455, Sree aditya Book Centre, 8332972720, 7013300914
ASSAM KATNI Shri Mahavir Agency, 9425363412
GUWAHATI Book Emporium, 9675972993, 6000763186, UJJAIN Shreenath Book Depot, 9827544045
Ashok Publication, 7896141127,
Kayaan Enterprises, (0361) 2630443, Orchid Book house, 9864624209, BHOPAL Gupta Brother, 9644482444
Newco, 9864178188
BIHAR MAHARASHTRA
PATNA Nova Publisher & Distributors, (0612) 2666404, Shri Durga Pustak Mandir, 9334477386, PUNE Natraj Book Depot, (020) 24485054, 9890054092, Vikas Book House, 9921331187,
Sharda Pustak Bhandar, 9334259293, Vikas Book Depot, 9504780402, Alka Book Agency, Pravin Sales, 9890683475, New Saraswati Granth Bhandar, 9422323859, Akshar
Books & Stationary, 7385089789, Vardhaman Educational, 9860574354, Yash Book
9835655005, Metro Book(E&C), Ishu Pustak Bhandar, 8294576789, Gyan Ganga Limited, Centre, 9890156763, Pragati Book Centre, (ISC), 9850039311, Praveen Sales, Pragati
6203900312, Ishu Pustak Bhandar, ( E & C ), 9334186300/8294576789 Book Centre, Pune ( E & C ), 9850039311
MUZAFFARPUR Pustak Bhandar, 7870834225 AURANGABAD Shree Sainath Agencies, 7350294089, Maya Book Centre, (ISC), 9372360150
CHATTISGARH MUMBAI Vidyarthi Sales Agencies, 9819776110, New Student Agencies, 7045065799, Shivam
Books & Stationery, 8619805332
AMBIKAPUR Saini Brothers, 9425582561, M.P Department Stores, 9425254264 JALGAON Sharma Book Depot & Stat. (ISC), 9421393040
BOKARO Bokaro Student Friends Pvt. Ltd, Bokaro, 7277931285 LATUR Yash Book House, 9637936999, Shri Ganesh Pustakalay, 9730172188
BHILAI Anil Book Depot, 9425234260 KOLHAPUR Granth the Book World, 9922295522
DURG Bhagwati Bhawani Book Depot, 0788-2327620, 9827473100 NAGPUR Laxmi Pustakalay and Stationers, (0712) 2727354, Vijay Book Depot, 9860122094
KORBA Kitab Ghar, Korba ( E & C ), 9425226528 Renuka Book distributor, 9765406133, Novelty Book Depot, 9657690220, Karamveer
Book Depot, 9923966466, Arun Book & Stationers, 9423110953
RAIPUR Shri Ramdev Traders, 9981761797, Gupta Pustak Mandir, 7974220323, NANDED Abhang Pustakalaya, 9823470756/9175940756
Anil Publication, 9691618258/7999078802 NASHIK Rahul Book Centre, 9970849681, New India Book House, 9623123458
RAIGARH Sindhu Book Deopt, 9981935763 DHULE Navjeevan Book Stall, 7020525561
DELHI YAVATMAL Shri Ganesh Pustkalaya, 9423131275
DELHI Mittal Books, (011) 23288887, 9899037390, Singhania Book & Stationer, 9212028238, VASAI Prime Book Centre, Vasai, 9890293662
AoneBooks, New Delhi, 8800497047, Radhey Book Depot, 9818314141, Batheja
Super Store, 9871833924, Lov Dev & Sons, Delhi ( E & C ), 9811182352, Zombozone, ODISHA
9871274082, LDS Marketing, 9811182352/9999353491
CUTTACK A. K. Mishra Agencies, 9437025991, 9437081319
GUJARAT
AHMEDABAD Patel Book, 9898184248, 9824386112, 9825900335, Zaveri Agency, 9979897312, BHUBANESHWAR M/s Pragnya, 8847888616, 9437943777, Padmalaya, 9437026922, Bidyashree,
9979890330, Hardik Book Agency, (ISC) 079-24110043, 9904659821 9937017070, Books Godown, 7894281110
BHAVNAGAR Samir Book Stall, Bhavnagar (ISC) 9586305305 BARIPADA Trimurti Book World, 9437034735
DAHOD Collegian Book Corner, 9925501981 KEONJHAR Students corner, 7008435418
VAPI Goutam Book Sellers, 9081790813 PUNJAB
VALSAD Mahavir Stationers, 9429474177 AMBALA Bharat Book Depot, 7988455354
NAVSARI College Store, (ISC) NO CALL 02637-258642, 9825099121 PATIALA Goel Sons, 9463619978, Adarsh Enterprises, 9814347613
SURAT Shopping Point, 9824108663 JALANDHAR Cheap Book Store, 9872223458, 9878258592, City Book Shop, 9417440753, Subhash
Book Depot, 9876453625, Paramvir Enterprises, 9878626248
VADODARA Umakant Book Sellers & Stationer, 9624920709 FEROZPUR Sita Ram book Depot, 9463039199, 7696141911
HARYANA LUDHIANA Amit Book, 9815807871, Gupta Brothers, 9888200206, Bhatia Book Centre,
9815277131
ROHTAK Manish Traders, 9812556687, Swami Kitab Ghar, 9355611088, CHANDIGARH Mohindra Book Depot, 9814920226
Babu Ram Pradeep Kumar, 9813214692
REWARI Sanjay book depot, 9255447231 RAJASTHAN
BALLABGARH Kashi Ram Kishan lal, 9289504004, 8920567245 AJMER Laxmi General Store, Ajmer, 0145- 2428942 9460652197
HISAR Natraj Book Distributors, 7988917452 KOTA Vardhman Book Depot, 9571365020, 8003221190 Raj Traders, 9309232829
BHUNA Khurana Book Store, 9896572520 BHILWARA Nakoda Book Depot, (01482) 243653, 9214983594,
Alankar Book Depot, 9414707462
JAMMU JAIPUR Ravi Enterprises, 9829060694, Saraswati Book House, (0141) 2610823, 9829811155,
Goyal Book Distt., 9460983939, 9414782130
Sahitya Sangam, 9419190177 UDAIPUR Sunil Book Store, 9828682260
JHARKHAND JODHPUR Second Hand Book Stall, 9460004745
BOKARO Bokaro Student Friends, (0654) 2233094, 7360021503, Bharati Bhawan Agencies, TRIPURA
9431740797
RANCHI Crown Book Distributor & Publishers, (0651) 2213735, 9431173904, AGARTALA Book Corner, 8794894165, 8984657146, Book Emporium, 9089230412
Pustak Mandir, 9431115138,
DUMKA Vidyarthi Pustak Bhandar, 9431310228 TAMIL NADU
KARNATAKA COIMBATORE Majestic Book House, (0422) 2384333, CBSC Book Shop, 9585979752
HUBLI Renuka Book Distributor, (0836) 2244124, Vidyamandir Book Distributors, 9980773976 CHENNAI Arraba Book Traders, (044) 25387868, 9841459105, M.R. Book Store (044) 25364596,
Kalaimagal Store, (044) 5544072, 9940619404, Vijaya Stores, 9381037417, Bookmark
BANGLORE Krishna book house, 9739847334, Hema Book Stores, 9986767000, It-Books & Stat. Store, 7305151653, M.K. Store, 9840030099, Tiger Books Pvt. Ltd.,
9710447000, New Mylai Stationers, 9841313062, Prince Book House, Chennai,
Sapna Book House Pvt. Ltd., 9980513242, Hema Book World, 0444-2053926, 9952068491, S K Publishers & Distributors, 9789865544, Dharma
(Chamrajpet) (ISC) 080-40905110, 9945731121 Book Shop, 8667227171
BELLERI Chatinya book centre, 9886064731 PUDUCHERRY Sri Lakshmi Book Seller, 7871555145
KERALA SALEM Pattu book centre, 9894816280
ERNAKULAM Academic Book House, (0484) 2376613, H & C Store, 9864196344, TRICHY P.R.Sons Book Seller, 9443370597, Rasi Publication, 9894816280
Surya Book House, 9847124217, 9847238314
KOTTAYAM Book Centre, (0481) 2566992 THENI Maya Book Centre, 9443929274
TRIVANDRUM Academic Book House, (0471) 2333349, 9447063349, Ponni Book Stall, 9037591721 MADURAI Selvi Book Shoppe, 9843057435, Jayam Book Centre, 9894658036
CALICUT Aman Book Stall, (0495) 2721282, VELLORE G.K book centre and collections, 9894517994
MADHYA PRADESH TELANGANA
CHHINDWARA Pustak Bhawan, ( E & C ), 8982150100 HYDERABAD Sri Balaji Book Depot, (040) 27613300, 9866355473, Shah Book House, 9849564564
GWALIOR Agarwal Book Depot, 9425116210 Vishal Book Distributors, 9246333166, Himalaya Book World, 7032578527
0808
Exclusive School Books Suppliers
UTTARAKHAND GORAKHPUR Central Book House, 9935454590, Friends & Co., 9450277154, Dinesh book depot,
9125818274, Friends & Co., 9450277154
DEHRADUN Inder Book Agencies, 9634045280, Amar Book Depot , 8130491477, Goyal Book Store, JHANSI Bhanu Book Depot, 9415031340
9897318047, New National Book House, 9897830283/9720590054
MUSSORIE Ram Saran Dass Chanda kiran, 0135-2632785, 9761344588 KANPUR Radha News Agency, 8957247427, Raj Book Dist., 9235616506, H K Book Dis-
tributors, 9935146730, H K Book Distributors, 9506033137/9935146730
UTTAR PRADESH LUCKNOW Vyapar Sadan, 7607102462, Om Book Depot, 7705871398, Azad Book Depot Pvt. Ltd.,
7317000250, Book Sadan, 9839487327, Rama Book Depot(Retail), 7355078254,
Ashirwad Book Depot, 9235501197, Book.com, 7458922755, Universal Books,
9450302161, Sheetla Book Agency, 9235832418, Vidyarthi Kendra Publisher &
Distributor Pvt Ltd, (Gold), 9554967415, Tripathi Book House, 9415425943
AGRA Sparsh Book Agency, 9412257817, Om Pustak Mandir, (0562) 2464014, 9319117771, MEERUT Ideal Book Depot, (0121) 4059252, 9837066307
Sanjay Publication, 8126699922 Arti book centre, 8630128856, Panchsheel Books,
9412257962, Bhagwati Book Store, (E & C), 9149081912
ALLAHABAD Mehrotra Book Agency, (0532) 2266865, 9415636890 NOIDA Prozo (Global Edu4 Share Pvt. Ltd), 9318395520, Goyal Books Overseas Pvt.Ltd.,
1204655555 9873387003
AZAMGARH Sasta Sahitya Bhandar, 9450029674 PRAYAGRAJ Kanhaiya Pustak Bhawan, 9415317109

ALIGARH K.B.C.L. Agarwal, 9897124960, Shaligram Agencies, 9412317800, New Vimal Books, MAWANA Subhash Book Depot, 9760262264
9997398868, T.I.C Book centre, 9808039570
BULANDSHAHAR Rastogi Book Depot, 9837053462/9368978202
WEST BENGAL

BALRAMPUR Universal Book Center, 8933826726 KOLKATA Oriental Publishers & Distributor (033) 40628367, Katha 'O' Kahini, (033)
22196313, 22419071, Saha Book House, (033), 22193671, 9333416484, United
Book House, 9831344622, Bijay Pustak Bhandar, 8961260603, Shawan Books
Distributors, 8336820363, Krishna Book House, 9123083874
BAREILLY Siksha Prakashan, 9837829284 RENUKOOT Om Stationers, 7007326732

HARDOI Mittal Pustak Kendra, 9838201466

DEORIA Kanodia Book Depot, 9415277835 COOCH BEHAR S.B. Book Distributor, Cooch behar, 9002670771

VARANASI Gupta Books, 8707225564, Bookman & Company, 9935194495/7668899901 KHARAGPUR Subhani Book Store, 9046891334

MATHURA Sapra Traders, 9410076716, Vijay Book House , 9897254292 SILIGURI Agarwal Book House, 9832038727, Modern Book Agency, 8145578772

FARRUKHABAD Anurag Book Agencies, 8844007575 DINAJPUR Krishna Book House, 7031748945

NAJIBABAD Gupta News Agency, 8868932500, Gupta News Agency, ( E & C ), 8868932500 MURSHIDABAD New Book House, 8944876176
DHAMPUR Ramkumar Mahaveer Prasad, 9411942550

Entrance & Competition Distributors


BIHAR CUTTAK A.K.Mishra Agencies, 9437025991

PATNA Metro Books Corner, 9431647013, Alka Book Agency, 9835655005, Vikas Book BHUBANESH- M/s Pragnya, 9437943777
Depot, 9504780402 WAR

CHATTISGARH PUNJAB

KORBA Kitab Ghar, 9425226528, Shri Ramdev Traders, 9981761797 JALANDHAR Cheap Book Store, 9872223458, 9878258592

DELHI RAJASTHAN

DELHI Singhania Book & Stationer, 9212028238, Radhey Book depot, 9818314141, The KOTA Vardhman Book Depot, 9571365020, Raj Traders, 9309232829
Book Shop, 9310262701, Mittal Books, 9899037390, Lov Dev & Sons, 9999353491
NEW DELHI Anupam Sales, 9560504617, A ONE BOOKS, 8800497047 JAIPUR Goyal Book Distributors, 9414782130

HARYANA UTTAR PRADESH

AMBALA Bharat Book Depot, 7988455354 AGRA BHAGWATI BOOK STORE, 9149081912, Sparsh Book Agency, 9412257817,
Sanjay Publication, 8126699922
JHARKHAND ALIGARH New Vimal Books, 9997398868

BOKARO Bokaro Student Friends Pvt. Ltd, 7360021503 ALLAHABAD Mehrotra Book Agency, (532) 2266865, 9415636890

MADHYA PRADESH GORAKHPUR Central Book House, 9935454590

INDORE Bhaiya Industries, 9109120101 KANPUR Raj Book Dist, 9235616506

CHHINDWARA Pustak Bhawan, 9827255997 LUCKNOW Azad Book Depot PVT LTD, 7317000250, Rama Book Depot(Retail), 7355078254
Ashirwad Book Depot , 9235501197, Book Sadan, 8318643277, Book.com ,
7458922755, Sheetla Book Agency, 9235832418
MAHARASHTRA PRAYAGRAJ Format Center, 9335115561, Garg Brothers Trading & Services Pvt. Ltd.,
7388100499
NAGPUR Laxmi Pustakalay and Stationers, (0712) 2727354 UTTAR PRADESH

PUNE Pragati Book Centre, 9850039311 DEHRADUN Inder Book Agancies, 9634045280

MUMBAI New Student Agencies LLP, 7045065799 WEST BENGAL

ODISHA KOLKATA Bijay Pustak Bhandar Pvt. Ltd., 8961260603, Saha Book House, 9674827254
United Book House, 9831344622, Techno World, 9830168159
BARIPADA Trimurti Book World, 9437034735
0808

( 16 )
UNIT-I NUMBER SYSTEMS Study Time:
Maximum time: 3:15 Hrs
Maximum questions: 112
CHAPTER

1
REAL
NUMBERS

Review of representation of natural numbers, integers and rational numbers on


Syllabus the number line. Rational numbers as recurring/terminating decimals. Opera-
tions on real numbers.

Examples of non-recurring/non-terminating decimals. Existence of non-rational numbers (irrational numbers) such as


2 , 3 and their representation on the number line. Explaining that every real number is represented by unique point
on the number line and conversely viz. every point on the number line represents a unique real number and Definition
of nth root of a real number.

1 1
Rationalization (with precise meaning) of real numbers of the type and (and their combinations)
a+b x x+ y
where x and y are natural numbers and a and b are integers.
Recall of laws of exponents with integral powers. Rational exponents with positive real bases (to be done by particular
cases, allowing learner to arrive at the general laws).

List of Topics
Topic-1: Rational Numbers
 Page No. 1
Topic-2 : Irrational Numbers
Topic-1 Rational Numbers  Page No. 7
Topic-3 : nth Root of a Real Number
 Page No. 9
Topic-4 : Laws of Exponents with
Integral Powers
Revision Notes Page No. 12
Topic-5 : Rationalization of Real
Numbers Page No. 16
 Rational Number : A number ’r’ is called a rational number, if it can be
written in the form p/q, where p and q are integers and q ≠ 0, denoted
by ‘Q’.
1 3 4 2
e.g., , , , − etc. are all rational numbers.
2 4 5 3

p 
Symbolically, Q =  q , q ≠ 0 and p, q ∈ I 
 
Scan to know
 Decimal Expansion of Real Numbers : The decimal expansion of real more about
number is used to represent a number on the number line. this topic

If the decimal expansion of a real number is either terminating or non-terminating (recurring),


then the real number is called a rational number.
 Cases in Rational Number:
p Introduction
Case 1 : When Remainder becomes Zero : Every rational number , (q ≠ 0) can be expressed as of Rational
q and Irrational
Numbers
a decimal. On dividing p by q, when the remainder becomes zero, then the decimal is called a
terminating decimal.
2 Oswaal CBSE Question Bank Chapterwise & Topicwise, MATHEMATICS, Class-IX
REAL NUMBERS 3
1
e.g., = 0.5
2
On dividing 1 by 2, we get value 0.5 i.e., remainder equal to zero, so 0.5 is a terminating decimal.
P
 Terminating decimal : If a rational number (¹ integer) can be expressed in the form n m P Î Z, (n, m) Î W, the
2 ×5
rational number will be a terminating decimal.
5 5 5
For Example : = 3 , is terminating decimal.
8 2 × 50 8

p
 Non-terminating decimal : If in a rational number , prime factors of q are other than 2 and 5, then the rational
q
number is Non-terminating decimal.
4 4 4
For Example : = , is non terminating decimal.
45 3 2 × 2 0 × 51 45
p
Case 2 : When remainder never becomes Zero — A rational number expressed in the form of
or division of
q
p by q, when remainder never becomes zero and set of digits repeat periodically then the decimal is called non-
terminating recurring or repeating decimal. It is denoted by the bar over it.
1
e.g., = 0.333.... = 0.3
3
On dividing 1 by 3, we get 3 again and again in the decimal part of the quotient i.e., remainder never becomes
zero, so 0.3 is a non-terminating repeating decimal.
 Every integer is a rational number.
 There are infinitely many rational numbers between any two given rational numbers.
 If x and y are any two rational numbers, then :
(i) x + y is a rational number
(ii) x – y is a rational number
(iii) x × y is a rational number
(iv) x ÷ y is a rational number, (y ≠ 0).

Example 1
p On multiplying eqn. (i) by 10, we get
Express 0.5 in the form of . 10x = 5.555..... ...(ii)
q
Step III : Subtracting eqn. (i) from (ii), we get
Solution:
10x – x = 5.555....... – 0.555......
Step I : Assume the given decimal expansion
as x and count the number of digits which are or 9x = 5
repeated. 5
Let x = 0.5 or x=
9
or x = 0.555...... ...(i) 5
Here, 1 digit is repeated that is, 5. Hence, 0.5 =
9
Step II : Multiply both sides by 10 (because one
digit is repeating)

Mnemonics
1. We use R symbol. Think R as in ‘Real’. The quick way to remember real numbers is that they’re the numbers that
truly exist and can be represented on number line.
N : Naturally (Natural numbers)
I : Involve (Integers)
R : Relation (Rational Numbers)
I : Insanity (Irrational Numbers)
Rational Numbers
These numbers can be expressed as a ratio of two integers hence the name rational numbers.
Denoted by symbol ‘Q’ - as in quotient so ratio means on dividing we get quotient.
4 Oswaal CBSE Question Bank Chapterwise & Topicwise, MATHEMATICS, Class-IX

SUBJECTIVE TYPE QUESTIONS


Very Short Answer Type
Commonly Made Error
Questions (1 mark each)
Q. 1. Find two rational numbers between 4 and 5. While inserting rational number between two
U [Board Term I, 2016] rational numbers, students do not convert the
rational numbers into like fractions.
4 5
Sol. × 5 and 5 = × 5
4=
5 5
20 25
Answering Tip
i.e., 4= and 5 =
5 5
While inserting rational number between two
21 22 . rational number, students should convert the
The numbers are and 1
5 5 rational numbers into like fractions.
[CBSE Marking Scheme, 2016] Q. 5. Write the sum of 0.3 and 0. 4 . U
58 Sol. 0.3 + 0.4 = (0.333.....) + (0.444.....)
Q. 2. Find the decimal expansion of .
1000
= 0.777......
R [Board Term I, 2015] Let, x = 0.777.......
58 10x = 7.777.......
Sol. = 0.058 (Decimal point is shifted three places
1000 or, 10x – x = (7.777.......) – (0.777.......)
to the left.)  1 or, 9x = 7
7
or, x= 1
Commonly Made Error 9

Students make an error while placing decimal Short Answer Type


while dividing a number by denomination of Questions-I (2 marks each)
10’s. Q. 1. Is zero (0) a rational number ? Justify your
answer. R [Board Term I, 2015]
Sol. Yes, zero is a rational number.
Answering Tip 0 0 0
Zero can be expressed as , , etc,
5 26 100
Students should remember while multiplying,
decimal is placed to the right counting the p
which are in the form of where p and q are
number of zeros and while dividing to the left q
as many zeros as are there.
integers and q ≠ 0. 2
98
Q. 3. Is a rational number or not ? U Q. 2. If 7x = 1, then find the decimal expansion of x.
2
U [Board Term I, 2012]
98 98
Sol. = = 49 = 7
2 2 1
Sol. x =
So, it is a rational number. 1 7
2 0.142857
Q. 4. Insert three rational numbers between − and
3 )
7 1.000000
1 –7
− . U [Board Term I, 2016] 30
3 – 28
1 4 20
Sol. − = − – 14
3 12 60
– 56
2 8 40

and − = −
3 12 – 35
50
So three rational numbers are – 49
5 6 7 1
− , − and − . 1
12 12 12  \ x = 0.142857 2
[CBSE Marking Scheme, 2012]
REAL NUMBERS 5
p 5 9
Q. 3. Express – 0.00875 in the form , where p and q are Hence, three rational numbers between and are:
q 7 11
integers and q ¹ 0. 56 57 58
U [Board Term I, 2016] , , 1
77 77 77
875 Q. 2. Find six rational numbers between 3 and 4.
Sol. –0.00875 = -
100000
U [Board Term I, 2014]
- 35 -7
= = 2 Sol. Let a = 3 and b = 4
4000 800
Here, we find six rational numbers, i.e., n = 6
[CBSE Marking Scheme, 2016]
b−a 4−3 1
2157 So, d= = =
Q. 4. Express in the decimal form and state n+1 6+1 7
625
whether it is terminating or not. 1 22
1st rational number = a + d = 3 + = ½
R [Board Term I, 2012] 7 7
Sol. 3.4512 2 23
2nd rational number = a + 2d = 3 + = ½
625) 2157 1½ 7 7
–1875
3 24
2820 3rd rational number = a + 3d = 3 + = ½
– 2500 7 7
3200 4 25
4th rational number = a + 4d = 3 + = ½
–3125 7 7
750
5 26
–625 5th rational number = a + 5d = 3 + = ½
7 7
1250
– 1250 6 27
6th rational number = a + 6d = 3 + = ½
× 7 7
2157 So, six rational numbers are:
= 3.4512 (Terminating) ½
625 22 23 24 25 26 27
Q. 5. Find the rational numbers between , , , , and
7 7 7 7 7 7
0.121221222122221 ..... and 0.141441444144441 .... in
p 1 1
the form, where p and q are integers and q ¹ 0. Q. 3. Find four rational numbers between and .
q 6 5

R [Board Term I, 2016] U [Board Term I, 2012]
Sol. Since, LCM of 5 and 6 is 30.
Sol. Two rational numbers between 0.1212212221 ... 1 1 5 5 5 25
and 0.1414414441 ... are 0.13 and 0.14. \ = × = × = ½
6 6 5 30 5 150 
13 14
i.e., and 1 1 6 6 5 30
100 100 and = × = × = ½
5 5 6 30 5 150 
13 7 1 1
or, and . 2 Hence, four rational numbers between and
100 50 6 5
[CBSE Marking Scheme, 2016] are :
26 2627 2728 2829 29
, ,, , ,and, ½+½+½+½
Short Answer Type 150 150 150
150 150 150 150 150 
Questions-II (3 marks each)
Q. 4. Express 0·328 in the form of
p
, where p and q are
5 9 q
Q. 1. Find three rational numbers between and .
7 11 integers and q ¹ 0. A [Board Term I, 2012]
U [Board Term I, 2014] Sol. Let x = 0·328 = 0·3282828....... ½
Sol. Since, LCM of 7 and 11 is 77. or, 10x = 3·282828..... ½
5 5 11 55 or, 1000x = 328·282828........ ½
\ = × = 1 or, 1000x – 10x = 328·2828..... – 3·2828..... ½
7 7 11 77
or, 990x = 325·000
9 9 7 63 325 65
and = × = 1 or, x = = 1
11 11 7 77 990 198 
6 Oswaal CBSE Question Bank Chapterwise & Topicwise, MATHEMATICS, Class-IX

or, 100x – 10x = (132.222.......) – (13.222.....)


Long Answer Type
or, 90x = 119
Questions (5 marks each)
119
or, x= 2
Q. 1. Give two rational numbers whose 90

(i) difference is a rational number, Again, let y = 0.35 = 0.353535.......

(ii) sum is a rational number, or, 100y = 35.3535........
(iii) product is a rational number, or, 100y – y = (35.3535......) – (0.3535.......)
(iv) division is a rational number. or, 99y = 35
35
Justify also. R [Board Term I, 2015] or, y= 2
99
Sol. Any example and verification of example :
119 35
4 9 \ 1.32 + 0.35 = x + y = +
Let m = , n = 90 99
5 2
119 × 11 + 35 × 10
=
9 4 37 990
(i) Difference= − = (Rational Number) 1½
2 5 10 1309 + 350
=
45 - 8 37 990
=
10 10 1659
=
4 9 53 990
(ii) Sum = + = (Rational Number)
5 2 10 553
= 1
84+ 459 553 330
+ = 
510 2 10
2 1½ p
Q. 4. Express 0.6 +0.7 +0.47 in the form , where p
q
4 9 36 18
(iii) Product = × = =
5 2 10 5
 (Rational Number) 1 and q are integers and q ≠ 0. Ap

9 4 45 Sol. Let x = 0. 7 = 0.777... …(i)


(iv) Division = ÷ = (Rational Number) 1
2 5 8 Multiply both sides by 10
3
10x = 7.777... …(ii)
Q. 2. Arrange in descending order 2, 4 5, 6
7 and
Subtracting (i) from (ii),
12
3. U [Board Term I, 2012]
9x = 7

Sol. LCM of 3, 4, 6 and 12 is 12 7


x= 1½
1 4 9
3
=
2 2= 2 3 12
= 12
16
1 3 Let x = 0.47 = 0.477... …(iii)
4
=
5 5= 5 4 12
= 12
125
Multiply both sides of (iii) by 10
1 2
6
=
7 7= 7 6 12
= 12
49 10x = 4.777... ...(iv)
1 Multiply both sides of (iv) by 10
=
12
3 3= 12 3 12

100x = 47.777... ...(v)
Descending order is
Subtracting (iv) from (v),
12
125 , 12
49 , 12
16 and 12
3.
 90x = 43
i.e.,
4
5, 6
7, 3
2 and 12
3. 1+1+1+1+1 43
 x=
[CBSE Marking Scheme, 2012] 90  1½
6 7 43
p \
0.6 + 0. 7 + 0.47 = + +
Q. 3. Express 1.32 + 0.35 in the form , where p and q are 10 9 90
q
54 + 70 + 43
integers and q ≠ 0. A [Board Term I, 2012] =
90
Sol. Let, x = 1.32 = 1.32222.......
167
or, 10x = 13.222.......... =  2
90
or, 100x = 132.222......
REAL NUMBERS 7

Topic-2 Irrational Numbers

Revision Notes
 Irrational Number : If a number cannot be written in the form of p/q, where q ≠ 0 and p, q ∈ I, then it is called an
irrational number.

e.g., 2 , 3 , 5 , 2 + 5 , 3 − 7 , π etc. are all irrational numbers.
 The decimal expansion of an irrational number is non-terminating and non-recurring.
 The addition, subtraction, multiplication and division of rational and irrational number is an irrational number.
i.e., If x and y are two real numbers where x is rational and y is an irrational, then
(i) x + y is an irrational number.
(ii) x – y is an irrational number.
(iii) x × y is an irrational number.
(iv) x ÷ y is an irrational number.

Example 2

5 is an irrational number. 5q2 = p2 ...(i)
Prove that 5 divides 5q2 so 5 divides p2 also.
Solution: \ 5 divides p.
Let us assume 5 be rational number, Let p = 5a where a is an integer
On squaring both sides
p
then 5 = p2 = 25a2 ...(ii)
q Substituting value of p2 from (i) into (ii) we get
(where p and q are integers and q ¹ 0) 5p2 = 25a2
and p, q have no common factor other than 1. q2 = 5a2
p (5 divides 5a2 so 5 divides q2 as well)
So, 5 = \ 5 divides q
q
Thus 5 is common factor of p and q.
p2 But this contradicts the fact that p and q have no
or 5= (on squaring both sides) common factor other than 1.
q2
Hence, 5 is irrational number.

SUBJECTIVE TYPE QUESTIONS


Sol. No, it may be rational or irrational. 1
Very Short Answer Type Q. 3. Write the sum of 2 5 and 3 7 . R
Questions (1 mark each)
Sol. Sum of 2 5 and 3 7 = 2 5 + 3 7 .  1
Q. 1. Identify an irrational number among the following
Q. 4. Calculate the irrational number between 2 and 2.5.
numbers : 0.13 , 0.1315 , 0.1315 , 0.3013001300013 ...  U
R [Board Term I, 2014] Sol. Irrational number = 2×2.5 = 5
Sol. 0.13 is a terminating number. So, it is not an
irrational number. Since, 5 = 2.236....
0.1315 = 0.131515......, is repeating continuously, so Hence, the irrational number between 2 and 2.5 is
it is not an irrational number. 5. 1
0.1315 = 0.13151315...., is repeating continuously,
( 2 + 5) .
2

so, it is not an irrational number. Q. 5. Simplify the number A


0.3013001300013...., is non-terminating and non-
( 5) = ( 2) + ( 5) + 2 ×
2 2 2
recurring decimal. Hence, it is an irrational number. Sol. 2+ 2× 5
So, 0.3013001300013... is an irrational number. 1 = 2 + 5 + 2 10
Q. 2. Is the product of two irrational numbers always an
irrational number ? R = 7 + 2 10 (Irrational Number) 1
8 Oswaal CBSE Question Bank Chapterwise & Topicwise, MATHEMATICS, Class-IX

Short Answer Type So, 3 – r is rational, so 2 is also rational which


Questions-I (2 marks each) contradicts the statement that 2 is an irrational
Q. 1. Find any two irrational numbers between 0.1 and number. 1

0.12. R [Board Term I, 2014] \ 3 - 2 is irrational number.


1
Hence Proved
Sol. Required two irrational numbers are :
Q. 3. Simplify:
(i) 0.10100100010000 ......... and 1
(a) 6 5 ´ 2 5
(ii) 0.10200200020000 ........ 1
(b) 8 15 ¸ 2 3
Q. 2. Find any two irrational numbers between 0.5 and
0.55. R [Board Term I, 2012] Sol. (a) 6 5 ´ 2 5
Sol. Required two irrational numbers are : = ( 6 ´ 2 ) 5 ´ 5 1½
(i) 0.5101001000100001..... and 1 = 12 × 5 = 60
(ii) 0.502002000200002.... 
1 (b) 8 15 ¸ 2 3
1 2
Q. 3. Find an irrational number between and , = 8 5 ´ 3 ¸ 2 3 1½
7 7
1 8 5´ 3
when it is given that = 0.142857 = 4 5
7 2´ 3
U [Board Term I, 2012] 2 1
Q. 4. Insert an irrational number between - and .
1 5 2
Sol. Given, = 0.142857142857......... ½
7
2 1
2 Sol. One irrational number between - and
\ ½
= 0.285714285714...... 5 2
7  5) 2 (0.4
Hence, required irrational number lies between –0
0.142857 ...... 0.285714 20
It can 0.142858 or 0.20203 20
0
Short Answer Type 2
Questions-II (3 marks each) - = –0.4 1
5
5 9 2) 1 (0.5
Q. 1. Find three irrational numbers between and .
7 11 –0
U [Board Term I, 2016] 10
10
5 0
Sol. = 0·714285
7 1
= 0.5 1
9 2
= 0·81 1
11 So, there are infinite irrational numbers between
5 2 1
Hence, three irrational numbers between and - and
7 5 2
9 one irrational number among them can be
can be :
11 0.1010010001
0·727227222........... 1
0·737337333...........
Long Answer Type
0·747447444........... 1 Questions (5 marks each)
[CBSE Marking Scheme, 2016] Q. 1. Give an example of two irrational numbers whose:
Q. 2. 2 is an irrational number. Prove that 3 - 2 is (i) difference is an irrational number,
also an irrational number (ii) sum is an irrational number,
(iii) product is an irrational number,
Ap [Board Term I, 2013] (iv) division is an irrational number.
Sol. 2 is an irrational number (given) Justify also. R [Board Term I, 2015]
Let us assume 3 - 2 be a rational number ‘r’ Sol. Let two irrational numbers be :

So
3- 2 =r 1 6 and
3 , 1

3–r= 2 (i) 6 − 3
We know that r is rational = Difference is an irrational number. 1
REAL NUMBERS 9

(ii) 6 + 3 7 6
= is an irrational number
= sum is an irrational number. 1 12
(iii) 6 × 3 = 18 = 3 2 3 1
(d) +
= product is an irrational number. 1 8 2
6
(iv) = 2 3 1
3 = +
2´2´2 2
= division is an irrational number. 1
3 1
Q. 2. Simplify : = + 1½
2 2 2
(a) 45 - 3 20 + 4 5
3+2
(b) 3 3 + 2 27 + 7 3 = (By taking LCM)
2 2
24 54 5
(c) + =
8 9 2 2
3 1
(d) + 5 2
8 2 = ´ (By rationalizing the denominator)
2 2 2
Sol. (a) 45 - 3 20 + 4 5
5 2
3´3´5 - 3 2´2´5 + 4 5 1 = is an irrational number
4
3 5 - 3´2 5 + 4 5
3 5 -6 5 +4 5 Q. 3. Examine whether 2 is rational or irrational.

7 5 -6 5 Ap [Board Term I, 2016]

Sol. If possible let 2 be rational and let its simplest
= 5 is an irrational number
a
(b) 3 3 + 2 27 + 7 3 form be . 1
b
3 3 + 2 3´3´3 + 7 3 1 where a and b are integers having no common
3 3 + 2´3 3 +7 3 factor other than 1 and b ¹ 0.

a a2
3 3 +6 3 +7 3 Now, 2 = or, 2 = 2 [on squaring both sides] 1
b b
(3 + 6 + 7 ) 3
or, 2b2 = a2 ...(i)
= 16 3 is an irrational number or, 2 divides a2 [∵ 2 divides 2b2]
24 54 or, 2 divides a [∵ 2 is prime and
(c) + divide a2]  1
8 9
Let a = 2c for some integer c
2´2´2´3 2´3´3´3 Putting a = 2c in (i), we get
= + 2b2 = 4c2 or, b2 = 2c2
8 9
or, 2 divides b2 [∵ 2 divides 2c2] 1
2 6 3 6 or, 2 divides b [2 is prime and 2 divides b2]
= +
8 9 Thus, 2 is a common factor of a and b.
6 6 But this contradicts the fact that a and b have no
= + 1½ common factor other than 1.
4 3 The contradiction arises by assuming that 2 is
By taking LCM
rational.
3 6 +4 6
= Hence, 2 is irrational. 1
12

Topic-3 nth Root of a Real Number

Revision Notes
 Definition: In an = b, a and b are real numbers and n is a positive integer,.
(i) a is an nth root of b.
n
(ii) It can also be written as b = a.
10 Oswaal CBSE Question Bank Chapterwise & Topicwise, MATHEMATICS, Class-IX

(iii) It is also known as radical.



4
Example : (i) 3 is fourth root of 81 i.e., 34 = 81 or 3 =
81
nd
 Square root: The “2 ” root is the square root.
 Cube root: The “3rd” root is the cube root.

 a × a = a : Square root is used two times in a multiplication to get the original value.
3
 a × 3 a × 3 a = a : Cube root is used three times in a multiplication to get the original value.
n
× n
 a × ..........
a n
a = a : The nth root is used n times in a multiplication to get the original value.
n terms

 Identities used for radicals : Identities for two positive real numbers r and s : Scan to know
more about
r r this topic
(i)
rs = r · s (ii) =
s s

(iii)
( r+ s )( r − s =r−s) ( )(
(iv) r + s r − s = r 2 − s )
nth root of
( )
2
(v) r− s =r−2 r s+s number

 Laws of radicals : Laws for two positive real numbers a and b :
mn
(i)

n n
a = a (ii) a =nma
n
a na
(iii)

n
a n b = n ab , (a, b > 0 be real number) (iv) =
n
b b
p n
a p
(v) p m
= an − m (vi)
p n
a × am = an + m
p

a
1
( an )
p m p −m
(vii) = anm (viii) a = a m


Examples :
(i) 2 × 2 = 2 × 2 = 2

( )( ) ( 2) − ( 3)
2 2
(ii) 2+ 3 2− 3 = = 2 − 3 = −1

( ) = ( 5) + ( 7)
2 2 2
(iii) 5+ 7 + 2 5 7 = 5 + 7 + 2 35 = 12 + 2 35

9 9 3
(iv) = =
4 4 2

Example 3

Simplify : 4
81 − 8 3 216 + 15 5 32 + 225 Step III : Multiplying the powers
1 1 1 1
Solution: 4×
− 8.( 6 )

+ 15.( 2 )

+ (15)

i.e., ( 3) 4 3 5 2
Step I : Write the exponents in the form of powers
1 1 1 1 = 3 – 8 (6) + 15 (2) + 15
i.e., ( 81) 4 − 8.( 216 ) 3 + 15.( 32 ) 5 + ( 225) 2
Step IV : Solving the expression
Step II: Factorize the radical 3 – 48 + 30 + 15 = 0
1 1 1 1
Here, ( 3 4 ) 4 − 8.( 6 3 ) 3 + 15( 2 5 ) 5 + (15 2 ) 2 48 – 48 = 0
REAL NUMBERS 11

SUBJECTIVE TYPE QUESTIONS


Q. 3. Simplify : 3 3 40 − 4 3 320 − 3 5 .
Very Short Answer Type
U [Board Term I, 2012]
Questions (1 mark each)

(
Q. 1. Simplify : 5 + 5 5 − 5 . )( ) Sol.
3
40 =
3
23 × 5 = 2 3 5 ½
3 3
320 = 4 5 ½
U [Board Term I, 2014]
3 3 3 3 3 3
∴ 3 40 − 4 320 − 5 = 3 × 2 5 − 4 × 4 5 − 5

Sol. (5 + 5 ) (5 − 5 ) {2
= 5 − ( 5) }
2
½
= −113 5
 [CBSE Marking Scheme, 2012]
1

= 25 – 5 Detailed Solution :
= 20 ½ 3
40 = 3
2 × 2 × 2 × 5 = 23 5
[CBSE Marking Scheme, 2014] 3
320 = 3 2 × 2 × 2 × 2 × 2 × 2 × 5
Q. 2. Write the equivalent of 12 × 8 . U
= 2 × 23 5 = 43 5
Sol. 12 × 8 = 2 3 × 2 2 = 4 3 × 2 = 4 6 . 1 3 3 3
\ 3 40 − 4 320 − 5
Q. 3. Calculate the value of 4 28 √ 3 7 . U = 3 × 23 5 − 4 × 43 5 − 3 5
8 1
Sol. 4 28 ÷ 3 7 = 4 × 2 7 ÷ 3 7 = = 6 3 5 − 16 3 5 − 3 5 = −113 5 .
3
Q. 4. If b > 0 and b2 = a, then find the value of a . R Q. 4. Find the product of 5 2 3 + 2 5 + 2 . ( )( )
2
Sol. b = a or, a = b. 1 U [Board Term I, 2012]
Q. 5. Simplify : 72 + 800 − 18 . (
Sol. 5 2 3 + 2 5 + 2 )( )
 U [Board Term I, 2016]

(
= 5 2 15 + 3 2 + 5 2 + 2 ) 1
Sol. 72 + 800 − 18
= 5 2 (17 + 8 2 )

= 36 × 2 + 400 × 2 − 9 × 2
= 85 2 + 40 × 2
= 6 2 + 20 2 − 3 2 = 23 2 1
= 85 2 + 80 1
 [CBSE Marking Scheme, 2016]
Short Answer Type
Short Answer Type Questions-II (3 marks each)
Questions-I (2 marks each) Q. 1. Simplify : 3 45 − 125 + 200 − 50 .
Q. 1. Simplify : 8 3 − 2 3 + 4 3 .  U [Board Term I, 2014]
U [Board Term I, 2015] Sol. 3 45 − 125 + 200 − 50

3 (8 − 2 + 4 ) = 9 5 − 5 5 + 10 2 − 5 2 1½
Sol. 8 3 −2 3+4 3 =
= 4 5+5 2  1½
= 10 3 2
−1
 [CBSE Marking Scheme, 2015] Q. 2. Find the value of (729 ) . U [Board Term I, 2012]
6

Q. 2. Simplify : 50 − 98 + 162 .
1
(729 ) 6 = ( 36 )
−1 −
U [Board Term I, 2012] Sol. 6 = 3 −1
 2
Sol. 50 - 98 + 162 1 1
1 =
= 5×5×2 − 7×7×2 + 3×3×3×3×2 ½ 3 
[CBSE Marking Scheme, 2012]
= 5 2 −7 2 +9 2 ½
Q. 3. Evaluate : 5 + 2 6 + 8 − 2 15 .
= 7 2 [CBSE Marking Scheme, 2012] U [Board Term I, 2012]
12 Oswaal CBSE Question Bank Chapterwise & Topicwise, MATHEMATICS, Class-IX

3 6 +4 6
Sol. 5+2 6 = 3+2+2 6 = 1
12
( )
2
3+ 2 7 6
= = 1
12
= 3+ 2 1
Evaluate
Q. 2. 4
Ap
8 − 2 15 = 5 + 3 − 2 15 12 × 7 6

Sol.
4
12 × 7 6
( )
2
= 5− 3 = 5− 3 1
1 1

∴ 5 + 2 6 + 8 − 2 15 = (12 ) 4 × ( 6 ) 7 1

= 3+ 2+ 5− 3 ½ 1 1

= ( 2 × 2 × 3) × ( 2 × 3) 4 7
1
= 2+ 5 ½
1 1 1 1 1
+ + +
[CBSE Marking Scheme, 2012] = ( 2 ) 4 4 7
× ( 3) 4 7
1
9 11
Long Answer Type = ( 2 ) × ( 3) 14 28
1
Questions (5 marks each)
= 14 9
2 × 3 28 11
1
24 54 U Q. 3. Simplify :
Q. 1. Simplify : + 4
16 − 6 343 + 18 243 − 196 .
3 5

8 9
U [Board Term I, 2014]
24 54
Sol. + Sol. 4
16 = 4
2×2×2×2 =2
8 9
3 3
4×6 9×6 343 = 7×7×7 =7
= + 1 5 5
8 9 243 = 3×3×3×3×3 = 3
2 6 3 6 196 = 14 1
= + 1
8 9 4 3 5
\ 16 − 6 343 + 18 243 − 196
6 6 = 2 – 6 × 7 + 18 × 3 – 14
= + 1
4 3 = 2 – 42 + 54 –14
= 56 –56 = 0 1

Topic-4 Laws of Exponents with Integral Powers

Revision Notes
 Let a > 0 be a real number and ‘r ’ and ‘s’ be rational numbers, then
(i) ar.as = ar+s (ii) (ar)s = ars
r Scan to know
a
(iii) = ar–s, r > s (iv) ar br = (ab)r more about
as this topic
r
1
(v) a–r = r (vi) a s = (ar)1/s = (a1/s)r
a
r −r r
 a ar  a  b
(vii)   = r (viii)   =   Laws of
 b b  b  a exponents

(ix) a0 = 1
Examples :

( 4 )7
(i) (3)4 × (3)3 = 34+3 = 37 (ii) = (4)7–2 = 45
( 4 )2
REAL NUMBERS 13
−2 2
 3  5
(iii) (3)2 × (4)2 = (12)2 (iv)   = 
5  3
1
(v) (9)–2 =
92

Example 4
3 40 + 339 + 338 338 − 39 ( 9 + 3 + 1)
Find the value of i.e.,
3 41 + 3 40 − 339 ( 9 + 3 − 1)
Solution: Step III : Solving the expression
Step I : Taking common factor from numerator and
denominator as possible we can. 3 −1 × 13
i.e., =
11
3 40 + 339 + 338 338 ( 3 2 + 31 + 1)
i.e., 41 = 13 13
3 + 3 − 3
40 39
339 ( 3 2 + 31 − 1)
= =
3 × 11 33
Step II : Shifting the common factor which in
denominator and solving the expression which are in
bracket.

SUBJECTIVE TYPE QUESTIONS


⇒ 23x–1 + 10 = 42
Very Short Answer Type ⇒ 23x–1 = 42 – 10
Questions (1 mark each) ⇒ 23x–1 = 32
−2 −2 −2 ⇒ 23x–1 = (2)5
1 2 3
Q. 1. Simplify :   +  +  U ⇒ 3x – 1 = 5
2 3 4  ⇒ 3x = 6
−2 −2 −2 2 2 ⇒ x=2 1
1 2 3 2 3 4
Sol.   +  +  = (2) +   +  
2 3 4 2 3 Short Answer Type
= 4+
9 16
+ Questions-I (2 marks each)
4 9 Q. 1. Find the value of (13 + 23 + 33)–3/2.
144 + 81 + 64 289 U [Board Term I, 2012]
= = 1
36 36
−4 3x (13 + 23 + 33)–3/2 = (1 + 8 + 27)–3/2
Sol. ½
 7   7 
= (36)–3/2 ½
Q. 2. Find the value of x for which   ×  =
 12   12  = [(6)2]–3/2 = 6–3 ½
5 1 1
 7  U
= 3 = ½
 12  . 6 216
 
[CBSE Marking Scheme, 2012]
Sol. Given,
−4 3x 5 Q. 2. If a = 2 and b = 3, then find the value of :
 7   7   7 
⇒   ×  =   (i) (ab + ba)–1
 12   12   12 
(ii) (aa + bb)–1  U [Board Term I, 2012]
−4 + 3 x 5
 7   7 
⇒  12  =   Sol. Given, a = 2 and b = 3.
   12 
(i) (ab + ba)–1 = (23 + 32)–1 ½
⇒ – 4 + 3x = 5
= (8 + 9)–1 = 17–1
⇒ 3x = 9 1
⇒ x=3 1 = ½
17
Q. 3. If (23x – 1 + 10) ÷ 7 = 6, then find the value of x. 
U (ii)
(aa + bb)–1 = (22 + 33)–1 ½

3 x−1 = (4 + 27)–1 = 31–1
2 + 10
Sol. Given, =6 =
1
7 ½
31
3x–1
⇒ 2 + 10 = 6 × 7 [CBSE Marking Scheme, 2012]
14 Oswaal CBSE Question Bank Chapterwise & Topicwise, MATHEMATICS, Class-IX

5 5
 81 

4  25 

2 Þ (xab–ac–ba+bc) ÷ (xbc–ac)
Q. 3. Simplify :   ×  .
 16   9 Þ x(bc–ac)÷ x(bc–ac)
U [Board Term I, 2012] Þ x0 = 1 = RHS 1
5 5 [CBSE Marking Scheme, 2014]
5 5 − −
− −  3  4  4  5  2  2
 81  4  25  2
Sol.   ×  =    ×    ½ Short Answer Type
 16   9  2    3   
−5 −5 Questions-II (3 marks each)
 3  5
× 
=   ½
( ) + (3 ) −( )
2 2 2
 2  3 Q. 1. Simplify : 2 2 − 5 2+ 3 2 −1 .

5 5
 2  3 U [Board Term I, 2012]
=   × 
 3  5

(2 ) + (3 ) −( )
2 2 2
Sol. 2 −5 2+ 3 2 −1
25
= 5
5
( ) ( )
2 2
= 2 2 – 2(2 2 )(5) + (5)2 + 3 2
32
= 1
( 2)
3125  2
+ 2(3 2 ) ( 3 ) + ( 3 )2 – – (1)2 +2( 2 ) (1)
1
Commonly Made Error = 8 + 25 – 20 2 + 18 + 3 + 6 6 – 2 – 1 + 2 2
1
While solving the real numbers with integral
powers, the bases of numbers should be made = 51 – 18 2 + 6 6 1
the same to solve but students sometimes do [CBSE Marking Scheme, 2012]
it directly and simply reduce it to lowest term, 2

( x)
− 1
( xy )

thereby marks are not awarded to them inspite Q. 2. Simplify : 3
y4 ÷ 2 .
of correct answer.
U [Board Term I, 2012]
2 1

Answering Tip  1 3 1
 1
− 2
( )
2 1
( x)

3 y4 ÷ (xy ) − 2
=  x  . y4 2 ÷ ( xy ) 2 
Sol. 2
   
The students should be careful in making the
−1 1
base of exponent same and then solve according −
= x 3 ·y 2 ÷ ( xy ) 4
to the law of exponents.
1 −1 1
  1 1 3
 4
= x 3 ·y 2 × ( xy ) 4
Q. 4. Simplify : 5 8 + 27   .
3 3
    −1 1 1
+ 2+
= x 3 4 ·y 4 2
U [Board Term I, 2012]
1 1 −1 9
  1 1 3  4   1 1 3  4 = x 12 ·y 4
Sol.  5 8 3 + 27 3    5 ( 2 3 ) 3 + ( 33 ) 3  
=    
   1 9
      
 y4
1 1 = 1 1
3 3
=  5( 2 + 3)  4 =  5( 5)  4 x 12

1 a+ b b+ c c+a
x  a  xb   xc 
= ( 54 )=
4 1
5= 5 1 Q. 3.  b 
x 
× c 
x 
× a 
x 
 R [KVS 2019]
     
( )
c
x a( b − c )  xb 
Q. 5. Show that : ÷  a  = 1.  xa 
a+b
 xb 
b+c
 xc 
c+a
x
b( a − c )
( )
 x  Sol.  b  × c 
x 
× a 
x 
x     
U [Board Term I, 2014]
= (xa – b)a + b × (xb – c)b + c × (xc – a)c + a 1
( )
c
x a( b − c )  xb 
a2 − b 2 b2 − c2 c 2 − a2
Sol. LHS = ÷ a  = x ×x ×x
x b( a − c )
 ( )
 x 

1
2
− b 2 + b 2 − c 2 + c 2 − a2
= x a  1
ab − ac bc
x x
Þ ba − bc
÷ ac = x0 = 1 1
x x
REAL NUMBERS 15
7 5 3 3
−  3
 5−1 × 7 2  2  5−2 × 7 3  2  16  4  9  2  2  
Q. 4. Simplify :  . =   ×   ÷  1
 × 3   81   25   5 
 52 × 7 −4   5 × 7 −5    
U [Board Term I, 2012] 3  3 
 2 4  4  32  2  2  3 
7

5 =  4  ×  2  ÷    ½
 5 −1 × 7 2  2  5 −2 × 7 3  3  5  5
 
2
Sol. 
 2  × 3  ½ 
 5 × 7 −4   5 × 7 −5 
3  3 
7

5  2  4  4   3  2  2  5  3 
 74 × 72  2  75 × 73  2 =    ×     ×    1
=  2 1 × 3 2 1  3    
  5 
 2

5 ×5  5 ×5   
3  3  3  5  3 
1 1 1  2
 7 42  2  525  2  7 42 525  2 =   ×   ×    ½
=  21  ×  40  =  21 × 40  ½  3  5   2 

5  7  5 7  3  3 3
2 3 5
½
1 = 3 ×  3 × 3 
2 4 3  5 2 
= (7 × 5 ) 2 ½
= 7 × 52 = 7 × 25 ½ 23 33
= 3
× 3 ½
= 175 3 2 

[CBSE Marking Scheme, 2012] = 1 ½
x 2x
2  3 81
Q. 5. Find x, if  
-x
·  = A [Board Term I, 2012] 2x −1
2
 3  2 Q. 3. If a = , b = x+1 and a – b = 0, find the value
16  2x−2 2
x 2x of x. A [Board Term I, 2016]
 2  3 81
Sol.   ·  = 1 Sol. a – b = 0
3 2 16
2 x −1 2−x
2 x 32 x 34 or, − x +1 = 0 1
or, · = 4 1 2x −2 2
x 2x
3 2 2
or, 2x–2x.32x–x = 34.2–4 or, 2 x – 1 – (x – 2) – 2 – x – (x + 1) = 0
or,
x–2x = – 4 or, 2x–1–x+2 – 2–x–x–1 = 0 1
\
x =4 1 or, 21 – 2 – 2x – 1 = 0
[CBSE Marking Scheme, 2012] or, 2– 2x –1 = 21
or, -2x – 1 = 1 1
or, -2x = 2
Long Answer Type or, x = –1 2
Questions (5 marks each) [CBSE Marking Scheme, 2016]
Q. 1. Simplify : 2 4 81 − 8 3 216 + 15 5 32 + 225 − 4 16 Q. 4. If x is positive real number and exponents are
U [Board Term I, 2016] rational number then simplify :
b+ c − a c+ a−b a+ b− c
 xb   xc   xa 
Sol. 2 4 81 − 8 3 216 + 15 5 32 + 225 − 4 16  xc  × a × b
1 1 1 1
  x  x 
( ) ( )
= 2 34 4 − 8 6 3 3 + 15 2 5 5 + 15 − 2 4 4

2 ( ) ( ) U [Board Term I, 2016]

= 2 × 3 – 8 × 6 + 15 × 2 + 15 – 2 1 b+c−a c+ a−b a+b−c

= 6 – 48 + 30 + 15 – 2  1 Sol.
 xb   xc   xa 
 xc  × a × b

= 51 – 50 = 1  1 x  x 
 
[CBSE Marking Scheme, 2016, 2019 ] b+c−a c+ a −b a+b−c
=  x  ×  x a − b 
b−c
×  x c − a  1
−3  −3 
3
81  9  2  5
Q. 2. Evaluate :   ×   ÷    .
4
b 2
− c 2 − ab + ac
× xc
2
− a 2 − bc + ab
× xa
2
− b 2 − ac + bc 2
 16   25   2  = x
  1
b2 − c 2 − ab + ac + c 2 − a 2 − bc + ab + a 2 − b2 − ac + bc
= x
Ap [Board Term I, 2016] = x0 = 1 [CBSE Marking Scheme, 2016] 1
−3 3
 −3 
Sol.  81  4   9  2  5 
×   ÷  ½
 16   25   2 
 
16 Oswaal CBSE Question Bank Chapterwise & Topicwise, MATHEMATICS, Class-IX

Topic-5 Rationalization of Real Numbers

Revision Notes
 Rationalization: If a given number is transformed into an equivalent form, such that the denominator is a rational
number then the process is known as rationalization.
 Rationalizing the denominator : If the denominator of a fraction contains a term with root (a number under
a radical sign), the process of converting it to an equivalent expression with rational denominator is called as
rationalizing the denominator.
 List of Rationalization Factors.
Term Rationalizing Factor
1
r
r

1
r +s
r −s

1
r −s
r +s

1
r+ s
r− s

1
r− s
r+ s

Example 5
7 Step III : Multiply the numerator and denominator
Rationalize the denominator of of x by the conjugate of denominator and rationalize
5- 2
it.
Solution:
7 5+ 2
To rationalize the denominator, we will multiply x = ×
5− 2 5+ 2
the numerator and denominator by its conjugate to
remove the radical sign from the denominator. 7( 5 + 2 )
Step I : Assume the given fraction as x and write =
( 5 )2 − ( 2 )2
the denominator.

7 7( 5 + 2 )
Let x = and denominator = 5- 2 =
5−2
5- 2
Step II : Find the conjugate of denominator. 7
= ( 5 + 2)
Here, the conjugate of denominator ( 5 - 2 ) is 3
( 5 + 2)

SUBJECTIVE TYPE QUESTIONS


1 1
Very Short Answer Type Sol.
50
=
5×5×2
Questions (1 mark each)
1 2 2
= × = 1
1 5 2 2 10
Q. 1. Write the rationalizing factor of . R
50 
So, rationalizing factor is 2.
REAL NUMBERS 17
2 Short Answer Type
Q. 2. Rationalize the denominator of . U [NCERT]
3 3  Questions-II (3 marks each)
2 2 3 1 1
Sol. = × Q. 1. If x = and y = , then find the
3 3 3 3 3 3−2 2 3+2 2
value of x + y+ xy. A [Board Term I, 2014]
2 3 2 3
= = 1 1 3+2 2
3×3 9  Sol. x = ×
3−2 2 3+2 2
Short Answer Type 3+2 2
Questions-I (2 marks each) = = 3+2 2 1
9−8
1
Q. 1. If 2 = 1.414, then, find the value of . 1 3−2 2
2 +1 and y = ×
3+2 2 3−2 2
U [Board Term I, 2012]
1 1 ( 2 − 1) 3−2 2
= × = =3−2 2 1
Sol. 1 9−8
2 +1 ( 2 + 1) ( 2 − 1)

x + y+ xy = 3+2 2 + 3 – 2 2 + (3+2 2 )
( 2 − 1)
=
( 2 )2 − (1)2 (3 – 2 2 ) ½
= 6 + 9 – 8
2 − 1 1.414 − 1 = 7 ½
= = = 0.414
2 −1 1 1
2 1
Q. 2. If x = 2 + 3 , then find the value of x + .
1 x2
Q. 2. Taking 2 = 1.414 and r = 3.141, evaluate +r
2 A [Board Term I, 2015]
upto three places of decimal.
Sol. x =2+ 3
U [Board Term I, 2012]
1 1
=
x 2+ 3
1 1 2
Sol. = × (rationalizing)
2 2 2 1 2− 3
= ×
2 1.414 2+ 3 2− 3
= = = 0.707 1
2 2 = 2− 3

1 1
\ + p = 0.707 + 3.141 =2– 3 1
2 x

= 3.848 1 1
\ x+ =4
[CBSE Marking Scheme, 2012] x

1 Squaring both sides, we get
Q. 3. If x = 3 − 2 2 , find the value of x+ .
x 
1
x2 + + 2 = 16 1
A [Board Term I, 2014] x2
Sol. x = 3−2 2 1
or, x2 + = 14 1
or, x = 1+ 2 − 2 2 x2
1 2 2
or, x = ( 2 − 1)
2
½ Q. 3. If a = 2 + 5 and b = , find a + b .
a

or, x = 2 −1 ½ A [Board Term I, 2016]
1 1 2 +1
or, = × 1
x 2 −1 2 +1 Sol. b =
2+ 5
= 1 + 2 ( ) ½ 1 2− 5
= × 1
1 2+ 5 2− 5
\ x+ = 2 −1+ 2 +1
x 2− 5
= = −2 + 5
= 2 2 ½ −1
18 Oswaal CBSE Question Bank Chapterwise & Topicwise, MATHEMATICS, Class-IX


( ) ( ) ( 2 − 1)
2 2 2
a2 = 2 + 5 =9+4 5  ½ 2 +1 −
= a+ 2b 1

( 1) ( 2 + 1)
2 −
( )
2
b2 = − 2 + 5 =9−4 5 ½
 2+1+ 2 2 − 2 −1+ 2 2
=a+ 2b
\ a2 + b2 = 9 + 4 5 + 9 − 4 5 2 −1
= 18  1
[CBSE Marking Scheme, 2016] 4 2 = a+ 2b

 1
3 On comparing a = 0, b = 4 1
Q. 4. If x = 2 – 1, then find the value of  x −  .
 x [CBSE Marking Scheme, 2016]

A [Board Term I, 2014]

Sol. x= 2 –1 Commonly Made Error


1 1 2 +1
or, = × 1 Generally, students make mistake while
x 2 −1 2 +1 adding or subtracting rational numbers while
1 2 +1 rationalizing them.
or, = = 2 +1 ½
x 2 −1

 1
3
Answering Tip
\  x −  = ( 2 − 1 − 2 − 1)3 ½
x
The student should be slow and precise while
= (–2)3
working with rational numbers.
= – 8 1

Q. 5. Find a and b if 1 − 3 = a + b. Long Answer Type


1+ 3
Questions (5 marks each)
 A [Board Term I, 2016] Q. 1. Prove that :
1 1 1 1 1
1− 3 − + − + =5
Sol. a+b = 3 − 8 8 − 7 7 − 6 6 − 5 5 −2
1+ 3
U [Board Term I, 2014]
1− 3 1− 3 Sol. LHS
= × 1
1 + 3 1 − 3 1 1 1 1 1
= − + − +
3− 8 8− 7 7− 6 6− 5 5 −2
(1 − 3 )
2

= 3+ 8 8+ 7 ( 7 + 6)
1−3 = − +
( 3 )2 − ( 8 )2 ( 8 )2 − ( 7 )2 ( 7 )2 − ( 6 )2
1+3− 2 3
= 1 ( 6 + 5) ( 5 + 2) 1
−2 − +
( 6 )2 − ( 5 )2 ( 5 )2 − ( 2 )2
4−2 3 3 + 8 ( 8 + 7 ) ( 7 + 6) ( 6 + 5)
= = − + − 1
−2 9−8 8−7 7−6 6−5

a + b = –2 + 3 ( 5 + 2)
+ [ a2 – b2 = (a – b) (a + b)] 2
5−4
\ a = –2 or, a = 3
= 3+ 8 − 8 − 7 + 7 + 6 − 6 − 5 + 5 +2
and b= 3 b=–2 1 = 3 + 2 = 5 = RHS 1
2
[CBSE Marking Scheme, 2016]  1
Q. 2. If x = 4 – 15 , then find the value of  x +  .
Q. 6. Find the value of a and b if  x 
2 +1 2 −1 U [Board Term I, 2014]
− =a+ 2 b Sol. x = 4 – 15
2 −1
2 +1
A [Board Term I, 2016] 1 1 4 + 15
or, = × 1
x 4 − 15 4 + 15
2 +1 2 −1
Sol. − =a+ 2b 1 1 4 + 15
2 −1 2 +1 = 1
x 16 − 15
REAL NUMBERS 19
1 5 + 2 and b = 5− 2
= 4 + 15 1 Q. 5. If a = . Find the value
x 5+ 2
5− 2
2
 1
\  x +  = ( 4 − 15 + 4 + 15 )
2
1 a2 + ab + b2
x of . A
a2 − ab + b2 
= (8)2
= 64 1 Sol. a2 + ab + b2 = (a + b)2 – ab
2 −1 2 +1 a2 – ab + b2 = (a – b)2 + ab
Q. 3. If x = and y = then find the value of
2 +1 2 −1
5+ 2 5+ 2
x2 + 5xy + y2. A [Board Term I, 2016] a= ×
5− 2 5+ 2
Sol. x2 + 5xy + y2 = (x + y)2 + 3xy ( )
2
5+ 2
= 1
2
5−2
é 2 -1 2 + 1ù 2 -1 2 +1
ê + ú +5 ´ 2
êë 2 + 1 2 - 1 úû 2 +1 2 -1 5 + 2 + 2 10 7 + 2 10

= =
3 3
2
2 +1−2 2 + 2 +1+ 2 2
5− 2 5− 2
= 
 2 −1  ++33 1 b= ×
5+ 2 5− 2

= (6)2 +3
1 7 − 2 10 1
=

= 36 + 3 = 39 1 3
2
[CBSE Marking Scheme, 2016]  7 + 2 10 7 − 2 10 
(a + b)2 – ab =  +  ½
 3 3
5+1 5 −1
Q. 4. If x = and y = , then find the value of
5 −1 5+1  7 + 2 10   7 − 2 10 
−   
x2 + y2. U [Board Term I, 2016]  3 3

5 +1 2
Sol. x = 14 49 − 40 
5 −1 =   −  196 9
 3  9  = 9 − 9

2
 5 + 1 6+2 5 3+ 5 187
x = 2  = = 1 = ½
 5 − 1  6−2 5 3− 5 9

2
5 −1  7 + 2 10 7 − 2 10 
y = (a – b)2 + ab =  −  ½
5 +1  3 3

 5 − 1
2
6−2 5 3− 5  7 + 2 10   7 − 2 10 
2 = = 1 +   
y =   6+2 5 3+ 5  3 3
 5 + 1
2
3+ 5 3− 5  4 10  49 − 40
2 2 + =  +
\ x +y =
3− 5 3+ 5
1  3  9

160 9 169
(3 + 5 ) + (3 − 5 )
2 2
= + = ½
\ x2 + y2 = 1 9 9 9
(3 − 5 ) (3 + 5 )
a 2 + ab + b 2 ( a + b )2 − ab
\ =
2
a − ab + b 2 ( a − b )2 + ab
9+5+6 5 +9+5−6 5
=
9−5 187
28 = 9
= 169
4

9
\ x2 + y2 = 7 1 187
= 1
[CBSE Marking Scheme, 2016] 169
20 Oswaal CBSE Question Bank Chapterwise & Topicwise, MATHEMATICS, Class-IX

OBJECTIVE TYPE QUESTIONS (1 mark each)

Explanation: The product of any two irrational


A Multiple Choice Questions numbers is sometimes rational and sometimes
irrational.
Q. 1. Every rational number is Q. 6. The decimal expansion of the number 2 is
(A) a natural number
(A) a finite decimal
(B) an integer
(B) 1.41421
(C) a real number
(C) non-terminating recurring
(D) a whole number A [NCERT Exemp.]
Ans. Option (C) is correct. (D) non-terminating non-recurring
Explanation: We know that rational and irrational Ans. Option (D) is correct.
numbers taken together are known as real numbers. Explanation: The decimal expansion of the number
Therefore, every real number is either a rational is 1.41421…… non- terminating non - recurring
number or an irrational number. Hence, every Q. 7. Which of the following is irrational?
rational number is a real number. Therefore, (c) is
the correct answer. 4 (B) 12
(A)
Q. 2. Between two rational numbers 9 3
(A) there is no rational number (C) 7 (D) 81
(B) there is exactly one rational number Ans. Option (C) is correct.
(C) there are infinitely many rational numbers Explanation:
(D) there are only rational numbers and no 4 2
irrational numbers (A) = which is a rational number.
A [NCERT Exemp.] 9 3
Ans. Option (C) is correct.
(B) 12 4× 3
Explanation: Between two rational numbers there = = 2 which is a rational number.
are infinitely many rational number, for example, 3 3
between 4 and 5 there are 4.1, 4.2, 4.22, 4.223............ (C) 7 is an irrational number.
Q. 3. Decimal representation of a rational number
cannot be (D) 81 = 9 which is a rational number.
(A) terminating Q. 8. Which of the following is irrational?
(B) non-terminating
(A) 0.14 (B) 0. 1416
(C) non-terminating / repeating
(D) non-terminating / non-repeating (C) 0. 1416 (D) 0.4014001400014...
 A [NCERT Exemp.] Ans. Option (D) is correct.
Explanation: A number is irrational if and only if
Ans. Option (D) is correct.
its decimal representation is non-terminating and
Explanation: The decimal representation of a non-recurring.
rational number cannot be non-terminating and
non-repeating. (A) 0.14 is a terminating decimal and therefore
cannot be an irrational number.
Q. 4. A rational number between 2 and 3 is
2+ 3 2− 3 (B) 0. 1416 is a non-terminating and recurring
(A) (B)
2 2 decimal and therefore cannot be irrational.
(C) 1.5 (D) 1.8
(C) 0. 1416 is a non-terminating and recurring
 A [NCERT Exemp.] decimal and therefore cannot be irrational.
Ans. Option (C) is correct.
(D) 0.4014001400014... is a non-terminating and
Explanation: We know that non-recurring decimal and therefore is an
=2 1=
.4142135.... and 3 1.732050807....
irrational number.
We see that 1.5 is a rational number which lies Q. 9. 2 3 + 3 is equal to
between 1.4142135….. and 1.732050807….
(A) 2 6 (B) 6
Q. 5. The product of any two irrational numbers is
(C) 3 3 (D) 4 6
(A) always an irrational number
(B) always a rational number A [NCERT Exemp.]
(C) always an integer Ans. Option (C) is correct.
(D) sometimes rational, sometimes irrational Explanation: 2 3 + 3 = 3 3
Ans. Option (D) is correct.
REAL NUMBERS 21
Q. 10. 10 × 15 is equal to Explanation:
(A) 6 5 (B) 5 6 2560.16 ´ 2560.09 = 2560.16 + 0.09 = 2560.25
(C) 25 (D) 10 5 1 1

1
= 256 4 = ( 44 ) 4 = 4 4
=4
A [NCERT Exemp.]
Ans. Option (B) is correct. 1
Explanation: We have 10 × 15 = 10 × 15 = 5 6 Q. 17. is equal to
9− 8
32 + 48
Q. 11. The value of is equal to 1 1
8 + 12 (A) (3 - 2 2 ) (B)
(A) 2 (B) 2 2 3+2 2
(C) 4 (D) 8
(C) 3 - 2 2 (D) 3 + 2 2 A
 A [NCERT Exemp.]
Ans. Option (B) is correct. Ans. Option (D) is correct.
Explanation: Explanation:
32 + 48 4 2 + 4 3 4( 2 + 3 ) 4
= = = = 2 1 1 1 3+ 8 3+ 8
8 + 12 2 2 + 2 3 2( 2 + 3 ) 2 = = × =
9− 8 3− 8 3− 8 3+ 8 9−8
Q. 12. The value of 6 ´ 27 is
= 3+ 2×4 = 3+ 2 2
(A) 3 3 (B) 9 3
(C) 9 2 (D) 2 2 Q. 18. The number obtained on rationalizing the
1
Ans. Option (C) is correct. denominator of is
7 −2
Explanation:
6 ´ 27 = 2 ´ 3 ´ 3 ´ 3 ´ 3 7 +2 7 -2
(A) (B)
3 3
Þ 3´3 2 = 9 2
7 +2 7 +2
(C) (D) A
Q. 13. 4 3
2 2 equal 5 45
−1
Ans. Option (A) is correct.
(A) 2 6 (B) 2–6
1 Explanation:
(C) 2 6 (D) 26
1 1 7+2 7+2 7+2
 A [NCERT Exemp.] = × = =
7 −2 7 −2 7 +2 7−4 3
Ans. Option (C) is correct.
1

= (2 )
2 4 2 1
×
1 7
Explanation: 4 3 2
2 = (2 )
4 2 1/3 3
= 23 4
= 26 Q. 19. After rationalizing the denominator of ,
3 3 −2 2
3 4 12
Q. 14. The product 2. 2. 32 equals
we get the denominator as
(A) 2 (B) 2
(A) 13 (B) 19
(C) 12 2 (D) 12 32
 A [NCERT Exemp.] (C) 5 (D) 35 A

Ans. Option (B) is correct. Ans. Option (B) is correct.


Explanation: We have, Explanation:
1 1 5 1 1 5 12
+ +
3 4 12
2. 2 32 = 2 .2 .2 = 2 3 4 12 3 4 12
=2 =2
12 7 7 (3 3 + 2 2 )
= ×
3 3−2 2 3 3−2 2 3 3+2 2
Q. 15. Value of 81−2 is
4

1 1 3 3+2 2 3 3+2 2
2 2 ) 7( 3 3=+ 2 2 )
7( 3 3 + =
(A) (B) = 27= − 8 19
9 3 27 - 8 19
1
(C) 9 (D)
81 Therefore, we get the denominator as 19.
 A [NCERT Exemp.]
Ans. Option (A) is correct. 2 −1
Q. 20. If 2 = 1.4142... then is equal to
1
1 2 +1
æ æ 1 ö 2´ 2 ö
4 4´
1 4 1
Explanation:
4
81-2 = çç ÷ ÷ = = (A) 2.4142 (B) 5.8282
çè 9 ø ÷ 9 9
è ø
(C) 0.4142 (D) 0.1718 AE
Q. 16. Value of 256 0.16 × 256 0.09 is
(A) 4 (B) 16 Ans. Option (C) is correct.
(C) 64 (D) 256.25 Explanation: After rationalisation, we get
 A [NCERT Exemp.]
2 - 1 = 1.4142 – 1 = 0.4142
Ans. Option (A) is correct.
22 Oswaal CBSE Question Bank Chapterwise & Topicwise, MATHEMATICS, Class-IX

= 5.82842......
B Assertion & Reason
\ Assertion is true.
Directions: In the following questions, a statement In case of Reason (R):
of assertion (A) is followed by a statement of A number whose decimal expansion terminates is a
reason (R). Mark the correct choice as: rational number is true.
(A) Both assertion (A) and reason (R) are true and reason Both A and R are true but R is not correct explanation
(R) is the correct explanation of assertion (A). of A.
(B) Both assertion (A) and reason (R) are true but reason
(R) is not the correct explanation of assertion (A). Q. 3. Assertion (A): 3 is an irrational number.
(C) Assertion (A) is true but reason (R) is false. Reason (R): Square root of a positive integer which
(D) Assertion (A) is false but reason (R) is true. is not a perfect square is an irrational number.
Ans. Option (A) is correct.
2
Q. 1. Assertion (A): Three rational numbers between Explanation: In case of Assertion (A):
5
3 has non terminating and non-recurring decimal
3 9 10 11
and are , and . expansion.
5 20 20 20
\ Assertion is true.
Reason (R): A rational number between two
1 In case of Reason (R):
rational numbers p and q is ( p + q ). Square root of a positive integer which is not perfect
2
square has a non terminating and non-recurring
Ans. Option (B) is correct. decimal expansion.
Explanation: In case of Assertion (A): \ Reason is true.
2 3 Therefore, Both A and R are true and R the
Multiplying and dividing and by 4, we get correction explanation of A.
5 5
Q. 4. Assertion (A): e is an irrational number.
2 4 8 Reason (R): p is an irrational number.
× =
5 4 20 Ans. Option (B) is correct.
Explanation: In case of Assertion (A):
3 4 12
× = e = 2.71828...
5 4 20
\ e is an irrational number, hence, Assertion is true.
8 12 In case of Reason (R):
Now, rational number are and .
20 20 p = 3.14159...
\ p is irrational number, hence, Reason is true.
9 10 11
\ , , are three rational numbers between Both A and R are true but R is not correct explanation
20 20 20
of A.
2 3
and . Q. 5. Assertion (A): Value of 5 + 2 6 is an irrational
5 5
number.
\ Assertion is true
Reason (R): Product of a rational number and
In case of Reason: an irrational number is irrational, when rational
Now, rational number between two rational numbers number is not equal to 0.
Ans. Option (B) is correct.
1
= ( p + q ) . where p and q are rational numbers. Explanation: In case of Assertion:
2
5+2 6 = ( 2 )2 + ( 3 )2 + 2 2 3
\ Reason is true
Therefore, Both A and R are true but R is not the = ( 2 + 3 )2 = 2 + 3

correction explanation of A.
In case of Reason:
Q. 2. Assertion (A): On simplification, 3 + 2 2 gives an
Product of rational and irrational number is
irrational number.
irrational when rational number is not equal to 0.
Reason (R): A number whose decimal expansion is
terminating is rational. Both A and R are true but R is not correct explanation
of A.
Ans. Option (B) is correct.
Explanation: In case of Assertion: 1 1
Q. 6. Assertion (A): Value of 7–2 = =
72 49
3 + 2 2 = 3 + 2 × (1.41421. .......)
1
= 3 + 2.82842...... Reason (R): According to laws of radicals a–m =
am
REAL NUMBERS 23
Ans. Option (A) is correct. p
Explanation: In case of Assertion (A): Rational number is in the form of where p and q
q
1 1 1 are not equal to 0.
7–2 = 2
= =
7 7 ´ 7 49 Both A and R are true and R is the correct
In case of Reason (R): explanation of A.
1 Q. 9. Assertion (A): The rationalizing factor of 2 + 3 3 is
a–m =
am 2 - 3 3.
satisfies the laws of radicals for positive real number Reason (R): If the product of two irrational numbers
Hence, Both A and R are true but R is the correct is rational then each one is called the rationalizing
explanation of A. factor of the other.
Ans. Option (A) is correct.
Q. 7. Assertion (A): Value of (32)2/5 + (–7)0 is a rational
Explanation: In case of Assertion (A):
number.
( 2 + 3 3 )( 2 - 3 3 )
Reason (R): (a)0 ¹ 1; where a > 0 and a is a real
number. Þ 4 - 6 3 + 6 3 - 27
Ans. Option (C) is correct.
= 4 – 27
Explanation: In case of Assertion:
= –23 is a rational number
(32)2/5 + (–7)0 = (25)2/5 + (–7)0
\ Assertion is true
2

= 2 5 +1
In case of Reason (R):
If product of two irrational is rational then they are
= 22 + 1
rationalizing factor of each other.
= 4 + 1 = 5
\ A and R are true and R is correct explanation of A.
\ Assertion is true
In case of Reason: Q. 10. Assertion (A): To rationalize the denominator of
(a)0 = 1 1
, we should multiply and divide it by 3 - 4.
\ Reason is false. 4+ 3

Q. 8. Assertion (A): Value of 3


( 343)-2 is a rational number. Reason (R): To rationalize the denominator, we will
multiply the numerator and denominator by its
p conjugate.
Reason (R): Rational number is of the form
q Ans. Option (D) is correct.
Explanation: In case of assertion (A):
where p and q are not equal to zero.
To rationalize the denominator
Ans. Option (A) is correct.
Explanation: In case of assertion (A): 1 4- 3 4- 3
´ =
4´ 3 4- 3 16 - 3
3
( 343)-2 = [(343)–2]1/3 = (343)–2/3
4- 3
1
3 3
× −2 =
= (7 )

13
 2 \ Assertion is False.
 3 ×− 
= 7  3  = 7–2 In case of Reason (R):
1 1 To rationalize the denominator, multiply numerator and
= 2 = denominator by its conjugate.
7 49
\ Reason (R) is True:
\ Assertion is true.
Therefore A is False and R is true.
In case of Reason:

COMPETENCY BASED QUESTIONS (4 marks each)

is to VOTE. In India, every citizen over 18 years of


A Case based MCQs age has the right to vote. Instead of enjoying it as
a holiday, one must vote if he/she truly wants to
Read the following passage and answer any four contribute to the nation-building process and bring
questions. about a change.
I. Democracy has given people a powerful right- that
24 Oswaal CBSE Question Bank Chapterwise & Topicwise, MATHEMATICS, Class-IX

Q. 5. If r is rational and s is irrational, then which


statement is false? 1
(A) r + s is irrational number.
(B) r – s is rational number.
(C) rs is irrational number.
r
(D) is irrational number.
s
Ans. Option (B) is correct.
II. In a school 5 out of every 7 children participated in
‘Save wild life’ campaign organised by the school
authorities.
A survey was done in a small area in which Q. 1. What fraction of the students participated in the
5 campaign. 1
9 + 2 x − 2 x voters were men and 2 5
9 + 2x (A) (B)
voters were women. 7 7

Q. 1. Find x, if number of men is equal to number of 4 7


(C) (D)
women. 1 7 7
(A) 6 (B) 5 Ans. Option (B) is correct.
(C) 8
(D) 9 5
Q. 2. What is the value of ? 1
Ans. Option (C) is correct. 7
5 (A) 0.714285 (B) 1.4
\ 9 + 2x − 2x =
9 + 2x (C) 1.414 (D) 1.7142
⇒ =5
9 + 2 x [ 9 + 2 x − 2 x ] Ans. Option (A) is correct.
⇒ ( 9 + 2 x )2 − 2 x ( 9 + 2 x ) = 5 Q. 3. What kind of decimal expansion it has. 1
(A) Terminating

9 + 2x – 5 = 2 x( 9 + 2 x ) (B) non terminating
(C) terminating repeating

4 + 2x = 2 x( 9 + 2 x )
(D) non terminating repeating
Squaring both sides,
Ans. Option (D) is correct.
( )
2
(4 + 2x)2 = 2 x( 9 + 2 x ) Q. 4. How many rational numbers are there between 5
and 7. 1
⇒ 16 + 4x2 + 16x = 18x + 4x2
⇒ 16 = 2x (A) 0 (B) 1
⇒ x = 8 1 (C) 2 (D) infinite
Q. 2. Which mathematical concept is used here ? 1 Ans. Option (D) is correct.
(A) Number system (B) Circle
Q. 5. Every rational number is a .................. number.1
(C) Area (D) Statistics
(A) Prime (B) Composite
Ans. Option (A) is correct.
(C) real (D) even.
Q. 3. ap . aq =  1
(A) ap + q (B) ap – q Ans. Option (C) is correct.
pq
(C) a (D) (pq)a
Ans. Option (A) is correct. B Subjective Based Questions
ap.aq = ap + q
1
Read the following passage and answer the
75 following questions:
Q. 4. 1
= 1
I. In a classroom activity on real numbers, the students
73  have to give answers of some questions framed by
-2 -1 their teacher on basis of number cards picked up by
(A) 7 15 (B) 7 15 first 3 Roll numbers.
2 -4
Q. 1. Reena picked up 7 and question asked by
(C) 7 15
(D) 7 15 teacher was, whether 7 is rational or irrational
Ans. Option (A) is correct. number.1
1 Ans. 7 is an irrational number as decimal expansion of
1 1 3−5 −2
75 −
an irrational number is non-terminating and non-
1
= 75 3 = 7 15 = 7 15
recurring.
73
REAL NUMBERS 25
Q. 2. Rajiv picked up card on which it is written Then x has a non-terminating repeating decimal
15 - 10 and again teacher asked whether it is expansion.
rational or irrational. 1 29
Q. 1. Solve 2 3
5 ´2 
Ans. 15 - 10 Þ Irrational number
29 29 29

Because difference of two irrational numbers is Ans. Þ = = 0.145 1
52 ´ 23 25 ´ 8 200
irrational.
441
1 Q. 2. What will be the decimal expansion of
Q. 3. Saumya picked third card on which was 2 ´ 57 ´ 7 2
2

45
is.
written. Now teacher asked students to write the
441
1 Ans. Decimal expansion of will be non
rationalizing factor of . 2 2 ´ 57 ´ 7 2
45  1
terminating repeating as it is not of the form
1 1 1 5 p
Ans. = Þ ´ 1
45 3´3´5 3 5 5 2 ´ 5m
n

5 Q. 3. Express both of them in decimal form and say


=
15 what kind of decimal expansion each has?
49 2
So, Rationalizing factor is 5.
(a) (b)
100 15 
II. Decimal Form of Rational numbers can be classified 49 49
into two types. Let x be a rational number whose Ans. (a) = 2 2 = 0.49
100 2 ´5
decimal expansion terminates.
p so, it has terminating decimal expansion since
Then x can be expressed in the form where p and p
q it is of the form n m 1
2 ´ 5
q are co-prime and the prime factorisation of q is of
the form 2n5m, where n and m are non-negative and 2 2
(b) = 1 1 = 0.13
p 15 5 ´3
vice-versa. Let x = be a rational number, such
q so, it has non terminating decimal expansion,
that the prime factorisation of q is not of the form p
since it is not of the form n m 1
2n5m where n and m are non-negative integers, 2 ´5

  
SELF ASSESSMENT PAPER - 01

Time: 1 hour MM: 30

UNIT-I
I. Multiple Choice Questions [1×6 = 6]
1
1. is a/an ............. number.
2
(A) Rational
(B) Irrational (C) Fractional (D) None of these
6x 2
2. 5 = 125 , then x = ...............
(A) 2 (B) 3 (C) 1 (D) 4
3. The sum of 2 5 and 3 7 is .............
(A) 5 12
(B) 2 5 + 3 7 (C) 2 + 3 5 + 7 (D) 5 5 + 7

4. Value of 710 × 810 is ...........


(A) (15)10 (B) (56)10 (C) (15)20 (D) (56)20
II. Assertion and Reason Based MCQs [1×6 = 6]

Directions: In the following questions, A statement of Assertion (A) is followed by a statement of Reason (R).
Mark the correct choice as.
(A) Both A and R are true and R is the correct explanation of A.
(B) Both A and R are true but R is NOT the correct explanation of A.
(C) A is true but R is false.
(D) A is false and R is True.
1. Assertion (A): 2 is an irrational number.
Reason (R): The sum of a rational number and an irrational number is an irrational number.
p 29
2. Assertion (A): A number 0.57 in the form of rational number is .
q 99
29
Reason (R): Decimal Expansion of is non terminating and recurring.
99

III. Very Short Answer Type Questions


 [1×5 = 5]
225
1. Write the simplest form of a rational number .
1250
p
2. Represent 0.239 in the form of , where p and q are integers and q ≠ 0.
q
3. If a = 3 and b = 2, then find the value of ab + ba. 
196
4. Is a rational number or not ?
2
1
5. Find 32 5 .
IV. Short Answer Type Questions–I
 [2×2 = 4]
1. Find the Product of 5 3 (3 + 3 ) ( 5 + 3 ) . 
1 1
2. If 23x × 4x = (8 ) 3 × (64 ) 6 , then find the value of x.
SELF ASSESSMENT PAPER 27
V. Short Answer Type Questions-II
 [3×2 = 6]
1. If x + y = ( 2 − 3 ) , then find the values of x and y.
2

7
2. what would be the denominator after rationalizing ?
5 3 -5 2 
VI. Long Answer Type Question [5×1 = 5]
.9 × 3 × ( 3
n 2 )
− n / 2 −2 n
− ( 27 ) 1
1. If 3m 3
= , prove that m – n = 1.
3 ×2 27

VII. Case Based Questions [1×4 = 4]



Attempt any four sub parts from the given questions.

Read the following text and answer the following questions on the basis of the same:


To judge the preparation of class IX students on the topic ‘ Real Numbers’, a maths teacher ask the following
questions.
1. Which of the following is an irrational number?
(A) 3.245245 (B) 3.245 (C) 3.245245245… (D) 3.245224522245....
2. What is the decimal form of 2/11?
(A) 0.81
(B) 0.18 (C) 0.17 (D) 0.71
3. Decimal form of 2/11 is
(A) terminating (B) non-terminating
(C) non-terminating and non-recurring (D) non-terminating and recurring

4. Write p/q form of 0.38 .


(A) 38/99 (B) 7/99 (C) 11/18 (D) 1/18
5. If p/q form of 0.38 is m/n then find the value of m + n.
(A) 137
(B) 90 (C) 140 (D) 130

qq
UNIT-II ALGEBRA Study Time:
Maximum time: 3:15 Hrs
Maximum questions: 64
CHAPTER

2 POLYNOMIALS

Definition of a polynomial in one variable, with examples and counter examples.


Syllabus Coefficients of a polynomials terms of a polynomial and zero polynomial. Degree of a
polynomial. Constant, linear, quadratic, cubic polynomials. Monomials, binomials,
trinomials. Factors and multiples. Zeroes of a polynomial. Motivate and state the
Remainder Theorem with examples. Statement and proof of the Factor Theorem. Factorization of ax2 + bx + c, a ≠ 0,
where a, b and c are real numbers, and of cubic polynomials using the Factor Theorem.
Recall of algebraic expressions and identities. Verification of identities:
(x + y + z)2 = x2 + y2 + z2 + 2xy + 2yz + 2zx,
(x± y)3 = x3 ± y3 ± 3xy(x ± y),
x3 ± y3 = (x ± y)(x2 ∓ xy + y2),
x3 + y3 + z3 – 3xyz = (x + y + z)(x2 + y2 + z2 – xy – yz – zx)
and their use in factorization of polynomials.

List of Topics
Topic-1: Polynomials
Topic-1 Polynomials  Page No. 28
Topic-2 : Factor Theorem
 Page No. 33
Topic-3 : Algebraic Identities
Revision Notes  Page No. 36

 Polynomial : The algebraic expression in which the variables involved have only non-negative integral exponent
is called ‘Polynomial’.
A polynomial p(x) in one variable x is an algebraic expression in x of the form
p (x) = anxn + an–1xn–1+an–2xn–2 + … + a2x2 + a1x + a0.
where a0, a1, a2, …, an are real numbers and an ≠ 0. Here, a0, a1, a2, … +a are respectively the co-efficients of x0, x1,
n
x2, …, xn and n is called the degree of the polynomial. Scan to know
more about
This form of polynomial is known as the “Standard form of Polynomial”. this topic
e.g., : 2x3 – 4x2 + 5x – 7 is a polynomial in one variable (x).
 Constant Polynomial : A polynomial of degree zero is called a constant polynomial.
7 3

e.g.,: 4, – , are constant polynomials. Polynomials
5 4
 Zero Polynomial : A zero polynomial is a polynomial whose whole value is zero. The coefficients of the variables
are equal to zero and the constant term is zero so the degree of zero polynomial is not defined.
 Degree of a Polynomial : Highest power of variable in a polynomial is called the ‘degree of a polynomial’.

l In polynomial of one variable, the highest power of the variable is called the degree of the polynomial.
e.g., : 4x7 – 3x3 + 2x2 + 3x – 6 is a polynomial in x of degree 7.

POLYNOMIALS 29
30 Oswaal CBSE Question Bank Chapterwise & Topicwise, MATHEMATICS, Class-IX
l In a polynomial of more than one variable, the sum of the powers of variables in each term is taken up and the
highest sum so obtained is called the degree of the polynomial.
e.g.,: 7x3 – 4x2y2 + 3x2y – 3y + 9 is a polynomial in x and y of degree 4.
 Types of Polynomials :
l On the basis of terms
Term : In a polynomial x2 + 3x + 4, the expressions x2, 3x and 4 are called terms.
e.g.,: Polynomial x2 + 3x + 7 has three terms.
(I) Monomial : A polynomial of one non-zero term, is called a monomial.
e.g.,: 2x, –4x2, 7x3, 10x are monomials.
(II) Binomial : A polynomial of two non-zero terms, is called a binomial.
e.g.,: (4x2 + 8), (7y2 – 3y), (3x – 6), (10x2 – 4) are binomials.
(III) Trinomial : A polynomial of three non-zero terms, is called a trinomial.
7
e.g.,: (x2 + 2x + 4), (4x2 + x+14), (3x2+3 3 x + 3 ) are trinomials.
5
l On the basis of degree

(I) Linear Polynomial : A polynomial of degree 1 is called a linear polynomial. It is expressed in the form of
ax + b, where a and b are real constants and a ≠ 0.
e.g.,: 3x + 6 is a linear polynomial in x.
(II) Quadratic Polynomial : A polynomial of degree 2 is called a quadratic polynomial. It is expressed in the form

of ax2 + bx + c, where a, b and c are real constants and a ≠ 0.
e.g.,: 3x2 + 4x + 1 is a quadratic polynomial in x.

(III) Cubic Polynomial : A polynomial of degree 3 is called a cubic polynomial. It is expressed in the form of

ax3 + bx2 + cx + d, where a, b, c and d are real constants and a ≠ 0.
e.g.,: 4y3 + 3y2 + 7y + 4 is a cubic polynomial in y.

 Zeroes of a Polynomial : A polynomial is when equal to zero then all the value of x are called zeroes of the
polynomial.

Zero of a polynomial p(x) is a number c such that p(c) = 0.
(i) ‘0’ may be a zero of a polynomial.
(ii) Every linear polynomial in one variable has a unique zero of a polynomial.
(iii) A non-zero constant polynomial has non zero of a polynomial.

(iv) Every real number is a zero of the zero polynomial.
(v) Maximum number of zeroes of a polynomial is equal to its degree.
Example 1 : Find the zero of the polynomial p (x) = 3x + 1.

Solution : Finding the zero of p (x), is the same as solving the equation

p (x) = 0
−1 −1
Now, 3x + 1 = 0, Þ x = . So, is the zero of the polynomial 3x + 1.
3 3

Example 1

Verify whether 3 and 0 are zeroes of polynomial p(3) = (3)2 – 3(3)


x2 – 3x. = 9 – 9
Solution: = 0
Step I : Write the given polynomial equal to p(x) Step III : Similarly, putting 0 in place of x and
p(x) = x2 – 3x ...(i) solving it.
Step II : Putting 3 in place of x and solve it to find Putting x = 0 in (i), we get
the value of p(3). If the value of p(3) is zero, then 3 p(0) = 02 – 3(0)
will be zero of the given polynomial. = 0
Putting x = 3 in (i), we get So, 3 and 0 are zeroes of the given polynomial p (x).
POLYNOMIALS 31

Mnemonics
1. Memorising by ‘SOAP’ Mnemonic
Same opposite Always Positive
a3 + b3 = (a + b) (a2 – ab + b2)

same
opposite Always
Positive
2. We can check the long division using mnemonic
Dirty Monkeys Smell Bad
(i) Divide the leading term of the dividend by the leading term of the divisor. Write this quotient directly above
the term you just divided into.
(ii) Multiply: Multiply the quotient from step by the entire divisor and write it under the dividend so the like
terms are lined up.
(iii) Subtract: Change the sign of the subtrahend and subtract.
(iv) Bring down the next term and Repeat steps (i) to (iv).

SUBJECTIVE TYPE QUESTIONS


Q. 5. If p(x) = x2 – 3x + 2, then what is the value of
Very Short Answer Type p(0) + p(2) ? U
Questions (1 mark each)
Sol. Putting, x =0
Q. 1. What is the degree of the polynomial (x3 + 5) p(0) = 0 – 3 × 0 + 2 = 2
(4 – x5) ? U Putting, x =2
Sol. Degree of x3 + 5 = 3 p(2) = 22 – 3 × 2 + 2
= 4 – 6 + 2 = 0
Degree of 4 – x5 = 5
Thus, p(0) + p(2) = 2 + 0 = 2 1
Now, (x3 + 5)(4 – x5) = 4x3 – x8 + 20 – 5x5
= –x8 – 5x5 + 4x3 + 20 Short Answer Type
3 5
Degree of (x + 5)(4 – x ) = 8. 1 Questions-I (2 marks each)
π Q. 1. Classify the following as linear, quadratic and
Q. 2. In the expression x2 + x – 7, what is the
2 cubic polynomials : R [Board Term I, 2013]
coefficient of x ? U (a) x2 + x (b) x – x3
π π
Sol. Coefficient of x in expression x2 + x – 7 is . 1 (c) 1 + x (d) 7x3
2 2
Sol. Linear polynomial ® 1 + x; degree = 1 ½
Q. 3. If –4 is a zero of the polynomial
2
p(x)= x2 + 11x + k, then calculate the value of k.
Quadratic polynomial ® x + x; degree = 2
½
3 3
Cubic polynomial ® x – x , 7x ; degree = 3
1
U
Sol. Given, p(x) = x2 + 11x + k Q. 2. Find the value of the polynomial
Since, – 4 is a zero of polynomial
p(x) = x3 – 3x2 – 2x + 6 at x = 2

p(– 4) = 0
or, (– 4)2 + 11 × (– 4) + k = 0 U [Board Term I, 2014]
or, 16 – 44 + k = 0 3 2
Sol. Given, p(x) = x – 3x – 2x + 6 ½
\ k = 28 1
Q. 4. Write the zeroes of the polynomial  Then, p( 2 ) = ( 2 )3 − 3( 2 )2 − 2( 2 ) + 6 ½

p(x) = x(x – 2)(x – 3). U = 2 2 − 6 − 2 2 + 6 ½
Sol. For zeroes, put p(x) = 0

=0 ½
\ x(x – 2)(x – 3) = 0,
Therefore, x = 0, 2, 3 1
32 Oswaal CBSE Question Bank Chapterwise & Topicwise, MATHEMATICS, Class-IX
Q. 3. If y = 2 and y = 0 are the zeroes of the polynomial 2
 1  1  1 49
f(y) = 2y3 – 5y2 + ay + b, find the value of a and b. and f   =   − 5  + 7 = 1
 3  3  3 9
Ap [Board Term I, 2016]
Sol. Given, f(y) = 2y – 5y2 + ay + b
3
 1 49 −59
Thus, f(2) – f(–1) + f   = 1 − 13 + = 1
\ f(2) = 2(2)3 – 5(2)2 + a(2) + b = 0  3 9 9
or, 16 – 20 + 2a + b = 0 1
Q. 2. Find the value of the polynomial x2 – 3x + 6 at
or, 2a + b = 4 ...(i)
(i) x = 2 , (ii) x = 3. U
and f(0) = b = 0
From (i), 2a + 0 = 4 Sol. Given, p(x) = x2 – 3x + 6
or, a =2
(i) When x= 2
\ a = 2, b = 0 1
[CBSE Marking Scheme, 2016]
Then, p( 2 ) = ( 2 )2 − 3 × 2 + 6 1

Short Answer Type = 2 – 3 2 + 6
Questions-II (3 marks each)
= 8 – 3 2 2
Q. 1. If f(x) = 3x + 5, evaluate f(7) – f(5). U
Sol. Given, f(x) = 3x + 5 (ii) When x =3
\ f(7) = 3 × 7 + 5 = 26 1 Then, p(3) = 32 – 3 × 3 + 6
and f(5) = 3 × 5 + 5 = 20 1 = 9 – 9 + 6
\ f(7) – f(5) = 26 – 20 = 6 1
= 6 2
Q. 2. If f(x) = x3 – 3x2 + 3x – 4, find f(2) + f(– 2) + f(0). 4 3 2
Q. 3. If f(x) = x – 4x + 3x – 2x + 1, then check whether
U [Board Term I, 2016]
f(0) × f(– 1) = f(2) is true or not.
Sol. Given, f(x) = x3 – 3x2 + 3x – 4
U [Board Term I, 2016]
\ f(2) = (2)3 – 3(2)2 + 3(2) – 4
= 8 – 12 + 6 – 4 Sol. Given, f(x) = x – 4x + 3x2 – 2x + 1
4 3

f(2) = – 2 1 \ f(0) = 1 1
and f(– 2) = (– 2)3 – 3(– 2)2 + 3(– 2) – 4 and 4 3 2
f(– 1) = (– 1) – 4(– 1) + 3(– 1) – 2(– 1) + 1
= – 8 – 12 – 6 – 4
= 1 + 4 + 3 + 2 + 1 = 11
f(– 2) = – 30
and f(0) = – 4 1 and f(2) = (2)4 – 4(2)3 + 3(2)2 – 2(2) + 1
\ f(2) + f(– 2) + f(0) = – 2 – 30 – 4 = – 36 1 = 16 – 32 + 12 – 4 + 1
Q. 3. If f(x) = 5x2 – 4x + 5, find f(1) + f(– 1) + f(0). = 29 – 36 = – 7 1
U [Board Term I, 2016] \ f(0) × f(– 1) = 11
...(i)
Sol. Given, f(x) = 5x2 – 4x + 5
f(2) = –7 ...(ii)
\ f(1) = 5 – 4 + 5
= 6 1 From (i) and (ii) we get
and f(– 1) = 5(– 1)2 – 4(– 1) + 5 (11) ¹ (–7) 2
= 5 + 4 + 5 = 14 \ f(0) × f(– 1) ¹ f(2) (Not true)

and f(0) = 5 1
[CBSE Marking Scheme, 2016]
\ f(1) + f(– 1) + f(0) = 6 + 14 + 5 = 25 1

Long Answer Type Commonly Made Error


Questions (5 marks each)
Students commit errors while substituting and
 1 calculating values.
Q. 1. If f(x) = x2 – 5x + 7, evaluate f(2) – f(–1) + f   .
 3

U [Board Term I, 2014] Answering Tip


Sol. Given, f(x) = x2 – 5x + 7 1
Practice to substitute the values in polynomial
Then, f(2) = 22 – 5 × 2 + 7 = 1 1 properly.
and f(–1) = (– 1)2 – 5 (–1) + 7 = 13 1
POLYNOMIALS 33

Topic-2 Factor Theorem

Revision Notes

 If p(x) is a polynomial of degree x ³ 1 and a is any real number, then


(i) Graph of linear equation is a straight line, while graph of quadratic equation is a parabola.
(ii) Degree of polynomial = Number of zeroes of polynomial.
(iii) If remainder r(x) = 0, then g(x) is a factor of p(x).
(iv) (x + a) is a factor of polynomial p(x), if p(– a) = 0. Scan to know
more about
(v) (x – a) is a factor of polynomial p(x), if p( a) = 0. this topic
(vi) (x – a)(x – b) is a factor of polynomial p(x), if p(a) = 0 and p(b) = 0.
(vii) (ax + b) is a factor of polynomial p(x), if p(– b/a) = 0.
(viii) (ax – b) is a factor of polynomial p(x), if p(b/a) = 0.
 Factorization of a Polynomial : Factor theorem

 By Splitting the Middle Term :


Let a quadratic polynomial be x2 + lx + m, where l and m are constants.
Factorize the polynomial by splitting the middle term lx as ax + bx, so that ab = m. Then,
x2 + lx + m = x2 + ax + bx + ab
= x(x + a) + b(x + a).
= (x + a) (x + b).
 By using Factor Theorem :

Consider a quadratic polynomial ax2 + bx + c, where a, b and c are constants. It has two factors (x – a) and (x – b).
\ ax2 + bx + c = a(x – a)(x – b)
or, ax2 + bx + c = ax2– a(a + b)x + aab
−b c
On equating the coefficient of x and constant term, we get a + b = and ab = .
a a
On simplifying, we get the value of a and b.
Example 1. Factorize 6x2 + 17x + 5 by splitting the middle term, and by using the Factor Theorem.
Solution :
(i) By Splitting the Middle Term :
If we find the two numbers a and b, such that a + b = 17 and ab = 6 × 5 = 30, then we can get the factors. Factors
of 30 are 1 and 30, 2 and 15, 3 and 10, 5 and 6, of these pairs, 2 and 15 will give us a + b = 17.
So, 6x2 + 17x + 5 = 6x2 + (2 + 15)x + 5
= 6x2 + 2x + 15x + 5
= 2x(3x + 1) + 5(3x + 1)
= (2x + 5)(3x + 1)
(ii) By Factor Theorem :
 17 5
6x2 + 17x + 5 = 6  x 2 + x +  = 6p(x).
 6 6 

5
If a and b are the zeroes of p(x), then, new line 6x2 + 17x + 5 = 6(x – a)(x – b), and ab = .
6
Then, see some possibilities for a and b.
1 1 5 5
They could be ± , ± , ± , ± , ± 1.
2 3 3 2
 1 1 17  1  5
Now, p   = +   + ≠ 0 ,
 2 4 6  2 6
 −1   1
But, p   = 0. So,
 3  x +  is a factor of p(x).
3
 5
Similarly, we will get  x +  as a factor of p(x).
 2
34 Oswaal CBSE Question Bank Chapterwise & Topicwise, MATHEMATICS, Class-IX

 1  5
\ 6x2 + 17x + 5 = 6  x +   x + 

 3  2
 3x + 1   2 x + 5 
= 6  = (3x + 1)(2x + 5).
 3   2 
In this example, the use of the splitting method appears more efficient.

Example 2
Factorize the cubic polynomial x3 + 6x2 + 11x + 6. = – 12 + 12 = 0
Solution: So, (x + 1) is a factor of p(x).
Step I : Consider the given cubic polynomial as p(x) Step IV : Now, write p(x) as the product of this
and find the constant term factor and a quadratic polynomial.
p(x) = x3 + 6x2 + 11x + 6 On dividing p(x) by (x + 1), we get quotient x2 + 5x + 6
Here,constant term = 6 So, p(x) = (x + 1)(x2 + 5x + 6)
Step II : Find all the factors of constant term of p(x). Step V : Now, use splitting method or factor
All possible factor of 6 are ± 1, ± 2, ± 3 and ± 6. theorem to find the factor of p(x).
Step III : Check at which factor, p(x) is zero by trial Now, by splitting the middle term of quotient, we get
method and get one factor of p(x). p(x) = (x + 1)[x2 + 3x + 2x + 6]
At x =–1 = (x + 1)[x(x + 3) + 2(x + 3)]
p( – 1) = (– 1)3 + 6(– 1)2 = (x + 1)(x + 2)(x + 3)
+ 11(– 1) + 6 Hence, the factors of given polynomial are
= – 1 + 6 – 11 + 6 (x + 1), (x + 2) and (x + 3).

SUBJECTIVE TYPE QUESTIONS


Q. 4. Factorize : 6 – x – x2 U [Board Term I, 2016]
Very Short Answer Type
Sol. 6 – x – x = 6 – 3x + 2x – x2
2
Questions (1 mark each)
= 3(2 – x) + x(2 – x)
 1
Q. 1. If f(x) be a polynomial such that f  −  = 0, then = (2 – x)(3 + x) 1
 3
[CBSE Marking Scheme, 2016]
calculate one factor of f(x). R
 1
Sol. Since, f −  = 0
 3 Commonly Made Error
1
\ − is a zero of polynomial f(x). While splitting the middle term in factorisation,
3
the students commit an error in putting
1
So, x + or 3x + 1 is a factor of f(x). 1 signs because in multiplication two negative
3 signs give positive product while adding two
Q. 2. Find the value of m, if x + 4 is a factor of the negative numbers, the result is negative.
polynomial x2 + 3x + m. U [Board Term I, 2015]
Sol. Given, x + 4 is a factor of x2 + 3x + m = p(x)
\ p(– 4) = 0
or, 16 – 12 + m = 0
Answering Tip
or, m =–4 1
Q. 3. Find the value of k, if 2x – 1 is a factor of the Students should be careful with signs and do
polynomial 6x2 + kx – 2. U [Board Term I, 2015] ample practice so that they do not make errors,
Sol. Since, 2x – 1 is a factor of p(x) = 6x2 + kx – 2 while splitting the middle term.

 1 Q. 5. Factorize : 8y3 – 125x3 U [Board Term I, 2016]


Thus, p  = 0
 2
Sol. 8y3 – 125x3 = (2y)3 – (5x)3
1 1 = (2y – 5x)(4y2 + 10xy + 25x2)
or, 6. + k . − 2 = 0
4 2 1
or, k =1 1 [CBSE Marking Scheme, 2016]
[CBSE Marking Scheme, 2015]
POLYNOMIALS 35
Q. 5. Find the value of ‘k’ if (x – 1) is a factor of
Short Answer Type
Questions-I (2 marks each) p(x) = 2x2 + kx + 2 . U [Board Term I, 2014]
2
Sol. Given, p(x) = 2x + kx + 2
Q. 1. Find the value of k, so that polynomial x3 + 3x2 – kx – 3
Since, (x – 1) is a factor of p(x), then p(1) = 0. ½
has one factor as x + 3. U [Board Term I, 2016]
Sol. Let f(x) = x3 + 3x2 – kx – 3 \ 2 (1)2 + k(1) + 2 =0 ½

Since, (x + 3) is a factor of f(x). or, 2+k+ 2 =0 ½


Then, f(– 3) = 0 1 or, k = –2 – 2 ½
3 2
or, (– 3) + 3(– 3) – k(–3) – 3 = 0
Short Answer Type
or, – 27 + 27 + 3k – 3 = 0
Questions-II (3 marks each)
or, 3k – 3 = 0
Q. 1. Factorize : x3 – 3x2 – 9x – 5
or, k =1 1
U [Board Term I, 2014], [NCERT]
Q. 2. Find the value of k, if x – 2 is a factor of
f(x) = x2 + kx + 2k. U [Board Term I, 2016] Sol. Let p(x) = x3 – 3x2 – 9x – 5,
Since, p (– 1) = – 1 – 3 + 9 – 5 = 0
Sol. Given, (x – 2) is a factor of f(x).
Therefore, (x + 1) is a factor of x3 – 3x2 – 9x – 5. ½
\ f(2) = 0 1 \ (x3 – 3x2 – 9x – 5) = (x + 1)(x2 – 4x – 5) 1
2
Þ (2) + k(2) + 2k = 0 Now, x2 – 4x – 5 = x2 – 5x + x – 5
Þ 4 + 2k + 2k = 0 = x(x – 5) + 1(x – 5)
Þ 4 + 4k = 0 x2 – 4x – 5 = (x + 1)(x – 5) 1½
\ p(x) = (x + 1)(x – 5)(x + 1)
Þ k =–1 1
[CBSE Marking Scheme, 2014]
[CBSE Marking Scheme, 2016]
Q. 2. Factorize : 2y3 + y2 – 2y – 1 U [NCERT]
Sol. Let p(y) = 2y3 + y2 – 2y – 1
Commonly Made Error Q p(– 1) = 2(– 1)3 + (– 1)2 – 2(– 1) – 1
= – 2 + 1 + 2 – 1
Sometimes students are confused between = 0 2
Remainder Theorem and Factor Theorem. \ (y + 1) is a factor of given polynomial.
\ 2y3 + y2 – 2y – 1 = (y + 1)(2y2 – y – 1)
Now, 2y2 – y – 1 = 2y2 – 2y + y – 1
= 2y(y – 1) + 1(y – 1)
Answering Tip = (2y + 1)(y – 1)
p(y) = (y + 1)(2y + 1)(y – 1) 1
Understand the concepts of both the theorems
and do adequate practice to solve problems Long Answer Type
based on them. Questions (5 marks each)
Q. 3. Find the value of ‘a‘ for which (x – 1) is a factor of
Q. 1. Using factor theorem, show that (m – n), (n – p) and
the polynomial a2x3 – 4ax + 4a – 1.
(p – m) are factors of
U [Board Term I, 2016]
m(n2 – p2) + n(p2 – m2) + p(m2 – n2)
Sol. Let f(x) = a2x3 – 4ax + 4a – 1 U [Board Term I, 2015]
Since, (x – 1) is a factor of f(x)
Then, f(1) = 0 1 Sol. Let f = m(n2 – p2) + n(p2 – m2) + p(m2 – n2)
Þ a2(1)3 – 4a(1) + 4a – 1 = 0 \ f(m) = n = n(n2 – p2) + n(p2 – n2) + p(n2 – n2)
Þ a2 – 4a + 4a – 1 = 0 = n(n2 – p2) – n(n2 – p2) + 0
Þ a2 – 1 = 0 = 0
Þ a =±1 1 So, m – n is a factor of f.
[CBSE Marking Scheme, 2016] Similarly, f(n = p) = 0 & f(p = m)= 0
Q. 4. For what value of k, is the polynomial p(x) = 2x3 – \ (m – n), (n – p) and (p – m) are factors of f. 5
kx2 + 3x+ 10 exactly divisible by (x + 2) ? U [CBSE Marking Scheme, 2015]
[Board Term I, 2014] Q. 2. Factorize : 9x3 – 3x2 – 5x – 1
Sol. Since, (x + 2) is a factor of p(x). ½ U [Board Term I, 2016]
Thus, p(–2) = 0 Sol. Let p(x) = 9x3 – 3x2 – 5x – 1
or, 2(–2)3 – k(–2)2 + 3(–2) +10 = 0 1
Factors of 1 = ± 1 1
or, – 16 – 4k – 6 + 10 = 0
p(1) = 9 – 3 – 5 – 1 = 0
k =–3 ½
36 Oswaal CBSE Question Bank Chapterwise & Topicwise, MATHEMATICS, Class-IX

i.e., x – 1 is a factor of p(x). 1 x2 – 22x + 120


\ 9x3 – 3x2 – 5x – 1 = 9x2(x – 1) + 6x(x – 1) + 1(x – 1) x – 1)x3 – 23x2 + 142x – 120
= (x – 1)(9x2 + 6x + 1) 1 x3 – x2
(–) (+)
= (x – 1)(3x + 1)2 1
– 22x2 + 142x – 120
\ p(x) = (x – 1)(3x + 1)(3x + 1) 1
– 22x2 + 22x
[CBSE Marking Scheme, 2016] (+) (–)
Q. 3. Factorize : x3 – 23x2 + 142x – 120. U [KVS, 2019] 120x – 120
3 2
Sol. x – 23x + 142x – 120 120x – 120
Put, x =1 (–) (+)
Then, x3 – 23x2 + 142x – 120 0
= (1)3 – 23(1)2 + 142(1) – 120 3 2 2
x – 23x + 142x – 120 = (x – 1)(x – 22x + 120)
= 1 – 23 + 142 – 120
= (x – 1)(x2 – 12x – 10x + 120)
= 143 – 143 = 0
\ Remainder = 0 = (x – 1)[x(x – 12) – 10(x – 12)]
\ (x – 1) is a factor of x3 – 23x2 + 142x – 120. \ x3 – 23x2 + 142x – 120 = (x – 1)(x – 10)(x – 12) 5

Topic-3 Algebraic Identities

Revision Notes
 Algebraic Identities :
An algebraic identity is an algebraic equation that is true for all values of the variables occurring in it.
 Some useful algebraic identities:
(i)
(x + y)2 = x2 + 2xy + y2 Scan to know
more about
(ii) (x – y)2 = x2 – 2xy + y2 this topic
(iii) x2 – y2 = (x + y)(x – y)
(iv) (x + a)(x + b) = x2 + (a + b)x + ab
(v) (x + y + z)2 = x2 + y2 + z2 + 2xy + 2yz + 2zx
Algebraic
(vi) (x + y)3 = x3 + y3 + 3xy(x + y) identities

(vii) (x – y)3 = x3 – y3 – 3xy(x – y)


(viii) x3 + y3 + z3 – 3xyz = (x + y + z)(x2 + y2 + z2 – xy – yz– zx)
(ix) x3 + y3 = (x + y)(x2 – xy + y2)
(x) x3 – y3 = (x – y)(x2 + xy + y2)
Example 1. Factorize the following :
(i) x2 + 4xy + 4y2 (ii) x3 – 8
(iii) 4x2 – 12xy + 9y2 (iv) x3 + 8y3 + 6x2y + 12xy2
Solution :
(i) x2 + 4xy + 4y2 = (x)2 + 2 × (x) × (2y) + (2y)2 = (x + 2y)2 = (x + 2y)(x + 2y)
(ii) x3 – 8 = x3 – 23 = (x – 2)(x2 + 2x + 22) = (x – 2)(x2 + 2x + 4)
(iii) 4x2 – 12xy + 9y2 = (2x)2 – 2(2x)(3y) + (3y)2 = (2x – 3y)2
= (2x – 3y)(2x – 3y)
(iv) x3 + 8y3 + 6x2y + 12xy2
= (x)3 + (2y)3 + 3(x)(2y)[x + 2y]
= (x + 2y)3 = (x + 2y)(x + 2y)(x + 2y)
POLYNOMIALS 37

Example 3
Evaluate (102)3 by using suitable identities. Step III : Use the identity (x + y)3 = x3 + y3 + 3xy
Solution: (x + y) to expand it.
Step I : Express the given number without power (102)3= (100 + 2)3
as the sum or difference of two numbers = (100)3 + (2)3 + 3(100)(2)(100 + 2)
Given number without power is 102. Since, it is Step IV : Simplify the above expression.
greater than 100, so it can be written as 100 + 2 (102)3= 1000000 + 8 + 60000 + 1200
\ (102)3 = (100 + 2)3 = 1000000 + 8 + 61200
Step II : Compare the expression with (x + y)3. = 1061208
On comparing (100 + 2)3 with (x + y)3, we get
x = 100 and y = 2

SUBJECTIVE TYPE QUESTIONS


3 3
1  2  1 2 1 2y 
Very Short Answer Type =  x −  y − 3 × x × y  x − 
3  
1
3  3 3 3 3
Questions (1 mark each)
Q. 1. Write the coefficient of x2 in the expansion of x 3 8 y 3 2 xy  x 2 y 
(x – 2)3. A = − −  − 
27 27 3 3 3 
Sol. (x – 2)3 = (x)3 – (2)3 – 3 × x × 2(x – 2)
= x3 – 8 – 6x2 + 12x
Coefficient of x2 in the expansion of (x – 2)3 = – 6 1 x 3 8 y 3 2 x 2 y 4 xy 2
= − − + 1
1 2 1 27 27 9 9
Q. 2. If x + = 4, then calculate the value of x + 2 .
x x Q. 3. Factorize : 8a3 + 8b3 U [Board Term I, 2014]
 U Sol. 8a3 + 8b3 = (2a)3 + (2b)3
2
1  1  1
= (2a + 2b)[(2a)2 + (2b)2 – (2a) × (2b)] 1
Sol. x 2 + 2 =  x +  − 2( x )  
x x x [ a3 + b3 = (a + b)(a2 + b2 – ab)]
= (4)2 – 2 = 16 – 2 = 14 1
= 2(a + b) × 4(a2 + b2 – ab)
833 + 17 3
Q. 3. Calculate the value of . = 8(a + b)(a2 + b2 – ab) 1
832 − 83 × 17 + 17 2 3
Q. 4. Factorize : 8x – (2x – y) 3
U [Board Term I, 2015]
Ap [NCERT Exemp.]
 Sol. 8x3 – (2x – y)3 = (2x)3 – (2x – y)3
Sol. 2
833 + 17 3
2 =
( )
( 83 + 17 ) 83 2 − 83 × 17 + 17 2
= [2x – (2x – y)][(2x)2 + (2x – y)2


83 − 83 × 17 + 17 (
83 2 − 83 × 17 + 17 2) + 2x(2x – y)]
[Since, (a3 – b3) = (a – b)(a2 + b2 + ab)]
[ a3 + b3 = (a+ b)(a2 – ab + b2)]
= 83 + 17 = 100 1 = y[4x2 + 4x2 + y2 – 4xy
+ 4x2 – 2xy]
Short Answer Type = y[12x2 + y2 – 6xy] 2
Questions-I (2 marks each) [CBSE Marking Scheme, 2015]
Q. 1. Expand by using identity (2x – y + z)2. Q. 5. Factorize : 64a3 – 27b3 – 144a2b + 108ab2
R [Board Term I, 2014] U [Board Term I, 2014], [NCERT]
Sol. By using the identity, (a + b + c)2
Sol. 64a3 – 27b3 – 144a2b + 108ab2
= a2 + b2 + c2 + 2ab + 2bc + 2ca
= (4a)3 – (3b)3 – 3 × (4a)2 × (3b) + 3 × (4a) × (3b)2 ½
(2x + (–y) + z) = (2x)2 + (–y)2 + z2 + 2(2x)(–y)
2
= (4a)3 – (3b)3 – 3 × 4a × 3b(4a – 3b) ½
 + 2(–y)(z) + 2(z)(2x)
= (4a – 3b)3  1
= 4x2 + y2 + z2 – 4xy – 2yz + 4xz
3
1 2 
Q. 2. Expand :  x − y U [Board Term I, 2013]
3 3  Commonly Made Error
3 While factorizing, the students factorize once
1 2 
Sol.  x − y and leave the answer without checking that
3 3 
few expressions can be further factorized.
38 Oswaal CBSE Question Bank Chapterwise & Topicwise, MATHEMATICS, Class-IX

1  1  2 1 1
Now, x3 + = x +   x + 2 − x· 
Answering Tip x3  x x x
 1  1 
Students should be particular and check =  x +   x 2 + 2 − 1
 x  x 
whether an expression can be further factorized
otherwise they tend to miss one step and = (3)(7 – 1)
thereby their marks. = 3 × 6 = 18 1
2 1 3 1
Q. 5. If x + 2 = 98 , then find value of x + 3 .
x x
Short Answer Type U [Board Term I, 2016]
Questions-II (3 marks each)
1
Q. 1. Using a suitable identity, find (98)3. Sol. x2 + = 98

x2
A [Board Term I, 2016]
2
Sol. (98)3 = (100 – 2)3  1

= (100)3 – (2)3 – 3 × 100 × 2(100 – 2) 1
or,  x + x  = 98 + 2 = 100 1
 

= 1000000 – 8 – 600 × (100 – 2) 1

= 1000000 – 8 – 60000 + 1200 1 x+ = ± 10

x
= 1000000 – 58808 = 941192 1
Q. 2. Evaluate : ( 2 + 3 )2 + ( 5 − 2 )2 1  1  2 1 1 
x3 + =  x +   x − x· + 2  1
U [Board Term I, 2013] x 3
 x  x x 
Sol. ( 2 + 3 )2 + ( 5 − 2 )2 = ( 2 )2 + ( 3 )2 + 2 × 2  2 1 
= ± (10 )  x + 2 − 1 
2
× 3 + ( 5) + ( 2) − 2 × 5 × 2 12
 x 

= ± (10)(98 – 1)
= 2 + 3 + 2 6 + 5 + 2 – 2 10 1
= ± 10 × 97 = ± 970 1
= 12 + 2 6 − 2 10 [CBSE Marking Scheme, 2016]
= 2( 6 + 6 − 10 ) 1
Q. 3. Simplify : (2a + 3b)3 – (2a – 3b)3
U [Board Term I, 2015] Commonly Made Error
Sol. Let (2a + 3b)3 – (2a – 3b)3= x3 – y3, where 2a + 3b =x
and 2a – 3b = y 1
Sometimes students take whole cube of z2 +
= (x –y)(x + xy + y ) 2 2 z2
= [(2a + 3b) – (2a – 3b)][(2a + 3b)2 + (2a + 3b) = 14, which leads to wrong calculation.
(2a – 3b) + (2a – 3b)2] 1
= 6b[(4a + 12ab + 9b2) + (4a 2 – 9b2)
2
Answering Tip
+ (4a2 – 12ab + 9b2)]
= 6b(12a2 + 9b2) 1 Students should use the identity
2 2 3
= 6b × 3 × (4a + 3b )  3 1  1  1
 z + z3  =  z + z  − 3  z + z 
= 18b(4a2 + 3b2) 1      
2 1 3 1
Q. 4. If x + 2 = 7. Find the value of x + 3 , taking
x x
1 Long Answer Type
only the positive value of x + . Questions (5 marks each)
x
U [Board Term I, 2016] 1 4 1
 1
2
1 1 Q. 1. If x −= 2, find x + 4 .
Sol.  x +  = x + x 2 + 2·x· x
2
x x
x U [Board Term I, 2015, Set-2]
1 Sol. On squaring both sides
= x + +2
2 2
x2  1 2
 x −  =2 1
 1  x
∵ x + x 2 = 7 
2
= 7 + 2 1 1
x2 + 2
− 2x × = 4
= 9 1 x x
1 1
 x + 2 =4+2=6
2
 x +  = ± 3 x
x
2
1  2 1  4 1
x+ = 3 [on taking +ve value] 1  x + 2  = x + 4  + 2

1
x x x 
POLYNOMIALS 39
1 1
Þ (6)2 = x 4 +
+2 1 Sol. RHS = (x + y + z)[(x – y)2 + (y – z)2 + (z – x)2] 1
x4 2
1 1
Þ 36 – 2 = x 4 + 4 1 = (x + y + z) [x2 + y2 – 2xy + y2 + z2 – 2yz + z2 + x2 – 2zx]
x 2
1 1
Þ x 4 + 4 = 34 1 = (x + y + z)[2x2 + 2y2 + 2z2 – 2xy – 2yz – 2zx] 2
x 2
Q. 2. Prove that (a2 – b2)3 + (b2 – c2)3 + (c2 – a2)3 = 1
3(a + b)(b + c)(c + a)(a – b)(b – c)(c – a). = (x + y + z). 2[x2 + y2 + z2 – xy – yz – zx]
2
U [Board Term I, 2016]
Sol. Let x = a2 – b2, y = b2 – c2, z = c2 – a2 = x3 + y3 + z3 – 3xyz
[Using identity x3 + y3 + z3 – 3xyz = (x + y + z)
Now, x + y + z = a2 – b2 + b2 – c2 + c2 – a2 (x2 + y2 + z2 – xy – yz – zx)] 2
= LHS
= 0 1

 x + y + z = 0 1 Q. 4. Factorize : x12 – y12. U [KVS, 2019]
Sol. x12 – y12
x3 + y3 + z3 = 3xyz 1
= (x6)2 – (y6)2
i.e., (a2 – b2)3 + (b2 – c2)3 + (c2 – a2)3
= (x6 – y6)(x6 + y6)
2
= 3(a – b2)(b2 – c2)(c2 – a2) 1 = {(x3)2 – (y3)2}(x6 + y6)

= 3(a + b)(a – b)(b + c)(b – c)(c + a)(c – a) = (x3 – y3)(x3 + y3)(x6 + y6)

= 3(a + b)(b + c)(c + a)(a – b)(b – c)(c – a) 1 = {(x)3 – (y)3}{(x)3 + (y)3}(x6 + y6)
3 3 3 1 = (x – y)(x2 + xy+ y2)(x + y)(x2 – xy+ y2)(x6 + y6)
Q. 3. Prove that x + y + z – 3xyz = (x + y + z)
2 = (x – y)(x + y)(x2 + xy + y2)(x2 – xy + y2)
[(x – y)2 + (y – z)2 + (z – x)2]  {(x2)3 + (y2)3}
A [Board Term I, 2015, Set-1] [NCERT] = (x – y)(x+ y)(x2 + xy + y2)(x2 – xy+ y2)(x2 + y2)
 (x4 – x2y2 + y4) 5

OBJECTIVE TYPE QUESTIONS (1 mark each)

Explanation : 2 = 2x 0
A Multiple Choice Questions Because exponent of x is 0.
Q. 1. Which one of the following is a polynomial ? Q. 3. Degree of the zero polynomial is
(A) 0 (B) 1
x2 2
(A) − (B) 2x − 1 (C) any natural number (D) not defined
2 x2
 A [NCERT Exemp.]
3
Sol. Option (D) is correct.
2 3x 2 x −1
(C) x + (D) Explanation : Because in zero polynomial the co-
x x +1 efficient of any variable is zero. So, we will not be
 A [NCERT Exemp.] able to determine the degree of the polynomial.
Sol. Option (C) is correct. Q. 4. If p ( x ) =x 2 − 2 2 x + 1, then p(2 2) is equal to
Explanation : It is a polynomial because degree of (A) 0 (B) 1
polynomial is whole number. (C) 4 2 (D) 8 2 +1
3x 3 / 2  A [NCERT Exemp.]
= x2 + 1/ 2
x Sol. Option (B) is correct.
3 1

= x 2 + 3x 2 2 Explanation :
3 −1 On putting x = 2 2 in given equation.
= x 2 + 3x 2
p( x ) =
x 2 − 2 2x + 1
= x 2 + 3x p(2 2=
) (2 2 )2 − 2 2 × 2 2 + 1
Q. 2. 2 is a polynomial of degree = 8−8+1= 9−8 = 1
(A) 2 (B) 0 Q. 5. One of the factors of (25x2 – 1) + (1 + 5x)2 is
1
(C) 1 (D) 2 (A) 5 + x (B) 5 – x
(C) 5x – 1 (D) 10x
 A [NCERT Exemp.]
Ans. Option (D) is correct.
Sol. Option (B) is correct. Explanation: (25x2 – 1) + (1 + 5x)2
40 Oswaal CBSE Question Bank Chapterwise & Topicwise, MATHEMATICS, Class-IX
[(5x)2 – (1)2] + (1 + 5x)2 Explanation :
[Using identity a2 – b2 = (a + b)(a – b)] x y
[(1 + 5x)(5x – 1)] + (1 + 5x)2 + = −1
y x
(1 + 5x)[5x – 1 + 1 + 5x]
(1 + 5x)(10x) x2 + y2
= −1
One of the factors = 10x xy

Q. 6. The factorisation of 4x2 + 8x + 3 is x2 + y2 =


− xy
(A) (x + 1) (x + 3) (B) (2x + 1) (2x + 3)
x 2 + y 2 + xy =
0 ...(i)
(C) (2x + 2) (2x + 5) (D) (2x – 1) (2x – 3)
Ans. Option (B) is correct. (
x 3 − y 3 = ( x − y ) x 2 + y 2 + xy )
Explanation:
4x2 + 8x + 3 (By splitting the middle term) x 3 − y 3 = ( x − y )( 0 ) [ Using equation(i)]
4x2 + 6x + 2x + 3
2x(2x + 3) + 1(2x + 3) x3 − y3 =
0
  
(2x + 1) (2x + 3)
( b)  1  1
2
Q. 7. Factorize 12a2b – 6ab2 Q. 11. If 49 x −
2
=  7 x +   7 x −  then the value
 2  2
(A) 6ab(2a – b) (B) 2ab(6a – 3b)
(C) 3ab(4a – 2b) (D) 6a(2ab – b) of b is
Ans. Option (A) is correct. 1
(A) 0 (B)
Explanation: 12a2b – 6ab2 2
= 6ab(2a – b) [By taking common] 1 1
(C) (D)
Q. 8. If (x + 2) is a factor of g(x) = 3x2 + x – k, then value 4 2
of k will be
 [NCERT Exemp.]
(A) 12 (B) 8
Ans. Option (C) is correct.
(C) 10 (D) 14
Explanation :
Ans. Option (C) is correct.
 1 
2

( )
2
Explanation: Since, x + 2 is a factor of g(x) 49x 2 − b = ( 7 x ) −   
2

then g(–2) = 0   2  
g(–2) = 3(–2)2 + (–2) – k = 0
= 12 – 2 – k = 0  Using identity a 2 − b 2 = ( a + b )( a − b ) 
k = 10
1
( b)
2
Q. 9. Which of the following is a factor of (x + y)3– (x3+ y3)? 49x 2 − =49x 2 −
4
(A) x2 + y2+ 2xy (B) x2 + y2 – xy
(C) xy 2
(D) 3xy On comparing both the sides
1
( )
2
 [NCERT Exemp.] − b = −
[(U) KVS 2019] 4
Ans. Option (D) is correct. On multiplying both sides by –1
Explanation : 1
( )
2
b =
(x + y ) − ( x3 + y3 ) 4
3

1
[Using identity a3 + b 3 = ( a + b )( a 2 + b 2 − 2ab )] b=
4
( )
= ( x + y ) − ( x + y ) x 2 + y 2 − xy 
3

Q. 12. If a + b + c = 0, then a3+ b3+ c3 is equal to


 (
= ( x + y ) ( x + y ) − x 2 + y 2 − xy 
2

 ) (A) 0 (B) abc


(C) 3abc (D) 2abc
[Using identity ( a + b )2 = a 2 + b 2 + 2ab]
 [NCERT Exemp.]
= ( x + y )  x 2 + y 2 + 2xy − x 2 − y 2 + xy  Ans. Option (C) is correct.
= (x + y )(3xy ) Explanation :
a+b+c =0
One of the factor of given polynomial is 3xy.
x y a+b =–c . ..(i)
Q. 10. If + = −1 ( x , y ≠ 0 ) the value of x3– y3 is On cubing both the sides
y x
(a + b) = ( −c )
3 3
(A) 1 (B) −1
a + b + 3ab ( a + b ) =−
( c)
3
1 3 3
(C) 0 (D)
2
a3 + b 3 + 3ab ( −c ) =( −c )
3

 [NCERT Exemp.]
Ans. Option (C) is correct. By using equation (i)
POLYNOMIALS 41
Here, p = 3, q = 2
( c)
3
a3 + b 3 − 3abc =−
Hence, Assertion is true
a3 + b 3 + c 3 =
3abc In reason x = 3 and 2 are the roots of x2 + 5x + 6
then, p(3) = 0 and p(2) = 0
px = x2 + 5x + 6
B Assertion & Reason x = 3 x =2
p(3) = (3)2 + 5 × 3 + 6 p(2) = (2)2 + 5 × 2 + 6
Directions: In the following questions, a statement p(3) = 9 + 15 + 6 p(2) = 4 + 10 + 6
of assertion (A) is followed by a statement of p(3) = 30 p(2) = 20
reason (R). Mark the correct choice as: \ p(3) ¹ 0 \ p(2) ¹ 0
(A) Both assertion (A) and reason (R) are true and reason Since, 3, 2 are not roots of x2 + 5x + 6
(R) is the correct explanation of assertion (A). Hence, reason is false.
(B) Both assertion (A) and reason (R) are true but reason
Q. 4. Assertion (A): If x + 2 is a factor of P(x) = 2x2 + 3x
(R) is not the correct explanation of assertion (A).
– k then value of k = 14
(C) Assertion (A) is true but reason (R) is false. Reason (R): (x – a) is a factor of polynomial P(x), if
(D) Assertion (A) is false but reason (R) is true. P(a) = 0.
Q. 1. Assertion (A): A polynomial may have more than Ans. Option (A) is correct.
one zero. Explanation: In case of assertion (A):
Reason (R): Every real number is zero of the zero Since, x – 2 is a factor of P(x)
polynomial. then P(2) = 0
Ans. Option (B) is correct. P(2) = 2 × (2)2 + 3 × 2 – k = 0
Explanation: In case of Assertion (A): 8+6–k =0
Maximum number of zeros of polynomial is equal k = 14
to its degree therefore polynomial may have more \ Assertion is true and R is correct explanation of A.
than one zero.
In case of Reason (R): Q. 5. Assertion (A): (3x + 4y)3 = 27x3 + 64y3 + 36xy(3x +
In zero polynomial 0(x – k) where k is a real number. 4y)
For determining the zero of the given polynomial Reason (R): (x + y)3 = x3 + y3 + 3xy(x + y)
put x – k = 0; x = k, where k can be any real number. Ans. Option (A) is correct.
Explanation: In case of Assertion (A):
Q. 2. Assertion (A): Degree of the polynomial 4x4 + 0x3
(3x + 4y)3= (3x)3 + (4y)3 + 3 × 3x × 4y(3x + 4y)
+ 0x5 + 5x + 7 is 4.
Þ 27x3 + 64y3 + 36xy(3x + y)
Reason (R): The highest power of the variable is 4,
Hence, Assertion is true and identity used in this
so the degree of polynomial is 4.
is (a + b)3 = a3 + b3 + 3ab(a + b) therefore reason is
Ans. Option (A) is correct.
also true and correct explanation of Assertion.
Explanation: In case of assertion (A):
Highest power of variable x is 4. Q. 6. Assertion (A): Value of (111)3 is 1367631
So, degree of the polynomial is 4 Reason (R): x3 + y3 = (x + y)(x2 – xy + y2)
Ans. Option (B) is correct.
Q. 3. Assertion (A): The equation x2 + 5x + 6 = (x + p)(x
Explanation: In case of assertion (A):
+ q) where p = 3, q = 2
(111)3 = (100 + 11)3
Reason (R): Roots of the polynomial x2 + 5x – 6 are
3, 2 = (100) + (11)3 + 3(100)2 × 11 + 3(100)(11)2
3

Ans. Option (C) is correct. = 1367631


Explanation: x2 + 5x + 6 = (x + p)(x + q) \ Assertion is True.
Þ x2 + 3x + 2x + 6 = (x + p)(x + q) In case of Reason (R) :
Þ (x + 3)(x + 2) = (x + p)(x + q) Identity used is true but not for Assertion.
Comparing the component, \ Both (A) and (R) are true but (R) is not correct
explanation of (A).

COMPETENCY BASED QUESTIONS (4 marks each)

A Case based MCQs


Read the following passage and answer any four
questions.
I. National Association For The Blind (NAB) aimed
to empower and well-inform visually challenged
population of our country, thus enabling them to
lead a life of dignity and productivity.
42 Oswaal CBSE Question Bank Chapterwise & Topicwise, MATHEMATICS, Class-IX

1  II. Beti Bachao, Beti Padhao (BBBP) is a personal


Ravi donated `  x 3 +  to NAB. When his cousin campaign of the Government of India that aims to
 x3 
generate awareness and improve the efficiency of
asks to tell the amount donated by him, he just gave welfare services intended for girls.
the below hint.
1
x + = 10
x
Answer the following questions :
Q. 1. (x + a)(x + b) = x2 + ................ x + ab 1
(A) a + b (B) ab
a
(C) a – b (D)
b
Ans. Option (A) is correct. In a school, a group of (x + y) teachers, (x2 + y2)
Q. 2. (x – y)3 = 1 girls and (x3 + y3) boys organised a campaign on
(A) x3 – y3 – 3xy (B) x3 – y3 – 3xy(x – y) Beti Bachao, Beti Padhao.
(C) x3 – y3 – 3xy(x + y) (D) x3 – y3
Q. 1. Which mathematical concept is used here? 1
Ans. Option (B) is correct.
(A) Linear equations (B) Triangles
Q. 3. Which mathematical concept is involved in the
(C) Polynomials (D) Area
above situation ? 1
(A) Polynomial (B) Circle Ans. Option (C) is correct.
(C) Lines and angles (D) Triangle Q. 2. Which is the correct identity? 1
2 2 2
Ans. Option (A) is correct. (A) (a + b) = a + b – 2ab
Q. 4. Find the amount donated by Ravi. 1 (B) (a + b)2 = a2 + b2 + 2ab
(A) ` 1000 (B) ` 850 (C) (a + b)2 = a2 – b2 – 2ab
(C) ` 970 (D) ` 900 (D) All are correct.
Ans. Option (C) is correct. Ans. Option (B) is correct.
Explanation: Q. 3. (x – y)3 =  1
1
x+ = 10 (A) (x2 – y2 – 3xy(x – y)) (B) (x3 – y3 – 3xy(x – y))
x (C) (x3 – y3 – 2xy(x – y)) (D) (x3 – y3 – 3xyx – y)
By cubing both sides we get Ans. Option (B) is correct.
3
æ 1ö Q. 4. If in the group, there are 10 teachers and 58 girls,
çx + x ÷ = (10)3
è ø then what is the number of boys? 1

1 1 1 (A) 300 (B) 360
x3 + + 3x  x +  = 1000
x3 x  x  (C) 350 (D) 370

3 1 Ans. Option (D) is correct.
(x + 3 ) + 3(10) = 1000
x Explanation:

1 No. of teachers = x + y = 10
(x3 + 3 ) = 1000 – 30 = 970 1
x By squaring both sides

1 Þ (x + y)2 = (10)2
Q. 5. Find the amount donated by Ravi if x + = 7. 1 Þ x + y2 + 2xy = 100
2
x
[Since (a + b)2 = a2 + b2 + 2ab]
(A) ` 522 (B) ` 422
No. of students = (x2 + y2) = 58
(C) ` 222 (D) ` 322
Þ 58 + 2xy = 100
Ans. Option (D) is correct.
Þ 2xy = 100 – 58
Explanation:
1 Þ 2xy = 42
x+ =7 42

x Þ xy =
2
By cubing both sides we get
3 Þ xy = 21½
æ 1ö
ç x + x ÷ = (7)
3 Now, since (x + y)3 = [x3 + y3 + 3xy(x + y)]
è ø Þ (10)3 = [x3 + y3 + 3 × 21(10)]
1 1 1 Þ 1000 = (x3 + y3 + 630)
x3 + 3 + 3x (x + )= 343
x x x Þ 1000 – 630 = (x3 + y3)
1 Þ (x3 + y3) = 370½
(x3 + 3 ) + 3(7) = 343
x Q. 5. Using part (iv), find (x2 – y2) if x – y = 23. 1
1 (A) 200 (B) 330
(x3 + 3 ) = 343 – 21 = 322. 1 (C) 120 (D) 230
x
POLYNOMIALS 43
Ans.Option (D) is correct. (4x – 3)(3x + 5) 1
Explanation: Share of Ashish and Amit are either (3x + 5) and
Given x – y = 23 (4x – 3) or (4x – 3) and (3x + 5) respectively.
Also, x + y = 10 Q. 3. Find value of x if their shares are equal 1
x2 – y2 = (x + y)(x – y)½ Ans. According to question if their share are equal.
= 10 × 23 4x – 3 = 3x + 5
= 230½
4x – 3x = 5 + 3
x = 8 1
B Subjective Based Questions
II. Two students in class of IX named Ria and Ravya
Read the following Passage and answer the were assigned a polynomial by their maths teacher.
questions. The polynomial was p(x) = x2 – 5x + 6.
I. Two brothers Ashish and Amit wanted to start They were asked to express this polynomial as
a business together. They decided to share their product of factors. Both applied factorisation by
amount depending upon the variable expenditure. splitting the middle term and got different answer.
The amount of two partners is given by the Riya’s answer : (x – 3)(x – 2)
expression 12x2 + 11x – 15, which is the product of Ravya’s answer : (x + 3)(x – 2)
their individual share factors. On basis of above information answer the following
On the basis of above information answer the questions:
following questions. [CBSE SAS] Q. 1. Find out whose answer is correct and show
Q. 1. Find the total Expenditure of Ashish and Amit factorisation 2
when x = Rs 100 1 Ans. p(x) = x2 – 5x + 6
Ans. Total Expenditure = 12x2 + 11x – 15 = x2 – 3x – 2x + 6 1
Put x = 100 = x(x – 3) – 2(x – 3)
= 12 × (100)2 + 11 × 100 – 15 = (x – 3)(x – 2) 1
= 120000 + 1100 – 15 Q. 2. Find the value of P(–1) 1
= Rs. 121,085 1 Ans. p(x) = x2 – 5x + 6
Q. 2. Find individual share factor of Ashish and Amit in p(–1) = (–1)2 – 5 × – 1 + 6 1
terms of x.2 =1+5+6
Ans. Total Amount = 12x2 + 11x – 15 = 12 1
By splitting the middle term, we get So, Riya answered correctly.
12x2 + 11x – 15 Q. 3. Which identity is used in the above situation. 1
12x2 + 20x – 9x – 15
1 Ans. Identity used in above situation is

4x(3x + 5) – 3(3x + 5) (x + a)(x + b) = x2 + (a + b)x + ab 1

  
Study Time:
Maximum time: 3:15 Hrs
Maximum questions: 23
CHAPTER
LINEAR

3 EQUATIONS IN
TWO
VARIABLES
Recall of linear equations in one variable. Introduction to the equation in two variables.
Syllabus Focus on linear equations of the type ax + by + c = 0. Explain that a linear equation
in two variables has infinitely many solutions and justify their being written as ordered
pairs of real numbers. Plotting them and showing that they lie on a line.

Revision Notes

 Linear equations in one variable are x + 1 = 0, x + 2 = 0 etc. we know that such equations have Scan to know
more about
unique solution and solution of these type of equations can be represented on number line. this topic
 Linear equation in two variables :
An equation of the form ax + by + c = 0, where a, b and c are real numbers, such that a and b are
both non zero, is called a linear equation in two variables.
Linear equations
e.g., x + y = 16, p + 4q = 7, 3 = 7x − y and 2l + m = 3 in Two Variables

All are linear equations in two variables.


 Solution of linear equation in two variables :
Any pair of values of x and y which satisfies the equation ax + by + c = 0, is called its solution. This solution can
be written as an ordered pair (x, y), first writing value of x and then value of y.
 Linear equation in two variables has infinitely many solutions.
For finding the solution of linear equation in two variables (i.e., ax + by + c = 0), we use following steps :
Step 1: Write the given equation in two variables, if not present.
Step 2: Put an arbitrary value (for convenience put x = 0 or y = 0) of x (or y) in the given equation and then it
reduces into linear equation of one variable, which gives a unique solution. Thus, we get one pair of solution of
given equation.
Step 3: Repeat step 2 for another arbitrary value of x (or y) and get another pair of solution of given equation.

Example 1

Find four different solutions of the equation 2x + y given equation and find corresponding value of y
= 7. (or x).
Solution: or, On putting x = 0 in eq. (i), we get
Step I : Write the given linear equation. 2(0) + y = 7
Given, linear equation in two variables is or, y=7
So, (0, 7) is a solution of the given equation
2x + y = 7 ...(i)
or, On putting y = 0 in eq. (i), we get
Step II : Put an arbitrary value of x (or y) in the
2x + 0 = 7
LINEAR EQUATIONS IN TWO VARIABLES 45
46 Oswaal CBSE Question Bank Chapterwise & Topicwise, MATHEMATICS, Class-IX

7 or, On putting y = 1 in eq. (i), we get


or, x= 2x + 1 = 7
2
or, 2x = 6
7 or, x=3
So, ( , 0) is also a solution of the given equation.
2 So, (3, 1) is also a solution of the given equation.
Step III : Repeat step 2 for other solutions. Step IV : Write all the solutions.
or, On putting x = 1 in eq. (i), we get 7
(0, 7), ( , 0), (1, 5) and (3, 1) are four solutions of
2(1) + y = 7 2
or, y=5 the given equation.
So, (1, 5) is also a solution of the given equation.

Mnemonics
1. Solving equations
“Don’t Call Me After Midnight”
(i) Distribute (multiply term outside parentheses by what’s inside)
x + 4(2 + 3x) = 21
(ii) Combine like terms
x + 8 + 12x = 21
(iii) Move variable
x + 12x
(iv) Add or subtract
21 – 8
(v) Multiply or divide
13x = 13
x = 13 = 1
13

SUBJECTIVE TYPE QUESTIONS


x
Very Short Answer Type Q. 3. Express − 3 y = 7 in the form of ax + by + c = 0.
4
Questions (1 mark each) R [Board Term II, 2017]
Q. 1. Find the value of k, if x = 2, y = 1 is a solution of x
the equation 2x + 3y = k. Sol. − 3y = 7
4
OR
Find two different solutions of the equation x − 12 y
=7 ½
3x + y = 19. A [KVS 2019] 4
Sol.  x = 2 and y = 1 is a solution of the equation x – 12y = 28
2x + 3y = k
\ 2×2+3×1 =k x – 12y – 28 = 0 ½
\ k = 7 1 [CBSE Marking Scheme, 2017]
OR Q. 4. Total number of legs in a herd of goats and hens is
3x + y = 19 40. Represent this in the form of linear equation of
When x = 1 y = 19 – 3x
two variables. A [Board Term II, 2014]
= 19 – 3(1) = 16
When x = 5 y = 19 – 3(5) Sol. Let the number of goats and hens in herd are x & y
= 19 – 15 = 4 respectively then,
Hence, solutions are (1, 16) and (5, 4). 1 4x + 2y = 40
Q. 2. In – 2y + 3x = 14, express y in terms of x.
Þ 2x + y = 20 1
R [Board Term II, 2017]
Q. 5. An equation of the form ax + by + c = 0 will be a
Sol. – 2y + 3x = 14 ½
3x – 14 = 2y linear equation in two variables, when a ¹ 0, b ¹ 0.
3x − 14 Is it True or False ? R
y= ½ Sol. True. 1
2
[CBSE Marking Scheme, 2017]
LINEAR EQUATIONS IN TWO VARIABLES 47

Short Answer Type Short Answer Type


Questions-I (2 marks each) Questions-II (3 marks each)
Q. 1. Find three solutions of linear equation 7x – 5y = 35 Q. 1. ABCD is a square. Co-ordinates of A and C
in two variables. U [Board Term II, 2014] are (–1, –1) and (1, 1) respectively. Write the
7 x − 35 coordinates of B and D. Also write the equations
Sol. So, when y= 1 of all the sides of square.
5
A [Board Term II, 2014]
x 5 0 10
Sol. Given, A(–1, –1) and C(1, 1)
y 0 –7 7 Then, B(1, –1) and D(–1, 1) 1
1 Also, equations of sides of square are,
Q. 2. Check which of the following is (are) the
AB : y = –1 ½
solution(s) of the equation 3y – 2x = 1.
BC : x =1 ½
(i) (4, 3) (ii) (2 2, 3 2 ) CD : y =1 ½
U [Board Term II, 2011] DA : x = –1 ½
Sol. (i) Put x = 4 and y = 3,
then 3y – 2x = 3(3) – 2(4) = 1
So, (4, 3) is the solution of the equation. 1
(ii) Again put x= 2 2
and y = 3 2 , then,
3y – 2x = 3( 3 2 ) − 2( 2 2 )

= 5 2 ≠ 1
So, ( 2 2 , 3 2 ) is not a solution of the given
equation. 1
Q. 3. Express y in terms of x from the equation 3x + 2y = 8
and check whether the point (4, – 2) lies on the
line. U [NCERT] Q. 2. For what value of k, the linear equation 2x + ky =
Sol. 2y = 8 – 3x 8 has x = 2 and y = 1 as its solution ?
8 − 3x If x = 4, then find the value of y.
or, y= 1
2
U [Board Term II, KVS 2014]
For x =4 ½ Sol. The linear equation is 2x + ky = 8
8−3×4 At x = 2, y = 1,
y=
2 2(2) + k(1) = 8 1
8 − 12 −4 or, 4+k =8
= = =–2 \ k =4 1
2 2
If x = 4, then
\ (4, – 2) lies on the line. ½ or, 2(4) + 4y = 8
Q. 4. Find the point at which the equation 3x – 2y = 6 or, 8 + 4y = 8
meets the x-axis. U [NCERT] or, 4y = 0 \
Sol. On x-axis, y co-ordinate is zero. \ y =0 1
So, put y = 0 in 3x – 2y = 6, we get 1 Q. 3. For what value of p; x = 2, y = 3 is a solution of
3x – 0 = 6 (p + 1) x – (2p + 3)y – 1 = 0 and write the
6 equation. U [Board Term II, 2013]
or, x= =2 ½
3 Sol. Given equation is (p + 1)x – (2p + 3)y – 1 = 0 ...(i)
\ 3x – 2y = 6 meets the x-axis at (2, 0). ½ If x = 2, y = 3 is the solution of the equation (i), then
Q. 5. If the point (2k – 3, k + 2) lies on the graph of the (p + 1)2 – (2p + 3)3 – 1 = 0
equation 2x + 3y + 15 = 0, find value of k. or, 2p + 2 – 6p – 9 – 1 = 0
U [Board Term II, 2011] or, – 4p – 8 = 0
Sol. Putting x = 2k – 3, y = k + 2 in 2x + 3y + 15 = 0, we get or, p =–2 2
2(2k – 3) + 3(k + 2) + 15 = 0 ½ Put the value of p in equation (i), then
or, 4k – 6 + 3k + 6 + 15 = 0 ½ –x + y – 1 = 0
or, x – y + 1 = 0, is the required equation. 1
−15
or, k = 1
7
48 Oswaal CBSE Question Bank Chapterwise & Topicwise, MATHEMATICS, Class-IX

or, y = 25 – x 2½
Commonly Made Error x 0 10 15
y 25 15 10
While opening the bracket the students tend to (ii) 14 + y = 25
forget all the terms inside the bracket with one y = 25 – 14 2½
outside and to change the sign too. y = 11 years
Hence, Akhil’s age = 11 years
Q. 3. Given below a linear equation that converts
Answering Tip Fahrenheit to Celsius and vice-versa.
9
F = C + 32
Students have to be careful when they multiply 5
a term with another and be particular about
(i) If the temperature is 30°C, what is the temperature
signs.
in Fahrenheit?
Q. 4. Find the equations of any two lines passing (ii) It the temperature is 95°F, what is the temperature
through the point (– 1, 2). How many such lines in Celsius ?
can be here ? R [Board Term II, 2017] (iii) If the temperature is 0°C, what is the temperature
Sol. Equation of two lines passing through the point in Fahrenheit and if the temperature is 0°F, what is
(– 1, 2) are the temperature in Celsius ?
x+y =1 1 (iv) Is there a temperature which is numerically the
2x + y = 0 1 same in both Fahrenheit and Celsius ? If yes, find
Infinite lines can pass through the point (– 1, 2).1 it. A + U [KVS, 2019]
[CBSE Marking Scheme 2017] 9
Sol. F = C + 32
5
Long Answer Type (i) C = 30°
Questions (5 marks each) 9
F= × 30 + 32
Q. 1. The auto rickshaw fare in a city is charged ` 10 5
for first kilometre and @ ` 4 per kilometre for F = 54 + 32 = 86 1
subsequent distance covered. Write the linear 30°C = 86°F
equation to express the above statement and find (ii) F = 95°
3 solutions to the equations. 9
95° = × C + 32
A [Board Term II, KVS 2016] 5
Sol. Total distance covered = x km. 1 9
95 – 32 = ×C
Total fare = ` y 5
Fare for the first kilometre = ` 10 63 × 5
=C
Subsequent distance = (x – 1) km 9
\ Fare for the subsequent distance = ` 4(x – 1) 1½ C = 35 1
According to question, 95°F = 35°C
y = 10 + 4(x – 1) (iii) if, C =0
or, y = 10 + 4x – 4 9
then, F = C + 32
or, y = 4x + 6 5
\ Required linear equation 9
y = 4x + 6 where x ³ 1 F= × 0 + 32
5
x 1 2 3 F = 32
y 10 14 18 2½ 0°C = 32°F
Q. 2. A student Amit of class IX is unable to write in his If, F =0
examination, due to fracture in his arm. Akhil a 9
then, F = C + 32 1
student of class VI writes for him. The sum of their 5
ages is 25 years. A [Board Term II, 2013]
9
(i) Write a linear equation for the above situation and or, 0= C + 32
find three solutions to the equations. 5
(ii) Find the age of Akhil, when age of Amit is 14 years. 9
– 32 = C
Sol. Let Age of Amit = x years 5
Age of Akhil = y years
(i) According to the question the linear equation for 5 × −32
C=
the above situation is 9
x + y = 25
LINEAR EQUATIONS IN TWO VARIABLES 49
−160 9x
= x− = 32
9 5
−160 5x − 9 x
\
0°F = °C 1 = 32
9 5
(iv) Let °C = °F = x −4 x
= 32
9C 5
F= + 32
5
32 × 5
9x Þ
x= − = – 40°
x= + 32 4
5
\
– 40°C = – 40°F 1

OBJECTIVE TYPE QUESTIONS (1 mark each)

Explanation: 2 x + 0 y + 9 = 0
A Multiple Choice Questions 2x + 9 = 0
2 x = −9
Q. 1. The linear equation 2x – 5y = 7 has
(A) a unique solution 9
x= −
(B) two solutions 2
(C) infinitely many solutions  9 
And y can be any real number. Hence,  − , m 
(D) no solution U  2 
Ans. Option (C) is correct.
is the required form of solution of the given linear
Explanation: In given equation 2x − 5y = 7, for
equation.
every value of x, we get a corresponding value of
y and vice-versa; therefore, the linear equation has
infinitely many solutions. B Assertion & Reason
Q. 2. The equation 2x + 5y = 7 has a unique solution, if
x, y are Directions: In the following questions, a statement
of assertion (A) is followed by a statement of
(A) natural numbers
reason (R). Mark the correct choice as:
(B) positive real numbers
(C) real numbers (A) Both assertion (A) and reason (R) are true and reason
(D) rational numbers. U (R) is the correct explanation of assertion (A).
Ans. Option (A) is correct. (B) Both assertion (A) and reason (R) are true but reason
Explanation: In natural numbers, there is only one (R) is not the correct explanation of assertion (A).
pair, i.e., (1, 1) which satisfies the given equation (C) Assertion (A) is true but reason (R) is false.
but in positive real numbers, and rational numbers,
(D) Assertion (A) is false but reason (R) is true.
there are many pairs to satisfy the given linear
equation. Q. 1. Assertion (A): The point of the form (a, –a) lies on
Q. 3. If (2, 0) is a solution of the linear equation 2x + 3y the line x + y = 0.
= k, then the value of k is Reason (R): Any point which satisfies the euqation
(A) 4 (B) 6 ax + by + c = 0 is the solution of the equation.
(C) 5 (D) 2 A Ans. Option (A) is correct.
Ans. Option (A) is correct. Explanation: In case of Assertion (A):
Explanation: Putx = 2, y = 0 in the equation Let x = a and y = –a
2x + 3y = k x + y = a + (–a) Þ a – a = 0
2 (2) + 3 (0) = k \ Assertion is true.
\ In case of Reason (R):
4+0 =k
Any pair (x, y) satisfies ax + by + c = 0 then (x, y) is
Hence, k  = 4 the solution.
Q. 4. Any solution of the linear equation : 2x + 0y + 9 = 0 \ Reason is true.
in two variables is of the form Both (A) and (R) are true and (R) is the correct ex-
 9   9 planation of (A).
(A)  − , m  (B)  n, 
 2   2 2
Q. 2. Assertion (A): x + 2 y - = 0 is a linear equation.
9
 9
(C)  0,  (D) (–9, 0) A Reason (R): A linear equation in two variables is of
 2
the form ax + by + c = 0
Ans. Option (A) is correct. Ans. Option (A) is correct.
50 Oswaal CBSE Question Bank Chapterwise & Topicwise, MATHEMATICS, Class-IX

Explanation: In case of Assertion (A):


\ Assertion is true.
2
x + 2y - = 0 In case of Reason (R):
9 A linear equation in two variables is of the form ax
9 x + 18 y - 2 + by + c = 0 which a ¹ 0 and b ¹ 0.
=0
9 Hence, Both (A) and (R) are true and (R) is the cor-
rect explanation of (A).
9x + 18y – 2 = 0

COMPETENCY BASED QUESTIONS (4 marks each)

Q. 4. If both contributed equally, then how much is


A Case based MCQs contributed by each?
(A) ` 50, ` 150 (B) ` 100, ` 100
Read the following passage and answer any four (C) ` 50, ` 50 (D) ` 120, ` 120
questions of the following :
Ans. Option (C) is correct.
I. Prime Minister’s National Relief Fund (also called
PMNRF in short) is the fund raised to provide Explanation: If x = y then x + x = 200½
support for people affected by natural and man- 2x = 200
made disasters. Natural disasters that are covered 200
x = = 100 ½
under this include flood, cyclone, earthquake etc. 2
Man-made disasters that are included are major Thus, each contributed is ` 100.
accidents, acid attacks, riots, etc.
Q. 5. Which is the standard form of linear equation
x=–5?
(A) x + 5 = 0 (B) 1.x – 5 = 0
(C) x + 0.y + 5 = 0 (D) 1.x + 0.y = 5
Ans. Option (C) is correct.
Explanation: Since, x = –5
Þ x+5 =0
Thus, standard form of x = – 5 is
Two friends Sita and Gita, together contributed 1.x + 0.y + 5 = 0. 1
` 200 towards Prime Minister’s Relief Fund. II. Rainwater harvesting system is a technology that
Q. 1. Which out of the following is not the linear collects and stores rainwater for human use.
equation in two variables ? Anup decided to do rainwater harvesting. He
(A) 2x = 3 (B) 4 = 5x – 4y collected rainwater in the underground tank at
the rate of 30 cm3/sec.
(C) x2 + x = 1 (D) x − 2 y = 3
Ans. Option (C) is correct.
Explanation: x2 + x = 1 is not linear as highest
power is 2. Also, it is an equation in one variable.
Thus, it is not a linear equation in two variables.1
Q. 2. How to represent the above situation in linear
equations in two variables ?
(A) 2x + y = 200 (B) x + y = 200
(C) 200x = y (D) 200 + x = y
Ans. Option (B) is correct.
Q. 1. What will be the equation formed if volume of
Explanation: Here, x represents Sita’s contribution water collected in x seconds is taken as y cm3 ?1
and y represents Gita’s contribution. 1 (A) 30x = y (B) x = 30y
Q. 3. If Sita contributed ` 76, then how much was (C) 30 – x = y (D) 30 + y = x
contributed by Gita ? Ans. Option (A) is correct.
(A) ` 120 (B) ` 123 Q. 2. What is the type of solution of the equation
(C) ` 124 (D) ` 125 formed? 1
Ans. Option (C) is correct. (A) a unique solution
(B) only two solutions
Explanation: If x = 76 then 76 + y = 200 ½
(C) no solution
y = 200 – 76
(D) infinitely many solutions
y = 124½
LINEAR EQUATIONS IN TWO VARIABLES 51
Ans. Option (D) is correct. Q. 1. Write an equation to express the above situation. 1
Explanation: Because for every value of x, there Ans. y is directly proportional to x.
is a corresponding value of y and vice versa. 1 Þ y = cx where c is a constant. 1
Q. 3. Write the equation in standard form. 1 Q. 2. Find acceleration produced in the body, if force
(A) 30x – y + 0 = 0 (B) 30x + y + 0 = 0 applied is 10 newton.
(C) 30x – y – 0 = 0 (D) 30x – y = 0 Ans. Given,
Ans. Option (A) is correct. Force applied = 10 newtons
Explanation: Standard form of a linear equation c = 10
in two variables is ax + by + c = 0. 1 Since, y = cx
Q. 4. How much water will be collected in 60 sec ? 1 Þ 10 = 10x
Þ x=1
(A) 1500 cm3 (B) 2 cm3
3 Thus, acceleration produced = 1 m/sec2 1
(C) 1800 cm (D) 1 cm3
Q. 3. Find force applied, if acceleration produced is
Ans. Option (C) is correct.
2.5m/sec2.
Explanation: Since, y = 30x Ans. Given, acceleration produced = 2.5 m/sec2
If x = 60, then, y = 30 × 60
y = cx,
= 1800
y = 10 × 2.5
Required volume is 1800 cm3 1
= 25 newton 1
Q. 5. How much time will it take to collect water in 900
cm3 ? 1 II. Temperature is a measure of the warmth or
coldness of an object or substance with reference
(A) 20 sec (B) 50 sec
to some standard value.
(C) 40 sec (D) 30 sec
In countries like USA and Canada, temperature is
Ans. Option (D) is correct. measured in Fahrenheit, whereas in countries like
Explanation: Since, y = 30x India, it is measured in Celsius.
If y = 900, then, 900 = 30x An equation that converts Fahrenheit to Celsius is:
x = 900 = 30 æ 9ö
F = ç ÷ C + 32
30 è 5ø

Required time is 30 sec. 1
On reading the above information, some queries
come in Aman’s mind. Help him to resolve his
B Cased Based Subjective following queries:
Questions Answer the following :
Read the following passage and answer the Q. 1. How many variables are there in the equation,
following questions: æ 9ö
F = ç ÷ C + 32 ? 1
I. The force applied on a body is directly proportional è 5ø

to the acceleration produced in the body. Assume
x be acceleration produced in the body and y be Ans. There are two variables, C and F. 1
the force. Take constant as 10. (S.I unit of force is Q. 2. If the temperature is 30°C, what is the temperature
Newton and S.I. unit of acceleration is m/sec2. in Fahrenheit ?  1
9
Ans. F =   30 + 32
5
= 54 + 32
= 86°F 1
Q. 3. What is a temperature which is numerically the
same in both Fahrenheit and Celsius ? 2
9
Ans. F =   x + 32
5
9
x =   x + 32
5
5x = 9x + 160
5x – 9x = 160
– 4x = 160
x = – 40 2
52 Oswaal CBSE Question Bank Chapterwise & Topicwise, MATHEMATICS, Class-IX

Artificial Intelligence
AI
PARAMETERS DESCRIPTION CONCEPTS
INTEGRATED
Chapter Covered Chapter 4: Linear Equations in Two Variables
Name of the book Mathematics, Class 9, NCERT
Subject and Artificial Understanding the concept of Linear equations in two variables
Intelligence Integrated Introductory video:

Geogebra
AI Applications
How google map and Uber app are interconnected.
Learning Objectives To understand concept of Linear equation of 2 variables Autodraw

To understand concept of
Framing of equations in two variables
To find the solutions
Represent graphically linear equation in two variable on graph paper.
Students can practice of graph in Geogebra of different equations.
Time Required 5 periods of 40 minutes each
Classroom Arrangement Flexible
Material Required Pen, paper, Black Board chalk, Graph paper Laptops and Internet connection
Pre-Preparation Activities Students will asked to arrange graph paper one day prior·
Previous Knowledge Students are given an idea of linear equations in one variable and its
graph representation in class VIII
Methodology Activity 1. Teacher will assign the task in groups to the students. Each
group will create word problems by their own based on Linear equation in
two variables and then they will plot the graph on graph paper.
Activity II: Practice Activity Google map, Uber App
Ask students to apply their understanding of Linear equations in two
variables in solving ex 4.1 to ex 4.3.
Activity III
The points of intersection of two graphs represent common solutions to
both equations.

Activity IV:
At the End of the chapter students were asked to gather a information on
Uber or OLA App. How AI works in these apps. How these app calculating
Distance And total Fare.
Learning Outcomes Students will understand concept of Linear equation of 2 variables
Students will able to frame of equations in two variables.
They will able to find the solutions.
Students will able to Represent linear equation in two variable on graph
paper.
Students will able to draw graph of different linear equations in two
variables in Geogebra
Follow up Activities Students will give the presentation in groups and teacher will give their
inputs. In this way teacher will evaluate students’ task in groups.
Reflections Ask the students to explore more AI based applications app. Where linear
equations has used.
Explore different app like Arogya app (predicting infectious people)
Swiggy app. Zomato App.

  
SELF ASSESSMENT PAPER - 02

Time: 1 hour MM: 30

UNIT-II
I. Multiple Choice Questions  [1 × 6 = 6]
1. The linear equation 3x – 4y = 9 has
(A) a unique solution (B) two solutions
(C) infinitely many solution s (D) no solution
2. The equation x = 5, in two variables can be written as
(A) 1.x + 1.y = 5 (B) 1.x + 0.y = 5 (C) 0.x + 1.y = 5 (D) 0.x + 0.y = 5
3. x = 5, y = 2 is a solution of the linear equation.
(A) x + 2y = 7 (B) 5x + 2y = 7 (C) x + y = 7 (D) 5x + y = 7
4. The equation of x-axis is of the form
(A) x = 0 (B) y = 0 (C) x + y = 0 (D) x = y
II. Assertion and Reason Based MCQs

Directions: In the following questions, A statement of Assertion (A) is followed by a statement of Reason (R).
Mark the correct choice as.
(A) Both A and R are true and R is the correct explanation of A.
(B) Both A and R are true but R is NOT the correct explanation of A.
(C) A is true but R is false.
(D) A is false and R is True.
1. Assertion (A): (4, 3) is a solution of 3y – 2x = 1.
Reason (R): Any pair of values of x and y which satisfies the equation ax + by + c = 0 is the solution of the
equation.
2. Assertion (A): A linear equation in two variables has infinitely many solutions.
Reason (R): An equation x + 2y = 6 has solutions (2, 2) and (6, 0).
III. Very Short Answer type Questions  [1 × 5 = 5]
1
1. Is + 5x + 7 a polynomial? Justify your answer.
5x −2
2. Write the coefficient of x2 in (x – 1) (3x – 4).
3. Write the degree of the given polynomial 4 – y2.
4. The graph of the linear equation x + 2y = 7 passes through the point (0, 7). Is this statement True or False? Justify
your answer.
5x
5. Solve for x if =x–2
6
IV. Short Answer Type questions–I  [2 × 5 = 10)
1. If f(x) = 4x + 7, evaluate f(5) – f(2).
2. Find the remainder when x3 + x2 + x + 5 is divided by x + 1, using remainder theorem.
3. Find the value of k, if x – 4 is a factor of p(x) = x2 – kx + 2k.
1 1
4. Find the value of x 2 + 2 , if x − = 5 .
x x
5. If x = 5 and y = 2 is the solution of the linear equation x + 2y + k = 0, find the value of k.
V. Short Answer Type questions-II  [3 × 2 = 6]
2
1. Show that the points A(3, 0), B(0, – 2) and C(6, 2) lie on the graph of the linear equation x − y = 2 .
3
2. If x + y + z = 12 and x2 + y2 + z2 = 50, then find xy + yz + zx and x3 + y3 + z3 –3xyz.
54 Oswaal CBSE Question Bank Chapterwise & Topicwise, MATHEMATICS, Class-IX

VI. Long Answer Type questions  [5 × 1 = 5]


1. Ravi took a cab to go to his office. The cab fare is as follows:
For the first kilometre, the fare is Rs 50 and for the subsequent distance it is Rs 10 per kilometre.
Take the distance covered as x km and total fare as Rs y.
(i) Form a linear equation in two variables 3
(ii) If Ravi covered 7 km, then how much fare he has to pay. 2
VII. Case Based Questions  [1 × 4 = 4]
From the following graph, ABCD is a rectangle.

Give the answer of the following questions :



1. Equation of the side AB is
(A) x = – 1 (B) y = – 4 (C) y = 0 (D) x = 1
2. The area of rectangle ABCD is
(A) 16 sq. units (B) 15 sq. units (C) 20 sq. units (D) 12 sq. units
3. The area of DABC is
(A) 6 sq. units (B) 3 sq. units (C) 7 sq. units (D) 8 sq. units
4. Equation of the side CD is
(A) y = 0 (B) y = – 4 (C) x +1 (D) x +4 = 0

qq
CO-ORDINATE Study Time:
UNIT-III GEOMETRY
Maximum time: 2:30 Hrs
Maximum questions: 31

CHAPTER

4
CO-ORDINATE
GEOMETRY
The Cartesian plane, co-ordinates of a point, names and terms associated with the co-
Syllabus ordinate plane, notations..

List of Topics
Topic-1: Cartesian System
 Page No. 55
Topic-2 : Co-ordinate Plane
Topic-1 Cartesian System  Page No. 58

Revision Notes
 Cartesian System : The system by which we can describe the position of a point in a plane is called Cartesian
System.
 The horizontal line XOX’ is called the X–axis and vertical line YOY’ is called Y–axis.

Scan to know
more about
this topic

Introduction
to Coordinate
Geometry

 The point where XOX’ and YOY’ intersect is called the origin, and is denoted by O.
 Location of a point P in cartesian system, written in the form of ordered pair say P(a, b) in above figure.
 a is the length of perpendicular of P(a, b) from Y-axis and is called abscissa of P.
 b is the length of perpendicular of P(a, b) from X-axis and is called ordinate of P.
 Positive numbers lie on the directions OX and OY, are called the positive directions of the X-axis and the Y-axis,
respectively. Similarly, OX’ and OY’ are called the negative directions of the X-axis and the Y-axis respectively.
 How to write the co-ordinates of a point :
 x – co-ordinate (or abscissa) = perpendicular distance of a point from Y-axis.
 y – co-ordinate (or ordinate) = perpendicular distance of a point from X-axis.
 If abscissa of a point is x and ordinate is y, then the co-ordinates of the point are (x, y).
 The abscissa of every point on Y-axis is zero.
56 Oswaal CBSE Question Bank Chapterwise & Topicwise, MATHEMATICS, Class-IX
CO-ORDINATE GEOMETRY 57
 The ordinate of every point on X-axis is zero.
 Co-ordinate of a point on X-axis are of the form (x, 0).
 Co-ordinate of a point on Y-axis are of the form of (0, y).
 X-axis and Y-axis intersect at origin, represented by O and its co-ordinates are (0, 0).
Example. On which axes do the given points lie.
A(0, 2), B(– 3, 0), C(0, – 3), D(0, 4), E(6, 0), F(3, 0).
Solution : On X-axis : B(–3, 0), E(6, 0), F(3, 0)
On Y-axis : A(0, 2), C(0, – 3), D(0, 4)

Example 1
Write the quadrant in which each of the following Hence, (i) the point (– 2, – 3) lies in III quadrant
points lie : because its both co-ordinates are negative.

(i) (– 2, – 3), (ii) (3, – 4), (iii) (– 1, 2) (ii) The point (3, – 4) lies in IV quadrant because
its x-co-ordinate is positive and y-co-ordinate
Sol. If both (abscissa and ordinate) co-ordinates are
is negative.
positive, then point lie in I quadrant. If abscissa is
negative and ordinate is positive, then point lies in (iii) The point (– 1, 2) lies in II quadrant because its
II quadrant. If both co-ordinates are negative then x-co-ordinate is negative and y-co-ordinate is
point lies in III quadrant. If abscissa is positive and positive.
ordinate is negative, then point lies in IV quadrant.

SUBJECTIVE TYPE QUESTIONS


Very Short Answer Type Short Answer Type
Questions (1 mark each) Questions-I (2 marks each)
Q. 1. Find one solution of y – 5 = 0 in a cartesian plane. Q. 1. Write ordinates of following points : R
 R [Board Term II, 2017] (3, 4), (4, 0), (0, 4), (5, –3) R [Board Term I, 2015]
Sol. 4, 0, 4, –3. 2
Sol. y–5=0 ½
[CBSE Marking Scheme, 2015]
y=5 ½
Q. 2. Find distances of points C(– 3, – 2) and D(5, 2) from
\ (0, 5), (1, 5), (2, 5) any one x-axis and y-axis. R [Board Term I 2016]
[CBSE Marking Scheme, 2017] Sol. C(– 3, – 2), distance from X-axis = 2 units  ½
Q. 2. In which quadrants the points P(2, – 3) and distance from Y-axis = 3 units ½
Q(–3, 2) lie ? D(5, 2), distance from X-axis = 2 units ½
R [Board Term-I, 2015]
distance from Y-axis = 5 units ½
Sol. P(2, – 3) and Q(–3, 2) lie in IV and II quadrants
respectively. 1 Short Answer Type
Q. 3. If (a, b) = (0, – 2). Find the value of a and b. Questions-II (3 marks each)
R [Board Term I, 2016] Q. 1. In which quadrant or on which axis does each of
Sol. (a, b) = (0, – 2) the following points lie  R
or, a = 0, b = – 2 1 (– 5, 3), (4, – 3), (5, 0), (6, 6), (– 5, – 4) ?
Q. 4. The points P(a, b) lies in the IV quadrant. Find Sol. In point (– 5, 3), x < 0 and y > 0
which of a or b is greater ? R [Board Term I, 2016] \ Point (– 5, 3) lies in II quadrant. ½
In point ( 4, – 3), x > 0 and y < 0
Sol. Since, P(a, b) lies in IV quadrant
\ a > 0, and b < 0 \ Point (4, – 3) lies in IV quadrant. ½
\ a >b 1 In point (5, 0), x > 0 and y = 0.
[CBSE Marking Scheme, 2016] \ Point (5, 0) lies on X-axis ½
In point (6, 6), x > 0 and y > 0
Q. 5. Find the reflection of the points (–3, –2) in y-axis.
\ Point (6, 6) lies in I quadrant. ½
A [Board Term-I, 2015] In point (– 5, – 4), x < 0 and y < 0.
Sol. (3, –2) 1 \ Point (– 5, – 4) lies in III quadrant 1
[CBSE Marking Scheme, 2015]
58 Oswaal CBSE Question Bank Chapterwise & Topicwise, MATHEMATICS, Class-IX

Q. 2. Write the co-ordinates of each point P, Q, R, S, T


and O from the figure given below.
U [NCERT Exemplar]

2
Q. 2. See figure and write the following :
(i) The co-ordinate of B.
Sol. Co-ordinates of P = (1, 1) ½
(ii) The point identified by the coordinates (–3, –2).
Co-ordinates of Q = (– 3, 0) ½
Co-ordinates of R = (– 2, – 3) ½ (iii) The abscissa of the point D.
Co-ordinates of S = (2, 1) ½ (iv) The ordinate of the point C.
Co-ordinates of T = (4, – 2) ½
Co-ordinates of O = (0, 0) ½

Long Answer Type


Questions (5 marks each)
Q. 1. Write the quadrant in which each of the following
points lie :
(i) (– 3, – 5) (ii) (2, – 5) (iii) (– 3, 5)
Also, verify by locating them on the cartesian plane.
 A [NCERT Exemplar]
Sol. (i) (– 3, – 5) lies in III quadrant, as x < 0 and y < 0.
(ii) (2, – 5) lies in IV quadrant, as x > 0 and y < 0.
(iii) (– 3, 5) lies in II quadrant, as x < 0 and y > 0. A [Board Term I, 2014]
½+½+½ Sol. (i) The co-ordinates of B = (–2, 3) 1
(ii) E is the point which is identified by the co-ordinates
Verification : The points (– 3, – 5), (2, – 5) and
(–3, –2) 1
(– 3, 5) are plotted as shown in figure : (iii) The co-ordinates of the point D is (6, 2) 1½
Result is verified : (i) (– 3, – 5) lies in III quadrant. \ Abscissa is 6.
(ii) (2, – 5) lies in IV quadrant. (iv) The co-ordinates of the point C is (3, –1) 1½
\ Ordinate is –1.
(iii) (– 3, 5) lies in II quadrant. 1½

Topic-2 Co-ordinate Plane

Revision Notes
 When two mutually perpendicular lines (one horizontal and other vertical) intersect each other they locate the
position of a point or an object in a plane.
 The plane is called the co-ordinate plane and the lines are called the co-ordinate axes.
 The coordinate axes divide the plane into four parts called quadrants (one-fourth part) numbered I, II, III and IV
anti-clockwise from OX.
CO-ORDINATE GEOMETRY 59

Example 2
If the perpendicular distance of a point p from the the perpendicular lies on the negative direction of
x-axis is 5 units and the foot of the perpendicular x-axis, so perpendicular distance can be measure in
lies in the negative direction of x-axis, then the y- II and III quadrant.
coordinate of p are? So, the point p has y-coordinate = 5 or –5.
Solution:
The perpendicular distance of a point from the
x-axis gives y-coordinate of that point. The foot of

SUBJECTIVE TYPE QUESTIONS


Q. 2. In the co-ordinate plane, draw a square of side 3
Very Short Answer Type units, taking origin as one vertex. Also, write the
Questions (1 mark each) co-ordinates of its vertices.
R [Board Term I, 2015]
Q. 1. From the figure given below find the co-ordinates Sol.
of point Q. R

Vertices are (0, 0), (3, 0), (3, 3), & (0, 3). 2
Q. 3. In which quadrant or on which axis do the points
Sol. The co-ordinate of point Q = (– 3, – 3.5) 1
(–2, – 4), (2, 4), (0, –2) and (4, – 6) lie ?
Short Answer Type A [Board Term I, 2014]
Questions-I (2 marks each) Sol. The given points are : (–2, – 4) = A
Q. 1. Which of the following point lie (2, 4) = B
(i) on X-axis ? (0, –2) = C
(ii) on Y-axis ? (4, – 6) = D
A(0, 2), B(5, 6), C(23, 0), D(0, 23), E(0, 4), F(6, 0), G(3, 0) The point A will lie in III quadrant.
R [Board Term-I, 2016]
The point B will lie in I quadrant.
Sol. (i) on X-axis = C, F, G 1
The point C will lie on Y-axis (i.e., x = 0)
(ii) on Y-axis = A, D, E 1
The point D will lie in IV quadrant. 1
[CBSE Marking Scheme, 2016]
60 Oswaal CBSE Question Bank Chapterwise & Topicwise, MATHEMATICS, Class-IX

Short Answer Type Since, DABC is an equilateral triangle with side 2a


Questions-II (3 marks each) units, therefore

Q. 1. Solve the equation 2x + 1 = x – 3 and represent the AB = BC = CA = 2a units


solution(s) on
(i) The number line
O is the mid-point of AC, then 1
(ii) The Cartesian plane. U [KVS 2019] 1
Sol. (i) 2x + 1 = x – 3 OA = AC
Þ 2x – x = – 3 – 1 2
Þ x =–4 1
(i) number line 1
= (2a) = a units 1
2
–8 –7 –6 –5 –4 –3 –2 –1 0 1 2 3 1
(ii) Cartesian plane Now, in right DAOB
Y
5
OB = AB2 − OA 2
4
3
= ( 2 a )2 − a 2
2
1

X' –7 –6 –5 –4 –3 –2 –1 0 1 2 3 4 5 X = 4 a2 − a2
–1
–2
3a 2 = a 3 units 1
–3 =
x =–4 –4
–5 Thus, co-ordinates of B are (0, a 3 ) 1
Y'  1
Q. 2. If the coordinates of a point A are (–2, 9) which can
also be expressed as (1 + x, y2) and y > 0, then find
Long Answer Type
in which quadrant do the following points lie :
Questions (5 marks each)
P(y, x), Q(2, x), R(x2, y – 1), S(2x, – 3y)
Q. 1. In the given figure, DABC and DADC are Sol. Here, A(–2, 9) can also be expressed as (1 + x, y2)
equilateral triangles on common base AC, each Þ (–2, 9) = (1 + x, y2) where y > 0
side of triangles being 2a units. Vertices A and C \ 1 + x = –2, y2 = 9 1
lies on X-axis, vertices B and D lies on Y-axis. O is
the mid-point of AC and BD. Find the co-ordinates x = – 3, y = 9
of the point B. A [NCERT Exemplar] y=3 ( y > 0)
Now, P(y, x) = (3, –3), it lies in IV quadrant 1
Q(2, x) = (2, –3), it lies in IV quadrant 1
R(x2, y – 1) = [(–3)2, (3 – 1)]
= (9, 2), it lies in I quadrant
5(2x, –3y) = [2 × (–3), (–3 × 3)]
= (–6, –9), it lies in III quadrant 1
Q. 3. (i) Find values of a and b, if two ordered pairs (a –
3, –6) and (4, a + b) are equal.
(ii) Find distances of point (a, b) obtained from
x-axis and y-axis.
(iii) Find in which quadrant they lie.
Sol. (i) Here, two ordered pairs are equal
Sol. \ a – 3 = 4; a + b = 6
a = 4 + 3; 7 + b = – 6
(substituting value of 'a')
a = 7; b = –6 – 7
b = –13 2
Hence a = 7 and b = –13
(ii) Distances of point (7, –13) from x-axis is, 13 units in
negative direction and from y-axis it is 7 in positive
direction. 2
1 (iii) Clearly point (7, –13) lie in IV quadrant. 1
CO-ORDINATE GEOMETRY 61

OBJECTIVE TYPE QUESTIONS (1 mark each)

Ans. Option (B) is correct.


A Multiple Choice Questions Explanation: Abscissa of P = –2
Abscissa of Q = –3
Q. 1. Point (–3, 5) lies in the (Abscissa of P) – (Abscissa of Q) = (–2) – (–3)
(A) first quadrant (B) second quadrant Þ –2 + 3 = 1
(C) third quadrant (D) fourth quadrant R Q. 7. If P (5, 1), Q (8, 0), R (0, 4), S (0, 5) and O (0, 0) are
Ans. Option (B) is correct. plotted on the graph paper, then the point(s) on the
Explanation: In point (–3, 5), x-coordinate is x-axis are
negative and y-coordinate is positive. So, the point (A) P and R
lies in second quadrant. (B) R and S
Q. 2. Signs of the abscissa and ordinate of a point in the (C) Only Q
second quadrant are respectively (D) Q and O A
(A) +, + (B) –, – Ans. Option (D) is correct.
(C) –, + (D) +, – R Explanation: A point lies on x-axis, if its y-coordinate
Ans. Option (C) is correct. is zero. Here, Q(8, 0) and O(0, 0) has y-coordinate
Explanation: In second quadrant, x-axis is negative zero.
and y-axis is positive. So, sign of abscissa of a point is
Q. 8. Abscissa of a point is positive in
negative and sign of ordinate of a point is positive.
(A) I and II quadrants (B) I and IV quadrants
Q. 3. Point (0, –7) lies
(C) I quadrant only (D) II quadrant only R
(A) on the x–axis
Ans. Option (B) is correct.
(B) in the second quadrant
Explanation: Abscissa of a point is positive in I and
(C) on the y-axis
IV quadrants.
(D) in the fourth quadrant R
Ans. Option (C) is correct.
Explanation: In the point (0, –7), x-coordinate is
B Assertion & Reason
zero, So, it lies on y-axis and y-coordinate is negative
so, the point lies on y-axis in the negative direction. Directions: In the following questions, a statement
of assertion (A) is followed by a statement of
Q. 4. The point at which two coordinate axes meet is reason (R). Mark the correct choice as:
called the ............... .
(A) Both assertion (A) and reason (R) are true and
(A) abscissa (B) ordinate
reason (R) is the correct explanation of assertion (A).
(C) origin (D) quadrant R
(B) Both assertion (A) and reason (R) are true but reason
Ans. Option (C) is correct.
Explanation: It is the point at which both the axes; (R) is not the correct explanation of assertion (A).
i.e., x-axis and y-axis meet and its co-ordinates are (C) Assertion (A) is true but reason (R) is false.
(0, 0). (D) Assertion (A) is false but reason (R) is true.
Q. 5. Points (1, –1), (2, –2), (4, –5), (–3, –4) Q. 1. Assertion (A): The point P(–3, 0) lies on x-axis.
(A) lie in II quadrant Reason (R): Every point on x-axis is of the form (x, 0).
(B) lie in III quadrant Ans. Option (A) is correct.
(C) lie in IV quadrant
Explanation: In case of assertion (A):
(D) do not lie in the same quadrant
The point P(–3, 0) lies on x-axis as ordinate is 0.
Ans. Option (D) is correct.
\ Assertion is true.
Explanation: In points (1, –1), (2, –2) and (4, –5)
In case of reason (R):
x-coordinate is positive and y-coordinate is negative.
Every point on x-axis has ordinate 0(zero).
So, they all lie in IVth quadrant. \ Reason is true.
In point (–3, –4), x-coordinate and y-coordinate Therefore, Both A and R are true and R is the correct
both are negative, so it lies in III quadrant. So, all Explanation of A.
the points do not lie in same quadrant.
Q. 2. Assertion (A): The point P(–6, –4) lies in the
Q. 6. If the coordinates of the two points are P (–2, 3) and quadrant III.
Q (–3, 5), then (abscissa of P) – (abscissa of Q) is Reason (R): The signs of points in quadrants I, II, III,
(A) –5 IV are respectively (+, +), (–, +), (–, –), (+, –).
(B) 1 Ans. Option (A) is correct.
(C) –1 Explanation: In case of assertion (A):
P(–6, –4) lies in III quadrant as the points having
(D) –2 A
negative signs lies in quadrant III.
62 Oswaal CBSE Question Bank Chapterwise & Topicwise, MATHEMATICS, Class-IX

In case of reason (R): Any point lying in I quadrant has both the
This statement is fact which is always true. co-ordinates positive.
Therefore, Both A and R are true and R is the correct \ Reason is true.
Explanation of A. Q. 4. Assertion (A): Point (0, 5) lies on y-axis.
Q. 3. Assertion (A): Point (–5, –4) lie in quadrant II. Reason (R): Every point on y-axis is of the form
Reason (R): Point lying in quadrant I has x > 0 and (0, y).
y > 0. Ans. Option (A) is correct.
Ans. Option (D) is correct. Explanation: In case of assertion (A):
Explanation: In case of assertion (A): Point (0, 5) lies on y-axis as x-coordinate is zero.
In point (–5, –4) x < 0 and y < 0 In case of reason (R):
\ (–5, –4) lies in III quadrant. Every point on y-axis has x-coordinate zero is fact.
\ Assertion is false Therefore, Both A and R are true and R is the correct
In case of reason (R): Explanation of A.

COMPETENCY BASED QUESTIONS (4 marks each)

Ans. Option (C) is correct.


A Case based MCQs Explanation: As polygon formed on joining five
points will be pentagon.1
Read the following passage and answer any four Q. 4. Name the point lying in the third quadrant. 1
questions of the following :
(A) R (B) P
I. Five friends playing a game in which they are (C) Q (D) T
standing at different positions, P, S, T, R and Q. Ans. Option (A) is correct.
Explanation: In third quadrant, both x-co-ordinate
and y-co-ordinate are negative.1
Q. 5. (x, y) = (y, x), if 1
(A) x > y (B) x < y
x 1
(C) =1 (D) x =
y y
Ans. Option (C) is correct.
x

Explanation: Since, =1
y
Þ x =y ½
Also, (x, y) = (y, x), if x = y ½
II. Sohan draws a gate of a temple on the graph paper.
He has following points :
(– 1, 0), (1, 0), (1, 1), (– 1, 1) and (0, 2)
Rohan is watching them playing. Few questions
came to Rohan's mind while watching the game.
Give answer to his questions by looking at the
figure.
Q. 1. What are the co-ordinates of P? 1
(A) (– 1, 1) (B) (1, – 1)
(C) (1, 1) (D) (– 1, – 1)
Ans. Option (C) is correct.
Explanation: Co-ordinates of P(1, 1)1
Q. 2. Name the point whose y-co-ordinate is zero : 1
(A) P (B) Q
(C) R (D) S Q. 1. Name the closed figure obtained. 1
Ans. Option (B) is correct. (A) Triangle (B) Quadrilateral
Explanation: As point Q lies on x-axis. Therefore, its (C) Pentagon (D) Hexagon
y-co-ordinate is zero.1 Ans. Option (C) is correct.
Q. 3. Name the polygon formed on joining all these five Explanation: A pentagon has five sides. 1
points in an order. 1 Q. 2. In which quadrant (– 1, 1) lies ? 1
(A) Quadrilateral (B) Hexagon (A) 1st quadrant (B) 2nd quadrant
(C) Pentagon (D) Triangle (B) 3rd quadrant (D) 4th quadrant
CO-ORDINATE GEOMETRY 63
Ans. Option (B) is correct. Q. 1. Find in which Quadrant does the Banyan plant
Explanation: In 2nd quadrant, x-co-ordinate is lie?
negative and y-co-ordinate is positive. 1 Ans. Co-ordinates of Banyan plant are (–3, 4) so, it lies in
Q. 3. Write the ordinate of the point (1, 0). 1 II quadrant. 1
(A) 1 (B) 0 Q. 2. Which plant lie on x-axis?
(C) 2 (D) – 1 Ans. Coordinates of Amla are (–2, 0) and that of mango
Ans. Option (B) is correct. are (2, 0) so these both plants lie on x-axis. 1
Explanation: y-co-ordinate of a point also called
Q. 3. Find the co-ordinates of location point of Orange
ordinate.1
Q. 4. Write the abscissa of the point (0, 2). 1 tree. Also, write that in which quadrant do these
(A) 0 (B) 2 coordinates lie?
(C) – 2 (D) 1 Ans. Co-ordinates of Orange tree are (3, 4).
Ans. Option (A) is correct. These co-ordinates lie in Ist quadrant as both
Explanation: x-co-ordinate of a point also called abscissa and ordinate are positive. 2
abscissa. 1
II. Observing the figure given below, Answer the
Q. 5. Which point from the following lies on Y-axis ? 1
following questions.
(A) (1, 1) (B) (1, 0)
(C) (0, 2) (D) (– 1, 1)
Ans. Option (C) is correct.
Explanation: On Y-axis, x-co-ordinate is zero. 1

B Case Based Subjective


Questions
Read the following passage and answer the
following questions:
I. On Environment day class 9 students got five
plants of mango, silver oak, Orange, banyan and
Amla from soil department. Students planted
the plants and noted their locations as (x, y).
Observing the graph given below, Answer the
following questions. Q. 1. In a cartesian plane find the coordinates of point
[CBSE SAS] Q.
Ans. Coordinates of point Q are (2, –5)  1
Q. 2. Which point lie in I quadrant.
Ans. Coordinates of point R are (5, 4) since both are
positive, therefore point 'R' lies in Ist quadrant. 1
Q. 3. What will be the value of (abscissa of P) – (ordinate
of Q)?
Ans. Coordinates of P are (–4, 2) and that of Q are (2, –5).
\ Abscissa of P-Ordinate of Q
Þ –4 – (–5)

Þ –4 + 5 = 1 
2
64 Oswaal CBSE Question Bank Chapterwise & Topicwise, MATHEMATICS, Class-IX

Artificial Intelligence
AI
PARAMETERS DESCRIPTION CONCEPTS
INTEGRATED
Chapter Covered Chapter 3: Coordinate Geometry
Name of the book Mathematics, Class 9 NCERT
Subject and Artificial Understanding the concept of Coordinate Geometry. Using Geogebra and Google Maps
Intelligence Integrated Google Maps.
Learning Objectives The students will able to
● To explore the idea of location of an object with a frame of reference.
● Understand the elements of the Cartesian system i.e., x-axis, y-axis,
origin, quadrants.
● To find a point in any quadrants with reference to x-axis and y-axis.
● The students will understand the concept of Coordinate Geometry
using the Google AI App.
● To solve the real life problems of coordinate geometry.
Time Required 3 period of 40 minutes each
Classroom Arrangement Flexible
Material Required Internet connection, laptop / desktop / smartphone, Graph Paper, Pen,
paper.
Pre-Preparation Activity The Students will be asked to locate points on the Number Line on the
graph.
Previous Knowledge The students will be made to recall about
● Knowledge of how to locate a point on the number line.
● Basic knowledge of x-axis and y-axis.
● Using (AI) demos basic knowledge of x-axis and y-axis will be checked.
● Also knowledge of how to locate a point on the number line will be
checked.
Students will use the Geogebra to locate points on the Number Line.

Geogebra
Methodology Strategy Used:
Brainstorming
Brainstorming on the previous knowledge would be checked using
mentimeter.
Motivation: Google Maps
Motivate the students by explaining the importance of Coordinate
geometry day to day life.
Use of coordinates and coordinate geometry in Google maps to calculate
shortest distance between two destination points.
Recapitulation: Oral test of basic concepts discussed in the class.
Activity-1 Seating Plan given in the NCERT Textbook will be discussed.
Discussion of the topic through Collaborative Learning (Inductive Method)
Guided Practice: Students will be given questions from the Textbook.
Independent Practice: Students would Practice the given questions for
Homework
Closer: MCQ based test will be taken on Google forms/MS forms.
Activity II: Practice Activity students will apply their understanding of
Coordinate Geometry to attempt the questions of NCERT Textbook.
CO-ORDINATE GEOMETRY 65
Learning Outcomes ● At the end of the topic, students will be able to:
● Demonstrate their knowledge by locating an object with a frame of
reference.
● Describe the elements of the Cartesian plane
● Identify the x-axis and y-axis of any point on
● Cartesian plane
● Analyze the position of any point on the Cartesian system.
● Apply the concept of coordinate geometry in real life situations.
Follow up Activities Students will find a hidden picture by plotting and joining the various
points with given coordinates in a Cartesian plane through an activity in
Geogebra.

Geogebra
Home assignment:
Crossword puzzles based on discussed topics in the class will be given.
Reflections ● Ask the students to analyze the real-life problems in their daily life and
apply the concept of Coordinate Geometry.
● Discussion with Students on the role of AI application: Ask and take
feedback on the scope of the AI tools used and any problems they
encountered while using the AI tool.
● Any other AI application that can be used as an alternative.

  
SELF ASSESSMENT PAPER - 03

Time: 1 hour MM: 30

UNIT-III
I. Multiple Choice Questions. [1 × 6 = 6]
1. The point where x-axis and y-axis intersect is called the
(A) Origin (B) Ordinate (C) Abscissa (D) Ordered pair
2. The point (2, 7) lies in .............. quadrant
(A) II (B) I (C) III (D) IV
3. The ordinate of a point is its distance from the
(A) origin (B) x-axis (C) y-axis (D) None of these
4. The equation representing y-axis is
(A) x = 0 (B) y = 0 (C) y = a (D) x = 6
II. Assertion and Reason Based MCQs [1×6 = 6]

Directions: In the following questions, A statement of Assertion (A) is followed by a statement of Reason (R).
Mark the correct choice as.
(A) Both A and R are true and R is the correct explanation of A.
(B) Both A and R are true but R is NOT the correct explanation of A.
(C) A is true but R is false.
(D) A is false and R is True.
1. Assertion (A): The point O(0, 0) lies in quadrant I.
Reason (R): The point O(0, 0) lies on both the axis.
2. Assertion (A): Point (a, b) lies in quadrant IV so a > b.
Reason (R): Point (0, 0) lies on y-axis.
III. Very Short Answer Type Questions [1 × 5 = 5]

Observe the points plotted in the figure and give answer the following questions :

1. The co-ordinates of the point P is


(A) (1, 1) (B) (– 1 , 3) (C) (1 , 3) (D) (0, 0)
2. The coordinates of the point S is
(A) (2, 1) (B) (4, – 2) (C) (– 2 , – 3) (D) (3, 1)
SELF ASSESSMENT PAPER 67
3. The coordinates of the point T is
(A) (4, – 2) (B) (4 , 2) (C) (– 2, 4) (D) (2, 4)
4. The abscissa of P- abscissa of S is
(A) 2 (B) – 2 (C) 4 (D) – 4
5. The ordinate of P + ordinate of S is :
(A) 4 (B) – 4 (C) 5 (D) – 5
IV. Short Answer Type questions–I [2 × 2 = 4]
1. From the given figure, write the following :
(i) Find the coordinates of P.
(ii) The abscissa of the point Q.
(iii) The ordinates of the point R.
(iv) The points whose abscissa is O.

2. Locate and write the coordinates of a point :


(i) Above X-axis lying on Y-axis at a distance of 3 units from origin.
(ii) Below X-axis lying on Y-axis at a distance of 5 units from origin.
V. Short Answer Type questions-II [3 × 2 = 6]
1. In which quadrant or on which axis does each of the following points lie (– 4, 2), (5, – 4), (6, 0), (8, 8), (– 1, – 3)?
2. Find the coordinates of the point :
(i) Whose abscissa is 4 and ordinate is 2.
(ii) Whose abscissa is 5 and which lies on X-axis.
(iii) Whose ordinate is 7 and which lies on Y-axis.
VI. Long Answer Type questions [5 × 1 = 5]
1.

 [CBSE SAS]

In a cartesian plane given below.

A child is standing at a certain Point P and his mother is standing at a point O.
(i) What are the co-ordinates of mother? 1
(ii) What are the co-ordinates of child? 1
(iii) In which Quadrant a child is standing? 1
(iv) What is the distance between the child and his mother? 2
68 Oswaal CBSE Question Bank Chapterwise & Topicwise, MATHEMATICS, Class-IX

VII. Case Based (Attempt any 4 parts) [1 × 4 = 4]


Read the following text and answer the following questions on the basis of the same:
Priya draws a kite on the graph paper. She has following points: (0, 4), (3, 0), (–3, 0), (0, –4).

1. Name the closed figure obtained.


(A) Triangle (B) Circle (C) Pentagon (D) Quadrilateral
2. What is ordinate of the point (–3, 0)?
(A) 0 (B) 1 (C) 2 (D) 3
3. What is abscissa of the point (3, 0)?
(A) 0 (B) – 3 (C) – 4 (D) 3
4. What is the mirror image of (0, 3)?
(A) (0, 4) (B) (0, –3) (C) (2, 3) (D) (0, – 4)
5. Name the point where x-co-ordinate is positive and y-co-ordinate is zero.
(A) (0, 4) (B) (3, 0) (C) (–3, 0) (D) (0, 0)

qq
UNIT-IV GEOMETRY Study Time:
Maximum time: 2:30 Hrs
Maximum questions: 40
CHAPTER

5
INTRODUCTION
TO EUCLID’S
GEOMETRY
History : Geometry in India and Euclid’s geometry. Euclid’s method of formalizing
Syllabus observed phenomenon into rigorous Mathematics with definitions, common/obvious
notions, axioms/postulates and theorems. The five postulates of Euclid. Showing the
relationship between axiom and theorem, for example :
(Axiom) 1. Given two distinct points, there exists one and only one line through them.
(Theorem) 2. (Prove) Two distinct lines cannot have more than one point in common.

Topic-1 Euclid’s Geometry List of Topics


Topic-1: Euclid’s Geometry
 Page No. 69
Topic-2 : Euclid’s Postulates
Revision Notes  Page No. 73

 Axiom : Axioms are the assumptions which are obvious universal truths. They are not proved.
 Euclid’s Axioms :
 Things which are equal to the same thing are equal to one another. Scan to know
      more about
e.g., If AB = PQ and PQ = XY , then AB = XY . this topic

 If equals are added to equals, the wholes are equal.



e.g., If m∠1 = m∠2, then
Euclid Geometry

m∠1 + m∠3 = m∠2 + m∠3.
 If equals are subtracted from equals, the remainders are equal.

e.g., If m∠1 = m∠2, then

m∠1 – m∠3 = m∠2 – m∠3.
 Things which coincide with one another are equal to one another.
   
e.g., If AB coincides with XY , such that A falls on X and B falls on Y, then AB = XY
 The whole is greater than the part.

e.g., If m∠1 = m∠2 + m∠3, then m∠1 > m∠2 & m∠1 > m∠3.
 Things which are double of the same thing are equal to one another.

e.g., If a = 2c and b = 2c, then a = b.
 Things which are halves of the same thing are equal to one another.
c c
e.g., If a = and b = , then a = b
2 2
70 Oswaal CBSE Question Bank Chapterwise & Topicwise, MATHEMATICS, Class-IX
INTRODUCTION TO EUCLID’S GEOMETRY 71

Mnemonics

Postulates are true assumptions specific to Geometry (PG-Post Graduate) Axioms are true assumption, not
specifically linked to geometry (Requires no proof) (AP–Andhra Pradesh).

SUBJECTIVE TYPE QUESTIONS


AB = AD
Very Short Answer Type
AC = AD
Questions (1 mark each)
AB = AC 1
Q. 1. Explain when a system of axioms is called Things which are equal to the same thing are
consistent. R [Board Term I, 2015] equal to one another. 1
Sol. A system of axioms is called consistent, when [CBSE Marking Scheme, 2012]
it is impossible to deduce from these axioms, a
Q. 3. Solve the equation x + 4 = 10 and state Euclid’s
statement that contradicts any axiom or previously
proved statement. 1 axiom used. R [Board Term I, 2016]
[CBSE Marking Scheme, 2015] Sol. x + 4 = 10
Q. 2. Express in variables the things which are double x + 4 – 4 = 10 – 4 1
of the same thing are equal. R Þ x =6
Sol. Let, First thing = x = 2a If equals are subtracted from equals, the
Second thing = y = 2a remainders are equal. 1
then, x =y 1
[CBSE Marking Scheme, 2016]
Q. 3. What does a theorem require ? R
Sol. Theorem requires a proof. 1 Q. 4. In a triangle PQR, X and Y are the points on PQ
and QR respectively. If PQ = QR and QX = QY,
Short Answer Type show that PX = RY.  R [Board Term I, 2016]
Questions-I (2 marks each) Sol. Here PQ = QR
Q. 1. In the given figure AC = DC, CB = CE, show that QX = QY
AB = DE. R [Board Term I, 2012]


If equals are subtracted from equals, the remainders
Write Euclid’s axiom to support this.
are also equal.
Sol. AC = DC  (Given)
CB = CE We have PQ – QX = QR – QY
Adding, AC + CB = DC + CE ½ PX = RY 1
AB = DE ½
Q. 5. P and Q are the centres of two intersecting circles.
If equals are added to equals, the wholes are equal. 1
Prove that PQ = QR = PR. U
Q. 2. In the given figure, we have AB = AD and
AC = AD. Prove that AB = AC. State the Euclid’s [Board Term I, 2016; 2013; 2012]
axiom to support this. R [Board Term I, 2012]
Sol.
72 Oswaal CBSE Question Bank Chapterwise & Topicwise, MATHEMATICS, Class-IX

Sol. In a circle having centre at P, we have Sol. Given, ∠ABC = ∠ACB


PR = PQ = radius ½ or, ∠1 + ∠4 = ∠2 + ∠3 1
or, ∠1 + ∠4 – ∠4 = ∠2 + ∠3 – ∠3
In a circle having centre at Q, we have
 (As, ∠3 = ∠4) 1
QR = QP = radius ½ If equals are subtracted from equals then remainders
Euclid’s first axiom : Things which are equal to the
are also equal.
same thing are equal to one another. ½ \ ∠1 = ∠2. 1
Q. 4. Prove that every line segment has one and only

PR = PQ = QR. ½
one mid-point. U [KVS 2019]
Short Answer Type Sol. Suppose C and D are two mid-points of the line
segment AB.
Questions-II (3 marks each)
A C D B
Q. 1. In the given figure, if ∠1 = ∠3, ∠2 = ∠4 and ∠3 =
∠4, write the relation between ∠1 and ∠2, using 1
\ AC = CB = AB
Euclid’s axiom. 2
3 1
4 \
AD = DB = AB
2

1
\ AC = AD
2 [Things which are equal to the same thing are equal
to one another]
R [NCERT Exemplar]
\ This is possible only when C and D coincide each other.
Sol. Here, ∠3 = ∠4 and ∠1 = ∠3 and ∠2 = ∠4. Euclid’s Hence, every line segment has one and only one
first axiom says, the things which are equal to same midpoint. Hence Proved 3
thing are equal to one another. 2
Given, ∠3 = ∠4 and ∠1 = ∠3 Long Answer Type
\ ∠1 = ∠4 Questions (5 marks each)
Again given ∠2 = ∠4
Q. 1. In the figure we have Ð1 = Ð3 and Ð2 = Ð4. Show
and ∠1 = ∠4 (Proved above)
\ ∠1 = ∠2. 1 that ÐA = ÐC. State which axiom you used here.
Also give two more axioms other than the axioms
Q. 2. In a triangle ABC, X and Y are the points on AB
used in the above situation.
and BC such that BX = BY and AB = BC. Show that
AX = CY. State the Euclid’s Axiom used.
A [Board Term I, 2015]
Sol.

AB = BC (given) U [Board Term I, 2016] [NCERT Exemplar]


BX = BY (given) 1 Sol. Since, ∠1 = ∠3 and ∠2 = ∠4, therefore adding both
If equals are subtracted from equals, then equations
remainders are also equal. 1
∠1 + ∠2 = ∠3 + ∠4 ½
AB – BX = BC – BY
\ AX = CY 1 Þ
∠BAD = ∠BCD
Q. 3. In the given figure, we have ∠ABC = ∠ACB, Þ
∠A = ∠C 1
∠3 = ∠4. Show that ∠1 = ∠2. If equals are added to equal, the wholes are equal 1
R [NCERT Exemplar]
Two more axioms :
(i) Things which are equal to the same thing are equal
to one another.  1

e.g., if AB = PQ and PQ = XY , then AB = XY  ½

(ii) If equals are subtracted from equals, the remainders


are equal. 
e.g., if m∠1 = m∠2 then
mÐ1 – m∠3 = m∠2 – m∠3 1
INTRODUCTION TO EUCLID’S GEOMETRY 73
1 1 Two more axioms :
=
Q. 2. In the fig., if OX = XY , PX XZ and OX = PX,
2 2 (i) Things which coincide with one another are equal
to one another.  1
Show that XY = XZ. State which axiom you used
here. Also give two more axioms other than the e.g., if AB coincide with XY , such that A falls on X
axiom used in the above situation. and B falls on Y, then AB = XY .
(ii) The whole is greater than the part 1
e.g., if mÐ1 = mÐ2 + mÐ3, then mÐ1 > mÐ2 and
mÐ1 > mÐ3.
Q. 3. Using Euclid’s axiom, compare length AD and AF.
State which axiom you used here. Also give two
more axioms other than the axiom used in the
above situation.
U [Board Term I, 2016] [NCERT Exemplar]
1
Sol. Here OX = XY ½  R [Board Term I, 2016]
2  Sol. AD is part of AF
1 \ AD < AF
PX = XZ 1
2  As whole is greater than the part. 2
Two more axioms :
Also OX = PX (Given) (i) If equals are added two equals, the wholes are
1 1 equal. 1½
\ XY = XZ ½
2 2 e.g., if mÐ1 = mÐ2, then

mÐ1 + mÐ3 = mÐ2 + mÐ3
XY = XZ 1 (ii) If equals are subtracted from equals, the remainders
Thing which are equal to half of same thing are are equal. 1½
equal to one another. e.g., if mÐ1 = mÐ2, then
mÐ1 – mÐ3 = mÐ2 – mÐ3

Topic-2 Euclid’s Postulates

Revision Notes

 Postulates : The basic facts which are taken for granted, without proof and which are specific to geometry are
called postulates.
 Plane : A plane is a surface such that the line obtained by joining any two points in it will be entirely in the plane.
 Incidence Axioms on lines :
(i) A line contains infinitely many points.
(ii) Through a given point A, (infinite) lines can be drawn.

(iii) One and only one line can be drawn to pass through two given points A and B.

 Collinear points : Three or more points are said to be collinear, if there is a straight line which passes through all
of them.
74 Oswaal CBSE Question Bank Chapterwise & Topicwise, MATHEMATICS, Class-IX


Figure I Figure II
In figure I; A, B, C are collinear points, while in figure II; P, Q, R are non-collinear points.
 Intersecting lines : Two lines which cut at one point are said to be intersecting lines. The point P common to two
given line segments AB and CD is called their point of intersection.

 Concurrent lines : Three or more lines intersecting at a same point are said to be concurrent.
l

m P
n

 Parallel lines : Two lines l and m in a plane are said to be parallel, if they have no point in common then we write
l || m. l

m
The distance between two parallel lines always remains the same.
 Two distinct lines cannot have more than one point in common.
 Parallel Line Axiom : If l is a line and P is a point not on the line l, there is one and only one line (say m) which
passes through P and parallel to l.

 If two lines l and m are both parallel to the same line n, they will also be parallel to each other.
l
m
n
 If l, m, n are lines in the same plane such that l intersects m and n || m, then l also intersects n.
l

 If l and m are intersecting lines, l || p and q || m, then p and q also intersect.


l p

 If line segments AB, AC, AD and AE are parallel to a line l, then points A, B, C, D and E will be collinear.
 Betweenness : Point B is said to lie between the two points A and C, if :

(i) Points A, B and C are collinear, and


(ii) AB + BC = AC.
INTRODUCTION TO EUCLID’S GEOMETRY 75
 Mid-point of a Line-segment : For a given line segment AB, a point M is said to be the mid-point of AB, if :

(i) M is an interior point of AB, and


(ii) AM = MB.
 Euclid’s Five Postulates :
Postulate 1 : A straight line can be drawn from any one point to another point.

Postulate 2 : A terminated line can be produced indefinitely i.e., ‘A line segment can be extended on either side to

form a line’.

Postulate 3 : A circle can be drawn with any centre and any radius.

Postulate 4 : All right angles are equal to one another.


If ∠XYZ = 90° and ∠PQR = 90°, then ∠XYZ = ∠PQR.  [congruent angles]
Postulate 5 : If a straight line falling on two straight lines makes the interior angles on the same side of it taken
together less then two right angles, then the two straight lines, if produced indefinitely, meet on that side on
which the sum of angles is less than two right angles.
 Non-Euclidean geometries :
All the attempts to prove the Euclid’s fifth postulate using the first 4 postulates failed. But they led to the discovery
of several other geometries, called non-Euclidean geometries.
 Theorems : Theorems are statements which are proved using definitions, axioms, previously proved statements
and deductive reasoning.

SUBJECTIVE TYPE QUESTIONS


Q. 3. Give any one example of a geometrical line from
Very Short Answer Type your surroundings. A
Questions (1 mark each) Sol. Meeting place of two walls. 1
Q. 1. How many lines can be passed through two Q. 4. What is a surface ? R
distinct points ? R Sol. A surface is that which has length and breadth. 1
Sol. Only one line passes through two distinct points. y

B
1
Surface breadth (y)
A
length (x) x
Q. 2. What is a straight line ? R
Sol. Two planes intersect each other to form a straight
line. 1
76 Oswaal CBSE Question Bank Chapterwise & Topicwise, MATHEMATICS, Class-IX

Q. 5. How can we identify parallel lines ? R (ii) How would you rewrite Euclid’s fifth postulate so
Sol. Lines are parallel if they do not intersect on being that it would be easier to understand ?
extended. A [NCERT]
For example : Sol. (i) Since, it is true for things in any part of universe
so, this is a universal truth. 1
(ii) If the sum of the co-interior angles made by a
transversal intersect two straight lines at distinct
points is less than 180°, then the lines cannot be
parallel. 1
1 Q. 5. Consider two ‘postulates’ given below :
Lines A and B are parallel lines or they have no (i) Given any two distinct points A and B, there exists
common point. a third point C which is in between A and B.
(ii) There exist at least three points that are not on the
Short Answer Type same line.
Questions-I (2 marks each) Do these postulates contain any undefined term ?
Are these postulates consistent ?
Q. 1. How many planes can be made to pass through :
Do they follow with Euclid’s postulates ? Explain.
(i) Three collinear points. U [NCERT]
(ii) Three non-collinear points. Sol. There are two undefined terms, line and point.
R [Board Term I, 2012] They are consistent, because they deal with two
Sol. (i) Infinite, if they are collinear. different situations.
(ii) Only one, if they are non-collinear. 1+1 (i) Says that given two points A and B, there is a point C,
Q. 2. Show that of all the line segments drawn from a lying on the line which is in between them. 1
given point to a line, not on it, the perpendicular (ii) Says that given A and B, we can take C not lying on
line segment is the shortest. the line passing through A and B.
U [Board Term I, 2011] These ‘Postulates’ do not go with Euclid’s postulates.
1

Long Answer Type


Questions (5 marks each)
Q. 1. A square is a polygon made up of five line segments,
out of which, length of three line segments are
equal to the length of fourth of one and all its
angles are right angles” Define the term used in
Sol. Let AB be perpendicular to a line l and AP is any
this definition which have been highlighted/
other line segment.
underlined. R [Board Term I, 2016]
In right ∆ABP, ∠B > ∠P, (Q ∠B = 90°) 1
Sol. Polygon : A simple closed figure made up of three
or, AP > AB or AB < AP. 1 or more line segments. 1½
Q. 3. In figure, AE = DF, E is the mid-point of AB and Line Segment : Part of a line with two end points. 1
F is the mid-point of DC. Using an Euclid’s axiom, Angle : A figure formed by two rays with a common
show that AB = CD. A [Board Term I, 2011, 2010] initial point.  1½
Right angle : Angle whose measure is 90°. 1
Q. 2. Two salesmen make equal sales during the month
of June. In July, each salesmen doubles his sale
of the month of June. Compare their sales in July.
State which axiom you use here. Also give two
more axioms other than the axiom used in the
above situation. R [Board Term I, 2015]
Sol. Their sales in July will also be equal as things which
are double of the same thing are equal to one
Sol. AB = 2AE another.  1½
(E is the mid-point of AB) ½
Two other axioms :
CD = 2DF
(i) The whole is greater than the part. 1½
 (F is the mid-point of CD) ½
(ii) Things which are halves of the same thing are equal
Also, AE = DF (Given)
to one another. 2
Therefore, AB = CD (Things which are double of
Q. 3. It is known that if a + b = 10, then a + b – c = 10 –
the same thing are equal to one another) 1
c. Write the Euclid’s axiom that best illustrates this
Q. 4. (i) Why is Axiom 5, in the list of Euclid’s axioms, statement. Also give two more axioms other than
considered a ‘universal truth’ ? (Note that the the axiom used in the above situation.
question is not about the 5th postulate.)
R [Board Term I, 2015]
INTRODUCTION TO EUCLID’S GEOMETRY 77
Sol. Axiom : If equals are subtracted from equals, the
remainders are equal.  2 Answering Tips
Two more axioms :
(i) Things which are halves of the same thing are equal
To prove anything the student must be familiar
to one other. 1½
with Axioms, postulates and properties of lines,
(ii) The whole is greater than the part. 1½
triangles, circle etc.
Students should memorize the Euclid’s Axioms
Commonly Made Error by heart.

Sometimes students make mistakes or find


difficulty in proving statements and equations.

OBJECTIVE TYPE QUESTIONS (1 mark each)

Explanation: ‘Lines are parallel if they do not


A Multiple Choice Questions intersect’ is the form of a definition.
Q. 6. Attempts to prove Euclid’s fifth postulate using the
Q. 1. The number of dimension, a point has other postulates and axioms led to the discovery of
(A) 0. (B) 1. several other geometries. State whether the given
(C) 2. (D) 3. statement is true or false? Justify your answer.
 R [NCERT Exemp.]
 R [NCERT Exemp. Ex. 5.2, Q. 9, Page 49]
Ans. Option (A) is correct. Ans. True, these geometries are far different from
Euclidean geometry and are called non-Euclidean
Explanation: According to Euclid, a point is that
geometries.
which has no part,that is, no length, no breadth
and no height. Therefore, it has no dimension. Q. 7. The basic facts which are taken for granted,
without proof and which are specific to geometry
Q. 2. A pyramid is a solid figure, the base of which is
are called
(A) only a triangle. (B) only a square.
(A) Axiom (B) Postulates
(C) only a rectangle. (D) any polygon.
(C) Theorem (D) Definition
 R [NCERT Exemp.] Ans. Option (B) is correct.
Ans. Option (D) is correct. Explanation: Postulates means basic facts without
Explanation: A pyramid is a solid figure, the base proof.
of which is any polygon. Q. 8. Three or more lines intersecting at a same point
are said to be
Q. 3. It is known that if x + y = 10 then x + y + z = 10 +
z. The Euclid’s axiom that illustrates this statement (A) Parallel lines (B) Intersecting lines
is (C) Concurrent lines (D) Straight line
(A) First Axiom. (B) Second Axiom. Ans. Option (C) is correct.
(C) Third Axiom. (D) Fourth Axiom.
 R [NCERT Exemp.] B Assertion & Reason
Ans. Option (B) is correct.
Explanation: If x + y = 10 and then x + y + z = 10 Directions: In the following questions, a statement
+ z. The Euclid’s second axiom states that if equals of assertion (A) is followed by a statement of
are added to equals, the wholes are equal. reason (R). Mark the correct choice as:
Q. 4. Which of the following needs a proof? (A) Both assertion (A) and reason (R) are true and
reason (R) is the correct explanation of assertion (A).
(A) Theorem (B) Axiom
(B) Both assertion (A) and reason (R) are true but reason
(C) Definition (D) Postulate
(R) is not the correct explanation of assertion (A).
 R [NCERT Exemp.] (C) Assertion (A) is true but reason (R) is false.
Ans. Option (A) is correct.
(D) Assertion (A) is false but reason (R) is true.
Explanation: Theorem needs a proof.
Q. 1. Assertion (A): According to Euclid’s 1st Axiom-
Q. 5. ‘Lines are parallel if they do not intersect’ is stated ”Things which are equal to the same thing are also
in the form of equal to one another.”
(A) an axiom. (B) a definition.
Reason (R): If AB = PQ and PQ = XY, then AB =
(C) a postulate. (D) a proof. XY.
 R [NCERT Exemp. Ex. 5.1, Q. 22, Page 47] Ans. Option (A) is correct.
Ans. Option (B) is correct.
78 Oswaal CBSE Question Bank Chapterwise & Topicwise, MATHEMATICS, Class-IX

Explanation: In case of Assertion (A):


\ Reason is true.
Euclid’s 1st Axiom is given which is an assumption
Therefore Assertion and Reason are correct and
and is universal truth.
Reason is correct explanation of Assertion.
\ Assertion is correct.
Q. 3. Assertion (A): Through two distinct points there
In case of Reason (R): It is correct explanation of
can be only one line that can be drawn.
Euclid’s 1st Axiom.
Reason (R): From these two points we can draw
\ Reason is true.
only one line.
Hence, Assertion and Reason are true and R is
correct explanation of A. A B
Q. 2. Assertion (A): According to Euclid’s Axiom, when Ans. Option (A) is correct.
equals are added to equals, wholes are equal.
Q. 4. Assertion (A): If ‘l’ is a line and P is a point not on
Reason (R): If Rita and Rivi are of same age that is the line l, there is one and only one line (say m)
10 years then after 6 years also they will have the which passes through P and parallel to l.
same age.
Ans. Option (A) is correct.
Explanation: In case of Assertion (A):
Euclid’s Second Axiom is given which is an Reason (R): If two lines ‘l’ and ‘m’ are both parallel
assumption and is universal truth. to the same line n, they will also be parallel to each
\ Assertion is correct. other.
In case of Reason (R): If Rita and Rivi are 10 years Ans. Option (B) is correct.
old. Explanation: Assertion is true and Reason is also
After 6 years true but Assertion is saying about only two lines
10 + 6 = 10 + 6 whereas Reason is saying about more than two
16 = 16 lines.

COMPETENCY BASED QUESTIONS (4 marks each)

(A) 13 chapters (B) 12 chapters


A Case based MCQs (C) 11 chapters
(D) 9 chapters

Read the following passage and answer any four Ans. Option (A) is correct.
questions of the following : II. If a point C be the midpoint of a line segment AB,
I. Two salesmen make equal sales during the month then the relationship among AB, BC and AC can be
of June. In July each salesmen doubles his sale of explained by:
the month of June.
Q. 1. What will their sales be in July?
(A) Double (B) Half Q. 1. Relationship Between BC and AC is
(C) Equal (D) One third 1 1
(A) AC =
BC
XY (B) AC = AB
XY
Ans. Option (C) is correct. 2 2
Q. 2. Which axiom is used here? (C) AC = BC
(D) All of these
(A) Things which are halves of same thing are equal
Ans. Option (D) is correct.
to one another.
(B) All right angles are equal to one another 1

Explanation: AC = AB
XY
(C) Things which coincide are equal 2
(D) None of the above ( C is midpoint of AB)
Ans. Option (D) is correct.
1
Q. 3. Euclid’s stated that all right angles are equal to BC = AB
XY
2
each other in the form of
(A) An axiom (B) a definition \
AC = BC
(C) a postulate (D) a proof
Q Thing’s which are halves of the same thing are
Ans. Option (C) is correct. equal to one another.
Q. 4. What does a theorem require ? Q. 2. Which Euclid Axiom’s state the required result.
(A) Meaning (B) Conclusion (A) All right angles are equal to one another.
(C) Theory
(D) Proof (B) Things which are equal to the same thing are
Ans. Option (D) is correct. equal to one another.
Q. 5. Euclid’s divided his famous treatise ‘The Elements‘ (C) Things which are halves of the same thing are
into equal to one another.
INTRODUCTION TO EUCLID’S GEOMETRY 79
(D) None of the above
Q. 1. How many straight lines can be drawn from A to
Ans. Option (C) is correct. C?
Q. 3. Euclid’s belong to the category. Ans. One and only one line can be drawn from A to C. 1
(A) Babylonia
(B) Egypt Q. 2. State the Euclid Axiom which states the required
(C) Greece
(D) India result.
Ans. Option (C) is correct. Ans. According to Euclid’s Postulate,
Q. 4. Here, A, B and C are
“A straight line can be drawn from any point to any
(A) Points of intersection (B) Collinear Points other point.” 1
(C) Non Collinear Points (D) None of these
Q. 3. Give one more Postulate.
Ans. Option (B) is correct. Ans. Another Postulate, “A circle can be drawn with any
Q. 5. For given line segment AB, C is midpoint of AB, if centre and any radius.” 2
II. Rohan’s sister has two daughter’s of same age. Both
(A) C’ is an exterior point of AB
of them have equal number of dolls. Rohan on his
(B) C is an interior point of AB

birthday plans to give both of them same number of
(C) C is not lying on AB
dolls.
(D) None of the above
Q. 1. How many dolls will each one of them have after
Ans. Option (B) is correct. Rohan’s birthday?
Ans. Both of them will have equal number of dolls. 1
Case based Subjective Q. 2. Which Euclid’s axiom is used to answer this
B Questions
question?

Read the following passage and answer the Ans. Euclid’s Axiom 2, “If equals are added to equals,
following questions: then the whole are equal, is used in this question. 1
I. Three light house towers are located at points A, B Q. 3. Write one more Euclid’s axiom?
and C on the section of a national forest to protect Ans. According to Euclid’s Axiom 3, “If equals are
animals from hunters by the forest department as subtracted from equals, then the remainders are
shown in figure. equal. 2

  
Study Time:
Maximum time: 3:30 Hrs
Maximum questions: 45
CHAPTER

6
LINES AND
ANGLES
(Motivate) If a ray stands on a line, then the sum of the two adjacent angles so formed
Syllabus is 180° and the converse.
(Prove) If two lines intersect, vertically opposite angles are equal.\
(Motivate) Lines which are parallel to a given line are parallel.

List of Topics
Topic-1 Types of Angles Topic-1: Types of Angles
 Page No. 80
Topic-2 : Exterior Angle and In-
tersecting Lines Page No. 86
Revision Notes
Line : Line is a collection of points which has only length neither breadth nor thickness.
Line Segment : A line with two end points.
Ray : A part of line with one end point.
Angle : An angle is formed when two rays originate from the same end point. The rays making an angle are called
the arms and the end point is called the vertex.
 Types of Angles :
(i) Acute Angle : 0° < x < 90°
An angle whose measure is more than 0° but less than 90° is called an Acute angle.

Scan to know
more about
this topic

(ii) Right Angle : x = 90°


An angle whose measure is 90°, is called a right angle.

Lines and
Angles

(iii) Obtuse Angle : 90° < x < 180°


An angle whose measure is more than 90° but less than 180° is called an obtuse angle.
LINES AND ANGLES 81
82 Oswaal CBSE Question Bank Chapterwise & Topicwise, MATHEMATICS, Class-IX

(iv) Straight Angle : x = 180°


An angle whose measure is 180° is called a straight angle.

(v) Reflex Angle : 180° < x < 360°


An angle whose measure is more than 180° but less than 360° is called a reflex angle.

(vi) Complete Angle : x = 360°


An angle whose measure is 360° is called a complete angle.

 Complementary Angles : Two angles whose sum is 90° are called complementary angles.
e.g., Complement of 30° angle is 60° angle.
 Supplementary Angles : Two angles whose sum is 180° are called supplementary angles.
e.g., Supplement of 70° angle is 110° angle.
 Adjacent Angles : Two angles are called adjacent angles, if :
(i) they have the same vertex,
(ii) they have a common arm, and
(iii) uncommon arms on opposite side of the common arm.
In the figure, ∠AOP and ∠BOP are adjacent angles.
 Vertically Opposite Angles : When two straight lines intersect each other four angles are formed. The pair of
angles which lie on the opposite sides of the point of intersection are called vertically opposite angles.

In figure, ∠AOC and ∠BOD are vertically opposite angles and ∠AOD and ∠BOC are also vertically opposite
angles.
Vertically opposite angles are always equal.
 Linear Pair of Angles : Two adjacent angles are said to form a linear pair of angles, if their non-common arms are
two opposite rays.
OR
When the sum of two adjacent angles is 180°, then they are called linear pair of angles.

In figure, ∠AOC and ∠BOC form a linear pair of angles.


Examples: Find x in the figure given below
LINES AND ANGLES 83
Solution : 6x + 3x = 180°  [Linear pair]
9x = 180°
x = 180° = 20°
9

Mnemonics
1. To draw an angle with the help of protractor we use the mnemonic ‘LIRO’—left inner right outer to draw angle
on left side of a line we use inner scale of protractor to draw from right side we use outer scale.
2. Easy way to learn supplementary and complementary angles.

Supplementary Complementary
= 180° = 90°
(It makes 8) (It makes 9)

Example 1
In the given figure, two straight lines PQ and RS 105°
intersect each other at O. If ÐPOT = 75°, find the or, b = = 15°
7
values of a, b, c.
a + 2c = 180° ...(i)
Step-II : Use the property which gives relation for a
and 4b find the value of a.
Since, vertically opposite angles are equal
\ a = 4b
or, a = 4 × 15°
or, a = 60° ...(ii)
Step-III : Substitute value of a in eq. (i) to find the
value of c.
Solution: From eqn. (i) we get
Step-I : Identify the straight line and use the a + 2c = 180°
suitable property to find the value of b. or, 60° + 2c = 180°
Here, ROS is a straight line. So by property that or, 2c = 180° – 60° = 120°
sum of all angles on a straight line is 180°, we get 120°
or, c= 2
ÐROP + ÐPOT + ÐTOS = 180°
4b + 75° + 3b = 180° = 60°
or, 7b + 75° = 180° Hence, a = 60°, b = 15°, c = 60°
or, 7b = 180° – 75 ° = 105°

SUBJECTIVE TYPE QUESTIONS


Angle x = Complement of x
Very Short Answer Type or, x = 90° – x
Questions (1 mark each) 2x = 90°.
90°
Q. 1. Two supplementary angles are in ratio 2 : 7. Find x=
the measures of angles.  U [Board Term I, 2016] 2
or x = 45° 1
Sol. 2x + 7x= 180° Þ x = 20° Q. 3. Two angles measure (30° – a) and (125° + 2a). If
So, the angles are each one is the supplement of the other, then find
2x = 2 × 20° = 40° the value of a. A
7x = 7 × 20° = 140° Sol. Angles (30° – a) and (125° + 2a) are supplementary
So, two angles are 40° and 140°. 1 of each other, then
[CBSE Marking Scheme, 2016] 30° – a + 125° + 2a = 180°
Q. 2. What is the measure of an angle which is or, a = 180° – 155°
complement of itself ? R = 25°. 1
Sol. Let the angle be x, then
84 Oswaal CBSE Question Bank Chapterwise & Topicwise, MATHEMATICS, Class-IX

Q. 4. Write the angle which is one-fifth of its complement. R Sol. a + b = 180° (Linear pair) ½
Sol. Let the angle be x, then a – b = 80° (Given)
1 Adding, 2a = 260° ½
By given condition, x = (90° –x)
5 or, a = 130° ½
or, 5x = 90° – x and b = 180° – a = 180° – 130°
or, x = 15°. 1 = 50°. ½
Q. 5. In the figure below, AOB is a straight line. [CBSE Marking Scheme, 2012]
Calculate the measure of ∠COD. U Q. 4. In the given figure, ∠AOB : ∠BOC = 2 : 3. If ∠AOC
= 75°, then find the measure of ∠AOB and ∠BOC.
C D  U [Board Term I, 2012]

2x – 20°
x+ 20° 60°
A O B
Sol. ∵ x + 20° + 2x – 20° + 60° = 180°
(∵ Straight line makes an angle of 180°)
or, 3x = 180° – 60° = 120°
or, x = 40°
Thus, ∠COD = 2x – 20° = 80° – 20° = 60°. 1

Short Answer Type


Sol. Let, ∠AOB = 2x
Questions-I (2 marks each) and ∠BOC = 3x
Q. 1. If (3x – 15°) and (x + 5°) are complementary angles, ∠AOB + ∠BOC = ∠AOC
find the angles. R [Board Term I, 2015] or, 2x + 3x = 75°
Sol. (3x – 15°) + (x + 5°) = 90° 75°
or 4x = 90° + 10° = 100° or, x= = 15°
5
100°
or x= ∴ ∠AOB = 2x = 2 × 15° = 30° 1
4
and ∠BOC = 3x = 3 × 15 = 45°. 1
or, x = 25°
1st angle = 3x – 15 [CBSE Marking Scheme, 2012]
= 3(25) – 15 = 60° Q. 5. In figure, ∠DOB = 87° and ∠COA = 82°. If ∠BOA
2nd angle = (x + 5) = 35°, then find ∠COB and ∠COD.
= 25 + 5 = 30°
Angles are 60° and 30°. 2
Q. 2. In the figure, lines XY and MN intersect at O. If
∠POY = 90° and a : b = 2 : 3, find the value of c.

U [Board Term I, 2012]


U [Board Term I, 2012] Sol. Given, ∠COA = 82°
Sol. Let, ∠a = 2x and ∠b = 3x ½ or, ∠COB + ∠BOA = 82°
Then, ∠a + ∠b = 90°
or, 2x + 3x = 90° or, ∠COB + 35° = 82° (∵ ∠BOA = 35°)
∴ x = 18° ½ or, ∠COB = 82° – 35° = 47° 1
a = 2 × 18° = 36°
b = 3 × 18° = 54° Similarly, ∠DOB = ∠DOC + ∠COB
∴ c = 180° – b = 180° – 54° or, 87° = ∠DOC + 47°
= 126°. 1
or, ∠DOC = 87° – 47° = 40°. 1
[CBSE Marking Scheme, 2012]
[CBSE Marking Scheme, 2012]
Q. 3. In the given figure, ∠AOC and ∠BOC form a line
AB. If a – b = 80°, find the values of a and b.
Short Answer Type
Questions-II (3 marks each)
Q. 1. In the figure PQ and RS intersect each other at
point O. If ÐPOR : ÐROQ = 2 : 3, Find ÐPOR and
ÐROQ.
U [Board Term I, 2012]
LINES AND ANGLES 85

ÐAOC + ÐAOD = ÐAOD + ÐBOD


[Each equal to 180°]
\ ÐAOC = ÐBOD
Similarly ÐAOD = ÐBOC Hence Proved 1
[CBSE Marking Scheme, 2016]
Q. 4. Prove that bisectors of pair of vertically opposite
angles are in the same straight line.
U [Board Term I, 2016] A [Board Term I, 2016]
Sol. Given : Two lines AB and CD intersect at point O.
Sol. ÐPOR = 2x = ÐROQ = 3x Also, OM and ON are the bisectors of ÐAOC and
ÐPOR + ÐROQ = 180° (straight line angle) ÐBOD respectively.
or, 2x + 3x = 180°
x = 36°
1
\ ÐPOR = 2x = 2 × 36° = 72°
ÐROQ = 3x = 3 × 36° = 108° 3
[CBSE Marking Scheme, 2016] To prove : MON is a straight line.
Q. 2. In the given figure, lines AB, CD and EF meet Prove : Since, the sum of all the angles around a
at O. Find the value of x, hence find all the three point O is 360°, we have
indicated angles. ÐAOC + ÐBOC + ÐBOD + ÐAOD = 360°
or, 2ÐMOC + 2ÐBOC + 2ÐBON = 360° 1
[∵ ÐBOC = ÐAOD ( vertically : opp. angles OM is
bisector of ÐAOC, ON is bisector of ÐBOD]
or, ÐMOC + ÐBOC + ÐBON = 180°
or, ÐMON = 180°
\ ÐMON is a straight angle
Hence, MON is a straight line. 1
Q. 5. In the figure, if x + y = w + z, then prove that AOB
is a straight line.
U [Board Term I, 2016]

Sol. ÐCOF = 2x
(vertically opposite angles)
\
3x + 2x + 5x = 180 (straight line angle)
or, 10x = 180 Þ x =18
ÐAOC = 3x = 3 × 18 = 54
ÐBOF = 5x = 5 × 18 = 90 R [Board Term I, 2015] [NCERT]
ÐDOE = 2x = 2 × 18 = 36 3 Sol. x + y + w + z = 360°
or, 2(x + y) = 360° (∵ x + y = w + z)
[CBSE Marking Scheme, 2016] or, x + y = 180°
Q. 3. Prove that if two lines intersect, vertically opposite ∴ AOB is a straight line. 3
angles are equal.  U [Board Term I, 2016]
[NCERT Exemp.] Long Answer Type
Sol. Given : Two lines AB and CD intersect at a point O. Questions (5 marks each)
Q. 1. In figure, m and n are two plane mirrors perpendicular
to each other. Show that incident ray CA is parallel to
reflected ray BD. A

1
To Prove
(i) ÐAOC = ÐBOD
(ii) ÐAOD = ÐBOC
Proof \ Ray OA stand on line CD
\ ÐAOC + ÐAOD = 180° [Linear pair] ...(i)
Again ray OD stand on line AB 1
\ ÐAOD + ÐBOD = 180° [Linear pair]...(ii)
from eqn. (i) and (ii),
 [Board Term I, 2016, NCERT Exemplar]
86 Oswaal CBSE Question Bank Chapterwise & Topicwise, MATHEMATICS, Class-IX

Sol. Let normals at A and B meet at P. Sol. 84° + 2x = 180° (linear pair)
or, 2x = 96° 2
or, x = 48°
y + 75° = 2x (VOA)
or, y = 2 × 48° – 75°
= 96° – 75° = 21° 1
75° + 21° + z = 180° (straight line angles) 2
z = 180° – 96°
z = 84°
Q. 3. AB, CD and EF are three concurrent lines passing
1 through the point O such that OF bisect ÐBOD. If
ÐBOF = 35°, find ÐBOC and ÐAOD.
As mirrors are perpendicular to each other there- Sol.
fore BP || OA and AP || OB.
So BP ^ PA,
i.e., ÐBPA = 90° 1
Therefore Ð3 + Ð2 = 90° ...(i)
(Angle sum property)
Also Ð1 = Ð2 and Ð4 = Ð3  1
(Angle of incidence = Angle of reflection)
Therefore Ð1 + Ð4 = 90° ...(ii) [from (i)]
Adding (i) and (ii), we have
AB, CD and EF are three concurrent lines passing
Ð1 + Ð2 + Ð3 + Ð4 = 180°
through the point O.
i.e., ÐCAB + ÐDBA = 180°
OF bisects ÐBOD
Hence, CA || BD 2
Þ ÐBOF = ÐFOD = 35° 1½
Q. 2. In the given figure, lines AB and CD intersect each But ÐBOD = ÐBOF + ÐFOD
other at O. Find the values of x, y and z. = 35° + 35°
= 70° 1½
ÐBOD + ÐAOD = 180° (Linear pair)
ÐAOD = 180° – 70°
= 110°
ÐBOC = ÐAOD
(Vertically opposite angles are equal)
\ ÐBOC = 110° 2
Hence, ÐAOD = 110° and ÐBOC = 110°.

U [Board Term I, 2014]

Topic-2 Exterior Angle and Intersecting Lines

Revision Notes
 Intersecting Lines : Two lines are said to be intersecting when the perpendicular distance between the two lines
is not same everywhere. They meet at one point.
 Non-Intersecting lines : Two lines are said to be non-intersecting lines when the perpendicular distance between them
is same every where. They do not meet. If these lines are in the same plane these are known as parallel lines.

THEOREM
 Theorem 1 : the exterior angle so formed is equal to the sum of
Statement : Lines which are parallel to the same line the two interior opposite angles.
are parallel to each other. Given : A Triangle ABC with interior angles x, y and

 Theorem 2 : z, and exterior angle ‘e’.
Statement : If a side of a triangle is produced, then To Prove :

LINES AND ANGLES 87
Proof : In the figure above :
x + y + z = 180° ...(i)
(Angle Sum Property)
e + z = 180° ...(ii)
(Linear Pair)
Comparing equations (i) and (ii),
x+y+z =e+z
therefore, x + y = e, Hence Proved.

Example 2
In given figure find value of x?
In DABC ÐA = 50°
Exterior ÐACE = 127°
ÐA + ÐB = 127°
(Exterior angle property)
50° + ÐB = 127°
x = 127° – 50°
Hence, x = 77°
Solution:

SUBJECTIVE TYPE QUESTIONS


ÐCOF = 5y
Very Short Answer Type Now AB is a straight line
\ ÐAOC + ÐCOF + ÐFOB = 180°
Questions (1 mark each) (Straight line angle)
Q. 1. What will be measure of an angle which is five 5y + 5y + 2y = 180°
times its complement? 12y = 180°
Sol. Let the angle be x 180
y= = 15° 1
Complement of x = (90 – x) 12
\ x = 5(90 – x) Q. 4. In the figure given below, Find ÐACE?
\ x = 450° – 5x
6x = 450°
x = 75° 1
Q. 2. What will be measure of an angle whose
supplement is one-fourth of itself?
Sol. Let the angle be x
1
Supplement of x = x
4 Sol. In DABC ÐACE = exterior angle
1 \ ÐACE = ÐBAC + ÐABC
\ x + x = 180°
4 ( exterior angle is equal to sum of interior opposite
4x + x angles)
= 180°
4 ÐACE = 42° + 56°

180° ´ 4 = 98° 1
x=
5 Q. 5. What can you say about lines l and n if l||m and
x = 144° 1 m||n in figure given below?
Q. 3. In the figure given below, find the value of y.
U [Board Term I, 2015, 2012]

Sol. l||m and m||n (given)


\ l||n
Because, Lines which are parallel to the same line
are parallel to each other.

Sol. ÐCOF = ÐEOD


(Vertically opposite angles are equal)
88 Oswaal CBSE Question Bank Chapterwise & Topicwise, MATHEMATICS, Class-IX

Short Answer Type Short Answer Type


Questions-I (2 marks each) Questions-II (3 marks each)
Q. 1. Two supplementary angles are in the ratio 3 : 7, Q. 1. In the fig. given below, An exterior angle of a
find the angles. R triangle is 130° and the two interior opposite angles
Sol. Let the two supplementary angles be 3x and 7x, are equal. Find each of these angles.
then 3x + 7x = 180°
10x = 180°
180
x= = 180° 1
10
Hence, the angles are 3 × 18 = 54°
7 × 18 = 126° 1 Sol. In a triangle ABC
Q. 2. In figure given below, calculate the value of angle x + x = 130°
q. U [CBSE Term-I, 2015, 2012]
(exterior angle is equal to the sum of interior
[CBSE SAS] opposite angles)
2x = 130°
x = 65° 1
\ ÐA = ÐB = 65°
ÐACB + ÐACD = 180° (Linear pair)
ÐACB + 130° = 180° 1
ÐACB = 50°
Hence, ÐABC = 65°; ÐBAC = 65° and ÐACB = 50° 1
Sol. AB is a straight line
Q. 2. In given figure PO ^ AB, if x : y : z = 1 : 3 : 5, then
\
x + 50° + 90° = 180° (straight line angles)
find the degree measure of x, y and z.
x = 180 – 140° 1
x = 40°
Now, x =q
(Vertically opposite angles are equal)
\
q = 40° 1
Q. 3. In figure, Prove that ÐAOB + ÐBOC + COD +
U [Board Term I, 2014, 2012]
ÐDOA = 360° A [Board Term I, 2012]
Sol. OP ^ AB ½
So, ÐPOA = 90°
Let x = ÐPOQ = a
y = ÐQOR = 3a
z = ÐROA = 5a ½
a + 3a + 5a = 90°
Sol. Extend AO to E, 9a = 90°
a = 10° ½
\ x = 10° ½
E
y = 3 × 10 = 30° ½
z = 5 × 10 = 50° ½
[CBSE Marking Scheme, 2012, 2014]
\ ÐAOB + ÐBOE = 180° (Linear Pair) ...(i) ½
Q. 3. In the figure given below ÐABD is an exterior
ÐEOC + ÐCOD + ÐDOA = 180°
angle of DABC.
(Adjacent angles) ...(ii) ½
Adding (i) and (ii) we get
ÐAOB + ÐBOE + ÐEOC +ÐCOD + ÐDOA
= 180° + 180°
ÐAOB + ÐBOC + ÐCOD + ÐDOA = 360° 1 Sol. ÐABD = ÐBAC + ÐACB
Hence Proved (exterior angle of a triangle is equal to sum of
[CBSE Marking Scheme, 2012] interior opposite angles)
3x + 2x + 1 = 2x2 + 1 + 3x + 6
2
LINES AND ANGLES 89
3x + 2x – 2x – 3x = 7 – 1
2 2
and, ÐAPD + ÐCPD = 100° ½
x2 – x – 6 = 0 Let ÐCPD = x ½
(x – 3) (x + 2) = 0 \ ÐBPC = 146° – x ½
x = 3 or –2 and ÐAPD = 100° – x ½
But angle cannot be in negative ÐAPD + ÐCPD + ÐBPC = 180°
\ x =3 2 (straight line angles) 1
ÐABC + ÐABD = 180° (Linear pair) 100° – x + x + 146° – x = 180°
ÐABC = 180° – (3 × 32 + 2 × 3 + 1) 246° – x = 180°
ÐABC = 180° – 34° x = 66° 1½
\ ÐABC = 146° 1 Hence, ÐCPD = 66°
Q. 4. In the figure AB ||CD, EF ^ CD and ÐGFC = 130°. Q. 2. In the figure given below, if AB and CD are straight
Find x, y and z. U [Board Term I, 2013, 2012] lines intersecting at O and OE is perpendicular to
CD, find the value of x and y.

E
Sol. According to figure,
Sol. EF ^ CD AB is a straight line
\ ÐCFE = 90° \ ÐAOD + ÐDOE + ÐEOB = 180°
1
90 + z = ÐCFG x + 90° + 5x = 180°
z = 130° – 90 = 40°
6x = 90°
ÐFEG = 90° ( FE ^ AB)
x = 15° 1
\ Exterior ÐBGF = Ðz + ÐFEG
Now, ÐAOC = ÐDOB
( exterior angle property)
x = 40° + 90° (vertically opposite angles)
x = 130° 1 ÐAOC = ÐDOE + ÐEOB 1
x + y = 180° (Linear pair) 1 y = 90 + 5 × 15
130° + y = 180° y = 165°
y = 50° Hence, x = 15° and y = 165° 2
[CBSE Marking Scheme, 2012, 2013] Q. 3. In the given figure, the side QR of ÐPQR is
produced to a point S. If the bisectors of ÐPQR and
ÐPRS meet at point T, then prove that ÐQTR =
Commonly Made Error 1
ÐPQR.
2
Students make a mistake in identifying and
applying exterior angle property.

Answering Tip

Students should be well versed with exterior


angle property.
Sol. In DPQR, QR is produced to point S,
\ ÐPRS = ÐP + ÐPQR
Long Answer Type (exterior angle property)
Questions (5 marks each)
So,
1 1
ÐPRS = ÐP + ÐPQR
1
1
Q. 1. In the figure given below ÐAPC = 100° and ÐBPD 2 2 2
= 146°. Find ÐCPD ? 1
ÐTRS = ÐP + ÐTQR ...(i)
2
( QT and RT are bisectors of ÐPQR and ÐPRS
respectively) 1
Now in DQTR,
Sol. According to figure ÐTRS = ÐT + ÐTQR ...(ii)
ÐBPC + ÐCPD = 146° ½ (exterior angle property of a triangle)
From (i) and (ii) we have
90 Oswaal CBSE Question Bank Chapterwise & Topicwise, MATHEMATICS, Class-IX

1 1
ÐP + ÐTQR = ÐTQR + ÐT 1 Hence, ÐQPR = ÐQTR
2 2
1 OR
Þ
ÐP = ÐT
2 1
ÐQTR = ÐQPR 1
2
Proved

OBJECTIVE TYPE QUESTIONS (1 mark each)

(A) 20° (B) 30°


A Multiple Choice Questions (C) 10° (D) 40° A
Ans. Option (C) is correct.
Q. 1. In the given figure, POQ is a line. The value of x is. Explanation: Angles (55° + 3a) and (115° – 2a) are
supplement of each other,
(A) 20° (B) 25° Then, 55° + 3a + 115° – 2a = 180°
(C) 30° (D) 35° A a + 170° = 180°
a = 10°
Q. 5. In the adjoining figure, AOB is a straight line. If x
: y : z = 4 : 5 : 6, then y = ?
(A) 60° (B) 80°
(C) 48° (D) 72°
Ans. Option (A) is correct.
Explanation:
40° + 4 x + 3x = 180° ( POQ is a straight line)
7 x = 180° - 40°
7 x = 140°
140°
x=
7
x = 20°
Q. 2. An exterior angle of a triangle is 105° and its two Ans. Option (A) is correct.
interior opposite angles are equal. Each of these Explanation: Let p bet the common ratio.
equal angles is \ x = 4p, y = 5p, 6z = 6p
1 1 x + y + z = 180°
(A) 37 ° (B) 52 ° [angles on a straight line]
2 2 4p + 5p + 6p = 180°
1 15p = 180°
(C) 72 ° (D) 75° A 180°
2 p= = 12
15
Ans. Option (B) is correct.
Explanation: Let each interiors be x° Now y = 5 × 12 = 60°
Exterior ∠s = 105° Q. 6. In the given figure, AOB is a straight line. If ÐAOC
Sum of interior opp. ∠s = Exterior ∠s = 4x° and ÐBOC = 5x° then find ÐAOC.
x + x = 105° (A) 40° (B) 60°
2 x = 105° (C) 80° (D) 100°
105°
x=
2
1
= 52.5° or 52 °
2
Q. 3. Write the complementary angle of 65°.
(A) 65° (B) 25° Ans. Option (C) is correct.
(C) 75° (D) 35° R Explanation: ÐAOC + ÐBOC = 180°
Ans. Option (B) is correct. [Angles on a straight line]
Explanation: Complementary angle of 65° 4x + 5x = 180°
= 90 – 65° 9x = 180°
= 25 ° 180°
x=
( Sum of complementary angles is 90°) 9
Q. 4. Two angles measure (55° + 3a) and (115° – 2a). If x = 20°
each is supplement of the other, then calculate the ÐAOC = 4x = 4 × 20° = 80°
value of a.
LINES AND ANGLES 91
(C) Assertion (A) is true but reason (R) is false.
Q. 7. In the given figure, AOB is a straight line, if ÐAOC
= (3x + 10)° and ÐBOC = (4x – 26)°, find ÐBOC. (D) Assertion (A) is false but reason (R) is true.
(A) 96° (B) 86° Q. 1. Assertion (A): If angles 'a' and 'b' form a linear pair
(C) 76° (D) 106° of angles and a = 40° then b = 150°.
Reason (R): Sum of linear pair of angles is always
180°.
Ans. Option (D) is correct.
Explanation: In case of Assertion (A):
'a' and 'b' are linear pair (given)
So a + b = 180°
But 40° + 150° = 190° ¹ 180°
\ Assertion is false.
In case of Reason (R):
Ans. Option (B) is correct. The sum of linear pair of angles is always 180°.
Explanation: ÐAOC + ÐBOC = 180° \ Reason is true.
[Angles on a straight line] Q. 2. Assertion (A): An angle is 14° more than its
(3x + 10)° + (4x – 26°) = 180° complementary angle, then angle is 52°.
3x + 10° + 4x – 26° = 180° Reason (R): Two angles are said to be supplementary
3x + 4x = 180° + 16° if their sum is 180°.
7x = 196° Ans. Option (B) is correct.
196° Explanation: In case of assertion (A):
x=
7 Let the angle be x°
x = 28° Complement of x° = (90 – x)°
ÐBOC = (4x – 26)° Since, the difference is 14°
= (4 × 28 – 26) = 86° \ x – (90° – x) = 14°
Þ x – 90° + x = 14°
Q. 8. In the given figure, AOB is a straight line. If ÐAOC
= (3x – 10)°, ÐCOD = 50° and ÐBOD = (x + 20)° Þ 2x = 14° + 90°
then find ÐAOC. 2x = 104°
(A) 40° (B) 60° x = 52°
(C) 80° (D) 50° \ Assertion is true.
In case of reason (R):
Sum of supplementary angles is 180°.
\ Reason is also true.
But Reason is not correct explanation of Assertion.
Q. 3. Assertion (A): Sum of the pair of angles is 120° and
60° is supplementary.
Reason (R): Two angles, the sum of whose is 90° are
Ans. Option (C) is correct. called supplementary angles.
Ans. Option (C) is correct.
Explanation: ÐAOC + ÐCOD + ÐDOB = 180°
Explanation: In case of assertion (A):
[Angles on a straight line]
Sum of 120° and 60° is
3x – 10° + 50° + x + 20° = 180°
120° + 60° = 180°
4x + 60° = 180° which is supplementary.
4x = 180° – 60° \ Assertion is true.
4x = 120° In case of reason (R):
120 Two angles whose sum is 90° are called
x= = 30°
4 complementary angles.
\ Reason is false.
ÐAOC = (3x – 10)°
= (3 × 30 – 10) Q. 4. Assertion (A): An exterior angle of a triangle is 110°
= 90 – 10 = 80° and its two interior angles are equal. Therefore each
of these equal angle is 55°.
Reason (R): Sum of two interior opposite angles is
B Assertion & Reason equal to the exterior angle.
Ans. Option (A) is correct.
Directions: In the following questions, a statement
Explanation: In case of assertion (A):
of assertion (A) is followed by a statement of
Given, interior angles are equal
reason (R). Mark the correct choice as: Let the interior angles be x.
(A) Both assertion (A) and reason (R) are true and x + x = 110°
reason (R) is the correct explanation of assertion (A). 2x = 110°
(B) Both assertion (A) and reason (R) are true but reason x = 55°
(R) is not the correct explanation of assertion (A). \ Assertion is true.
92 Oswaal CBSE Question Bank Chapterwise & Topicwise, MATHEMATICS, Class-IX

In case of reason (R):


\ Reason is also true.
We know that the exterior angle is equal to the sum
Hence, Both A and R are true and R is the correct
of two interior opposite angles.
Explanation of A.

COMPETENCY BASED QUESTIONS (4 marks each)

Ans. Option (D) is correct. 1


A Case based MCQs Q. 3. If cow at C and cow at D is 2 km apart, then what
is the distance between cow at A and cow at B?
Read the following passage and answer any four
(A) 1 km (B) 2 km
questions of the following :
(C) 3 km (D) 4 km
I. BSE stands for a disease called Bovine Spongiform
Encephalopathy. “Bovine” means that the disease Ans. Option (B) is correct.
affects cows, “spongiform” refers to the way Explanation: Since, p||q and l||m thus, ABCD is a
the brain from a sick cow looks spongy under a parallelogram.
microscope, and “encephalopathy” indicates that it
Also, since opposite sides of a parallelogram are
is a disease of the brain. This disease is commonly
called “mad cow disease.” equal.
So, AB = CD½
Given, distance between cow at C and cow at
D = CD = 2 km
⇒ AB = 2 km
Hence, distance between cow at A and cow at B is
2 km. ½
Q. 4. If ∠B = 45°, then ∠D = _________ .
(A) 50° (B) 40°
(C) 45° (D) 55°
Ans. Option (C) is correct.

A farmer has a field ABCD formed by two pair of
Explanation: Since, ∠B = 45
parallel roads as shown below in which l || m and
p || q. His four cows suffering from BSE. Thus, he ⇒ ∠D = 45°
tied them at four corners of the field ABCD. (opposite angles of a parallelogram are equal) 1
Q. 5. If we join BD such that BD meet AC at O and
∠BOC = 30°, then what is the measure of
∠AOD? 
(A) 90° (B) 60°
(C) 45° (D) 30°
Ans. Option (D) is correct.
Explanation: ∠BOC= ∠AOD = 30°
(vertically opposite angles are equal)  1
II. A plane mirror is a mirror with a flat reflective surface.
Q. 1. If ∠BAC = 30°, find ∠CAD.
(A) 60° (B) 90°
(C) 30° (D) 20°
Ans. Option (C) is correct.
Explanation: AC is diagonal.
\ ÐBAC = ÐCAD
( Diagonals of a parallelogram bisect one of its
angles)
Thus ÐCAO = 30° 1
Q. 2. ∠ABC + ∠BCD = 180° as : An incident ray is a ray of light that strikes a
(A) Corresponding angles are supplementary. surface. The reflected ray corresponding to a
(B) Alternate interior angles are supplementary. given incident ray, is the ray that represents the
(C) Alternate exterior angles are supplementary. light reflected by the surface.
(D) Angles on the same side of a parallelogram In figure, m and n are two plane mirrors
are supplementary. perpendicular to each other.
LINES AND ANGLES 93
Q. 1. Parallel lines straight road cuts the other at 30°.
(A) meet at a point Now using the given information, answer the
(B) never meet following questions.
(C) sometimes meet
(D) meet at right angle
Ans. Option (B) is correct.
Q. 2. Which statement is incorrect ? Q. 1. Find the measure of ÐBOD.
(A) Corresponding angles formed at Ans. ÐBOD = ÐAOC
corresponding corners.
(Vertical opposite angles)
(B) Alternate interior angles are equal
ÐAOC = 30° (given)
(C) Angles on the same side of the transversal
\ ÐBOD = 30° 1
are complementary
(D) Vertically opposite angles are equal Q. 2. Find the measure of ÐAOD
Ans. Option (C) is correct. Ans. ÐAOC + ÐAOD = 180° (Linear pair)
Q. 3. Incident ray CA is : 30° + ÐAOD = 180°
(A) parallel to AB ÐAOD = 150° 1
(B) perpendicular to BD Q. 3. Which property is used in this case.
(C) parallel to BD Ans. Intersecting lines property is used which states,
(D) perpendicular to AO that, "Two lines are said to be intersecting when the
Ans. Option (C) is correct. 1 perpendicular distance between the two lines is not
Q. 4. ∠BOA = same everywhere. They meet at one point." 2
(A) <60°° (B) = 90° II. To Protect poor people from cold weather, Ramlal,
(C) =180°° (D) >270°° has given his land to make a shelter home for them.
In the given figure sides PQ and RQ of DPQR are
Ans. Option (B) is correct. 1
Explanation: m and n are two plane mirrors produced to point S and T respectively. If ÐPQT
perpendicular to each other = 110° and ÐSPR = 135°, answer the following
So, AO is perpendicular to BO questions.
\ ∠BOA = 90°
Q. 5. If BO = 3 cm, AB = 5 cm then, AO = 
(A) 3 cm (B) 2 cm
(C) 4 cm (D) 6 cm
Ans. Option (C) is correct.  1
Explanation: m and n are two plane mirrors
perpendicular to each other.
Q. 1. Find ÐQPR.
So, AO is perpendicular to BO.
Ans. ÐSPR +ÐQPR = 180° (Linear pair)
Thus, triangle AOB is a right-angled triangle.
135° + ÐQPR = 180°
OA2 + OB2 = AB2
\ ÐQPR = 45° 1
OA2 + 32 = 52
OA2 + 9 = 25 Q. 2. Find ÐPRQ.
OA2 = 25 – 9 = 16 Ans. In DPQR, ÐPQT is an exterior angle
OA = 4 cm \ ÐPQT = ÐQPR + ÐPRQ
(exterior angle proeprty)
B Case based Subjective 110° = 45° + ÐPRQ
Questions 110 – 45° = ÐPRQ
\ ÐPRQ = 65° 1
Read the following passage and answer the Q. 3. Which property is used over here.
following questions: Ans. In this case exterior angle property is used which
I. Harry was going on a road trip with his father. They states that, "An exterior angle of a triangle is equal
were travelling on a straight road. After riding for to the sum of interior opposite angles." 2
some distance, they reach a cross road where one

  
Study Time:
Max. Time: 3 Hrs
Max. Questions: 42
CHAPTER

7 TRIANGLES

(Motivate) Two triangles are congruent if any two sides and the included angle of
Syllabus one triangle are equal to any two sides and the included angle of the other triangle
(SAS Congruence).
(Prove) Two triangles are congruent if any two angles and the included side of one triangle is equal to any two angles
and the included side of the other triangle (ASA Congruence).
(Motivate) Two triangles are congruent if the three sides of one triangle are equal to three sides of the other triangle
(SSS Congruence).
(Motivate) Two right triangles are congruent if the hypotenuse and a side of one triangle are equal (respectively) to the
hypotenuse and a side of the other triangle (RHS Congruence).
(Prove) The angles opposite to equal sides of a triangle are equal.
(Motivate) The sides opposite to equal angles of a triangle are equal.

List of Topics
Topic-1: Criteria for Congruence
of Triangles Page No. 94
Criteria for Congruence of
Topic-1 Triangles Topic-2 : Properties of Triangles
 Page No. 100

Revision Notes
 Congruence of Triangle : The geometrical figures of same shape and size are congruent to each other i.e., two
triangles DABC and DPQR are congruent if and only if their corresponding sides and the corresponding angles
are equal.


If two triangles DABC and DPQR are congruent under the correspondence A ¾® P, B ¾® Q and C ¾® R, then
symbolically it is expressed as
DABC @ DPQR Scan to know
more about
 SAS Congruence Rule : Two triangles are congruent if two sides and the included angle of one this topic
triangle are equal to the sides and the included angle of the other triangle.
 ASA Congruence Rule : Two triangles are congruent if two angles and the included side of one
triangle are equal to two angles and the included side of the other triangle.
 AAS Congruence Rule : Two triangles are congruent if any two pairs of angles and one pair of Triangles Part-I
corresponding sides are equal.
TRIANGLES 95
96 Oswaal CBSE Question Bank Chapterwise & Topicwise, MATHEMATICS, Class-IX

 SSS Congruence Rule : If three sides of a triangle are equal to the three sides of another triangle, then the two
triangles are congruent.
 RHS Congruence Rule : If in two right triangles, the hypotenuse and one side of a triangle are equal to the
hypotenuse and one side of the other triangle, then the two triangles are congruent.

Example 1
Since PQ = PR
Q. In an isosceles DPQR with PQ = PR, S and T are
So ÐR = ÐQ ...(iii)
points on QR such that QT = RS show that PS =
PT. [Property of isosceles triangle]
From eqn. (ii) we have QT = RS
Solution:
Þ QT – ST = RS – ST
Step I : Read the question carefully and write the
[Subtracting ST from both sides]
given conditions.
Þ QS = RT ...(iv)
Step III : Apply the suitable congruence rule in two
triangles.
In DPQS and DPRT, PQ = PR [From eqn. (i)]
ÐQ = ÐR [From eqn. (iii)]
and QS = RT [From eqn. (iv)]
\ DPQS @ ÐPRT
DPQR is an isosceles triangle in which (By SAS congruence rule)
PQ = PR ...(i) Step IV : Apply the property CPCT i.e.,
S and T are points on QR such that corresponding part of congruent triangles, to get
the required result.
QT = RS ...(ii)
Step II : Apply the theorems related to given As DPQS @ DPRT
conditions to find other information. Then PS = PT (CPCT)

Mnemonics
SSS has filled the form with SAS and AAS.
Concept: Congruency of a triangle
Interpretation
Side Side Side SSS (3 sides are equal)
Side Angle Side SAS (2 sides and included angles)
Angle Angle Side AAS (2 angles and corresponding side)

SUBJECTIVE TYPE QUESTIONS


Very Short Answer Type
Questions (1 mark each)
Q. 1. Write ASA congruence rule for two triangles.
R [Board Term I, 2015]
Sol. ASA congruence : Two triangles are congruent, if Therefore AC = DF. 1
two angles and the included side of one triangle are Q. 3. What do we call a triangle if the angles are in the
equal to two angles and the included side of other ratio 5 : 3 : 7 ? U
triangle. 1 Sol. Let the angles of triangle are 5x, 3x and 7x, then
Q. 2. In DABC and DDEF, AB = DE, ÐA = ÐD. What will 5x + 3x + 7x = 180°
be the condition in which the two triangles will be or, 15x = 180°
congruent by SAS axiom ? R Thus, x = 12°
Sol. Since AB = DE, ÐA = ÐD and DABC @ DDEF by \ Angles are 60°, 36°, 84°
SAS. ∵ Each angle is less than 90°
\ The triangle is an acute angled triangle. 1
TRIANGLES 97

It can also be a scalene triangle, since all angles are
\
ÐABD = ÐACD (CPCT) ½
of different measure. Hence, corresponding sides
Hence Proved
are unequal. Q. 2. In the figure below, the diagonal AC of quadrilateral
Q. 4. DABC @ DPQR, AB = PQ. Which statement has ABCD bisects ÐBAD and ÐBCD. Prove that BC =
been followed in this ? A CD. A [Board Term I, 2011, 2010]
Sol. If two triangles are congruent, then one side of a
triangle is equal to the corresponding side of the
other triangle.
Hence, AB = PQ 1
Q. 5. In the figure, if AB = DC, ÐABD = ÐCDB, which
congruence rule would you apply to prove DABD
@ DCDB ? R Sol. In DADC and DABC,
AC bisects ÐA and ÐC
So ÐDAC = ÐBAC ½
ÐDCA = ÐBCA (Given) ½
AC = AC (Common)
Hence, DADC @ DABC (By ASA rule) ½
\ CD = BC.  (CPCT) ½
Hence Proved.
Sol. AB = DC (Given)
ÐABD = ÐCDB (Given)
BD = BD (Common) Commonly Made Error
By SAS. DABD @ DCDB. Hence Proved 1
Students make an error in putting the condition
Short Answer Type AAS or ASA rule as both denote angle side or
Questions-I (2 marks each) angle side angle.
Q. 1. In the figure, DABC and DDBC are two isosceles
triangles on the same base BC. Prove that ÐABD =
ÐACD. A [Board Term I, 2014]
Answering Tip

ASA congruence rule states that two angles and


included side of one triangle are equal and AAS
congruence rule states that if any two pairs of angles
and one pair of corresponding sides are equal.
Q. 3. In the figure, OA = OB and OD = OC. Show that :
(i) DAOD @ DBOC, (ii) AD || BC.

Sol.

U [Board Term I, 2011]


Sol. (i) In DAOD and DBOC,
OA = OB (Given)
OD = OC (Given)
ÐAOD = ÐBOC
(Vertically opposite angles)
Join AD. So, by SAS criteria,
In DABD and DACD, DAOD @ DBOC 1
(ii) ÐCBO = ÐDAO (CPCT)
AB = AC (Given)
AD and BC are two lines intersected by AB and
BD = CD (Given) ÐCBA = ÐDAB (Proved above)
AD = AD (Common) 1 and they form a pair of alternate angles.
By using SSS Congruence Rule, \
AD || BC. Hence Proved 1
DABD @ DACD ½
98 Oswaal CBSE Question Bank Chapterwise & Topicwise, MATHEMATICS, Class-IX

Q. 4. In figure ÐB = ÐE, BD = CE and Ð1 = Ð2. Show


DABC @ DAED. U [Board Term I, 2011] Sol. In DBMR and DDNR,
BM = DN (Given)
\ ÐN = ÐM (each right angle) 1
ÐNDR = ÐMBR (Alternate angle)
\ DBMR @ DDNR (ASA Congruency)
BR = DR
or, R is mid-point of BD 1
\ AC bisects BD. Hence Proved 1
[CBSE Marking Scheme, 2016]

Sol. Let, ÐDAC be Ð3


Ð1 = Ð2 (Given) ½ Commonly Made Error
By adding Ð3 on both sides
Students get confused between the condition of
Ð1 + Ð3 = Ð2 + Ð3
congruency AAS and ASA and put the incorrect
ÐBAC = ÐEAD ...(i) ½ condition.
Given that, BD = CE
By adding DC on both sides
BD + DC = CE + DC ½ Answering Tip
or, BC = DE ...(ii)
ÐB = ÐE (Given) ...(iii) To prove the triangle students should be clear
From (i), (ii) and (iii), we get in mind about all the properties of triangles and
DABC @ DAED. (By AAS rule) ½ which condition of congruency will be applied
Hence Proved in which situation.

Q. 3. In the given figure AD = BC and BD = AC. Prove


Short Answer Type
that, ÐADB = ÐBCA and ÐDAB = ÐCBA.
Questions-II (3 marks each)
Q. 1. In the figure, if AF = CD and ÐAFE = ÐCDE, prove
that EF = ED.

A [Board Term I, 2014, 2012]


Sol. Proof :
In DABD and DBAC,
AD = BC (Given)
U [Board Term I, 2016] BD = AC (Given)
AB = AB (Common side) 2
Sol. In DAFE and DCDE,
DABD @ DBAC By SSS congruence
AF = CD (Given)
or, ÐADB = ÐBCA (CPCT) 1
\ ÐAFE = ÐCDE (Given)
ÐE = ÐE (Common) and ÐDAB = ÐCBA (CPCT)
\ ÐFAE = ÐDCE (Third angle of D)  Hence Proved.
\ DAFE @ DCDE. (ASA) Q. 4. In figure, PQRS is a square and SRT is an
Hence EF = ED (CPCT) Proved. equilateral triangle. Prove that :
[CBSE Marking Scheme, 2016] 3 (i) PT = QT
Q. 2. In the figure, BM and DN are both perpendicular to (ii) ÐTQR = 15° U [Board Term I, 2014; 2012, 2011, 2010]
AC and BM = DN. Prove that AC bisects BD.

U [Board Term I, 2016]


TRIANGLES 99
Sol. PQRS is a square.  (Given) A

B D E C

Sol. Step I : Read the question carefully and write the


given conditions.

(i) SRT is an equilateral triangle.(Given)


\ ÐPSR = 90°, ÐTSR = 60° ½
or, ÐPSR + ÐTSR = 150°.
Similarly, ÐQRT = 150°
In DPST and DQRT, we have PS = QR
ÐPST = ÐQRT = 150°
DABC is an isosceles triangle in which
and ST = RT ½
AB = AC ...(i)
By SAS, DPST @ DQRT D and E are points on BC such that
or, PT = QT  (CPCT) BE = CD ...(ii)
Hence Proved. Step II : Apply the theorems related to given
(ii) In DTQR, QR = RT conditions to find other information.
(Square and equilateral D on same base) ½ Since, AB = AC
or, ÐTQR = ÐQTR = x ½ So, ÐB = ÐC ...(iii)
[Property of isosceles triangle]
\ x + x + ÐQRT = 180°
From eqn. (ii) we have BE = CD
or, 2x + 150° = 180° Þ BE – DE = CD – DE
or, 2x = 30° [Subtracting DE from both sides]
\ x = 15°. Þ BD = CE ...(iv) 1
or, ÐTQR = 15°. Hence Proved. 1 Step III : Apply the suitable congruence rule in two
Q. 5. In a right angled triangle, if one acute angle is triangles.
double the other, then prove that the hypotenuse In DABD and DACE, AB = AC
is double the smallest side. A [Board Term I, 2016] ÐB = ÐC [From eqn. (iii)]
and BD = CE [From eqn. (iv)]
Sol. DABC is right angled at B. \ DABD @ DACE
(By SAS congruence rule) 1
Step IV : Apply the property CPCT i.e., corresponding
part of congruence triangles, to get the required result.
As DABD @ DACE
Then AD = AE (CPCT) 1

Long Answer Type


Let ÐCAB = x and ÐACB = 2x produce CB to D so Questions (5 marks each)
that BD = BC, join AD 1 Q. 1. In the given figure, if AC = BC, ÐDCA = ÐECB
In DABC and DABD, we have and ÐDBC = ÐEAC, then prove that BD = AE.
AB = AB (Common)
BC = BD (By Construction)
and ÐABC = ÐABD = 90° 1
\ DABC @ DABD (SAS)
\ AC = AD and ÐCAB = ÐDAB (CPCT)
Now, ÐCAD = x + x = 2x° = ÐACB
Then AD = CD
or AD = 2BC
or AC = 2BC 1
\ Hypotenuse AC is double the smallest side BC.
Hence Proved [CBSE Marking Scheme, 2016] U [Board Term I, 2014; 2012]
Q. 6. In an isosceles triangle ABC with AB = AC,
D and E are points on BC such that BE = CD (as Sol. Given, ÐDCA = ÐECB
given figure). Show that AD = AE Adding ÐDCE to both sides, we get
ÐDCA + ÐDCE = ÐECB + ÐDCE
U + A [KVS 2019]
or, ÐECA = ÐDCB 1½
100 Oswaal CBSE Question Bank Chapterwise & Topicwise, MATHEMATICS, Class-IX

(ii) DM || BC
In DACE and DBCD, \ ÐACB = ÐADM
AC = BC (Given)  (Corresponding angles)
ÐECA = ÐDCB (Proved)
Hence, MD ^ AC 1
ÐEAC = ÐDBC (Given)
(iii) In DADM and DDMC
\ DACE @ DBCD (By ASA cong.) 2
ÐADM = ÐMDC = 90° (Proved)
\ BD = AE. (CPCT) Proved. 1½
AD = DC(Proved)
[CBSE Marking Scheme, 2012]
MD = MD (Common)
Q. 2. ABC is a triangle right angled at C. A line through \ DADM @ DCDM (SAS Congruence rule)
the mid-point M of hypotenuse AB and parallel to \ AM = CM (CPCT)
BC intersects AC at D. Show that 1
(i) D is the mid-point of AC \ CM = AM = AB 2
2 
(ii) MD ^ AC
 Hence Proved.
1
(iii) CM = MA = AB U [KVS 2019] Q. 3. In the figure, OA = OB, OC = OD and
2 ÐAOB = ÐCOD. Prove that AC = BD.
Sol. Given : ABC is a triangle right angle at C and M is
the mid point of side AB.
A

D M

U [Board Term I, 2016]


Sol. ÐAOB = ÐCOD (Given)
C B
To prove ÐAOB – ÐCOB = ÐCOD – ÐCOB 1
(i) D is the mid point of AC ÐAOC = ÐBOD
(ii) MD ^ AC In DAOC and DBOD 1
1 AO = OB (Given) ½
(iii) CM = MA = AB
2 OC = OD (Given) ½
Proof (i) M is the mid point of side AB ÐAOC = ÐBOD (Proved above)
and MD || BC
\ DAOC @ DBOD (SAS Criterion) 1
\ D will be mid point of side AC (by converse of
AC = BD (CPCT) Hence Proved 1
mid point theorem)  2

Topic-2 Properties of Triangles

Revision Notes
 A triangle is isosceles if its any two sides are equal. Here, we will discuss some properties related to isosceles triangle.
(i) Angles opposite to equal sides of a triangle are equal.
In figure, ÐB = ÐC

Scan to know
more about
this topic

(ii) The sides opposite to equal angles of a triangle are equal. Properties of
In figure, AB = AC Triangles
 In an isosceles triangle, bisector of the vertical angle of a triangle bisect the base.
 The medians of an equilateral triangle are equal in length.
 A point equidistant from two intersecting lines lies on the bisector of the angles formed by the two lines.
TRIANGLES 101

Example 2
AB is a line segment. C and D are points on \ DCAD º DCBD
opposite side of AB such that each of them is [By SSS Congruence rule]
equidistant from the points A and B. Show that or, ÐACD = ÐBCD [By CPCT]
line CD is the perpendicular bisector of AB. or ÐACO = ÐBCO ...(iii)
Step IV : Show that DCAO and DCBO are congruent
and further use CPCT to find relation between
angles and sides.
In DCAO and DCBO
CA = CB [from eqn. (i)]
ÐACO = ÐBCO [from eqn. (iii)]
and CO = CO [Common side]
\ DCAO º DCBO
[By SAS Congruence rule]
or, AO = BO ...(iv) [By CPCT]
and ÐAOC = ÐBOC ...(v) [By CPCT]
Solution: Step V : Since, AB is a line segment, so we use the
Step I : Read the question carefully and write all property of linear pair and find the measure of
given conditions. ÐAOC or ÐBOC.
AB is a line segment. C and D are points on opposite AB is a line segment. So,
sides of AB such that ÐAOC + ÐBOC= 180°
CA = CB ...(i) or, ÐAOC + ÐAOC= 180° [from eqn. (v)]
and DA = DB ...(ii) or, 2ÐAOC = 180°
CD intersects AB at point O. 180°
Step II : Find what is given to show. or, ÐAOC =
CD is the perpendicular bisector of AB. 2
Step III : Show that DCAD and DCBD are congruent, or, ÐAOC = 90°
further use CPCT to find relation between angles. AO= BO [From eqn. (iv)]
In DCAD and DCBD ÐBOC = ÐAOC = 90° each
CA = CB [from eqn. (i)] Hence, CD is the perpendicular bisector of AB.
AD = BD [from eqn. (ii)]
and CD = CD [Common side]

SUBJECTIVE TYPE QUESTIONS


Very Short Answer Type
Questions (1 mark each)
Q. 1. DABC is an isosceles right angled triangle in which
ÐA = 90°. Calculate ÐB. U
Sol. AB = AC (given)
\ ÐC = ÐB PE = PE (Common)
(Angle opposite to equal side) \ DPEQ @ DPER (by ASA)
\
PQ = PR. (CPCT) 1
Hence Proved.

Short Answer Type


Questions-I (2 marks each)
Q. 1. ABC is an isosceles triangle with AB = AC. Draw
ÐA + ÐB + ÐC = 180° AP ^ BC. Show that ÐB = ÐC.
[Angle sum property] A [Board Term I, 2011]
or, 90° + ÐB + ÐB = 180° Sol.
or, 2ÐB = 90°
\ ÐB = 45° 1
Q. 2. In DPQR, PE is the perpendicular bisector of
ÐQPR, then prove that PQ = PR. U
Sol. In DPEQ and DPER,
ÐPEQ = ÐPER = 90°
ÐQPE = ÐRPE (Given) In DABP and DACP,
AB = AC (Given) ½
102 Oswaal CBSE Question Bank Chapterwise & Topicwise, MATHEMATICS, Class-IX

AP = AP (Common) or, ÐA = ÐB ....(2) ½


ÐAPB = ÐAPC = 90°, (AP ^ BC) ½ (Angles opp. to equal sides)
\
DABP @ DACP (By RHS rule) ½ From (1) and (2), we have
ÐA = ÐB = ÐC ½
or, ÐB = ÐC. (CPCT) Hence Proved ½ Also, ÐA + ÐB + ÐC = 180°, (Angle sum property)
Q. 2. PS is an altitude of an isosceles triangle PQR in \ ÐA + ÐA + ÐA = 180°
which PQ = PR. Show that PS bisects ÐP.
or, 3ÐA = 180° or, ÐA = 60°
A [Board Term I, 2011]
\ ÐA = ÐB = ÐC = 60°. 1
Sol. In DPQS and DPRS,
Thus, each angle of an equilateral triangle is 60°.
Hence Proved.
Q. 3. Triangle ABC is an isosceles triangle such that AB
= AC. Side BA is produced to D, such that AD =
AB. Show that ÐBCD is a right angle.

PQ = PR (Given) ½
PS = PS (Common)
ÐPSQ = ÐPSR = 90°
(PS is altitude) ½
By RHS rule,
DPQS @ DPRS ½
or, ÐQPS = ÐRPS (CPCT) ½ A [NCERT, Board Term I, 2014, 2012]
\ PS bisects ÐP. Hence Proved
Sol.
Short Answer Type
Questions-II (3 marks each)
Q. 1. In a triangle ABC, X and Y are the points on AB and
1 1
BC respectively. If BX = AB and BY = BC
2 2
and AB = BC. Show that BX = BY.
A [Board Term-I, 2015] In DABC, AB = AC or, Ð1 = Ð2 ...(1)
Angles opp. to equal sides are equal.
Sol. In DADC, AB = AD
\ AC = AD ½
In DBCD,
Ð3 = Ð4 ...(2) ½
Ð1 + (Ð2 + Ð3) + Ð4 = 180° ½
or, Ð2 + Ð2 + Ð3 + Ð3 = 180° ½
or, 2(Ð2 + Ð3) = 180°
1 or, Ð2 + Ð3 = 90° ½
AB = BC (Given) or, ÐBCD is a right angle. Hence Proved ½
1 1 [CBSE Marking Scheme, 2014, 2012]
or, AB = BC
2 2 Q. 4. In the figure, ABC is an isosceles triangle in which
1 AB = AC and LM is parallel to BC. If ÐA = 50°,
But AB = BX ...(i) ½
2 find ÐLMC.
But
1
BC = BY ...(ii) ½
2
From (i) and (ii), BX = BY Hence Proved 1
[CBSE Marking Scheme, 2011, 2015]
Q. 2. Prove that each angle of an equilateral triangle is
60°. U [Board Term I, 2012, NCERT]
Sol. Let DABC be an equilateral triangle, so that AB =
U [Board Term I, 2016]
AC = BC.
Sol. In DABC,
Now, AB = AC ½
AB = AC ½
or, ÐB = ÐC ....(1) ½ \ ÐABC = ÐACB = q
(Angles opp. to equal sides) or, ÐB = ÐC = q

CB = CA ÐA + ÐB + ÐC = 180°
TRIANGLES 103
50° + q + q = 180° AB = AC (Given) 1
2q = 180° – 50° ½
= 130° \ ÐABC = ÐACB
q = 65° (Angles opp. to equal sides)
\ ÐB = ÐC = 65° ÐABC + ÐACB = 137°, (ext. angle)
LM || BC (given) 1 137ϒ
\ ÐLMC + ÐBCM = 180° \ ÐABC = ÐACB = = 68.5° 1
2
[∵ ÐLMC and ÐBCM are co-interior angles]
ÐLMC + 65° = 180° CH = CB (Given)
ÐLMC = 180° – 65° or, ÐCBA = ÐCHB = 68.5° 1
ÐLMC = 115° 1 \ ÐHCB = 180° – 137° = 43° 1
Q. 5. In the given figure, AB = AC and BE and CF are ÐCHK = ÐHCB = 43°.
bisectors of ÐB and ÐC respectively. Prove that (Alternate angles) 1
DEBC @ DFCB. [CBSE Marking Scheme, 2013, 2012]
Q. 2. In the given figure, ABCD and BPQ are straight
lines. If BP = BC and DQ is parallel to CP prove
that :
(i) CP = CD
(ii) DP bisects ÐCDQ

U [Board Term I, 2016]

Sol. AB = AC (Given)
\ ÐABC = ÐACB ...(i)
BE and CF are the bisectors of ÐB and ÐC
1 1 U [Board Term I, 2016]
\ ÐEBC = ∠ ABC = ∠ACB
2 2 Sol. BP = BC (Given)
= ÐFCB \ ÐBCP = ÐBPC = y
ÐEBC = ÐFCB ...(ii) (Angle opp. to equal sides) 1
In DBEC and DCFB
\
Ð1 = y (Corresponding angles)

ÐABC = ÐACB

\ ÐECB = ÐFBC

ÐEBC = ÐFCB (Proved)

BC = BC (Common)

\ DBEC @ DCFB (By ASA)

DEBC @ DFCB Hence Proved 3
[CBSE Marking Scheme, 2016]

Long Answer Type


Questions (5 marks each)
Q. 1. In figure, AB = AC, CH = CB and HK||BC. If
ÐCAX = 137°, then find ÐCHK. 1
4x = 2y (external angle)1
U [Board Term I, 2013, 2012]
Ð2 = x (Alternate angles)
y = x + Ð3
4x
( 4x = 2y, = y)
2
2x = x + Ð3
or, x = Ð3 1
or, CP = CD
(side opp. to equal angles)
Also, we have Ð2 = x, x = Ð3 1
\ DP bisects ÐCDQ Hence Proved
[CBSE Marking Scheme 2016]
Q. 3. ABCD is a square and ABE is an equilateral triangle
Sol. ÐXAK + ÐKAH = 180° (Linear pair) 1
outside the square prove that ÐACE = ÐABE.
ÐKAH = 180° – 137° = 43° 2
(∵ ÐCAX = ÐXAK = 137°, Given)  U [Board Term I, 2016]
104 Oswaal CBSE Question Bank Chapterwise & Topicwise, MATHEMATICS, Class-IX

Sol. ∵ DABE is an equilateral triangle \


Ð1 + Ð2 = 180° – (60° + 90°)
\ ÐABE = ÐBEA = 180° – 150°
= ÐEAB = 60° 1 = 30° 1

or, 2Ð1 = 30°
or, Ð1 = 15°

or, Ð3 = 45° – 15°
= 30°
1 \
ÐACE = 30°,
ÐABE = 60°
1
Also ÐBAC = 45° \
ÐACE = ∠ABE Hence Proved 1
2
BE = BC
or, Ð1 = Ð2 1 [CBSE Marking Scheme, 2016]

OBJECTIVE TYPE QUESTIONS (1 mark each)

A F

A Multiple Choice Questions


Q. 1. Which of the following is not a criterion for
congruence of triangles?
(A) SAS (B) ASA
(C) SSA (D) SSS B C D E
 R [NCERT Exemp.] Therefore, both triangles are isosceles but not
Ans. Option (C) is correct. congruent. As we know that AAA is not a criterion
for congruence of triangles.
Explanation: Criterion for congruency are SAS (side
angle side), ASA (angle side angle), SSS (side side Q. 4. In triangles ABC and DEF, AB = FD and ÐA = ÐD.
side), AAS (angle-angle side). The two triangles will be congruent by SAS axiom
if
Q. 2. If AB = QR, BC = PR and CA = PQ, then
(A) BC = EF (B) AC = DE
(A) ∆ ABC ≅ ∆ PQR (B) ∆ CBA ≅ ∆ PRQ (C) AC = EF (D) BC = DE A
(C) ∆ BAC ≅ ∆ RPQ (D) ∆ PQR ≅ ∆ BCA Ans. Option (B) is correct.
 A [NCERT Exemp.] Explanation: Given, in ∆ABC and ∆DEF, AB = FD,
Ans. Option (B) is correct. ÐA = ÐD
Explanation: AB = QR & BC = PR Þ ∠B = ∠R ...(i) We know that, two triangles will be congruent
by SAS axiom if two sides and the included angle
BC = PR & CA = PQ Þ ∠C = ∠P …(ii)
between them are equal.
CA = PQ & AB = QR Þ ∠A = ∠Q …(iii)
From equations (i), (ii) and (iii),
we get ∆CBA ≅ ∆PRQ.
Hence, the option (b) is correct.
Q. 3. In triangles ABC and PQR, AB = AC, ∠C = ∠P and
∠B = ∠Q. The two triangles are
(A) isosceles but not congruent Q. 5. In ∆ABC, AB = AC and ÐB = 50°. Then ÐC is equal
(B) isosceles and congruent to
(C) congruent but not isosceles (A) 40° (B) 50°
(D) neither congruent nor isosceles (C) 80° (D) 130°
 A [NCERT Exemp.]  A [NCERT Exemp.]
Ans. Option (A) is correct. Ans. Option (B) is correct.
Explanation: In ∆ ABC, AB = AC [Given] Explanation : Given that AB = AC and ÐB = 50°
A
∠C = ∠B
[Angles opposite to equal sides are equal]
So, ∆ABC is an isosceles triangle
But it is given that
∠B = ∠Q
∠C = ∠P 50°
Therefore, ∠P = ∠Q [Since, ∠C = ∠B] B C
\ QR = PR In ∆ABC, AB =AC (given)
[Sides opposite to
ÐC = ÐB (angles opposites to equal sides are equal)
equal angles are equal]
So, ∆PQR is also an isosceles triangle. ÐC = 50°
TRIANGLES 105
Q. 1. Assertion (A): Each angle of an equilateral triangle
Q. 6. In ∆ABC, BC = AB and ÐB = 80°. Then ÐA is equal
is 60°.
to
Reason (R): Angles opposite to equal sides of a
(A) 80° (B) 40°
triangle are equal.
(C) 50° (D) 100° Ans. Option (A) is correct.
 A [NCERT Exemp.] Explanation: In case of assertion (A):
Ans. Option (C) is correct. We know that each angle of an equilateral triangle
is 60°.
Explanation : In ∆ABC, BC = AB, ÐB = 80°
\ Assertion is true.
ÐC =ÐA (angles opposite to equal sides are equal) In case of reason (R):
ÐA + ÐB +ÐC = 180° (sum of all angles of triangle) Angles opposite to equal sides are equal.
ÐA+80°+ÐA = 180° \ Reason is true.
2ÐA = 180° – 80° Hence, Both A and R are true and R is the correct
100° explanation of A.
∠A =
2 Q. 2. Assertion (A): In DABC and DPQR, AB = PQ, AC =
   ∠A = 50° PR and ÐBAC = ÐQPR, DABC @ DPQR.
Reason (R): Both the triangles are congruent by SSS
Q. 7. It is given that ∆ ABC @ ∆ FDE and AB = 5 cm, congruence.
ÐB = 40° and ÐA = 80°. Then which of the Ans. Option (C) is correct.
following is true? Explanation: In case of assertion (A):
(A) DF = 5 cm, ÐF = 60°
(B) DF = 5 cm, ÐE = 60°
(C) DE = 5 cm, ÐE = 60°
(D) DE = 5 cm, ÐD = 40° A [NCERT Exemp.]
Ans. Option (B) is correct.
Explanation : Given, ∆ ABC @ ∆ FDE and AB = 5 cm, In DABC and DPQR,
ÐB = 40°, ÐA = 80° AB = PQ (given)
Since, ∆FDE @ ∆ ABC AC = PR (given)
A F ÐBAC = ÐQPR
\ DABC @ DPQR (By SAS Rule)
80°
\ Assertion is true.
In case of reason (R):
The reason is false as the triangles are congruent by
40°
B C D E SAS and not SSS.
∴ DF = AB [by CPCT] Q. 3. Assertion (A): DABC and DDBC are two isosceles
DF = 5 cm triangles on the same base BC. Then ÐABD = ÐACD.
and ÐE = ÐC [by CPCT]
ÐE = ÐC = 180° – (ÐA + ÐB)
[By angle sum property of a ΔABC]
ÐE = 180° – (80° + 40°)
ÐE = 60°
 eason (R): The angles opposite to equal sides of a
R
B Assertion & Reason triangle are equal.
Ans. Option (A) is correct.
Directions: In the following questions, a statement
Explanation: In case of assertion (A):
of assertion (A) is followed by a statement of AB = AC (given)
reason (R). Mark the correct choice as: Þ ÐABC = ÐACB ...(i)
(A) Both assertion (A) and reason (R) are true and DC = DB (given)
Þ ÐDBC = ÐDCB ...(ii)
reason (R) is the correct explanation of assertion (A).
Subtracting (ii) from (i)
(B) Both assertion (A) and reason (R) are true but reason Þ ÐABC – ÐDBC = ÐACB – ÐDCB
(R) is not the correct explanation of assertion (A). \ ÐABD = ÐACD
(C) Assertion (A) is true but reason (R) is false. In case of reason (R):
(D) Assertion (A) is false but reason (R) is true. We know that the angles opposite to equal sides of
a triangle are equal.
106 Oswaal CBSE Question Bank Chapterwise & Topicwise, MATHEMATICS, Class-IX

Hence, Both A and R are true and R is the correct Explanation: In case of assertion (A):
Explanation of A. In DABD, AB + BD > AD ...(i)
Q. 4. Assertion (A): If AD is a median of DABC, then Similarly in DADC,
AB + AC + BC > 2AD. AC + CD > AD ...(ii)
Adding (i) and (ii), we get
AB + AC + BD + CD > AD + AD
Þ AB + AC + BD + DC > 2AD
Þ AB + AC + BC > 2AD
In case of reason (R):
We know that the sum of two sides is greater than
the third side.
Reason (R): In a triangle the sum of two sides is Hence, Both A and R are true and R is the correct
greater than the third side. Explanation of A.
Ans. Option (A) is correct.

COMPETENCY BASED QUESTIONS (4 marks each)

Ans. Option (B) is correct.


A Case based MCQs Explanation: An isosceles D has 2 sides equal 1
Read the following passage and answer any four Q. 4. If PQ = 6 cm and QR = 7 cm, then perimeter of
questions of the following : DPQR is
I. A children‘s park is in the shape of isosceles triangle (A) 19 cm (B) 20 cm
say PQR with PQ = PR, S and T are points on QR (C) 13 cm (D) 18 cm
such that QT = RS. Ans. Option (A) is correct.
P Explanation: Perimeter = sum of all 3 sides 1
PQ = PR = 6 cm,
QR = 7 cm
So P = (6 + 6 + 7) cm
= 19 cm 1
Q S T R Q. 5. If ÐQPR = 80°, find ÐPQR.
Q. 1. Which rule is applied to prove that congruency of (A) 20° (B) 100°
DPQS and DPRT. (C) 50° (D) 40°
Ans. Option (C) is correct.
(A) SSS (B) SAS
Explanation: let ÐQ = ÐR = x and ÐP = 80°
(C) AAS (D) RHS
In DPQR, ÐP + ÐQ + ÐR = 180°
Ans. Option (B) is correct.
(Angle sum property of D)
Explanation: In DPQS and DPRT 80° + x + x = 180°
PQ = PR  (Given) 2x = 180° – 80
QS = TR  (Given) 2x = 100°
ÐPQR =ÐPRQ
100°
(corresponding angles x=
2
of an isosceles D)
By SAS congurency = 50° 1
DPQS @ DPRT II. Neeraj has a plot in the shape of a triangle said ABC
Q. 2. In RHS rule ‘H‘ stands for with AD as the perpendicular bisector of BC such
(A) Height (B) Hypotenuse
that BD = DC.
(C) Heron‘s formula (D) Highest
Ans. Option (B) is correct.

Explanation: ‘H‘ stands for hypotenuse 1
Q. 3. An isosceles triangle has 
(A) 3 sides equal
(B) 2 sides equal
(C) None of these sides equal
(D) All angles equal
TRIANGLES 107
Q. 1. Which rule is applied to prove the congruency of Ans. Option (C) is correct.
DABD and DACD? Explanation: As DABD @ DACD
(A) RHS (B) SAS \ AB = AC
(C) SSS (D) AAS (Corresponding parts of congruent triangles)
Ans. Option (B) is correct. AC = 10 cm
Explanation: In DABD and DACD BC = BD + DC
BD = DC (given) Since BD = DC = 6 cm (given)
ÐADB = ÐADC = 90° (AD is altitude) \ BC = (6 + 6) cm = 12 cm
AD = AD (common) Perimeter of DABC = AB + BC + AC
\ DABD @ DACD (by SAS) = (10 + 12 + 10) cm
Q. 2. If ÐABD = 50°, then the value of ÐBAD. = 32 cm
(A) 60° (B) 50°
(C) 40° (D) 100° B Case based Subjective
Ans. Option (C) is correct. Questions
Explanation: Given ÐABD = 50° and ÐADB = 90°
Read the following passage and answer the
In DADB
following questions:
ÐBAD + ÐABD + ÐADB = 180°
(angle sum property)
ÐBAD + 50° + 90° = 180°
ÐBAD + 140° = 180°
ÐBAD = 180° – 140°
ÐBAD = 40°
Q. 3. In DADC find ÐACD.
I. As shown above:
(A) 55° (B) 20°
In Rajesh Village there was a big pole PC. This pole
(C) 60° (D) 50°
was tied with a strong wire of 10 m length. Once
Ans. Option (D) is correct.
there was a big spark on this pole, thus wires got
Explanation: ÐBAD = ÐCAD = 40°
damaged very badly. Any small fault was usually
(AD is the perpendicular bisector)
repaired with the help of a rope which normal
In DADC
board electricians were carrying on bicycles. This
ÐACD + ÐADC + ÐCAD = 180°
time electricians need a staircase of 10 m, so that
(angle sum property)
ÐACD + 90° + 40° = 180° it can reach at point P on the pole and this should
ÐACD + 130° = 180° make 60° with line AC.
ÐACD = 180° – 130° = 50° Q. 1. In DPAC and DPBC which side is common?
Ans. As shown in figure side PC is common. 1
Q. 4. If AB = 10 cm and BD = 6 cm then find perimeter
Q. 2. Find the value of Ðx?
of DABD.
(A) 50 cm (B) 24 cm Ans. In DPAC
ÐA = 60°, ÐPCA = 90° (given)
(C) 30 cm (D) 20 cm
ÐAPC + ÐPAC + ÐPCA = 180°
Ans. Option (B) is correct.
(Angle sum property)
Explanation: DABD is a right angled triangle
AB = 10 cm, BD = 6 cm. Þ x + 60° + 90° = 180°
Using Pythagoras Theorem x = 30° 1
(AB)2 = (AD)2 + (BD)2 Q. 3. In figure, DPAC and DPBC are congruent due to
(10)2 = (AD)2 + (6)2 which criteria?
100 = (AD)2 + 36 Ans. In DPAC and DPBC 2
(AD)2 = 100 – 36 = 64 PC = PC (Common)
AD = 8 cm ÐPCA = ÐPCB = 90°
Now, perimeter = AB + AD + BD AP = BP (given)
= (10 + 8 + 6) cm ( Length of Rope = Length of Stairs)
= 24 cm \ DPAC @ DPBC (By RHS case)
Q. 5. Find perimeter of DABC Hence, DPAC and DPBC are congruent due to 'RHS'
(A) 25 cm (B) 50 cm criteria.
(C) 32 cm (D) 24 cm
108 Oswaal CBSE Question Bank Chapterwise & Topicwise, MATHEMATICS, Class-IX

II. Raj is having a quadrilateral open space in his plot. 50° + 95° + ÐDCA = 180°
He divided the land into two parts by drawing the So, ÐDCA = 35° 1
boundary AC and AD = BC. ÐDAC = ÐBCA = 50° Q. 2. Find ÐBAC and ÐABC?
and ÐADC = 95° Ans. ÐDCA = ÐBAC (CPCT) ½
\ ÐBAC = 35°
ÐADC = ÐABC = 95° (CPCT)
\ ÐABC = 95° ½
Q. 3. Which rule is applied to prove the congruency? Is
AB = CD?
Ans. In DABC and DADC
Q. 1. In DACD find ÐDCA. BC = AD (given)
Ans. ÐDAC = 50° and ÐADC = 95° (given) ÐDAC = ÐBCA (given)
In DACD AC = AC (common) 1½
ÐDAC + ÐADC + ÐDCA = 180° (a) \ DABC @ DACD (SAS Criteria)
(Angle sum property) (b) AB = CD (CPCT) ½

  
Study Time:
Max. Time: 2:30 Hrs
Max. Questions: 35
CHAPTER

8 QUADRILATERALS

((Prove) The diagonal divides a parallelogram into two congruent triangles.


Syllabus (Motivate) In a parallelogram opposite sides are equal, and parallel and conversely.
(Motivate) In a parallelogram opposite angles are equal, and conversely.
(Motivate) A quadrilateral is a parallelogram if a pair of its opposite sides is parallel and equal.
(Motivate) In a parallelogram, the diagonals bisect each other and conversely.
(Motivate) In a triangle, the line segment joining the mid-points of any two sides is parallel to the third side and is half
of it and (motivate) its converse.

List of Topics
Topic-1: Properties of a Parallelo-
gram Page No. 109
Topic-2 : Mid-point Theorem
Topic-1 Properties of a Parallelogram  Page No. 114

Revision Notes
� Opposite sides of a parallelogram are parallel.
� Opposite sides of a parallelogram are equal.
� Opposite angles of a parallelogram are equal. Scan to know
more about
� Consecutive angles (conjoined angles) of a parallelogram are supplementary. this topic
� A diagonal of a parallelogram divides it into two congruent triangles.
� Diagonals of a parallelogram bisect each other.
� If each pair of opposite sides of a quadrilateral is equal and parallel then it is a parallelogram.
Properties of a
� If in a quadrilateral each pair of opposite angles is equal, then it is a parallelogram. Parallelogram

� If the diagonals of a quadrilateral bisect each other, then it is a parallelogram.


� If one pair of opposite side is equal and parallel then it is a parallelogram.
¾¾ Square, rectangle and rhombus are all parallelograms.
¾¾ Kite and trapezium are not parallelograms.
¾¾ A square is a rectangle.
¾¾ A square is a rhombus.
¾¾ A parallelogram is a trapezium.
¾¾ Every rectangle is a parallelogram; therefore, it has all the properties of a parallelogram. Additional properties of
a rectangle are :
 All the interior angles of a rectangle are right angles.
 The diagonals of a rectangle are equal.
110 Oswaal CBSE Question Bank Chapterwise & Topicwise, MATHEMATICS, Class-IX
QUADRILATERALS 111
¾¾ Every rhombus is a parallelogram; therefore, it has all the properties of a parallelogram. Additional properties of a
rhombus are :
 All the sides of a rhombus are equal.
 Diagonals of a rhombus bisect at right angles.
¾¾ Every square is a parallelogram; therefore, it has all the properties of a parallelogram. Additional properties of a
square are :
 All sides are equal.
 All angles are equal to 90°.
 Diagonals are equal.
 Diagonals bisect each other at right angle.
 Diagonals bisect the angles of vertex.

SUBJECTIVE TYPE QUESTIONS


1 1
or, ∠ADC + ∠BCD = 90° ½
Very Short Answer Type 2 2
Questions (1 mark each) or ∠1 + ∠2 = 90°
Q. 1. Two consecutive angles of a parallelogram are in the
ratio 1 : 3, then what will be the smaller angles ?

U [Board Term II, 2011]


Sol. Let the consecutive angles be x and 3x. ½
\ x + 3x = 180°
( consecutive angles are supplementary) In ∆ODC,
or, 4x = 180° ∠1 + ∠2 + ∠DOC = 180°
or, x = 45° ∴ ∠DOC = 90° Hence Proved [1]
\ Smaller angle = x Q. 2. The angle between the two altitudes of a
= 45° ½ parallelogram through the vertex of an obtuse
angle is 50°. Find the angles of a parallelogram.
Q. 2. Diagonals AC and BD of a parallelogram ABCD
intersect each other at O. If OA = 3 cm and OD = U [Board Term II, 2011]
2 cm, determine the lengths of AC and BD. Sol. AM ⊥ DC, AN ⊥ BC
 [NCERT Exemp. Ex. 8.2, Q. 1, Page 75] In quadrilateral AMCN,
∠MAN + ∠M + ∠C + ∠N = 360°
Sol. We know that the diagonals of a parallelogram
or ∠MAN + 90° + ∠C + 90° = 360°
bisect each other.
∴ ∠MAN + ∠C = 180°
Therefore,
or, 50° + ∠C = 180° or, ∠C = 130° 1
AC = 2 × OA = 2 × 3 cm = 6 cm
BD = 2 × OD = 2 × 2 cm = 4 cm
Therefore, AC = 6 cm and BD = 4 cm. [2×1=2]
Q. 3. Diagonals of a parallelogram are perpendicular to
each other. Is this statement true?
Give reason for your answer.
 [NCERT Exemp. Ex. 8.2, Q. 2, Page 75] In parallelogram, ∠A = ∠C = 130°
∠B = ∠D = 180° – 130°
Sol. This statement is not true. We know that the
= 50° 1
diagonals of a parallelogram bisect each other but
need not be perpendicular. [2×1=2] Q. 3. Two opposite angles of a parallelogram are (3x – 2)°
and (63 – 2x)°. Find all the angles of a parallelogram.
Short Answer Type  U [Board Term II, 2011]
Questions-I (2 marks each) Sol. Since, opposite angles of a parallelogram are equal.
Q. 1. In a parallelogram, show that the angle bisectors of ∴ 3x – 2 = 63 – 2x or 5x = 65
two adjacent angles intersect at right angle. or, x = 13° 1
 U [Board Term II, 2011] Angles of the parallelogram are :
Sol. ∠ADC + ∠BCD = 180° ½ (39 – 2)°, (180 – 37)°, (63 – 26)° and (180 – 37)°
( consecutive angles are supplementary) i.e., 37°, 143°, 37° and 143°. 1
112 Oswaal CBSE Question Bank Chapterwise & Topicwise, MATHEMATICS, Class-IX

Q. 4. In a parallelogram ABCD if ∠A = 115°, find ∠B,


∠C and ∠D. U [KVS, 2019] Answering Tip
Sol. D C
Students should learn the properties of
parallelogram thoroughly.
Q. 2. PQRS is a parallelogram and PL and RM are
perpendiculars drawn from the vertices P and R of
the parallelogram on diagonal SQ. Show that
115° (i) ∆PQL @ ∆RMS (ii) PL = RM
U [KVS 2014, NCERT]
A B
Sol. (i) In ∆RSM and ∆PQL,

In a||gm,

Consecutive angles are supplementary
\
∠A + ∠B = 180°
115° + ∠B = 180° ½
∠B = 180° – 115°
\
∠B = 65°
∠A = ∠C = 115° 1

 (Opposite angles are equal in ||gm) ∠RSM = ∠PQL
∠B = ∠D = 65° 1  (Alternate interior angles)
∠M = ∠L  (90° each)
Short Answer Type
SR = PQ 1
Questions-II (3 marks each)  (Parallel sides of parallelogram are equal)
Q. 1. Diagonal AC of a parallelogram ABCD bisects ∠A. By AAS, DRSM ≅ ∆PQL 1
Show that : (ii) PL = RM  (CPCT) ½
(i) it bisects ∠C also
Hence Proved
(ii) ABCD is a rhombus
U [Board Term II , 2012, NCERT] Q. 3. In the given parallelogram ABCD, two points
P and Q are taken on the diagonal BD such that
DP = BQ. Show that :

Sol.

(i) AB || CD, AD || BC
AC is transversal
So, Ð1 = Ð3 ...(i) (i) ∆APD ≅ ∆CQB
Ð2 = Ð4 (Alt. interior angles)
(ii) ∆AQB ≅ ∆CPD
But Ð1 = Ð2 ...(ii)
(Given, diagonal AC bisects ÐA) (iii) APCQ is a parallelogram A
Ð3 = Ð4 [Board Term II, 2012, NCERT]
(On comparing eqn. (i) and eqn. (ii))
Sol. (i) In ∆APD and ∆CQB
\ AC bisects ÐC, Proved 1½
(ii) As, Ð1 = Ð2 = Ð3 = Ð4 AD = BC (Opp. sides of a parallelogram)
Hence, Ð1 = Ð4 PD = QB  (Given)
\ AB = BC ∠ADP = ∠CBQ  (Alt. Angles)
(sides opp. to equal angles) 1
So, ∆APD ≅ ∆CQB (SAS Rule)
Hence, ABCD is a rhombus (in a parallelogram
if one pair of adjacent sides are equal then it is a or, AP = CQ (it is given) (CPCT) 1
rhombus). ½ (ii) In ∆AQB and ∆CPD
[CBSE Marking Scheme, 2012]
AB = DC (opp. sides of parallelogram)
BQ = DP (given)
Commonly Made Error and ∠ABQ = ∠PDC (Alt. angles)
∴ ∆AQB ≅ ∆CPD (SAS Rule)
Diagonal of a parallelogram bisects its vertex or, AQ = CP (CPCT) 1
angle. Students make a mistake that the (iii) In quad. APCQ,
two bisected angles are equal but it is not so, AP = CQ and AQ = CP (Proved above)
actually Ð1 ¹ Ð2, in case of rhombus but in case
of rectangle they are equal. \ APCQ is a || gm
Hence Proved. 1
QUADRILATERALS 113
∴ ABED is a parallelogram. Proved. 1
Long Answer Type
So, AD = BE and AD || BE. ...(i)
Questions (5 marks each)
(ii) In quadrilateral BCFE,
Q. 1. In the given figure, ABC is an isosceles triangle in BC = EF and BC || EF
which AB = AC, AD bisects the exterior angle PAC
i.e., one pair of opposite sides are equal and
and CD || AB. Show that :
parallel.
∴ BCFE is a parallelogram. Proved. 1
So, CF = BE and CF || BE. ....(ii)
(iii) From equations (i) and (ii), we get
AD = CF and AD || CF
\ ACFD is a parallelogram.
So, AC = DF and AC || DF Proved. 1
(iv) In ∆ABC and ∆DEF,
(i) ∠DAC = ∠BCA, and AB = DE (Given)
(ii) ABCD is a parallelogram. BC = EF (Given)
 U [Board Term II , 2012] [NCERT] and AC = DF (Proved above in part (iii))
Sol. (i) DABC is an isosceles triangle. So by SSS rule DABC ≅ DDEF. Hence Proved 1
So, ∠ABC = ∠BCA ½ Q. 3. ABCD is a trapezium in which AB || CD and
∠PAC = ∠ABC + ∠BCA ½ AD = BC, Show that :
(Q Sum of two interior opposite angles (i) ÐA = ÐB (ii) ÐC = ÐD
is equal to exterior angle) (iii) DABC @ DBAD
= 2∠BCA ...(i) 1
A [Board Term II 2012] [NCERT]
AD bisects ∠PAC
So, ∠PAC = 2∠DAC ...(ii)
From (i) and (ii), ∠BCA = ∠DAC 1
Sol.
(ii) ∠BCA = ∠DAC (Proved above)
These are alternate angles when lines BC and AD
are intersected by AC 1
\ BC || AD, (i) Through C draw CE || DA
Also, BA || CD (Given) \ AECD is a parallelogram
∴ ABCD is a parallelogram. Hence Proved 1 So, ÐA + Ð2 = 180° ...(i)
Q. 2. If DABC and DDEF are two triangles such that AB,
(Consecutive interior angles)
BC are respectively equal and parallel to DE, EF
then show that : AD = CE
( AECD is a ||gm)
(i) quadrilateral ABED is a parallelogram.
and AD = BC (Given)
(ii) quadrilateral BCFE is a parallelogram.
\ BC = CE
(iii) AC = DF Now in DBCE,
(iv) DABC ≅ DDEF. BC = CE (Proved above) 1
A [Board Term II, 2012] [NCERT] or, Ð1 = Ð2
Sol. Given : Two triangles ABC and DEF, such that Also, ÐABC + Ð1 = 180° (Linear Pair) ...(ii)
From (i) and (ii), ÐA = ÐB Hence Proved
(ii) Now, ÐA + ÐD = 180° ...(iii)
ÐB + ÐC = 180° ...(iv)
(Consecutive interior angles)
(on comparing eqn. (iii) and eqn. (iv))

ÐA + ÐD = ÐB + ÐC 1
1 So, ÐC = ÐD (Q ÐA = ÐB) 1
(iii) In D's ABC and BAD
AB = DE and AB || DE AB = BA (Common)
Also, BC = EF and BC || EF ÐB = ÐA (Proved above) 1
Proof : (i) In a quadrilateral ABED, BC = AD (Given)
So, DABC ≅ DBAD (By SAS) 1
AB = DE and AB || DE
Hence Proved
i.e., one pair of opposite sides are equal and
[CBSE Marking Scheme 2012]
parallel.
114 Oswaal CBSE Question Bank Chapterwise & Topicwise, MATHEMATICS, Class-IX

Topic-2 Mid-Point Theorem

Revision Notes
¾¾ Mid-Point Theorem : The line segment joining the mid-points of any two sides of a triangle is Scan to know
more about
parallel to the third side and equal to half of it. this topic
¾¾ Converse of mid-point theorem : The line drawn through the mid-point of one side of a triangle
parallel to the another side, bisects the third side.
¾¾ If there are three or more parallel lines and the intercepts made by them on a transversal are equal,
then the corresponding intercepts on any other transversal are also equal. Mid point

SUBJECTIVE TYPE QUESTIONS


Very Short Answer Type Short Answer Type
Questions (1 mark each) Questions-II (3 marks each)
Q. 1. In an equilateral triangle ABC, D and E are the Q. 1. ABCD is a quadrilateral in which P, Q, R and S are
mid-points of sides AB and AC respectively, then the mid-points of the sides AB, BC, CD and DA
respectively. Show that PQRS is a parallelogram.
find the length of DE.
A [NCERT][Board Term II , 2012]
R [Board Term II, 2011, NCERT] Sol. Join diagonal AC.
In ∆ADC,
Sol. S is the mid-point of AD.
R is the mid-point of DC.

Since, D and E are mid-points of sides AB and AC


1 ∴ SR || AC
respectively. So, by mid-point theorem, DE = BC.
2 1
and SR = AC (By mid-point theorem)
1 2
...(i) 1
Q. 2. D, E, F are the mid-points of sides BC, CA and AB
Similarly in ∆BAC,
of DABC. If perimeter of DABC is 12·8 cm, then
PQ || AC
find perimeter of DDEF. R [Board Term II, 2011]
1
A and PQ = AC ...(ii) 1
Sol. 2
From (i) and (ii),
SR || PQ
F E ½ and SR = PQ ½
∴ PQRS is a parallelogram. Hence Proved ½
Q. 2. D, E and F are the mid-points of sides PQ, QR and
C
RP respectively of an equilateral ∆PQR. Show that
B
D  ∆DEF is also an equilateral triangle.
Given, perimeter of DABC = 12·8 cm U [Board Term II, 2012]
1 Sol.
\ Perimeter of DDEF = (perimeter of DABC)
2
12·8
= cm
2
=6·4 cm ½
QUADRILATERALS 115

1
DE || PR and DE = PR
2
(By mid-point theorem)
1
Similarly, EF || PQ and EF = PQ
2
1
DF || QR and DF = QR 1½ Perimeter of DEF = DE + EF + DF 1
2
1
As PQ = QR = PR = (AC +AB + BC)
2
( DPQR is an equilateral triangle)
1 1
So, DE = EF = DF 1 = (7·8 + 6 + 7·2) = (21)
2 2
or, ∆DEF is an equilateral triangle. Hence Proved ½
= 10·5 cm 1
Q. 3. Prove that the quadrilateral formed by joining the
mid-points of the consecutive sides of a rectangle Q. 5. In the figure, ABCD is a parallelogram and E is the
is a rhombus.  A [Board Term II , 2012] mid-point of side BC. DE and AB on producing
meet at F. Prove that AF = 2AB.
OR
ABCD is a rectangle and P, Q, R and S are mid
points of the sides AB, BC, CD and DA respectively.
Show that the quadrilateral PQRS is a rhombus.
[NCERT]

U [Board Term II , 2012]


Sol. Sol. In ∆DCE and ∆FBE, ½
∠DCE = ∠FBE (alternate interior angles)
CE = BE (Given)
∠DEC = ∠BEF (VOA)
In DABC, P and Q are mid points of AB and BC \ ∆DCE ≅ ∆FBE (ASA)
respectively. or, DC = FB (CPCT) 1
1 DC = AB (opp. sides of ||gm)
\ PQ = AC and PQ || AC ... (i)
2 \ AB = FB ½
1 or, AF = AB + BF = AB + AB = 2AB
Similarly, RS = AC and RS || AC ...(ii) Hence Proved. 1
2
1 Long Answer Type
\ PQ || RS, PQ = RS = AC
2
Questions (5 marks each)
\ PQRS is a parallelogram 1
Q. 1. In the figure, ABCD is a parallelogram. E and F are
Also, AD= BC or, AS = BQ the mid-points of sides AB and CD respectively.
In DAPS and DBPQ, Show that the line segments AF and EC trisect the
AP = BP, AS = BQ and ÐA = ÐB = 90° 1 diagonal BD. U [Board Term II , 2013; 2012]
\ DAPS @ DBPQ (SAS Congnency)
Hence PS = PQ ...(iii) (CPCT)
\ PQRS is a parallelogram with PQ = PS
i.e., PQRS is a rhombus. Hence Proved. 1
[CBSE Marking Scheme 2012]
Q. 4. In ∆ABC, D, E and F are the mid-points of sides Sol. According to the question, E and F are the mid-
AB, BC and CA. If AB = 6 cm, BC = 7·2 cm and points of sides AB and CD.
AC = 7·8 cm, find the perimeter of ∆DEF. 1
R [Board Term II, 2012] ∴ AE = AB
2
1
Sol. DE = AC 1
2 and CF = CD 1½
2
1
EF = AB 1 In the parallelogram opposite sides are equal, so
2
AB = CD
1 1 1
DF = BC (By Mid-point theorem) AB = CD
2 2 2

116 Oswaal CBSE Question Bank Chapterwise & Topicwise, MATHEMATICS, Class-IX

∴ AE = CF AC = AF + FG + GC
Again, AB || CD = 3AF Hence Proved 2
1 1 Q. 3. Show that the line segments joining the mid-points
AB || CD
2 2 of the opposite sides of a quadrilateral bisect each
So, AE || FC 1 other. A [Board Term II, 2012] [NCERT]
Hence, AECF is a parallelogram. OR
In ∆ABP, P, Q, R, S are the mid-points of the sides AB,
E is the mid-point of AB and EQ || AP. BC, CD and DA respectively of the quadrilateral
∴ Q is the mid-point of BP. 1 ABCD. Prove that PR and QS bisect each other.
(By converse of mid-point theorem)  [Board Term II, 2012]
Similarly, P is the mid-point of DQ. Sol. Const. : Join SP, PQ, QR, RS and AC
\ DP = PQ = QB
∴ Line segments AF and EC trisect the diagonal
BD. Hence Proved 1½
Q. 2. In given fig., AD is the median of ∆ABC. E is the
mid-point of AD. DG || BF. Prove that AC = 3AF.

Proof : In ∆DAC,
RS || AC
1
and RS = AC (Mid-point theorem)
U [Board Term II, 2012] 2
Sol. In DADG, ...(i) 1
E is the mid-point of AD and EF || DG. In ∆ABC,
\ F is the mid-point of AG (converse of mid-point PQ || AC
theorem). 1
1
So, AF = FG ...(i) and PQ = AC (Mid-point theorem)
In DCBF, BF || DG, 2
...(ii) 1
D is the mid-point of BC.
From (i) and (ii), we get
So, G is the mid-point of FC. 1
RS || PQ and RS = PQ 1½
\ FG = GC ...(ii) 1
From (i) and (ii), AF = FG = GC or, PQRS is a parallelogram.
Since, diagonals of a parallelogram bisect each
other.
∴ PR and QS bisect each other. 1½

OBJECTIVE TYPE QUESTIONS (1 mark each)

A Multiple Choice Questions


Q. 1. A diagonal of a rectangle is inclined to one side of
the rectangle at 25°. The acute angle between the
diagonals is
(A) 55° (B) 50° Now, AC = BD
(C) 40° (D) 25°
[Diagonals of a rectangle are equal]
 A [NCERT Exemp.]
1 1
Ans. Option (B) is correct. \ AC = BD
2 2
Explanation: ABCD is a rectangle in which
diagonal AC is inclined to one side, AB of the or OA = OB
rectangle at an angle of 25°. In triangle AOB, we have
   OA = OB
QUADRILATERALS 117

\ ∠OBA =
∠BAO =25° Ans. Option (C) is correct.
(Angles opposite to equal sides) Explanation: AD is parallel to BC and AC cuts
them.
By angle sum property, we have
∠DAC = ∠ACB [Alt. int. angles]
So, ∠OBA + ∠AOB + ∠BAO = 180°
∠DAC = 32°
25° + 25° + ∠AOB
= 180° ⇒ ∠ACB = ∠DAC = 32°
= 180° − 50=
∠AOB ° 130°
      
We know that, ÐAOB and ÐAOD form linear pair.
So, ∠AOB + ∠AOD = 180°
130° + ∠AOD
= 180°
      ∠AOD= 180° − 130°= 50°
Therefore, the acute angle between the diagonal
is 50°. In triangle BOC, CO is produced to A
Q. 2. If bisectors of ÐA and ÐB of a quadrilateral ABCD Therefore,
intersect each other at P, of ÐB and ÐC at Q, of ÐC and Ext. ÐBOA = ÐOCB + ÐOBC
ÐD at R and of ÐD and ÐA at S, then PQRS is a [By exterior angle theorem]
(A) rectangle       70=
° 32° + ∠OBC
(B) rhombus ∠OBC = 70° − 32° = 38°
(C) parallelogram ⇒ ∠DBC = ∠OBC = 38°
(D) quadrilateral whose opposite angles are Q. 4. In the following figure, ABCD and AEFG are two
supplementary A [NCERT Exemp.] parallelograms. If ÐC = 60°, then ÐGFE is
Ans. Option (D) is correct.
Explanation:

(A) 30° (B) 60°


According to figure, (C) 90° (D) 120°
In DASD, Ans. Option (B) is correct.
DS bisects ÐD and AS bisects ÐA, Explanation: In parallelogram ABCD
1 1 ÐA = ÐC = 60°
\ ÐDAS + ÐADS = ÐA + ÐD
2 2 ( Opposite angles of a parallelogram are equal)
1 Similarly, In parallelogram AEFG
= (ÐA + ÐD)
2 ÐA = ÐF
1 (opposite angles of a||gm)
= × 180° \ ÐF = 60°
2
( Consecutive angles of a||gm are supplementary) Q. 5. The quadrilateral formed by joining the midpoints
So, ÐDAS + ÐADS = 90° of the sides of a quadrilateral PQRS, taken in order,
Also ÐDAS ++ ÐADS + ÐDSA = 180° is a rectangle, if :
(Angle sum property) (A) PQRS is a rectangle
90° + ÐDSA = 180° (B) PQRS is a parallelogram
So, ÐDSA = 90° (C) diagonals of PQRS are perpendicular
ÐPSR = ÐDSA (D) diagonals of PQRS are equal A [NCERT Exemp.]
(Vertically opposite angles) Ans. Option (C) is correct.
\ ÐPSR = 90°
Explanation: The diagonals of PQRS are
Similarly, ÐSPQ, ÐPQR and ÐQRS = 90°
perpendicular.
Hence, PQRS is a quadrilateral whose opposite
angles are supplementary. Q. 6. In fig. S is the mid-point of PQ and ST || QR then PT
Q. 3. The diagonals AC and BD of a parallelogram is equal to
ABCD intersect each other at the point O.
If ÐDAC = 32° and ÐAOB = 70°, then ÐDBC is
equal to
(A) 24° (B) 86°
(C) 38° (D) 32°
 A [NCERT Exemp.]
118 Oswaal CBSE Question Bank Chapterwise & Topicwise, MATHEMATICS, Class-IX

(A) SQ (B) PS
(C) TR (D) QR
B Assertion & Reason
Ans. Option (C) is correct.
Directions: In the following questions, a statement

Explanation: The line drawn through the mid-
of assertion (A) is followed by a statement of
point of one side of a triangle parallel to another
reason (R). Mark the correct choice as:
side, bisects the third side.
Therefore PT = TR (A) Both assertion (A) and reason (R) are true and
reason (R) is the correct explanation of assertion (A).
Q. 7. In fig. D and E are the mid-points of AB and AC
respectively. The length of DE is. (B) Both assertion (A) and reason (R) are true but reason
(R) is not the correct explanation of assertion (A).
(C) Assertion (A) is true but reason (R) is false.
(D) Assertion (A) is false but reason (R) is true.
Q. 1. Assertion (A): A parallelogram consists of two
congruent triangles.
Reason (R): Diagonal of a parallelogram divides it
(A) 8.2 cm (B) 4.1 cm into two congruent triangles.
(C) 4.9 cm (D) 5.1 cm Ans. Option (A) is correct.
Ans. Option (B) is correct. Explanation: According to properties of a
1 parallelogram a diagonal of a parallelogram
Explanation: DE = BC
2 divides it into two congruent triangles therefore
parallelogram has two congruent triangles.
(According to mid-point theorem)
Hence, A and R are true and R is correct explanation
1
DE = ´ 8.2 of A.
2
Q. 2. Assertion (A): Two opposite angles of a
= 4.1 cm
parallelograms are (3x – 8)° and (60 – x)°. The
Q. 8. In fig. PQRS is a rectangle X and Y are mid-points measure of one of the angle is 43°.
of PS and PQ respectively. The length of XY is
Reason (R): Opposite angles of a parallelogram are
supplementary.
Ans. Option (C) is correct.
Explanation: In case of assertion (A):
Opposite angles of parallelogram are equal
\ 3x – 8 = 60 – x
(A) 4 cm (B) 5 cm
3x + x = 60 + 8
(C) 2.5 cm (D) 2 cm
4x = 68
Ans. Option (C) is correct.
x = 17°
Explanation: In rectangle PQRS
By substituting 'x' we get
ÐR = 90°
(3x – 8)° = 43°
\ DSRQ is a right angled triangle (60 – x)° = 43°
Now, According to pythagoras theorem Hence, Assertion is true.
QS2 = SR2 + RQ2 In case of reason (R):
= 4 2 + 32 According to properties of parallelogram, opposite
= 16 + 9 angles of a parallelogram are equal.
\ Reason is false.
= 25
QS = 5 cm Q. 3. Assertion (A): ABCD and PQRC are rectangles and
Q is a midpoint of AC, then DP = PC.
X and Y are mid-points of RS and PQ
1
\ XY = QS
2
(By mid-point theorem)
1
XY = XS  eason (R): The line segment joining the midpoint
R
2 of any two sides of a triangle is parallel to third side
= 2.5 cm and is equal to half of it.
QUADRILATERALS 119
Ans. Option (B) is correct.
Explanation: In case of assertion (A):
Q is a midpoint of AC
So, P is also a midpoint of DC
(According to converse of midpoint theorem)
\ Assertion is true.
In case of reason (R):
It is a midpoint theorem
\ Reason is true but not correct explanation of Ans. Option (C) is correct.
Assertion. Explanation: In case of assertion (A):
Q. 4. Assertion (A): In DABC, median AD is produced to ABXC is a ||gm
 In a parallelogram Diagonals bisects each other.
X such that AD = DX. Then ABXC is a parallelogram.
\ Assertion is true.
Reason (R): Diagonals AX and BC bisect each other
In case of reason (R):
at right angles.
Diagonals of a ||gm do not bisect each other at right
angles.
\ Reason is false.
Hence, Assertion is true but Reason is false.

COMPETENCY BASED QUESTIONS (4 marks each)

Q. 3. If ∠RSP = 50°, then ∠SPQ = 


A Case based MCQs (A) 130° (B) 120°
(C) 110° (D) 100°
I. Read the following passage and answer any four
Ans.Option (A) is correct. 1
questions of the following :
Explanation: Adjacent angles of a parallelogram
Maths teacher of class 9th gave students coloured
are supplementary.
paper in the shape of quadrilateral and then ask
the students to make parallelogram from it by using Thus, ∠RSP + ∠SPQ = 180°
paper folding. 50° + ∠SPQ = 180°
∠SPQ = 180° – 50°

= 130°
Q. 4. If SP = 3 cm, then RQ = 

(A) 4 cm (B) 2 cm

(C) 3 cm (D) 5 cm
Ans.Option (C) is correct. 1
Q. 5. Which statement is incorrect about the
parallelogram ?

(A) Consecutive angles are supplementary

(B) Opposites sides are parallel
Q. 1. How can a parallelogram be formed by using (C) Diagonal bisects each other
paper folding ?
(D) Diagonals are equal in length.

(A) By joining any two vertices Ans. Option (D) is correct. 1
(B) By joining one pair of opposite vertices. II. Read the following passage and answer any four
(C) By joining mid points of sides of a questions of the following :
quadrilateral During maths lab activity, teacher gives four sticks

(D) None of the above of lengths 6 cm, 6 cm, 4 cm and 4 cm to each student
Ans. Option (C) is correct. 1 to make different types of quadrilateral.
Q. 2. If ∠RSP = 30°, then ∠RQP =  Q. 1. How many types of quadrilaterals can be
possible?

(A) 150° (B) 80°

(A) 3 (B) 4

(C) 50° (D) 30°
(C) 5 (D) 6
Ans.Option (D) is correct. 1
Ans. Option (A) is correct.
120 Oswaal CBSE Question Bank Chapterwise & Topicwise, MATHEMATICS, Class-IX

Q. 2. Write the name of quadrilateral that can be


formed with these sticks.
B Case based Subjective
Questions

(A) Kite, rectangle, rhombus
(B) Parallelogram, rectangle , trapezium Read the following passage and answer the

(C) Kite, rectangle, parallelogram following questions:
(D) Square, rectangle, kite I. ABCD is an area in the shape of rhombus in which
Ans. Option (C) is correct. ∠ADC = 120°. Samay and Tarun lived at D and C
Explanation: Opposite sides of a parallelogram and their school located at O.
and a rectangle are equal. Also, adjacent sides of kite
are equal. Thus, kite, rectangle and parallelogram
can be formed with the sticks of lengths 6 cm, 6
cm, 4 cm and 4 cm. Hence, option (C) is correct. 1
Q. 3. Which statement is incorrect ?

(A) Opposite sides of a parallelogram are equal

(B) A kite is not a parallelogram
(C) Diagonals of a parallelogram bisect each Answer the following questions:
other
Q. 1. Measure of ∠DCB is :

(D) A trapezium is a parallelogram.
Sol. ABCD is a rhombus and adjacent angles of a
Ans. Option (D) is correct.
rhombus are supplementary.
Explanation: A trapezium has only one pair of
parallel sides.
Thus, ∠CDA + ∠DCB = 180° ½

Thus, it cannot be a parallelogram. 1 ⇒
120° + ∠DCB = 180°
Q. 4. A
 student formed a rectangle with these sticks. ⇒
∠DCB = 180° – 120° = 60°
What is the length of the diagonal of the rectangle  ½
formed by the student ?
Q. 2. Who can reach school early?

(A) 6 13 cm (B) 3 13 cm
Sol. ∠CDO = 60° and ∠DCO = 30°

(C) 13 cm (D) 2 13 cm
Since, ∠CDO > ∠DCO
Ans. Option (D) is correct. ⇒
CO > DO.
Explanation: All the angles of a rectangle are right
 (side opposite to greater angle is greater.)
angle. ⇒ Samay will reach school early.
1
So, diagonal of a rectangle divides it into two right-
Q. 3. Measure of ∠CDO and ∠DCO are ______ and
angled triangles. Thus,
______ respectively.
Sol. Diagonal of a rhombus bisects the angles passing
from.
1

So, ∠CDO = ∠CDA
2

1
= (120)° = 60° 1

2

6 +4 =l
2 2 2


36 + 16 = l2
∠ADC + ∠DCB = 180°

l2 = 52
120° + ∠DCB = 180°

l = 2 13 cm 1 ⇒
∠DCB = 180° – 120° = 60°

Q. 5. A diagonal of a parallelogram divides it into two 1

∠DCO = ∠DCB
_______ triangles. 2

(A) Similar (B) Congruent
1
(C) Equilateral (D) Right angled
= (60°) = 30° 1
2
Ans. Option (B) is correct.1
QUADRILATERALS 121
II. Harish makes a poster in the shape of a parallelogram ⇒ 2y – 3 = 5 ½
on the topic SAVE ELECTRICITY for an inter school ⇒ 2y = 8
competition as shown in the follow figure. ⇒ y = 4 ½
Q. 2. Which mathematical concept is used here?
Sol. Properties of a parallelogram. 1
Q. 3. If ∠A = (4x + 3)° and ∠D = (5x – 3)°, then find the
measure of ∠B
Sol. Since, ABCD is a parallelogram.
∠A + ∠D = 180°
(consecutive of a||gm are supplementary)
\ (4x + 3)° + (5x – 3)° = 180°1
9x = 180°
x = 20°
∠D = (5x – 3)° = 97°
Q. 1. If AB = (2y – 3) and CD = 5 cm then what is the ∠D = ∠B
value of y ?
(opposite angles of a parallelogram are equal)
Sol. AB = CD
Thus, ∠B = 97°1
(opposite sides of a||gm are equal)

Artificial Intelligence
AI
PARAMETERS DESCRIPTION CONCEPTS
INTEGRATED
Chapter Covered Chapter 8: Quadrilaterals
Name of the book Mathematics, Text book for Class 9
Subject and Artificial To understand the concept of Quadrilaterals using AI tools.
Intelligence Integrated
Learning Objectives ● To understand the concept of Quadrilaterals.
- Quadrilaterals
- Angle sum property of a quadrilateral
- Types of Quadrilaterals
● Visualization of Quadrilaterals using AI tool Autodraw.
Autodraw
Time Required 2 periods of 40 minutes each.
Classroom Arrangement Flexible
Material Required Scrapbook, paper, pencil, scale, scissor, eraser, White Board. Laptops/
Desktops and Internet connection.
Pre-Preparation Activities Play the game -
Ask the students to collect the sticks:
Thirteen sticks of 6cm, one stick of 8 cm, 7 cm and 12cm each in length.
Using the sticks, construct:
● Quadrilateral with two pairs of parallel sides.
● Quadrilateral with four right angles.
● Quadrilateral with four congruent sides.
● Quadrilateral with exactly one pair of parallel sides.
Previous Knowledge ● The students are asked to recall the knowledge of parallel lines,
perpendicular lines, triangles, rectangle, and square.
● List the objects of quadrilateral shapes they faced in daily - life.
● Some questions will be asked related to quadrilaterals.
122 Oswaal CBSE Question Bank Chapterwise & Topicwise, MATHEMATICS, Class-IX

Methodology ● Ask students to go to www.kahoot.it


Teacher will create the quiz on www.create.kahoot.it Ask the students
to play the quiz, this will access previous knowledge about the topic.
The Teacher would get results instantly and they can assess the
students.
The students will be divided into four groups.
The teacher will provide each group with materials needed, each group
will do the activity
Activity 1.
● Ask students to take a scrap book and cut into two congruent triangles
and join them in such a way that they form a 4 sided closed figure.
● As they got the idea of a four sided closed figure introduced the term Autodraw.com
quadrilateral.
● Make perfect quadrilaterals with the help of AI tool Autodraw and ask
students to do the same thing.
Activity, ask the students to go to http://autodraw.com. Once they land
on this website, ask the students to select the first icon from the left
side toolbar. This icon activates the AI element of the tool. Now ask the
student to draw any quadrilateral shape
● Ask students to go to https://ncase.me/loopy/ once they land on
this activity, introduce the concept of similar factors and dissimilar
factors of various forms of quadrilateral with the parallelogram .And
then explain all properties of parallelogram and interrelation of
parallelogram with many forms of quadrilateral.
● Ask students to solve the exercises
Learning Outcomes Upon completion of the lesson , they are able
● To identify the all types of Quadrilateral
● To understand angle sum property of Quadrilateral
● Properties of parallelogram
● Properties of other quadrilaterals
● Visualization of interconnection of various quadrilaterals with
parallelogram using AI tool loopy.
● Visualization of Quadrilaterals using AI tools- Autodraw
Follow up Activities Make a table with the headings - know, want to know and learn about the
quadrilaterals using fontjoy- an AI element.
● Students will be divided into four groups. Students will construct their
own desired structures such as houses, schools and churches applying
the idea of quadrilateral around The World.
The students output will be assessed using constructed rubrics.
● Students would create their own quiz on www.create.kahoot.it and
play with their friends.

  
Study Time:
Max. Time: 3:30 Hrs
Max. Questions: 43
CHAPTER

9 CIRCLES
(Prove) Equal chords of a circle subtend equal angles at the centre and (motivate)
its converse.
Syllabus (Motivate) The perpendicular from the centre of a circle to a chord bisects the chord
and conversely, the line drawn through the centre of a circle to bisect a chord is
perpendicular to the chord.
(Motivate) Equal chords of a circle (or of congruent circles) are equidistant from the centre (or their respective centres)
and conversely.
(Prove) The angle subtended by an arc at the centre is double the angle subtended by it at any point on the remaining
part of the circle.
(Motivate) Angles in the same segment of a circle are equal.
(Motivate) If a line segment joining two points subtends equal angles at two other points lying on the same side of the
line containing the segment, the four points lie on a circle.
(Motivate) The sum of either of the pair of the opposite angles of a cyclic quadrilateral is 180° and its converse.

List of Topics
Topic-1: Basic Properties of Cir-
cles Page No. 123
Topic-2 : Cyclic Quadrilaterals
Topic-1 Basic Properties of Circles  Page No. 129

Revision Notes
 In a circle, equal chords subtend equal angles at the centre.
 The chords corresponding to congruent arcs are equal.
 If two arcs of a circle (or of congruent circles) are congruent, then the corresponding chords are equal.
 If two chords of a circle (or of congruent circles) are equal, then their corresponding arcs (minor, major or semi-
circular) are congruent.
 An infinite number of circles can be drawn through a given point P. Scan to know
more about
 An infinite number of circles can be drawn through the two given points. this topic
 Perpendicular bisectors of two chords of a circle intersect each other at the centre of the circle.
 The angle subtended by an arc at the centre is double the angle subtended by it at any point on
the remaining part of the circle.
Circle Theorems
 Angles in the same segment of a circle are equal.
 An angle in a semi-circle is a right angle.
 The arc of a circle subtending a right angle at any point of the circle in its alternate segment is a semi-circle.
124 Oswaal CBSE Question Bank Chapterwise & Topicwise, MATHEMATICS, Class-IX
CIRCLES 125

Example 1
Q. Two circles of radii 10 cm and 8 cm intersect and Step IV : Apply Pythagoras theorem in DOLP.

the length of the common chord is 12 cm. Find In right angle DOLP, we have

distance between their centres.
OP2 = OL2 + PL2
Solution:
Step I : Draw figure as per given information.
or, OL = OP 2 - PL2

Let O and O' be the centres of the circles of radii
(10)2 − (6)2
10 cm and 8 cm, respectively. Let PQ be their =
common chord.
= 64
= 8 cm
Step V : Apply Pythagoras theorem in DO'LP.

In right angle DO'LP, we have

O'P2 = PL2 + O'L2

O'L = (OP )2 - ( PL)2


= 82 − 62
Step II :Write the given information.
64 − 36

=

Given, OP = 10 cm, O'P = 8 cm and PQ = 12 cm.

Find OO'. = 28


Step III : For finding OO', first find PL.
= 5.29 cm.
1 \
OO' = OL + O'L
PL = PQ = 6 cm
2
= 8 + 5.29


( Perpendicular from the centre of a circle to a
= 13.29 cm
chord bisects the chord)

SUBJECTIVE TYPE QUESTIONS


Very Short Answer Type
Questions (1 mark each)
Q. 1. In the given figure, O is the centre of the circle and
PA = PB. Find ∠OPA. R [Board Term II, 2013]

Sol. ∠ACB = ∠ADB


[Angles in the same segment] ½
∴ ∠ADB = 40°
Now, in ∆ADB,
∠ADB + ∠DBA + ∠BAD = 180°
or, 40° + ∠DBA + 60° = 180° or, ∠DBA = 80° ½
Sol. Line drawn through the centre of circle to bisect a Q. 3. In the given figure, AD || BC and ∠BCA = 40°. The
chord is perpendicular to the chord. measure of ∠DBC is equal to ................... .
Given, PA = PB ; OP ^ AB R [Board Term II, 2012]
Hence, ∠OPA = 90° 1
Q. 2. In the given figure, A, B, C and D are the points on
a circle such that ∠ACB = 40° and ∠DAB = 60°, the
measure of ∠DBA is ....................... .
U [Board Term II, 2012]
126 Oswaal CBSE Question Bank Chapterwise & Topicwise, MATHEMATICS, Class-IX

Sol. ∠BDA = ∠BCA = 40° Sol. In ∆ADB,


(Angles in the same segment) ½ By angle sum property
Now, since AD || BC, ∠ABD + ∠ADB + ∠BAD = 180° ½
∠DBC = ∠BDA ∴ 50° + ∠ADB + 60° = 180°
(Alternate interior angles) ∴ ∠ADB = 180° – (50° + 60°)
∴ ∠DBC = 40° ½ = 70° 1
Q. 4. In the given figure, A, B, C and D are four points of ∴ ∠ACB = ∠ADB = 70°
a circle. AC and BD intersect at a point E such that (Q angles in the same segment of a circle are equal) ½
∠BEC = 130° and ∠ECD = 20°. Find ∠BAC. Q. 2. In the given figure, O is the centre of the circle and
 [Board Term II, 2012, KVS 2017, 2019, NCERT] chord AC and BD intersect at P such that ∠APB =
120° and ∠PBC = 15°, find the value of ∠ADB.
D
A U [KVS 2014]
E 

130°
20°

B C

Sol. ∠BEC = ∠EDC + ∠DCE Sol. In ∆PCB,


 (Exterior angle property) ∠PCB + ∠PBC = ∠APB (exterior angle of a D is
130° = ∠EDC + 20° equal to the sum of two Interior opposite angles) 1
∠EDC = 110° = ∠BDC ∠PCB + 15° = 120°
∠BAC = ∠BDC = 110° 1 ∴ ∠PCB = 105°
 (Angles in the same segment) or, ∠ACB = 105°
Q. 5. In the given figure, ∠ACP = 40° and ∠BPD = 120°, \ ∠ADB = ∠ACB = 105°
then ∠CBD = ....................... . [Angle in same segment] 1
U [Board Term II, 2012] Q. 3. If a line intersects two concentric circles with
common centre O, at A, B, C and D. Prove that
AB = CD. A [NCERT]
 [Board Term II, 2016; KVS 2014; 2012]

Sol. ÐADB = ÐACB = 40°


[Q Angles in the same segment are equal] ½
Now, in DDPB
Sol. Draw OP perpendicular to XY from the centre to a
ÐDPB + ÐDBP + ÐPDB = 180°
(Angle sum property) chord bisecting it.
or, 120° + ÐDBP + 40° = 180° OP ⊥ to chord BC.
or, ÐDBP = 180 – (120° + 40°) or, BP = PC ...(i) 1
or, ÐDBP = 20° Similarly, AP = PD ...(ii)
\ ÐCBD = ÐPBD = 20° ½ Subtracting eqn. (i) from eqn. (ii), we get ½
AP – BP = PD – PC
Short Answer Type or AB = CD ½
Questions-I (2 marks each)  Hence Proved
Q. 4. Prove that ‘‘equal chords of a circle subtend equal
Q. 1. In the figure, if ∠DAB = 60°, ∠ABD = 50°, then angles at the centres.’’ A [NCERT, KVS 2016, 2012]
find ∠ACB. U [KVS 2016] Sol. Given, AB and CD are the chords of a circle with
centre at O such that AB = CD
CIRCLES 127
To Prove : ∠AOB = ∠COD Q. 2. If O is the circumcentre of a ∆ABC and OD ⊥ BC,
Proof : In ∆AOB and ∆COD, then prove that ∠BOD = ∠BAC.
AO = CO (Radii of same circle) U [Board Term II, 2013]
AB = CD (Given)
BO = DO (Radii of same circle)
∆AOB @ ∆COD (SSS)
∴ ∠AOB = ∠COD (CPCT) 2
Hence Proved.
Q. 5. A chord of length 10 cm is at a distance of 12 cm
from the centre of a circle. Find the radius of the
circle. U [Board Term II, 2016] Sol. Given : OD ⊥ BC
Sol. Given, AB = 10 cm Proof : In ∆OBD and ∆OCD,
ON = 12 cm OB = OC (Radii) 1
OD = OD (Common)
∠ODB = ∠ODC (Each 90°)
\ ∆OBD ≅ ∠OCD (RHS rule)
So, ∠BOD = ∠COD (CPCT) 1
and, ∠BOC = 2∠BOD
But , ∠BOC = 2∠BAC
 (angle at centre is twice the
 angle at the circumference)
Also, ON ⊥ AB
From (i) and (ii)
and AN = BN
∠BOD = ∠BAC Hence Proved 1
( Perpendicular drawn from the centre of
Q. 3. In the given figure, AB and CD are two parallel
the circle to chord of circle bisects the chord) 1 chords of a circle with centre O and radius 5 cm
In ∆ONB, such that AB = 8 cm and CD = 6 cm. If OP is
OB2 = ON2 + NB2 perpendicular to AB and OQ is perpendicular to
(By pythagoras theorem) CD, determine the length of PQ.
∴ OB2 = 122 + 52 (Q BN = 5 cm) U [Board Term II, 2016]
= 144 + 25 = 169
∴ OB = 13 cm 1
Hence, the radius of the circle is 13 cm.

Short Answer Type


Questions-II (3 marks each)
Q. 1. In the given figure, AB and CD are two chords
of a circle with centre O such that MP = NP.
Sol. Construction : Join OA and OC.
If OM ⊥ AB and ON ⊥ DC, show that AB = CD.
Proof : Since, perpendicular from centre of the
U [Board Term II, 2014] circle to the chord bisects the chord.

Sol. Construction : Join OP.


Proof : In ∆OMP and ∆ONP, 1
∴ AP = PB = AB = 4 cm
∠OMP = ∠ONP = 90° (Given) 2
OP = OP (Common) 1 1
and, CQ = QD = CD = 3 cm 1
MP = NP (Given) 2
∴ ∆OMP @ ∆ONP (RHS) In ∆OAP,
∴ OM = ON (CPCT) 1 OP2 = OA2 – AP2
(Pythagoras theorem)
∴ AB = CD 
or, OP2 = 52 – 42
(Chords equidistant from the centre are equal) 1 = 25 – 16 = 9
Hence Proved ∴ OP = 3 cm ½
128 Oswaal CBSE Question Bank Chapterwise & Topicwise, MATHEMATICS, Class-IX

In ∆OCQ, ∴ OA bisects the chord CP, (Perpendicular from


OQ2 = OC2 – CQ2 the centre to the chord bisects the chord)
(Pythagoras theorem) 1
= 52 – 3 2 ∴ AP = CP
2
= 25 – 9
or, CP = 2AP ...(i) 1
= 16
Similarly, O'B ⊥ PD
∴ OQ = 4 cm ½
∴ PQ = OP + OQ 1
∴ BP = PD
=3+4 2
= 7 cm 1 or, PD = 2BP ...(ii)
[CBSE Marking Scheme, 2016] CD = CP + DP
Q. 4. If two equal chords of a circle intersect within a = 2AP + 2BP
circle, prove that the line segment joining the [from (i) and (ii)]
point of intersection to the centre makes equal = 2(AP + BP) ...(iii) 1
angles with the chords. = 2(AB)
In quadrilateral ABO'O,
U [NCERT][Board Term II, 2016]
OA = O'B
Sol. Let AB and CD be two equal chords intersecting (Two lines ⊥ to same parallel line CD)
at P. AB || OO' (Given)
Let O be the centre of the circle. ∴ ABO'O is ||gm and AB = OO'.
Construction: Draw OM ⊥ CD and ON ⊥ AB. (Opp. sides of parallelogram are equal)
In ∆OMP and ∆ONP, ∴ CD= 2AB = 2OO' Hence Proved 1
∠OMP = ∠ONP = 90° [CBSE Marking Scheme, 2015]
(By construction)
OM = ON Long Answer Type
Questions (5 marks each)
Q. 1. In the given figure, AB is a diameter of the circle
with centre O. If AC and BD are perpendiculars on
a line PQ and BD meets the circle at E, then prove
that AC = ED. U [Board Term II, 2013]

1
(equal chords are equidistant from the centre)
OP = OP (Common)
∴ ∆OPM @ ∆OPN (R.H.S.)
∴ ∠OPM = ∠OPN (CPCT) 2
Hence Proved Sol. Proof : ∠AEB = 90°  (Angle in semi-circle)
∠AEB + ∠AED = 180° (Linear pair) 1
[CBSE Marking Scheme, 2016]
\ ∠AED = 90°
Q. 5. Two circles whose centres are O and O' intersect at ∠EAC + ∠ACD + ∠CDE + ∠AED = 360°
P. Through P, a line parallel to OO', intersecting the (Sum of angles of a quad.)
circles at C and D is drawn as shown. Prove that or, ∠EAC + 90° + 90° + 90° = 360° 1
CD = 2OO' or, ∠EAC = 360° – 270° 1
= 90° 1
A [Board Term II, 2015, 2014, NCERT Exemplar]
Hence, each angle of quadrilateral is 90°.
∴ EACD is a rectangle.
∴ AC = ED. Hence Proved. 1
Q. 2. PQ and RS are two parallel chords of a circle
whose centre is O and radius is 10 cm. If
PQ = 16 cm and RS = 12 cm, find the distance
between PQ and RS when they lie,
Sol. Construction : Draw OA and O'B perpendicular
(i) On the same side of centre O.
to CD from O and O' respectively.
(ii) On the opposite sides of centre O.
A [Board Term II, 2015]
Sol. Given, OP = OR = 10 cm
(Radii of same circle)
PQ = 16 cm,
Proof : OA ⊥ CD
RS = 12 cm
CIRCLES 129
Q. 3. A circular park of radius 20 m is situated in a
Draw OL ^ PQ and OM ^ RS. 1 village. Three girls Rita, Sita and Gita are sitting at
Since, perpendicular from the centre to the chord equal distance on its boundary each having a toy
bisects the chord. telephone in their hands to talk to each other. Find
1 the length of the string of each phone. (There is no
\
PL = LQ = PQ = 8 cm slack in the string). A
2
1 OR
RM = MS = RS = 6 cm A circular park of radius 20 m is situated in a
2
colony. Three boys Ankur, Syed and Dayd are
In right triangle OLP,
sitting at equal distance on its boundary each
OP2 = OL2 + PL2 having a toy telephone in his hands to talk each
(By Pythagoras theorem) other. Find the length of the string of each phone.
100 = OL2 + 64
 A [NCERT]
OL = 100 − 64 = 36 1
OL = 6 cm
In right triangle OMR,
OR2 = OM2 + RM2 Sol. 1
(By Pythagoras theorem)
100 = OM2 + 36
OM = 100 - 36 = 64
OM = 8 cm Here, A, B and C are the three points where three
(i) If PQ and RS lie on same side of centre O. girls/boys are sitting.
DABC is an equilateral triangle.
In an equilateral triangle, the circumcentre is the
point of intersection of median. ½
∴ O divides AD in the ratio 2 : 1.  ½
Hence, if AO = 20 m ½
then, OD = 10 m
Distance between PQ and RS Also, median is same as the altitude for an equilateral
= LM = OM – OL triangle. ½
= 8 – 6 In right triangle ∆ODC,
= 2 cm 1 OC2 = OD2 + DC2
(ii) If PQ and RS lie on opposite sides of centre O or, 202 = 102 + DC2
or, DC2 = 400 – 100 = 300 ½
or, DC = 10 3 m
or, BC = 2DC
Perpendicular drawn from the centre to the chord
bisects the chord.
1 = 20 3 m 1
Distance between PQ and RS Length of the string of each phone = 20 3 m ½
= LM = OL + OM
= 6 + 8 cm
= 20 × 1.732
= 14 cm 1 = 34.64 m (Approx)

Topic-2 Cyclic Quadrilaterals

Revision Notes Scan to know


more about
this topic
 If a line segment joining two points subtends equal angles at two other points lying on the same
side of the line segment, the four points are concyclic, i.e., lie on the same circle.
 If the sum of any pair of opposite angles of a quadrilateral is 180°, then the quadrilateral is cyclic.
 Any exterior angle of a cyclic quadrilateral is equal to the interior opposite angle.
 Concentric Circles : Circles with a common centre are called concentric circles. Cyclic
Quadrilateral
130 Oswaal CBSE Question Bank Chapterwise & Topicwise, MATHEMATICS, Class-IX

 The degree measure of a semi-circle is 180°.


 The degree measure of a circle is 360°.
 The degree measure of a major arc is (360°– q), where q is the degree measure of the corresponding minor arc.
 Area of a circle = πr2 sq. units

Example 2
Prove that the quadrilateral formed (if 1
possible) by the internal angle bisectors of any = 180° – (ÐP + ÐQ) ...(i)
2
quadrilateral is cyclic.
[PB and QD are bisectors of ÐP and ÐQ,

respectively]
Similarly,
ÐBCD = ÐRCS
= 180° – (ÐCRS + ÐRSC)
1
= 180° – (ÐR + ÐS)
2
...(ii)
Solution:
Step I : Draw a figure according to given Step IV : Adding the results obtained in step III

information. and further use the property of a quadrilateral,
Let PQRS be a quadrilateral in which the angle which prove the required results.
bisectors PB, QD, RD and SB of internal angles
On adding eqs (i) and (ii), we get
P, Q, R and S, respectively form a quadrilateral 1
ÐBAD + ÐBCD = 180° – (ÐP + ÐQ) + 180°
ABCD 2
Step II : Write the proving statement. 1
– (ÐR + ÐS)
ABCD is a cyclic quadrilateral 2
i.e., ÐA + ÐC = 180° 1
or, ÐB + ÐD = 180° = 360° – (ÐP + ÐQ + ÐR + ÐS)
2
Step III : For proving the result, first find the
1
angles ÐBAD and ÐBCD. = 360° – × 360°
Since, ÐBAD = ÐPAQ 2
(Vertically opposite angles) (sum of angles of a quadrilateral is 360°)
Also, ÐPAQ = 180° – (ÐAPQ + ÐAQP) = 360° – 180°
( In ∆PAQ, using angle sum property = 180°
i.e., ÐPAQ + ÐAPQ + ÐAQP = 180°) \ ABCD is a cyclic quadrilateral because sum of
a pair of opposite angles of quadrilateral ABCD is
180°. Hence Proved

SUBJECTIVE TYPE QUESTIONS


Sol. Reflex angle POR = 360° – 110°
Very Short Answer Type = 250° ½
Questions (1 mark each) \ By degree measure theorem,
Q. 1. In the given figure, if ∠POR is 110°, then find the 1
ÐPQR = (reflex angle POR)
2
value of ∠PQR. R [Board Term II, 2012]
1
= (250°)
2
= 125° ½
Q. 2. The sum of the opposite angles of a cyclic
quadrilateral is : ............. R [Board Term II, 2011]
Sol. 180°. 1
CIRCLES 131
Q. 3. In the given figure, quadrilateral PQRS is cyclic. If Sol.
ÐP = 80°, then ÐR is equal to ................. .
R [Board Term I, 2011]

Since, sum of the opposite pairs of angles in a cyclic


quadrilateral is 180°.
Hence, ∠B + ∠D = 180°
or, ∠B = 180° – 70° = 110° 1
Sol. Since, quadrilateral PQRS is cyclic Again, AB || CD and AD is its transversal, so
\ ÐP + ÐR = 180° ½ ∠A + ∠D = 180°
(sum of opposite angles of a cyclic quadrilateral) (Consecutive interior angles)
or, 80° + ÐR = 180° ∠A + 70° = 180°
or, ÐR = 100° ½ or, ∠A = 180° – 70° = 110° ½
and ∠A + ∠C = 180°
Short Answer Type or, 110° + ∠C = 180°
∴ ∠C = 180° – 110 = 70° ½
Questions-I (2 marks each) Q. 4. If diagonals of a cyclic quadrilateral are diameters
Q. 1. In the given figure, find the value of x. of the circle through the opposite vertices of the
quadrilateral, prove that the quadrilateral is a
R [Board Term II, 2012] rectangle. R [Board Term II 2017]
Sol. ABCD is a cyclic quadrilateral in which AC and BD
are diameters

Sol. In a cyclic quadrilateral,


ÐA + ÐC = 180°
(opposite angles of cyclic quadrilateral are Since, AC is diameter
supplementary) \ ÐABC = ÐADC = 90°
or, 2x + 4° + 4x – 64° = 180° (Angles of semicircle) 1
or, 6x – 60° = 180° BD is the diameter
or, 6x = 180° + 60° = 240° 1 \ ÐBAD = ÐBCD = 90°
240ϒ (Angles of semicircle)
or, x= Since, all angles of quadrilateral ABCD are 90°. 1
6
\ ABCD is a rectangle. Hence Proved.
∴ x = 40° 1 Q. 5. In the given figure, O is the centre of the circle and
Q. 2. ABCD is a cyclic quadrilateral in which AC and BD BA = AC. If ÐABC = 50°, find ÐBOC and ÐBDC.
are its diagonals. If ∠DBC = 55° and ∠BAC = 45°,  A [Board Term II, 2017]
find ∠BCD. R [Board Term II, 2012]
Sol.

∠BAC = ∠BDC = 45°,


(Angles in the same segment) ½
Now, in DDBC, Sol. (i) AB = AC (Given)
∠DBC + ∠BCD + ∠CDB = 180° \ ÐABC = ÐACB = 50°
(Angle sum property) ½ (Isosceles D property)
or, 55° + ∠BCD + 45° = 180° By angle sum property of a triangle
or, ∠BCD = 80° 1 ÐBAC = 180° – ÐABC – ÐACB
Q. 3 ABCD is a cyclic quadrilateral in which AB || CD. If ÐBAC = 180° – 50° – 50° = 80°
∠D = 70°, find all the remaining angles. \ ÐBOC = 2ÐBAC 1
(Angle at the centre is twice the angle
R [Board Term II, 2012]
at the circumference)
132 Oswaal CBSE Question Bank Chapterwise & Topicwise, MATHEMATICS, Class-IX

= 2 × 80°
= 160° \ Ð1 + Ð3 = 180° [By (i) and (ii)]
(ii) ÐBDC + ÐBAC = 180° Ð2 + Ð4 = 180° [By (i) and (iii)] 1
(Opp. angles of cyclic quadrilateral) As these are opposite angles of a quadrilateral.
\ ÐBDC = 180° – 80° = 100° 1 \ BCED is a cyclic quadrilateral. 1
[CBSE Marking Scheme 2012]
Short Answer Type
Q. 3. In the given figure, PQ = QR = RS and ÐPQR =
Questions-II (3 marks each)
128°. Find ÐPTQ, ÐPTS and ÐROS.
Q. 1. In the given figure, AB is a chord equal to the
radius of the given circle with centre O. Find the U [NCERT]

values of a and b. U [Board Term II, 2014]

Sol. Given, PQ = QR = RS, ÐPQR = 128°


In DPQR
Sol. OB = OA (radii of circle)
OA = OB = AB (given)
\ DOAB is an equilateral triangle.
\ ÐAOB = 60° 1
(angles of equilateral D are 60° each)
\ a + ÐAOB = 180° (Linear pair)
\ a + 60° = 180°
\ a = 120°
Reflex angle BOD = 2ÐBCD 1
(angle subtended by an arc at the centre is (180° − 128°)
twice at the circumference) Ð1 = Ð2 =
360° – a = 2b 2
360° – 120° = 2b 52°
= = 26° 1
2b = 240° 2
b = 120° 1
[CBSE Marking Scheme, 2014] \
 ÐPTQ = ÐQRP = 26°
Q. 2. If a line is drawn parallel to the base of an isosceles  (Angle in same segment)
triangle to intersect its equal sides, prove that the (ii) ÐPTQ = ÐQTR
quadrilateral so formed is cyclic.
= ÐRTS = 26°
U [Board Term II , 2012, NCERT Exemplar]
(\ equal chords make equal angle at circumference
Sol. A or centre)
And ÐPTS = ÐPTQ + Ð4 + Ð3
= 26° + 26° + 26°
D E
1 1 = 78° 1
4
(iii) ÐROS = 2ÐRTS
2 3  (Angle subtended by an arc at the centre
B C
 is twice at circumference)
Given, DE || BC
= 2 × 26° = 52° 1
AB = AC
or, Ð2 = Ð3 ...(i) Long Answer Type
(Angles opposite to equal Questions (5 marks each)
sides to are always equal)
Q. 1. In the given figure, PQ is the diameter of the circle.
Since, DE is parallel to BC, the consecutive interior
If ∠PQR = 65°, ∠QPT = 60°, then find the measure
angles as supplementary
of :
Ð1 + Ð2 = 180° ...(ii)
(i) ∠QPR (ii) ∠PRS (iii) ∠PSR (iv) ∠PQT.
and Ð3 + Ð4 = 180° ...(iii)
U [Board Term II, 2012]
CIRCLES 133

1
ÐPUT = ÐPOT
2
(Angle subtended by an arc at the centre is
twice the angle at the remaining circle)
1
= × 150°
2
Sol. (i) ∠QRP = 90° (Angle in the semi-circle) 1
= × 150°
 1 2
∴ ∠QPR = 25° (By angle sum property) \ ÐPUT = 75° 1
(ii) ∠QPS= ∠QPR + ∠RPS = 25° + 25° = 50° [CBSE Marking Scheme, 2015]
∠QRS = 180° – 50° = 130° Q. 3. A chord of a circle is equal to the radius of the
(PQRS is a cyclic quad.) 1 circle. Find the angle subtended by the chord at a
∠PRS = 130° – ∠QRP 1 point on the minor arc and also at a point on the
= 130° – 90° = 40° major arc. U [Board Term II, 2012]
(iii) ∠PSR = 180° – 65° = 115°
Sol. According to the question,
 (PQRS is a cyclic quad. & ∠Q = 65°) 1
OA = AB = OB 1
(iv) ∠PTQ = 90°
∴ ∠PQT = 180° – (90° + 60°) = 30°1
Q. 2. In the given figure ÐSPQ = 45°, ÐPOT = 150° and O
is the centre of circle. Find the measures of ÐRQT,
ÐRTQ and ÐPUT. U [Board Term II, 2015]

∴ DOAB is an equilateral triangle


∠AOB = 60° 1
1
∠ACB = ∠AOB 1
2
Sol. In the given figure (Angle subtended by an arc at the circumference
ÐPOT + reflex ÐPOT = 360° is half of the angle at the centre of circle)
150° + reflex ÐPOT = 360° 1
ÐACB = × 60°
reflex ÐPOT = 210° 1 2
reflex ÐPOT = 2ÐPST ∠ACB = 30° 1
(Angle subtended by arc at the ∠ACB + ∠ADB = 180° (Opposite angles of cyclic
centre is twice at circumference) quadrilateral are supplementary)
210° = 2ÐPST or, ∠ADB = 180° – ∠ACB
\ ÐPST = 105° 1 ∠ADB = 180° – 30° = 150° 1
ÐPQT + ÐPST = 180°
(Opposite angles of cyclic quadrilateral PQTS are
supplementary) Commonly Made Error
ÐPQT = 180° – 105°
= 75° ½ Students fail to identify which angle is
ÐRQT + ÐPQT = 180° (Linear pair) subtended by the chord from which arc.
ÐRQT = 180° – 75°
= 105° ½
ÐRTQ = ÐSPQ Answering Tip
(Exterior angle of a cyclic quadrilateral is
equal to interior opposite angle)
= 45° 1 A chord has its both points on the circle.

OBJECTIVE TYPE QUESTIONS (1 mark each)

(A) 17 cm (B) 15 cm
A Multiple Choice Questions (C) 4 cm (D) 8 cm A [NCERT Exemp.]
Ans. Option (D) is correct.
Q. 1. AD is a diameter of a circle and AB is a chord. If
AD = 34 cm, AB = 30 cm, the distance of AB from
the centre of the circle is
134 Oswaal CBSE Question Bank Chapterwise & Topicwise, MATHEMATICS, Class-IX

Explanation: (A) 30° (B) 60°


(C) 90° (D) 45°
 A [NCERT Exemp.]
Ans. Option (D) is correct.
Explanation: As AOB is a diameter of the circle,
∠C=90 o [Angles in a semi-circle is 90o]
Now, AC = BC
∠A=∠B [Angles opposite to equal sides of triangle
Draw OP ⊥ AB are equal]
We know that the perpendicular from the centre of Using angle-sum property of a triangle, we have
the circle to the chord bisects the chord. ∠A + ∠B + ∠C =180o
∠A + ∠A + 90o =
180o
1 1
Therefore, AP = ×AB= ×30=15 cm 2∠A=180 o − 90 o
2 2 90 o
1 ∠ A= = 45o
Radius = OA = ×34=17 cm 2
2 Q. 4. In the given figure, BC is a diameter of the circle
In right DOPA, we have and ÐBAO = 60°. Then ÐADC is equal to
A
2 2
OP = OA − AP
60°
= 17 2 − 15 2
B O C
= 289 − 225
= 64 = 8 cm
D
Q. 2. If AB = 12 cm, BC = 16 cm and AB is perpendicular
(A) 30° (B) 45°
to BC, then the radius of the circle passing through
(C) 60° (D) 120°
the points A, O and C is
(A) 6 cm (B) 8 cm  A [NCERT Exemp.]
(C) 10 cm (D) 12 cm Ans. Option (C) is correct.
 Explanation: In DOAB,
A [NCERT Exemp.]
OA = OB [Radii of the same circle]
Ans. Option (C) is correct.
\ ∠ABO = ∠BAO
Explanation: [Angles opposite to equal sides are equal]
So, ∠ABO = ∠BAO = 60o  [Given]
Now, ∠ADC = ∠ABC = 60o

A
O . C
[Angles in the same segment of a circle are equal.]
Therefore, ∠ADC = 60o
Q. 5. In the given figure, O is the centre of a circle and
ÐBOD = 150°, then the value of x is
12

cm
16
cm

B
AB is perpendicular to BC, therefore, ABC is right
angle triangle.
In right angle triangle ABC, we have
AC = AB 2+BC 2 (By Pythagoras Theorem)
= 12 2+16 2
= 144+256 (A) 105° (B) 115°
= 400=20 (C) 100° (D) 110°
Therefore, AC = 20 cm (Diameter of circle) Ans. Option (A) is correct.
1 Explanation:
Radius = ×20=10 cm ÐBOD = 150°
2 1
Q. 3. In the given figure, if AOB is a diameter of the \ ÐBCD = ´ 150° = 75°
circle and AC = BC, then ÐCAB is equal to : 2
(Angle at centre is twice the angle at the
circumference)
Now, ABCD is a cyclic quadrilateral
ÐBCD + ÐBAD = 180°
(\ Sum of opposite angles of a quadrilateral)
75° + ÐBAD = 180°
ÐBAD = 105°
CIRCLES 135
Q. 6. In the given figure, O is the centre of a circle and
diameter AB bisects the chord CD at a point P such
that CP = PD = 10 cm and PB = 6 cm, then the
radius of the circle is

(A) 60° (B) 45°


(C) 40° (D) 70°
Ans. Option (A) is correct.
(A) 12.3 cm (B) 11.3 cm Explanation:
(C) 13 cm (D) 10.3 cm Here, ÐBAC = 30° (Given)
Ans. Option (B) is correct. Then, ÐBOC = 2 × ÐBAC
Explanation: Construction: Join OC and OD = 2 × 30° = 60°
Let the radius of the circle be r cm, then OC = OD  OB = OC = Radius
= OB = r cm, OP = (r – 6) cm \ ÐOBC = ÐOCB = x
But CP = PD = 10 cm (given) (Angle opposite to equal side)
Since, AB bisects the chord CD at a point P, Now, in DOBC,
ÐBOC + ÐOBC + ÐOCB = 180°
then ÐOPD = 90°
Þ 60° + x + x = 180°
\ In right triangle DOPD, 2x = 120°
OD2 = OP2 + PD2 Þ x = 60°
(Using Pythagoras theorem)
r2 = (r – 6)2 + (10)2 B Assertion & Reason
Þ r2 = r2 + 36 – 12r + 100
Þ 12r = 136 Directions: In the following questions, a statement
of assertion (A) is followed by a statement of
Þ r = 11.3 cm
reason (R). Mark the correct choice as:
Q. 7. In the given figure, O is the centre of a circle,
(A) Both assertion (A) and reason (R) are true and reason
ÐAOB = 50° and ÐBDC = 120°, then ÐOBC is: (R) is the correct explanation of assertion (A).
(B) Both assertion (A) and reason (R) are true but reason
(R) is not the correct explanation of assertion (A).
(C) Assertion (A) is true but reason (R) is false.
(D) Assertion (A) is false but reason (R) is true.
Q. 1. Assertion (A): In the given figure, if ÐABC = 20°,
then ÐAOC is 40°.

(A) 45° (B) 60°


(C) 35° (D) 90° O
Ans. Option (C) is correct. A B
20°
Explanation:
 ÐAOB = 50° (Given)
1 1 C
\ ÐDCB = ÐAOB = × 50°
2 2  eason (R): Angle subtended by an are of a circle at
R
= 25° the centre is double the angle subtended by it at any
Now, in DDBC, on the remaining part of the circle.
ÐBDC + ÐDCB + ÐDBC Ans. Option (A) is correct.
= 180° Explanation: In case of Assertion (A):
(Angle sum property) Arc AC subtends ÐAOC at the centre O and ÐABC
at a point B on the remaining part of the circle.
Þ 120° + 25° + ÐDBC = 180°
\ ÐAOC = 2ÐABC
Þ ÐDBC = 180° – (120° + 25°) ( Angle subtended by an arc at the centre of the
= 180° – 145° = 35° circle is double the angle subtended by it at any
other part of the circle.)
Q. 8. In the given figure, O is the centre of a circle and
Hence, ÐAOC = 2 × 20°
ÐBAC = 30°, then ÐOBC is: = 40°
136 Oswaal CBSE Question Bank Chapterwise & Topicwise, MATHEMATICS, Class-IX

 eason (R): Opposite angles of a cyclic quadrilateral


R
\ Assertion is True
In case of Reason (R): It is a theorem of circle which are equal.
is a correct explanation of Assertion. Ans. Option (C) is correct.
\ Reason is True Explanation: In case of Assertion (A):
Q. 2. Assertion (A): In the given figure, if OA = 5 cm, AB ÐA + ÐC = 180°
= 8 cm and OD is perpendicular to AB, then CD is \ ÐAOC = 2ÐABC
equal to 2 cm. ( Sum of opposite angles of a cyclic quadrilateral
is 180°.)
So, 2x + 40° + 3x + 20° = 180°
5x = 180° – 60°
x = 24°
\ Assertion is true
In case of Reason (R):
Opposite angles of a cylic quadrilateral are not
equal.
There sum is 180°
 eason (R): Perpendiculars bisectors of two chords
R \ Reason is false.
of a circle intersect each other at the centre of the Q. 4. Assertion (A): In the given figure, BOC is a diameter
circle. of a circle and ÐB = 45°, then ÐC is 25°.
Ans. Option (B) is correct.
Explanation: In case of Assertion (A):
The perpendicular drawn from the centre to a
chord bisects the chord,
1 1
\ AC = AB = ´ 8 = 4 cm
2 2
In right angle triangle ÐAOC
\ OC2 = OA2 – AC2

= 52 – 42

= 25 – 16
\ OC = 9 Reason (R): An angle in a semi-circle is a right angle.
OC = 3 cm Ans. Option (D) is correct.
Now, CD = OD – OC
Explanation: In case of Assertion (A):
= 5 – 3 = 2 cm
\ Assertion is true. ÐA = 90°
In case of Reason (R): Reason is true but not correct ( An angle in a semi circle is a right angle)
explanation of Assertion. ÐB = 45° (given)
Q. 3. Assertion (A): In given figure ABCD is a cyclic Then, ÐA + ÐB + ÐC = 180°
(Angle sum property)
quadrilateral in which ÐA = (2x + 40)° and ÐC =
90° + 45° + ÐC = 180°
(3x + 20)°, then the value of x is 24°
ÐC = 180° – 135°
A ÐC = 45°
\ Assertion is false
In case of Reason (R): Angle in a semicircle is right
(2x + 40)°
angle.
\ Reason is true
B D Hence, Assertion is false but Reason is true.

(3x + 20)°

COMPETENCY BASED QUESTIONS (4 marks each)

and CD in such a way that AB and CD are 6 cm and


A Case based MCQs 8 cm from the centre O. Now, he has some doubts
that are given below. Help him out by answering
Read the following passage and answer any four these questions:
questions of the following :
I. Rohan draws a circle of radius 10 cm with the help
of compass and scale. He also draws two chords, AB
CIRCLES 137
Q. 5. Which statement is not true ? 
(A) Equal chords of a circle subtend equal angles
at the centre
(B) The perpendicular from the centre of a circle
to a chord bisects the chord.
(C) Angles in the same segment of a circle are
equal.
(D) The sum of each pair of opposite angles of a
Q. 1. What is the length of AB ? cyclic quadrilateral is 90°.
(A) 12 cm (B) 11 cm Ans. Option (D) is correct.
(C) 16 cm (D) 8 cm Explanation: The sum of each pair of opposite
angles of a cyclic quadrilateral is 180°. 1
II. Three STD booth are placed at A, B and C in the
figure and these are operated by handicapped
persons. These three booth are equidistant from
each other as shown in figure given below.
[CBSE SAS]
A

Ans. Option (C) is correct.


Explanation: Length of AB = 16 cm O
In DAOP h2 = p2 + b2
(Pythagoras theorem)
102 = 62 + b2 B C
100 = 36 + b2
b = 64
= 8 cm
Q. 1. Value of ÐBAC will be?
AB = 8 + 8 (A) 30° (B) 60°
(Perpendicular drawn from the centre of a circle (C) 90° (D) 100°
to the chord bisects the chord) Ans. Option (B) is correct.
= 16 cm 1 Explanation: As AB = BC = CA (given)
Q. 2. What is the length of CD ? So, DABC is an equilateral triangle
(A) 10 cm (B) 12 cm \ ÐBAC = 60°
(C) 16 cm (D) 21 cm ( All angles of an equilateral triangle are equal)
Ans. Option (B) is correct. Q. 2. What is the value of ÐBOC?
Explanation: Length of CD = 12 cm (A) 60° (B) 120°
In DQOC h2 = p2 + b2 (C) 180° (D) 200°

(Pythagoras theorem) Ans. Option (B) is correct.
(10)2 = 82 + b2 Explanation: ÐBOC = 2ÐBAC
b2 = 100 – 64 = 120° 1
= 36 ( Angle subtended by an arc at the centre is
b = 6 cm double the angle subtended by it any point on the
CD = 6 + 6 remaining part of the circle.)
= 12 cm 1 Q. 3. If AB = 8 cm, value of BC + CA will be?
Q. 3. A circle divides the plane, on which it lies, in (A) 16 cm (B) 10 cm
_________ parts  (C) 8 cm (D) 18 cm
(A) 1 (B) 2 Ans. Option (A) is correct.
(C) 3 (D) 4 Explanation: DABC is an equilateral triangle
\ AB = BC = CA = 8 cm 1
Ans. Option (C) is correct.
Hence, BC + CA = 16 cm
Explanation: 3 parts (inside, outside and on the
Q. 4. What will be the value of ÐOBC?
circle) 1
(A) 60° (B) 120°
Q. 4. A quadrilateral is called cyclic if all the four
(C) 30° (D) 150°
vertices of it lie on a _________
Ans. Option (C) is correct.
(A) Circle (B) Quadrilateral
Explanation: OB = OC (equal radii)
(C) Pentagon (D) Triangle \ ÐOBC = ÐOCB = x°
Ans. Option (A) is correct. 1 (Angle opposite to equal side are equal.)
138 Oswaal CBSE Question Bank Chapterwise & Topicwise, MATHEMATICS, Class-IX

In DOBC ÐOPR + ÐORP + ÐPOR = 180°


ÐOBC + ÐOCB + ÐBOC = 180° x + x + 140° = 180°
x + x + 120° = 180° 1 ( ÐPOR = 2ÐPAR)
2x = 60° 2x = 40°
\ ÐOBC = 30° x = 20°
Q. 5. Which angle will be equal to ÐOBC?  Hence, ÐOPR = 20° 1
(A) ÐABC (B) ÐACB II. A farmer has a circular garden as shown in the
(C) ÐBOC (D) ÐOCB figure given below.
He has different type of trees, plants and flower
Ans. Option (D) is correct.
plants in his garden.
Explanation: OB = OC (equal radii)
In the garden, there are two mango trees A and B
\ ÐOBC = ÐOCB 1
at a distance of AB = 10 m. Similarly has two Ashok
(Angle opposite to equal side)
trees at the same distance of 10m as shown at C and
D. AB subtends ÐAOB = 120° at the centre O. The
B Case based Subjective perpendicular distance of AC from centre is 5m. the
Questions radius of the circle is 13m.
Read the following passage and answer the
A 10 cm
following questions: 13 B
cm
I. Four Friends Rima, Mohan, Sohan and Sita are
sitting on the circumference of a circular park. Their 120°
P O
locations are marked by points A, P, Q and R. 5 cm
Rohit joins them and sits at the centre of the circular
park, so he is equidistant from all the other friends. D
His position is marked as O. C 10 cm
They are sitting in such a way that ÐPQR = 110°.
Q. 1. What is the value of ÐADB?
[CBSE SAS]
Ans. ÐAOB = 2ÐADB
Q
( An angle subtended by an arc at centre is double
the angle subtended by it any point on circumference.)
P R 1
or, ÐADB = ÐAOB
2
O 1
= ´ 120°
2
\ ÐADB = 60° 1
Q. 2. What is the value of ÐACB ?
A Ans. ÐACB = ÐADB
Q. 1. What is measure of reflex ÐPOR? (Angle in same segment are equal.)
Ans. Reflex ÐPOR = 2ÐPQR \ ÐACB = 60°
( Angle subtended by an arc at the centre is double Q. 3. What is the distance between mango tree A and
the angle subtended by it in the remaining part of Ashok tree C?
the circle.) Ans. OP = 5 cm (Given)
\ Reflex ÐPOR = 2 × 110° OA = 13 cm (Radii)
= 220° OP ^ AC (Given)
Q. 2. What is the measure of ÐPAR? \ In right triangle AOP
Ans. ÐPAR + ÐPQR = 180° 1 OA2 = OP2 + AP2
( Sum of opposite angles of cyclic quadrilateral is
(Pythagoras theorem)
180°) 132 = 52 + AP2
\ ÐPAR = 180° – 110° 169 - 25
AP =
= 70° 1
= 12 cm
Q. 3. Find ÐOPR?
Now AC = 2AP 1
Ans. In DOPR
\ perpendicular drawn from the centre of a circle
OP = OR (equal radii)
to the chord bisects the chord
\ ÐOPR = ÐORP = x°
\ AC = 2 cm 1
(Angle opposite to equal sides are equal.)
CIRCLES 139

Artificial Intelligence
AI
PARAMETERS DESCRIPTION CONCEPTS
INTEGRATED
Chapter Covered Chapter 10: Circles
Name of the book Mathematics, Class 9 NCERT
Subject and Artificial Understanding the concept of Circles and integrating art and math with
Intelligence Integrated artificial intelligence
Learning Objectives ● To understand the concept of Circles
● To discover the relationship between angles subtended by the equal
chords at the center.
● To prove the theorem using triangle properties.
● To apply the theorem in solving problems.
AutoDraw
Time Required 2 periods of 40 minutes each
Classroom Arrangement Flexible
Material Required Pen, paper, Black Board chalk, scissors, glue, cardboard, geometry box,
Laptops/ desktops and Internet connection.
Pre-Preparation Activity Students will be asked to recall the circle shape, draw it and its related
terms radius, diameter, chord etc. using https://www.autodraw.com/ and
then see how many objects can you find showing the circle’s terms they
already know

AutoDraw
Previous knowledge Introduce the angle subtended by a chord at a point in a circle. Ask the
children to draw a circle and any two chords and measure the angles
made by them at the center.
Methodology Activity 1:
Ask students to paste a white paper on the cardboard and draw a circle
with center O on this paper. Now make two equal chords on a circle using
compasses. Joining end points of both the chords they will get 2 triangles.
Trace one of the triangles on the tracing paper. Place this obtained triangle
on the other triangle such that the chords overlap. They will observe that
both triangles completely overlap. Thus both triangles are congruent and Google Maps
angle subtended by both angles are equal. Thus equal chords subtend
equal angles at the center.
Activity 2:
Proof of theorem will be explained to the students.
Activity 3:
Ask students whether the converse is also true. Find by following the
above procedure but this time they will make equal angles not equal
chords.
Activity 4:
Ask students to apply their understanding of theorem to attempt
questions of Exercise 10.2.
Learning Outcomes ● The students will understand the concept of Circles.
● The students will discover the relationship between angles subtended
by the chords at the centre.
● The students will be able to prove the theorem using triangle
properties.
● The students will apply the theorem in solving problems.
Follow up Activities Activity: Ask students to go to https://goart.fotor.com/Create a beautiful
art by uploading a photo of a circle showing angles made by equal chords
at the centre and applying different art styles with this AI image generator.
It uses an algorithm inspired by the human brain. It uses the stylistic
elements of one image to draw the content of another.
Activity
Ask students to make a model showing the above theorem. Goart
Ask them to present to small groups.
Let them assess how correct they are in their presentations
Reflections Teacher will see how well the students are able to capture the concept
and gain confidence while presenting their model.
Teacher will discuss with students-
● How do you like the AI tool?
● Do you know any other tool that you can use in your concept?
● Try using the tools at home.

  
SELF ASSESSMENT PAPER - 04

Time: 1 hour MM: 30

UNIT-IV
I. Multiple Choice Questions [1×6 = 6]
1. ABCD is a rhombus such that ÐACB = 60°. Then ÐADB is
(A) 40° (B) 30° (C) 60° (D) 45°
2. If in the given figure ÐABC = 50°, then ÐADC = ?

(A) 40° (B) 30° (C) 50° (D) 20°


3. Find the supplement of 45°
(A) 45° (B) 145° (C) 135° (D) 125°
4. A triangle whose all three sides are unequal is called
(A) Scalene D (B) Isosceles D (C) Equilateral D (D) Right D
II. Assertion and Reason Based MCQs
Directions: In the following questions, A statement of Assertion (A) is followed by a statement of Reason (R).
Mark the correct choice as.
(A) Both A and R are true and R is the correct explanation of A.
(B) Both A and R are true but R is NOT the correct explanation of A.
(C) A is true but R is false.
(D) A is false and R is True.
1. Assertion (A): According to Euclid's 1st Axiom "Things which are equal to the same thing are also equal to one another."
Reason (R): If AB = PQ and PQ = XY, then AB = XY.
2. Assertion (A): In a cyclic quadrilateral ABCD, ÐA – ÐC = 60°, then the smaller of the two is 60°.
Reason (R): Opposite angles of cyclic quadrilateral are equal.
III. Very Short Answer Type questions [1×5= 5]
1. Solve the equation x + 5 = 12 and state Euclid‘s axiom used.
2. Write the complementary angle of 25°.
3. Three angles of a quadrilateral are 70°, 85° and 90°. Find the fourth angle.
4. In the given figure, if ∠DAB = 55°, ∠ABD = 45°, then find ∠ACB.

5. Calculate the value of x is the figure given below :


SELF ASSESSMENT PAPER 141
IV.Short Answer Type questions–I [2×2=4]
1.If angles of a quadrilateral are is the ratio 2 : 5 : 6 : 7. Find the measure of all the angles of a quadrilateral.
2.In a DABC, ∠A + ∠B = 70° and ∠B + ∠C = 150°. Find the angles.
V.Short Answer Type questions -II [3×2 = 6]
1.The diagonals AC and BD of a parallelogram, (AC, BD) intersect each other at O. If ∠DAC = 42° and ∠AOB = 80°,
find ∠DBC. 
2. In a right angled triangle, prove that the hypotenuse is the longest side.
VI. Long Answer Type questions [5×1 = 5]
1. If two equal chords of a circle intersect within the circle, prove that the segment of one chord are equal to
corresponding segments of the other chord.

VII. Case Study Based Questions (Attempt Any 4 parts) [1×4 = 4]


In the given figure, PQ = QR = RS and ∠PQR = 132°. Give the answer of the following questions :

1. ∠QPR is equal to
(A) 48° (B) 24° (C) 72° (D) 90°
2. What is the value of ∠PTQ ?
(A) 96° (B) 66° (C) 24° (D) 48°
3. The value of ∠PTS is
(A) 48° (B) 132° (C) 66° (D) 72°
4. The measure of ∠ROS is
(A) 48° (B) 96° (C) 90° (D) 42°
5. Which of the following relation is true ?
(A) OR > OS (B) OR < OS (C) OR = OS (D) None of these

qq
UNIT-V MENSURATION Study Time:
Max. Time: 2:30 Hrs
Max. Questions: 38
CHAPTER

10 AREAS

Area of a triangle using Heron‘s formula (without proof)


Syllabus

List of Topics
Topic-1: Area of Triangle
Page No. 142
Topic-2 : Heron's Formula
Topic-1 Area of Triangle Page No. 146

Revision Notes
 Parts of a Triangle
In ∆ABC, there are :
(i) three vertices, namely A, B and C.
Scan to know
(ii) three angles, namely ∠A, ∠B and ∠C. more about
this topic
(iii) three sides, namely AB, BC and CA.
1
 Area = × base × corresponding height.
2

 For an equilateral triangle of side ‘a’. Area of Triangle

3 2
(i) Area = a
4

(ii) Perimeter = 3a

3
(iii) Altitude = a
2
Example: Find the area of an equilateral triangle with side 9 cm.

3 3
Solution: Area of an equilateral triangle = × a2 = × (9)2
4 4
81 3
= cm2
4
For an isosceles triangle with length of two equal sides as ‘a’ and base ‘b’.
b
(i) Area = 4 a2 − b2
4

(ii) Perimeter = 2a+ b


AREAS 143
144 Oswaal CBSE Question Bank Chapterwise & Topicwise, MATHEMATICS, Class-IX

1
(iii) Altitude = 4 a2 − b2
2

 For right angled triangle, with ‘a’ and ‘b’ are the sides that includes the right angle.
1
(i) Area = × a × b
2

(ii) Perimeter = (a + b +
a2 + b2 )

(iii) Altitude = a

Example 1
The base of a right triangle is 15 cm and its AB2 = AC2 – BC2
hypotenuse is 25 cm, then calculate its area. = (25)2 – (15)2
Solution: = 625 – 225 = 400
Step I : We find the height (perpendicular) of AB = 20 cm
the right angled triangle by using pythagoras Step II : Now we find area of triangle by using
theorem.
1
Area = × base × height
2
é 1 ù
ê A = 2 ´ b ´ hú
ë û
1
= × 15 × 20
2
= 150 cm2

SUBJECTIVE TYPE QUESTIONS


or, 2x2 = 25 × 2
Very Short Answer Type or, x = 5 cm. 1
Questions (1 mark each) Q. 3. What is the area of DABC in which AB = BC = 4
cm and ∠B = 90° ? U
Q. 1. The sides of a triangular plot are in the ratio
Sol.
4 : 5 : 6 and its perimeter is 150 cm, then find its
sides. U
Sol. Let sides are 4x, 5x and 6x
Then, Perimeter = 4x + 5x + 6x = 150
or, 15x = 150
or, x = 10
1
\ Sides are 40 cm, 50 cm and 60 cm. 1 Area of DABC = ×4×4
2
Q. 2. Calculate the side of an isosceles right triangle of
= 8 cm2. 1
hypotenuse 5 2 cm. U
Q. 4. The base of a right triangle is 6 cm and hypotenuse
Sol. is 10 cm. What will be its area ?
U [NCERT Exemplar]
Sol.

In right triangle ABC.


( )
2
x2 + x2 = 5 2
AREAS 145
By Pythagoras theorem,
AB = AC 2 − BC 2 = 10 2 − 6 2
AC2 = AB2+ BC2
100 − 36 = 64 = 8 cm . = a2 + a2
=
= 2a2
1
Area of right triangle= × base × height or, AC = 2 a unit
2
1 Perimeter of ∆ABC = AB + BC + CA
= × 6×8
2 =a+a+ 2a
48 = 2a + 2a
= = 24 cm2 1
2 = a(2 + 2 ) unit 2
Q. 5. Find out the area of an isosceles triangle whose
base is ‘a’ and equal sides are of length ‘b’. A Long Answer Type
Sol. Let 'b' be the equal sides length and 'a' be the base. Questions (5 marks each)
Q. 1. From a point in the interior of an equilateral
triangle, perpendiculars are drawn on the three
sides. The length of the perpendiculars are 14
cm, 10 cm and 6 cm. Find the area of the triangle.
 A [NCERT Exemplar]
Sol. Join OA, OB and OC.
2
 a 4b2 − a2
x= b2 −   =
 2 2
1
Area of triangle ABC =
× base × height
2
1
= ×a×x
2
1 4b 2 − a2
×a× = ½
2 2
Let, sides of equilateral triangle be ‘a’ cm.
a 2 2
= 4 b − a (units)2 1 1
4 ar∆OAB = × AB × OP
2
Short Answer Type 1
= × a × 14
Questions-I (2 marks each) 2

Q. 1. If the area of an equilateral triangle is 81 3 cm . 2 = 7a cm2 ...(i) ½
Find its perimeter. U [Board Term I, 2012] 1
ar∆OAC = × AC × OR
2
3 2
Sol. Area of an equilateral triangle = a ½
4 1
= ×a×6
3 2 2
∴ a = 81 3 1
4 = 3a cm2 ...(ii) ½
or, a2 = 81 × 4
a = 9 × 2 = 18 cm 1
ar∆OBC = × BC × OQ
Perimeter of equilateral triangle 2
= 3a = 3 × 18 = 54 cm. ½
[CBSE Marking Scheme, 2012] 1
= × a × 10
Q. 2. For an isosceles right angled triangle having each 2

of equal sides a, find the perimeter. A = 5a cm2 ...(iii) ½
Sol. In right angled ∆ABC, Adding, (i), (ii) and (iii), we get
ar∆OAB + ar∆OAC + ar∆OBC
= (7a + 3a + 5a) cm2
= 15a cm2 1
∴ Area of equilateral DABC

= 15a cm2
146 Oswaal CBSE Question Bank Chapterwise & Topicwise, MATHEMATICS, Class-IX

3 2 3
( )
2
or, a = 15a ∴ Area = × 20 3
4 4

15 × 4 3 3
or, a = × = 20 3 cm 1 = × 400 × 3 = 300 3 cm2. 1
3 3 4

Topic-2 Heron’s Formula

Revision Notes
 Heron's Formula
Scan to know

Consider a triangle with sides a, b and c more about
this topic
Let AB = c, BC = a and CA = b

So, Its perimeter = a + b + c
a+b+c
Semi-perimeter, s=
2
Heron's Formula
Area of triangle = | s( s − a )( s − b )( s − c ) |

This formula is known as ‘Heron’s formula’.
This formula is applicable to all type of triangles whether it is a right triangle or an isosceles or an equilateral
triangle.

Example 2
The side of a triangle are 12 cm, 16 cm, and 20 cm. Step II : Now we find the area of triangle by using
Find its area. Heron's formula
Solution: Area = | s( s - a )( s - b )( s - c )|
Step I : We find the semi-perimeter of a triangle
by using Heron's formula = 24( 24 - 12 )( 24 - 16 )( 24 - 20 )
a+b+c = 24 ´ 12 ´ 8 ´ 4
s=
2
= 96 cm2
12 + 16 + 20
s= = 24 cm
2

SUBJECTIVE TYPE QUESTIONS


a+b+c 13 + 14 + c
Very Short Answer Type Sol. s = ⇒ 18 =
2 2
Questions (1 mark each)
or, c = 36 – 27
Q. 1. If a, b and c are the sides of a triangle and s = semi- = 9 cm. 1
perimeter, then calculate the area of triangle. R
Q. 4. The sides of ∆ABC are 8 cm, 7 cm and 5 cm
Sol. | s( s − a )( s − b )( s − c ) | unit2 1 respectively. Find out its semi-perimeter. R
a+b+c 8+7+5
Q. 2. Write the name of formula for finding the area of a Sol. s = =
2 2
triangle when sides are given. R [Board Term I, 2014]
= 10 cm. 1
Sol. Heron’s Formula. 1
Q. 5. The sides of a triangle are 7 cm, 24 cm and
Q. 3. Two sides of a triangle are 13 cm and 14 cm and
25 cm. What will be its area ? U
its semi-perimeter is 18 cm, then what will be the
third side of the triangle ? R
AREAS 147
7 + 24 + 25
Sol. s = Area of D = | s( s − a )( s − b )( s − c ) |
2
= 28 cm = | 42( 42 − 26 )( 42 − 28 )( 42 − 30 ) |

\
Area = | 28( 28 − 7 )( 28 − 24 )( 28 − 25) | = | 42 × 16 × 14 × 12 |

= 28 × 21 × 4 × 3 = | 14 × 3 × 16 × 14 × 4 × 3 |
= 84 cm2. 1 = 336 cm2 1
[CBSE Marking Scheme, 2016]
Short Answer Type
Q. 3. Sides of a triangle are 70 cm, 80 cm and 90 cm. Find
Questions-I (2 marks each)
its area. (Use 5 = 2.23)
Q. 1. Find the area of a triangle, two sides of which are 8 U [Board Term I, 2014]
cm and 11 cm and the perimeter is 32 cm.
a+b+c
U [KVS 2019] Sol. s =
2
Sol. Let the side AB(c) = 8 cm
70 + 80 + 90
Side AC(b) = 11 cm or, s =
A 2
= 120 cm
11 \
Area = | s( s − a )( s − b )( s − c ) |
cm ½
m
8c

= 120(120 − 70 )(120 − 80 )(120 − 90 )

= 120 × 50 × 40 × 30
B C
Perimeter of DABC= 32 cm = 40 × 3 × 5 × 10 × 4 × 10 × 3 × 10
a + b + c = 32 cm
= 40 × 10 × 3 × 5 ½
a + 8 + 11 = 32
a = 32 – 19 = 13 = 1200 × 2.23

\ a = 13 cm = 2676 cm2 1
Q. 4. The semi perimeter of a triangle is 132 cm and the
32 product of the differences of semi perimeter and
s= = 16
2 its respective sides (in cm) is 13200. Find the area
of triangle. U [Board Term I, 2016]
ar(DABC) = s( s − a )( s − b )( s − c )
Sol.
Here, s = 132 cm,
ar(DABC) = 16(16 − 13)(16 − 8 )(16 − 11) (s – a)(s – b)(s – c) = 13200
Area of D = | s( s − a )( s − b )( s − c ) | 1

= 16 × 3 × 8 × 5 = 132 × 13200

= 132 × 10
= 4×4×2×2×2×3×5 = 1320 cm2 1
[CBSE Marking Scheme, 2016]
= 4 × 2 2 × 3 × 5 Q. 5. The longest side of a right angled triangle is
125 m and one of the remaining two sides is 100 m.
\ ar(DABC) = 8 30 cm 2 2 Find its area using Heron’s formula.

U [Board Term I, 2015]
Q. 2. If the sides of a triangle are 26 cm, 28 cm and
30 cm. Find the area of triangle. Sol. Third side = (125)2 − (100 )2 = 75 1
R [Board Term I, 2016] 300  100 + 75 + 125 
Sol.
Here, a = 26 cm, b = 28 cm, c = 30 cm s=  
2  2 
a+b+c
s = = 150 cm
2
Area of ∆ = 150 × (150 - 125)(150 − 100 )(150 − 75)
26 + 28 + 30 84
= = 1 Area of ∆ = 150 × 25 × 50 × 75 = 3750 m2
2 2
[CBSE Marking Scheme, 2015] 1
= 42 cm
148 Oswaal CBSE Question Bank Chapterwise & Topicwise, MATHEMATICS, Class-IX

Short Answer Type 51 + 37 + 20 108


Sol. s= = 1
2 2
Questions-II (3 marks each)
= 54 m
Q. 1. An umbrella is made by stitching ten triangular
pieces of cloth, each measuring 60 cm, 60 cm and \
Area = | s( s − a )( s − b )( s − c ) |
20 cm. Find the area of the cloth required for the = 54( 54 − 51)( 54 − 37 )( 54 − 20 )
umbrella. A [Board Term I, 2014]

Sol. Area of cloth required = 10 × Area of cloth for one = 54 × 3 × 17 × 34
piece ½
= 9 × 3 × 2 × 3 × 17 × 17 × 2 1
Area of one piece of cloth having sides 60 cm, = 306 m2
60 cm and 20 cm. 306
60 + 60 + 20
No. of rose beds = = 51 1
s= = 70 cm ½ 6
2
Q. 4. The triangular side walls of a flyover have been used
\
Area =| 70(70 − 60 )(70 − 60 )(70 − 20 ) |
for advertisements. The sides of the walls are 122 m,
70 × 10 × 10 × 50 22 m and 120 m. The advertisements yield an earning
=
of Rs 5,000 per m2 per year. A company hired one of its
= 7 × 10 × 10 × 10 × 5 × 10 walls for 3 months. How much rent did it pay?
= 10 × 10 × 35
122m
= 100 35 cm 1
2
22m
Area of cloth required 120m
= 10 × 100 × 35
= 1000 35 cm 1
2

Q. 2. Find the percentage increase in the area of a  U [NCERT Ex. 12.1, Q. 2, Page 202]
triangle, if its each side is doubled. A Sol. = a 122 = m, b 22 = m, c 120 m
Sol. Let a, b, c be the given sides, a + b + c 122 + 22 + 120
Semi perimeter of original triangle = s =
2 2
a+b+c 264
s= = = 132 m
2 2
Area of original triangle : areaof triangle = s( s − a )( s − b )( s − c )

= | s( s − a )( s − b )( s − c ) | 1 = 132(132 − 122)(132 − 22)(132 − 120)



= 132 × 10 × 110 × 12
Again, 2a, 2b, 2c be the new sides, then
= 1320 m 2
Semi-perimeter of new triangle,
 [2]
2 a + 2b + 2 c
s' = Rent of 1 m2 area per year = ` 5,000
2
5000
2( a + b + c ) Rent of 1 m2 area per month =
= = 2s 12
2 Rent of 1320 m2 area for 3 months

\ New Area = | 2 s( 2 s − 2 a )( 2 s − 2b )( 2 s − 2 c ) |  5000 


= ` × 3 × 1320 [1]
 12 
= 2 × s × 2( s − a ) × 2( s − b ) × 2( s − c ) 
The company had to pay = ` 1,650,000
= 4 s( s − a )( s − b )( s − c ) Q. 5. There is a slide in a park. One of its side walls has

been painted in some colour with a message “KEEP
= 4 × original area 1 THE PARK GREEN AND CLEAN”. If the sides
∴ Increase in area = 4 × original area – original area of the wall are 15 m, 11 m and 6 m, find the area
= 3 × original area painted in colour.
3 × original area×100
% Increase in area = 6c
original area m
= 300% 1 m
11 c
Q. 3. The sides of a triangular field are 51 m, 37 m and KEEP THE PARK
20 m. Find the number of rose beds that can be GREEN AND CLEAN
prepared in the field if each rose bed occupies a
space of 6 sq. m. A [Board Term I, 2012] 15 cm
 Ap [NCERT Ex. 12.1, Q. 3, Page 203]
AREAS 149
Sol. a = 11 m, b = 6 m, c = 15 m So, total number of leaves in 2 circles = 2 × 6 = 12
11 + 6 + 15 32 ∴ Area of 12 leaves = (12 × 126) cm2 = 1512 cm2
=s = = 16 m
             2 2 [1] Hence, total area to be painted red
Area painted in colour = s( s − a )( s − b )( s − c ) = 1512 cm2 1½
Q. 2. The sides of a triangular park are 5 m, 7 m and 8 m
= 16(16 − 11)(16 − 6)(16 − 15))
respectively. Find the cost of levelling the park at
= 16 × 5 × 10 × 1
the rate of `10 per m2. (Use 3 = 1·73)
= 4×5× 2
A [Board Term I, 2016]
= 20 2 m 2
Sol. Here, a = 5 m, b = 7 m, c = 8 m
[2] a+b+c
s=
2
Long Answer Type
5 + 7 + 8 20
Questions (5 marks each) = = 1
2 2
Q. 1. Two identical circles with same inside design as s = 10 cm
shown in the figure are to be made at the entrance.
The identical triangular leaves are to be painted Area of D = | s( s − a )( s − b )( s − c ) |
red and the remaining are to be painted green. 10(10 − 5)(10 − 7 )(10 − 8 ) 1
=
Find the total area to be painted red. A
= 10 × 5 × 3 × 2

= 10 3 m ½
2

cm \ Cost of levelling = Rate × Area = 10 × 10 3 1



41 15 cm
28 cm = ` 100 3
= 100 × 1.73 = ` 173 1½
[CBSE Marking Scheme, 2016]
Q. 3. A triangular park has sides 60 m, 40 m and 26 m.
Gardener has to put a fence all around its boundary
Sol. For one identical triangular leaf, let and also plant grass inside.
a = 28 cm, b = 15 cm and c = 41 cm
Find the area in which grass will be planted. Also
a + b + c 28 + 15 + 41 calculate the cost of fencing it with barbed wire at
Also, s= =
2 2 the rate of ` 30 per meter, leaving a spare 2 m wide
84 for a gate on one side. A [Board Term I, 2016]
= = 42 cm 1
2 60 + 40 + 26
Sol. s=
2
41 cm 126
= = 63 1
28 cm 2
Area of D = | s( s − a )( s − b )( s − c ) |
15 cm
Using Heron’s formula, = 63 × 3 × 23 × 37 1

Area of one triangular leaf = 401 m2 (Approx.)


Length of wire needed for fencing
= | s( s − a )( s − b )( s − c ) |
= 60 + 40 + 26 – 2 1
= 42( 42 − 28 )( 42 − 15)( 42 − 41) = 124 m 1

\
Cost of fencing = 124 × 30
= 42 × 14 × 27 × 1 1
= ` 3720 1
= 3 × 14 × 14 × 3 × 9 = 9 × 14
[CBSE Marking Scheme, 2016]
= 126 cm2 1½
There are 6 leaves in a circle.
150 Oswaal CBSE Question Bank Chapterwise & Topicwise, MATHEMATICS, Class-IX

OBJECTIVE TYPE QUESTIONS (1 mark each)

Q. 4. If the area of an equilateral triangle is 16 3 cm2,


A Multiple Choice Questions then the perimeter of the triangle is
(A) 48 cm (B) 24 cm
Q. 1. An isosceles right triangle has area 8 cm2. The (C) 12 cm (D) 36 cm A
length of its hypotenuse is Ans. Option (B) is correct.
Explanation: Area of equilateral ∆ =16 3 cm 2
(A) 32 cm (B) 16 cm
(C) 48 cm (D) 24 cm A 16 3 × 4
=a2 = 64
Ans. Option (A) is correct. 3
Explanation: =a 64 8 cm
=
P = 3 × a = 3 × 8 = 24 cm
Q. 5. The sides of a triangle are 56 cm, 60 cm, and 52 cm
long. Then the area of the triangle is
(A) 1322 cm2 (B) 1311 cm2
(C) 1344 cm 2
(D) 1392 cm2
 A [NCERT Exemp.]
1
∵ Area of isosceles ∆ = × Base × height Ans. Option (C) is correct.
2 Explanation:
1 56 + 60 + 52 168
× ( B )2 = 8 = s = = 84 cm
2 2 2
1 Area = 84(84 − 56)(84 − 60)(84 − 52)
× ( B )2 = 8
2
= 84 × 28 × 24 × 32
B = 16
⇒ 1344 cm 2
= 4 cm
Q. 6. The sides of a triangle are 35 cm, 54 cm and 61 cm,
By Pythagoras theorem respectively. The length of its longest altitude is
( B )2 + ( P )2 =
( H )2 (A) 16 5 cm (B) 10 5 cm
)2 ( 4 )2 + ( 4 )2
( H= (C) 24 5 cm (D) 28 cm

H = 32 cm  A [NCERT Exemp.]
Ans. Option (C) is correct.
Q. 2. The perimeter of an equilateral triangle is 60 m.
Explanation:
The area is
(A) 10 3 m 2 (B) 15 3 m 2

(C) 20 3 m 2 (D) 100 3 m 2 A


Ans. Option (D) is correct.
Explanation: Perimeter of equilateral triangle
= 60 m
3a = 60 m 35 + 54 + 61
s= = 75 cm
a = 20 m 2
3 2 3 Area = s( s − a )( s − b )( s − c )
Its Area = a = × 20 × 20 = 100 3 m 2
4 4 = 75( 75 − 35)( 75 − 54)( 75 − 61)
Q. 3. The area of an equilateral triangle with side 2 3 cm = 75 × 40 × 21 × 14
is
= 420 5 cm 2
(A) 5.196 cm 2
(B) 0.866 cm 2

A
Area = 420 5
(C) 3.496 cm2 (D) 1.732 cm2
1
Ans. Option (A) is correct. × 35 × h = 420 5
Explanation: Area of equilateral triangle 2
h = 24 5 cm
3
( )
2
= × 2 3
4 Q. 7. The area of an isosceles triangle having base 2 cm
3 and the length of one of the equal sides 4 cm, is
= × 4 × 3= 3 3 15 cm2
4 (A) 15 cm 2 (B)
3 × 1.732 =
= 5.196 cm 2 2
AREAS 151
(C) 2 15 cm 2 (D) 4 15 cm 2 1
= × 8 cm × 18 cm
 A [NCERT Exemp.] 2
Ans. Option (A) is correct. = 72 cm2
Explanation: \ Assertion is correct.
A
In case of reason (R):
Reason is correct.
1
Area of a triangle = × Base × Height
2
Hence, Both A and R are true and R is the correct
B 1 cm D C
1 cm
Explanation of A.
= a 4= cm, b 4= cm, c 2 cm
Q. 2. Assertion (A): If the area of an equilateral triangle is
a+b+c 4+4+2
= s = = 5 81 3 cm2, then semi perimeter of triangle is 20 cm.
2 2
Reason (R): Semi-perimeter of a triangle is
Area = s( s − a )( s − b )( s − c )
a+b+c
s= where a, b, c are sides of triangle.
= 5(5 − 4)(5 − 4)(5 − 2) 2
= 5 ×1×1× 3 Ans. Option (D) is correct.
= 15 cm 2 Explanation: In case of assertion (A):
Q. 8. The edges of a triangular board are 6 cm, 8 cm, and 3 2
10 cm. The cost of painting it at the rate of 9 paise Area of equilateral triangle = a
4
per cm2 is
(A) ` 2.00 (B) ` 2.16 3 2
81 3 = a
(C) ` 2.48 (D) ` 3.00 4
 A [NCERT Exemp.] 81 × 4 = a2
Ans. Option (B) is correct.
Þ a = 18 cm
Explanation:
18 + 18 + 18
= a 6=cm, b 8= cm, c 10 cm Now s=
2
a + b + c 6 + 8 + 10 24
=
s = = = 12 cm = 27 cm
2 2 2
\ Assertion is false.
Area = s( s − a )( s − b )( s − c )
In case of reason (R):
= 12(12 − 6)(12 − 8)(12 − 10) a+b+c
as semi-perimeter = where a, b, c are sides
= 12 × 6 × 4 × 2 2
= 24 cm 2 of triangle.
Reason is correct
Cost of painting 24 cm2 = 0.09 × 24 = ` 2.16.
Q. 3. Assertion (A): Area of a rhombus whose side is 20
B Assertion & Reason cm and one diagonal is 24 cm is 384 cm2.
Reason (R): All sides of a rhombus are equal.
Ans. Option (A) is correct.
Directions: In the following questions, a statement
Explanation: In case of assertion (A):
of assertion (A) is followed by a statement of
reason (R). Mark the correct choice as:
(A) Both assertion (A) and reason (R) are true and reason
(R) is the correct explanation of assertion (A).
(B) Both assertion (A) and reason (R) are true but reason
(R) is not the correct explanation of assertion (A).
(C) Assertion (A) is true but reason (R) is false.
(D) Assertion (A) is false but reason (R) is true. Semi perimeter of DABD
Q. 1. Assertion (A): The height of a triangle is 18 cm and a+b+c
s=
its area is 72 cm2. Its base is 8 cm. 2
1 20 + 20 + 24
Reason (R): Area of a triangle = × Base × Height =
2 2
Ans. Option (A) is correct. 64
Explanation: In case of assertion (A): =
2
1
Area of D = × Base × Height = 32 cm
2
152 Oswaal CBSE Question Bank Chapterwise & Topicwise, MATHEMATICS, Class-IX

Area of DABD = s( s − a )( s − b )( s − c )

= 32( 32 − 24 )( 32 − 20 )( 32 − 20 )

= 32 × 8 × 12 × 12
= 8 × 2 × 12 = 192 cm2
\ Area of rhombus ABCD = 2 × Area of DABD
= 2 × 192 = 384 cm2
In case of reason (R):
16 + 16 + 16
Reason is true. s=
2
All sides of a rhombus are equal.
48
\ AB = BC = CD = AD = 20 cm = = 24 cm
2
Hence, Both A and R are true and R is the correct
Explanation of A. Area = s( s − a )( s − b )( s − c )

Q. 4. Assertion (A): Using Heron's formula, area of an = 24( 24 − 16 )( 24 − 16 )( 24 − 16 )


equilateral triangle with side 16 cm is 64 3 cm2. = 24 × 8 × 8 × 8
Reason (R): Heron's formula = s( s − a )( s − b )( s − c ) = 64 3 cm
2

a+b+c \ Assertion is true.


where, s = and a, b, c are sides a triangle.
2 Reason is correct.
Ans. Option (A) is correct. Area of triangle is calculated using Heron's formula
Explanation: In case of assertion (A): = s( s − a )( s − b )( s − c )
Side = 16 cm Hence, Both A and R are true and R is the correct
Explanation of A.

COMPETENCY BASED QUESTIONS (4 marks each)


2x = 20 – 4
A Case based MCQs
2x = 16

Read the following passage and answer any four


x = 16 = 8 cm ½
questions of the following : 2
I. Isosceles triangles were used to construct a bridge in Q. 2. What is the Heron's formula for the area of?
which the base (unequal side) of an isosceles triangle
is 4 cm and its perimeter is 20 cm. (A) | s( s + a )( s − b )( s − c ) |

(B) | s( s + a )( s + b )( s + c ) |

(C) | s( s − a )( s − b )( s − c ) |

(D) | s( s . a )( s . b )( s . c ) |

Ans. Option (C) is correct.
Q. 3. What is the semi perimeter of the highlighted
triangle ?
(A) 30 cm (B) 40 cm
(C) 10 cm (D) 50 cm
Q. 1. What is the length of equal sides ? Ans. Option (C) is correct.
(A) 2 cm (B) 3 cm Explanation:
(C) 8 cm (D) 10 cm Perimeter 20
Required semi perimeter = =
2 2
Ans. Option (C) is correct.
= 10 m
Explanation: Let x cm be the length of equal sides
Q. 4. What is the area of highlighted triangle ?
of the isosceles triangle.
So, x + x + 4 = 20 ½ (A) 4 15 cm 2
(B) 4 cm2

2x + 4 = 20
(C) 15 cm 2 (D) 20 cm2
AREAS 153
Ans. Option (A) is correct. Ans. Option (B) is correct.
Explanation: Since, semi perimeter, Explanation: P = sum of all 3 sides
s = 10 cm = (25 + 26 + 28) cm
Thus, area of the triangle = 79 cm  1
Now, semi-perimeter = 1 ×P
s( s − a )( s − b )( s − c ) = 10(10 − 8 )(10 − 8 )(10 − 4 ) ½ 2




= 10( 2 )( 2 )( 6 ) = 1 × 79
2

= 4 15 cm2 ½ = 39.5 cm
Q. 5. If the sides of a triangle are in the ratio 3 : 5 : 7 1
Q. 2. of AB =
and its perimeter is 300 m. Find its area.  2 
(A) (B) 500 2 m2
100 2 m
2
(A) QR (B) RP
(C) 1500 3 m2 (D) 200 3 m2 (C) QP (D) QC
Ans. Option (C) is correct.
Ans. Option (C) is correct. Explanation: 1 of AB = QP 1
Explanation: Let the sides of a triangle are 2

a = 3x, b = 5x, c = 7x Q. 3. What is the length of RQ ?

then a + b + c = 300 (A) 16 cm (B) 15 cm
(C) 13 cm (D) 14 cm

3x + 5x + 7x = 300
Ans. Option (D) is correct.

15x = 300 Explanation:

x = 20 Length of RQ = 1 of BC

So, a = 60, b = 100, c = 140 2
= 1 × 28

s = a+b+c 2
2
= 14 cm 1
= 300 Q. 4. If colourful rope is to be placed along the sides of

2 small DPQR. What is the length of the rope ?

= 150 km ½ (A) 34.5 cm (B) 39.5 cm
s( s − a )( s − b )( s − c ) (C) 32.5 cm (D) 31.5 cm

Area of triangle =
Ans. Option (B) is correct.
Explanation: Length of rope = Perimeter of DPQR
= 150(150 − 60 )(150 − 100 )(150 − 140 )

= (12.5 + 13 + 14) cm
= 150 × 90 × 50 × 10 = 39.5 cm 1
Q. 5. Area of DPQR = 
= 1500 3 m 2  ½ (A) | s( s − 12.5)( s − 13)( s − 14 ) | cm2

II. Shakshi prepared a Rangoli in triangular shape (B) | s( s − 25)( s − 26 )( s − 28 ) | cm2
on Diwali. She makes a small triangle under a big
triangle as shown in figure. (C) | s( s + 12.5)( s + 13)( s + 14 ) | cm2
(D) | s( s + 25)( s + 26 )( s + 28 ) | cm2

where s is the semi-perimeter of DPQR.
Ans. Option (A) is correct.
Explanation:
s( s − 12.5)( s − 13)( s − 14 ) cm 2 1
Area of DPQR = 

B Cased Based Subjective


Questions
Sides of big triangle are 25 cm, 26 cm and 28 cm.
Also, DPQR is formed by joining mid points of Read the following passage and answers the
sides of DABC. following questions:
Use the above data to help her in resolving below I. The School Principal gave a contract to a company
doubts. to plant grass in the park at the rate of ` 4500 per
Q. 1. What is the semi-perimeter of DABC ? hectare. The sides of a triangular park of a school
(A) 39 cm (B) 39.5 cm are 120m, 100m and 110m.
(C) 40 cm (D) 40.5 cm
154 Oswaal CBSE Question Bank Chapterwise & Topicwise, MATHEMATICS, Class-IX

5m

8m
0m

tor
5 m Inv m
110

6
en
Wa

12

m
re
ho
u
5 m se
B C 3m
100 m Canteen
4m
[CBSE SAS]
Q. 1. Find the area of Inventory? 1
Q. 1. What is the perimeter of the park? 1
Ans. Perimeter = AB + BC + CA 6 + 5 + 5 16
Ans. s= = =8m
2 2
= 120 + 100 + 110
= 330 m Area = s( s - a )( s - b )( s - c )
Q. 2. What is the area of the park? 2

= 8( 8 - 6 )( 8 - 5)( 8 - 5)
Ans. Area of the park = s( s - a )( s - b )( s - c )
= 8´2´3´3
a + b + c 330
s= = = 165 m = 12 m2
2 2
Q. 2. Find area allotted for canteen? 1
Area= 165(165 - 120 )(165 - 100 )(165 - 110 )
5 + 4 + 3 12
Ans. s= = =6m
= 165 ´ 45 ´ 65 ´ 55 2 2

= 825 39 Area = 6( 6 - 5)( 6 - 4 )( 6 - 3)

= 5152.12 m2 (approx)
= 6 ´1´ 2 ´ 3
Q. 3. How much does the school have to pay to the = 6 m2
company? Give you answer to the nearest ` 100. 1
Q. 3. Find cost of whole land at the rate of ` 500 per
Ans. Cost of planting = Area × Rate m2? 2
Area = 5152.12 m2 Ans. Area of land = Area of canteen + Area of inventory

= 0.5152 hectare + Area of warehouse
( 1 hect = 10,000 m2) 8 + 5 + 5 18
Area of warehouse = s = = =9
Cost = 4500 × 0.5152 2 2
= 2318

= 9( 9 - 8 )( 9 - 5)( 9 - 5)
= ` 2300 (to nearest hundred)
II. Ajay bought some land for carrying out his = 9 ´1´ 4 ´ 4
wholesale business as shown in the figure below. = 12 m2
He plans to divide this land into 3 parts for
Total Area = 12 + 6 + 12
warehouse, inventory, and canteen. Now using the
= 30 m2
given information, answer the following questions.
Cost = 30 × 500
= ` 15000

  
Study Time:
Max. Time: 2.30 Hrs
Max. Questions: 39
CHAPTER

11
SURFACE
AREAS AND
VOLUMES
Syllabus Surface areas and volumes of spheres (including hemispheres) and right cones.

List of Topics
Topic-1: Surface Area and Vol-
ume of Sphere (Including Hemi-
Surface Area and Volume of sphere) Page No. 155

Topic-1 Sphere (Including Topic-2 : Surface Area and Vol-


ume of Right circular Cone.
Hemisphere) Page No. 160

Revision Notes
 Sphere: A sphere is a perfectly round geometrical object in three-dimensional space, such as the shape of a round
ball.

 Hemisphere: A hemisphere is half of a sphere

 The total surface area of any object will be equal or greater than its curved surface area.
 Volume is the capacity or the space occupied by a body.
Scan to know
 The unit of measurement of both volume and capacity is cubic unit such as cubic feet, cubic cm more about
and cubic m, etc. this topic

 When an object of certain volume is recast into a new shape, the volume of the new shape, formed
will always be equal to the volume of the original object.
 The solids having the same curved surface do not necessarily occupy the same volume.
 When an object is dropped into a liquid, the volume of the displaced liquid is equal to the volume Surface area
of sphere and
of the object that is dipped. hemisphere
Important Formulae
 Sphere :
Surface area = 4pr2
4
Volume = pr3
3
156 Oswaal CBSE Question Bank Chapterwise & Topicwise, MATHEMATICS, Class-IX
SURFACE AREAS AND VOLUMES 157
 Hemisphere :
Curved surface area = 2pr2
Total surface Area = 3pr2
2
Volume = pr3
3

SUBJECTIVE TYPE QUESTIONS


If r denotes radius of a football and r’ that of a
Very Short Answer Type cricket ball, then we have
Questions (1 mark each) 2r = 5 × (2r’)
Q. 1. Compute the curved surface area of a hemisphere 2r
=5
whose diameter is 14 cm. 2r '
R [Board Term II, 2015, NCERT] r
Sol. Given diameter of hemisphere = 14 cm or =5 ½
r'
\ Curved surface area 1
Now, ratio of surface areas
= 2pr2 2
22 4 πr 2 r 25
= =  =
= 2 × × 7 × 7 = 308 cm2 4 π( r ')2  r '  1
7
Q. 2. If the number of square centimetres in the surface = 25 : 1 ½
area of a sphere is equal to the number of cubic cm in
its volume. Find the diameter of the sphere ? Short Answer Type
A [Board Term II, 2014] Questions-I (2 marks each)
Sol. Given, Area of sphere = Volume of Sphere Q. 1. Find the radius of a sphere whose surface area is
4 616 cm2. R [Board Term II, 2016]
4pr2 = pr3
3 Sol. Surface area of sphere = 4pr2
\ 4pr2 = 616
where r is the radius of sphere or, pr2 = 154 1
or, r = 3 cm [on solving]
154 × 7  22 
\ Diameter = 2r = 6 cm 1 or, r2 = ∵ π = 
22 7
Q. 3. Find the amount of water displaced by a solid
or, r2 = 49
spherical ball of diameter 4.2 cm, when it is \ r =7
completely immersed in water. Hence, the radius of sphere is 7 cm. 1
U [Board Term II, KVS 2016] Q. 2. Find the volume of a sphere whose surface area is
Sol. Amount of water displaced 154 cm2. R [Board Term II, 2015, NCERT]
= Volume of solid spherical ball
4 3 4 3
\ Volume of solid spherical ball = πr Sol. Volume = pr
3 3
But surface area = 4pr2 = 154 1
4.2
r= = 2.1 cm (given) 154 × 7  22 
2 154
r2 = = ∵ p = 7 
4p 4 × 22  
4
\ Volume of solid spherical ball = π(2.1)3
3 7
r=
2
4 22
= × × ( 2.1)3 cm3
3 7 539
\ Volume = or 179.67 cm3 (Approx) 1
3
38808
= cm 3 [CBSE Marking Scheme, 2015]
1000
\ Amount of water displaced = 38.808 cm3 1 Q. 3. The total surface area of a solid hemisphere is
5940 cm2. Find the diameter of the hemisphere.
= 38.808 ml ( 1 cm3 = 1 ml)
 U [Board Term II, 2012]
Q. 4. The diameter of a football is five times the diameter Sol. Let the radius of hemisphere be r
of a cricket ball. Ratio of surface areas of football and ∴ 3pr2 = 5940 1
cricket ball is .................. . A [Board Term II, 2013] 5940 × 7
or, r =
2
= 630
Sol. Given, diameter of football = 3 × 22
5 × diameter of cricket ball or, r= 630 or 3 70 cm.
158 Oswaal CBSE Question Bank Chapterwise & Topicwise, MATHEMATICS, Class-IX

So, d = 2r = 6 70 cm. 1 or, r2 = 39.69


Hence, diameter of the hemisphere is 6 70 cm. or, r = 6.3 m
\ Volume of air inside it
Q. 4. A solid shotput is a metallic sphere of radius 2
4.9 cm. Find the volume of the shotput. = πr 3 1
3
U [Board Term II, 2012]
2 22
OR = × × 6.3 × 6.3 × 6.3
3 7
A solid shotput is a metallic sphere of radius
4.9 cm. Find the mass of shotput, if density is 7.8 = 523.90 m3. 1
[CBSE Marking Scheme, 2015]
gm/cm3. [Board Term II, 2012, NCERT]
Sol. Given: r = 4.9 cm Q. 3. The internal and external diameters of a hollow
4 hemispherical vessel are 24 cm and 25 cm
Volume V = πr3 respectively. If the cost of painting 1 cm2 of the
3
surface area is ` 0.05, find the total cost of painting
4 22
= × × 4.9 × 4.9 × 4.9 1 the vessel all over. A [Board Term II, 2013]
3 7
Sol. Internal radius (r) = 12 cm
= 493 cm3 (approx.)

External radius (R) = 12.5 cm
Mass of the shot-put (V × d)
= 3845.44 gm S.A. = 2πr2 + 2πR2 + π(R2 – r2) 1
= 3.85 kg (Approx.) 1 = 2π(r + R ) + π(R – r)(R + r)
2 2

= 2π (144 + 156.25) + π (12.5 + 12)( 12.5 – 12)


Short Answer Type
22
Questions-II (3 marks each) = (600.50 + 12.25) ×
7
Q. 1. How many litres of milk can a hemispherical bowl
of diameter 10.5 cm hold ? (Use p = 3.14) = 1925.79 cm2 1
U [Board Term II, KVS, 2016] Cost of painting 1925.79 cm2 at the rate of ` 0.05/cm2
Sol. Given, diameter of hemispherical bowl = 10.5 cm = 1925.79 × 0.05
\ Radius of hemispherical bowl
= ` 96.29 1
10.5
r= = 5.25 cm 1 Q. 4. A hemispherical bowl made of brass has inner
2
diameter 0·105 m. Find the cost of tin-plating it on
2 3 the inside at the rate of ` 16 per 100 cm2.
Volume of hemispherical bowl = pr
3 U [Board Term II, 2017]
2 Sol. Diameter of hemispherical bowl
= × 3.14 × (5.25)3
3 = 0·105 m
= 302.91 \
Its radius = 0·0525 m = 5·25 cm
= 303 cm3 1
CSA of hemisphere = 2pr2 1
\ Amount of milk that the hemispherical bowl can
22 525 525
hold
= 2× × × cm2
7 100 100
= 0.303 litres 1
Q. 2. A dome of a building is in the form of a hemisphere

= 173·25 cm2 1
From inside, it was white washed at the cost of 16

Total cost = 173·25 × = ` 27·72
` 997.92. If the cost of white washing is 400 paisa 100
per square meter, find the volume of air inside the
1
22
dome.  Take π =  Q. 5. Metallic spheres of radii 6 m, 8 m and 10 m,
 7
respectively are melted to form a single solid
U [Board Term II, 2013, 2015] sphere. Find the radius of the resulting sphere.
Sol. Cost of white washing hemispherical dome  U [Board Term II, 2017]
= ` 997.92 Sol. Let r1 = 6 m, r2 = 8 m, r3 = 10 m
Cost of white washing per square meter
= 400 paise
Let the radius of resulting sphere be R. Then volume
=`4
of resulting sphere = sum of volume of small spheres
\ CSA = 997.92 ÷ 4 = 249.48 m2 4 3 4 4 4
2pr2 = 249.48 1
i.e., pR = πr 3 + πr 3 + πr 3 1
3 3 1 3 2 3 3
22 2
2× × r = 249.48 4 3 3 3
7 = π( r + r2 + r3 )

3 1
SURFACE AREAS AND VOLUMES 159
4 4 d
3
= π( 6 3 + 8 3 + 10 3 ) 1 = π 

3 3 2
4
= π( 216 + 512 + 1000 ) 1 4

3 = × πd 3 1
8 3
4
= π(1728) 1 4

3 × πd 3 ×
Volume of moon 512 3

or, R3 = 1728 =
Volume of earth 1 4 3
3 × πd

R = 1728 8 3

= 12 m 1
1
=
64
Long Answer Type
Questions (5 marks each) or, Volume of moon =
1
(Volume of earth)
Q. 1. The water for a industry is stored in a hemispherical 64
tank of internal diameter 14 m. The tank contains Volume of earth 64
40 kilolitres of water. Water is pumped into the or, = 1
Volume of moon 1
tank to fill it to full capacity. Calculate the volume
of water pumped into the tank. Q. 3. A hemispherical dome, open at base is made from
 U [Board Term II, 2016] sheet of fibre. If the diameter of hemispherical
Sol. Volume of hemispherical tank 13
dome is 80 cm and of sheet actually used was
2 3 170
= πr wasted in making the dome, then find the cost of
3
dome at the rate of ` 35/100 cm2
2 22
= × × 7 × 7 × 7 m3 A [Board Term II, 2014]
3 7
Sol. Given, Diameter = 80 cm
2156 3
= m ∴ r = 40 cm
3
= 718.67 m3
C.S.A. of the dome = 2πr2
= 718.67 kl. (Approx) 2½ 22

= 2× × 40 × 40
Volume of water already present = 40 kl 7
\Volume to be pumped = 718.67 – 40 70400
= 678.67 m3 (Approx) 2½ C.S.A. = cm2 1
7
[CBSE Marking Scheme, 2016]
13
Q. 2. The diameter of the moon is approximately one- since, of sheet was wasted,
170
fourth the diameter of earth. What fraction of
volume of earth is the volume of moon ? Area of sheet wasted
A [KVS 2014, NCERT] 13 70400
= ×
Sol. Let the diameter of earth be d. 170 7
d
∴ The radius of the earth will be, r1 = 1 915200
2 = cm2 1
1190
d
Diameter of moon will be and radius of moon 70400 915200
4 ∴ Total area = +
7 1190
d
(r2) = 1 = 10826.21 cm2 1
8
Cost of sheet per square metre
4 3
Volume of moon = πr2 35
3 = ` 1
100
3
4 d
= π  ∴Total cost of sheet
3 8
35
= × 10826.21
1 4 100
= × πd 3 × 1
512 3
= ` 3789.17. 1
4
Volume of earth = πr13
3
160 Oswaal CBSE Question Bank Chapterwise & Topicwise, MATHEMATICS, Class-IX

Topic-2 Surface Area and Volume of Right Circular Cone

Revision Notes
 Cone: Cone is a pyramid with a circular base. Scan to know
more about
this topic

Surface Area of
Cone

 Right circular cone : Scan to know


more about
Slant height (l) = h2 + r 2 this topic

Area of curved surface = prl = pr h 2 + r 2


Total surface area = Area of curved surface + Area of base
= prl + pr2 = pr(l + r) Volume of Cone
1
Volume = pr2h
3

Example 1
There are two cones. The ratio of their radii are r1
4 : 1 Also, the slant height of the second cone is or, = 4 and
r2 1
twice that of the former. Find the relationship
between their curved surface area. l1 1
l2 = 2l1 or, =
Solution: l2 2
Step I : First consider the unknown variables.
Let r1 and l1 be the radius and slant height of first CSA 1 πr1l1  r1   l1 
\ = =
cone. CSA 2 πr2l2  r2   l2 

Let r2 and l2 be the radius and slant height of
second cone. 41
=   
Step II : Write the formula for curved surface 12

area for both the cones.
Curved surface area of first cone (CSA1) = pr1l1 2
=
and curved surface area of second cone (CSA2) 1
= pr2l2
\ CSA1= 2CSA2 i.e., curved surface area of first

Step III : Use the given condition and simplify cone is twice of the second cone.
it.

According to the question,
r1 : r2 = 4 : 1

SUBJECTIVE TYPE QUESTIONS


1 22
Very Short Answer Type = × × ( 6 )2 × 7
3 7
Questions (1 mark each) 1 22
= × × 36 × 7
Q. 1. Find the volume of a right circular cone with 3 7
radius 6 cm and height 7 cm.
= 264 cm3 1
R [Board Term II, 2012, NCERT]
1 Q. 2. If the height and the radius of cone is tripled,
Sol. Volume of right circular cone = πr 2 h then find ratio of volume of new cone and that of
3
original. A [Board Term II, 2016]
SURFACE AREAS AND VOLUMES 161
Sol. Let h and r be the height and radius of original cone
and let h' and r' be the height and radius of new cone. \ l= r 2 + h2
Given, h' = 3h and r' = 3r = (12 )2 + (16 )2

1
Volume of original cone, V = πr 2 h = 144 + 256
3
= 400
2 1
Volume of new cone, V' = 3 π( r ') h ' = 20 cm 1
\ T.S.A. = pr(l + r) = 3.14 × 12 (20 + 12)
1 = 3.14 × 12 × 32
π( r ')2 h '
Volume of new cone 3 = 1205.76 cm2. 2
= 1 2
Volume of original cone πr h Q. 2. The radius and height of a right circular cone are
3 in the ratio 4 : 3 and its volume is 2156 cm3. Find the
curved surface area of the cone.
( r ')2 h ' ( 3r )2 3 h U [Board Term II, 2013]
= 2 =
r h r 2h Sol. Let the radius of the cone = 4x
27 and the height of the cone = 3x
= 1 Volume of the cone = 2156 cm3
1  1 2
Hence, the ratio of new cone to the original cone is or, πr h = 2156
3
27 : 1. 1 22
or, × × 4x × 4x × 3x = 2156
Short Answer Type 3 7
Questions-I (2 marks each) 22
or, × 16x3 = 2156 1
7
Q. 1. The radius and slant height of a cone are in the 3
ratio 4 : 7. If its curved surface area is 792 cm2, find 7 ×7 ×7 7 
or, x3 = = 
its radius. U [Board Term II, 2017] 2×2×2 2
Sol. Let the radius of a cone 7
∴ x= = 3.5 cm
r = 4x 2
and slant height ∴ Radius of the cone r = 4x = 4 × 3.5 = 14 cm
l = 7x Height of the cone h = 3x = 3 × 3.5 = 10.5 cm
∵ CSA = 792 cm2 1 Slant height of the cone
∵ prl = 792
l= h 2 + r 2 = 10.5 2 + 14 2
22
or, × 4 x × 7 x = 792 l = 17.5 cm
7
C.S.A of the cone = πrl
1
792 × 7 22

x2 = =9 = × 14 × 17.5 cm2
22 × 4 × 7 7
or, x = 3 cm = 44 × 17.5 cm2
\ radius = 4 × 3 \ C.S.A of the cone = 770 cm2. 1
= 12 cm 1 Q. 3. A joker's cap is in the form of right circular cone of
Q. 2. How much ice-cream can be put into a cone with base radius 7 cm and slant height 25 cm. Find the
base radius 3·5 cm and height 12 cm ? area of sheet required for 10 such caps.
U [Board Term II, 2017] U [Board Term II, KVS 2014]
Sol. r = 3·5 cm, h = 12 cm OR
1 A joker's cap is in the form of right circular cone of
\ Amount of ice-cream = πr 2 h 1 base radius 7 cm and height 24 cm. Find the area of
3
the sheet required to make 10 such caps. [NCERT]
1 22 Sol. Given, l = 25 cm, r = 7 cm
= × × 3·5 × 3·5 × 12

3 7 l2 = h2 + r2
or, h2 = l2 – r2

= 154 cm3 1
or, h2 = (25)2 – (7)2
Short Answer Type or, h = 24 cm
Area required = C.S.A of cone 1
Questions-II (3 marks each) = prl
Q. 1. The height of a cone is 16 cm and its base radius is 22
= × 7 × 25
12 cm. Find the total surface area of the cone. 7
(Use p = 3.14) U [NCERT] [Board Term II, KVS, 2016] = 550 cm2 1
Sol. Total Surface area of cone = pr(l + r) Area required to make 10 such caps
Given, r = 12 cm and h = 16 cm = 10 × 550 = 5500 cm2 1
= 0.55 m2.
162 Oswaal CBSE Question Bank Chapterwise & Topicwise, MATHEMATICS, Class-IX

Q. 4. Bhavya has a piece of canvas whose area is 552 Q. 2. A right triangle ABC with sides 5 cm, 12 cm and
m2. She uses it to make a conical tent with a base 13 cm is revolved about the side 5 cm. Find the
radius of 7 m. Assuming that all the stitching volume of the solid so obtained. If, it is revolved
margins and the wastage incurred while cutting about the side 12 cm, what would be the ratio of
amounts to approximately 2 m2. Find the volume volumes of two solids obtained in two cases ?
of the tent that can be made with it.
22  A [Board Term II, 2014, NCERT]
(Take π = ) A [Board Term II, 2014, 2012]
7 Sol. Case I : When revolved about the side 5 cm.
Sol. Curved surface area of the tent = 552 – 2 = 550 m2 ½ Here, r = 12 cm, h = 5 cm 1
Radius (r) = 7 m 1 2 1
∴ π × 7 × l = 550 ½ Volume = πr h = π × (12 )2 × 5 .
3 3
or, l = 25 m
= 240π cm3 1
 h= l2 − r2

∴ h= 252 − 7 2
5 13
= 24 m ½
1 22
Volume of the tent = × × 7 × 7 × 24 ½
3 7
= 1232 m3. 1
12
Long Answer Type Case II : When revolved about 12 cm,
Questions (5 marks each) r = 5 cm, h = 12 cm 1
Q. 1. A right angled DABC with sides 3 cm, 4 cm and 1 2
∴ Volume = πr h
5 cm is revolved about the fixed side of 4 cm. Find 3
the volume of the solid generated. Also, find the
total surface area of the solid.
A [Board Term II, 2015]
12 13
Sol.

r
5

1
= × 5 × 5 × 12 π
3
rcone = 3 cm = 100 π cm3 1
hcone = 4 cm ∴ Ratio of the two volume = 240π : 100π
lcone = 5 cm 1
= 12 : 5 1
Q. 3. What length of tarpaulin 3 m wide will be required
4 cm 5 cm to make conical tent of height 8 m and base radius
6 m ? Assume that the extra length of material that
will be required for stitching margins and wastage
r in cutting is approximately 20 cm (Use p = 3.14).
3 cm 1 Sol. Conical tent : height = 8 m
Above given cone is formed with radius 3 cm, base radius = 6 m
height 4 cm and slant height 5 cm when revolved
about the fixed side of 4 cm.
1
V = πr 2 h
3
1 22
= . .( 3)( 3)( 4 )
3 7
l2 = r2 + h2
= 37.71 cm3
l2 = 82 + 62
Total surface area = prl + pr2 = pr(l + r) 1
22 l= 64 + 36 = 10 m
= × 3( 5 + 3)
7 C.S.A of tent = prl unit2
= 75.43 cm2. 2 C.S.A. of Tent = 3.14 × 6 × 10 m2
[CBSE Marking Scheme, 2015]
= 188.4 m2
SURFACE AREAS AND VOLUMES 163
Area of Tarpaulin = C.S.A of tent Extra length required for stitching and wastage of
width × length of tarpaulin = 188.4 m2 cutting
3 × length of tarpaulin = 188.4 m2 = 20 cm = 0.20 m
188.4 \ Total length of tarpaulin = 62.8 + 0.2 = 63 m
length of tarpaulin = = 62.8 m
3  5

OBJECTIVE TYPE QUESTIONS (1 mark each)

4 3
Volume of sphere = pr
A Multiple Choice Questions 3
4 3
Q. 1. The radius of a hemispherical balloon increases pr1
4 3 64
from 6 cm to 12 cm as air is being pumped into it. \ Ratio = 3 = 3 =
4 3 3 27
The ratios of the surface areas of the balloon in the pr
two cases is 3 2
(A) 1 : 4 (B) 1 : 3 = 64 : 27
(C) 2 : 3 (D) 2 : 1 Q. 4. The volume of a hemisphere is 19404 cm3. The
 A [NCERT Exemp.] total surface area of the hemisphere is
(A) 16632 cm2 (B) 3696 cm2
Ans. Option (A) is correct.
(C) 4158 cm 2
(D) 8316 cm2
Explanation :
 Radius r = 6cm  A [NCERT Exemp.]
Surface area = 3pr2 = 3p62 = 3p × 6 × 6 Ans. Option (C) is correct.
    Radius R = 12 cm 2 3
Explanation : Volume of hemisphere = pr
Surface area = 3pR2 = 3p 122 3
= 3p × 12 × 12 2 3
So, pr = 19404
   Ratio = 3p × 6 × 6 : 3p × 12 × 12 = 1 : 4 3
Q. 2. The radius of a sphere is 2r, and then its volume 19404 ´ 7 ´ 3
will be r3 =
2 ´ 22
4 pr 3
(A) (B) 4pr3 r3 = 441 × 3 × 7
3
r = 21
8 pr 3 32 3
(C) (D) pr Total surface Area of hemisphere = 3pr2
3 3
22
= 3 ´ ´ 21 ´ 21
 A [NCERT Exemp.] 7
Ans. Option (D) is correct. = 4158 cm2
Explanation :
Q. 5. The total surface area of a cone whose radius is 2r
4 3 and slant height 2l is
Volume of the sphere = pr
3 (A) 4pr (l + r) (B) pr (l + 4r)
But radius r = 2r (C) pr (l + r) (D) 2prl
Therefore, volume of the sphere  A [NCERT Exemp.]
4 32 3 Sol. Option (A) is Correct :
= p(2r)3 = pr . Explanation :
3 3
   Radius = 2r
Q. 3. The surface areas of two spheres are in the ratio Slant height = 2l
16 : 9. The ratio of their volumes is     TSA = pr(l + r)
(A) 4 : 3 (B) 64 : 27         = p(2r) (2l + 2r)
(C) 16 : 9 (D) 163 : 93         = p(2r) × 2(l + r)
Ans. Option (B) is correct.         = 4pr(l + r)
Explanation : Q. 6. A cone is 8.4 cm high and the radius of its base is
Surface Aarea of sphere = 4pr2 2.1 cm. It is melted and recast into a sphere. The
radius of the sphere is
4 pr12 16
\ Ratio = = (A) 4.2 cm (B) 2.1 cm
4 pr22 9
(C) 2.4 cm (D) 1.6 cm
r1 16 4  A [NCERT Exemp.]
= =
r2 9 3 Ans. Option (B) is Correct :
164 Oswaal CBSE Question Bank Chapterwise & Topicwise, MATHEMATICS, Class-IX

Explanation.  eason (R): The total surface area of a hemisphere


R
Height h = 8.4 cm is 3pr2.
Radius r = 2.1 cm Ans. Option (A) is correct.
Volume of the sphere = Volume of cone Explanation : In case of Assertion (A):
4 1
pR3 = pr2h Total surface Area = 3pr2
  3 3
22
4 22 1 22 = 3 ´ ´ 7 ´ 7 = 462 cm2
× × R3 = × × 2.1 × 2.1 × 8.4 7
3 7 3 7
Cost of painting = `(465 × 5) = `2310
4 1
× R3 = 2.1 × 2.1 × 8.4 × \ Assertion is true and Reason is also true as well
  3 3 correct explanation of A.
  R3 = (2.1)3
  R = 2.1 cm. Q. 2. Assertion (A): If the volumes of two spheres are in
the ratio 27 : 8 then their surface areas are in the
Q. 7. In a right circular cone, the cross section made by a
plane parallel to the base is a ratio 3 : 2.
4
(A) Sphere (B) Hemisphere Reason (R): Volume of sphere = pr 3
(C) Circle (D) Semicircle 3
Ans. Option (C) is correct. Surface area of a sphere = 4pr2
Q. 8. A triangle having sides equal to 7 cm, 24 cm and 25 Ans. Option (D) is correct.
cm forms a cone when revolved about 24 cm side. Explanation :
What is the volume of a cone formed? 4
Volume of sphere = pr 3
(A) 1225 cm3 (B) 1232 cm3 3
(C) 4000 cm 3
(D) 3696 cm3
4 3
 A [NCERT Exemp.] pr1
27
Ratio of Volume = 3 =
Ans. Option (B) is correct. 4 3 8
pr
Explanation : As shown when the cone is formed, 3 2
its base radius is 14 cm and height is 24 cm.
r13 27
= =
r23 8
cm

r1 3
24 cm \ = =3:2
25

r2 2

4 pr12 32 9
Ratio of surface Area = = =
7 cm 4 pr22 22 4

1 2 = 9 : 4
Volume = pr h \ Assertion is false but Reason is true as volume of
3
4
1 22 sphere = pr 3 and Surface area = 4pr2
= ´ ´ 7 ´ 7 ´ 24 3
3 7
Q. 3. Assertion (A): If the height of cone is 24 cm and
= 1232 cm3
diameter of base is 14 cm, then the slant height of
cone is 25 cm.
Reason (R): If r be radius and h be the slant height
B Assertion & Reason
of cone then the slant height = æç h 2 + r 2 ö÷
è ø
Directions: In the following questions, a statement
of assertion (A) is followed by a statement of Ans. Option (A) is correct.
reason (R). Mark the correct choice as: Explanation : In case of Assertion (A):
(A) Both assertion (A) and reason (R) are true and reason 14
In cone h = 24 cm, r = = 7 cm
(R) is the correct explanation of assertion (A). 2
(B) Both assertion (A) and reason (R) are true but reason
(R) is not the correct explanation of assertion (A).
\ l2 = h2 + r 2
(C) Assertion (A) is true but reason (R) is false. = 24 2 + 7 2
(D) Assertion (A) is false but reason (R) is true.
= 625
Q. 1. Assertion (A): A hemisphere of radius 7 cm is to
be painted outside on the surface. The total cost of l = 25 cm
painting at `5 per cm2 is `2310. \ Assertion is true.
SURFACE AREAS AND VOLUMES 165
In case of Reason (R):
l2 = h2 + r 2
l2 = h2 + r 2
= 4 2 + 32
It is true and correct explanation of Assertion.
Hence, A and R are true and R is correct explanation = 25
of A. l = 5 cm
Q. 4. Assertion (A): The curved surface area of a cone of \ CSA = p × 3 × 5
base radius 3 cm and height 4 cm is (15p) cm2. = (15p) cm2
Reason (R): Volume of a cone = pr2h \ Assertion is true.
Ans. Option (C) is correct. In case of Reason (R) :
Explanation : In case of Assertion (A): 1
Curved surface area = prl Volume = pr 2 h
3
\ Reason is False.

COMPETENCY BASED QUESTIONS


Ans. Option (C) is correct.
A Case based MCQs Explanation : Circumference of the base of the
dome
Read the following passage and answer any four
questions of the following : = 2pr
I. Nikita has to make her project on 'Monument in = 2p(22)
India'. She decided to make her project on Gol = 44p 1
Gumbaz monument. She already knows following Q. 3. Find the cost of painting the dome, given the cost
things about it : of painting is ` 100 per cm2.
 It is located in a small town in Northern Karnataka. (A) ` 980p (B) ` 9690p
 It reaches up to 51 meters in height while the giant (C) ` 968000p (D) ` 9700p
dome has an external diameter of 44 meters, making
it one of the largest domes ever built. Ans. Option (C) is correct.
 At each of the four corners of the cube is a dome Explanation : Curved surface area of hemispherical
shaped octagonal tower seven stories high dome
with a staircase inside. = 968p m2
Cost of painting 100 cm2 = ` 10
Cost of painting 1 m2 = ` 1000 ½
Thus, cost of painting the dome = ` 1000 × 968p cm2
= ` 968000p½
II. A school organised an educational trip to Taj Mahal.
Mathematics teacher of the school took her 9th
standard students to it. The teacher had interest in
history as well. She narrated the facts to Taj Mahal
Help her in making project by answering the to students. Then the teacher said in this monument
following questions : one can find combination of solid figures. There
are 4 pillars which are cylindrical in shape. The Taj
Q. 1. What is the curved surface area of hemispherical
Mahal has a larger white dome surrounded by four
dome ?
smaller domes.
(A) 908p m2 (B) 968p m2
(C) 340p m 2
(D) 780p m2
Ans. Option (B) is correct.
Explanation :
Diameter = 44 m
Radius = 22 m
Curved surface area of hemispherical dome
= 2pr2½
= 2p(22)2
= 968p m2½
Q. 2. What is the circumference of the base of the dome ?
(A) 34p (B) 22p
(C) 44p (D) 55p
166 Oswaal CBSE Question Bank Chapterwise & Topicwise, MATHEMATICS, Class-IX

Q. 1. How much cloth material will be required to 1 2 4


cover 4 small domes each of radius 2 m? pr h = pr 3
3 3
(A) 99 m2 (B) 100.57 m2
(C) 98.23 m 2
(D) 75 m2 1 22 4 22
´ ´ 6 ´ 6 ´ 3 = ´ ´ r3
Ans. Option (B) is correct. 3 7 3 7
Explanation: Required cloth material = 4 × Curved 1 6´6´3´3
surface area of hemispherical domes ½ ´ = r3
3 4
= 4 × 2pr2
22 3 × 3 × 3 = r3
= 4 × 2 × × 22
7 r = 3 cm
22 \ Radius of sphere = 3 cm
2

= 8× ×4
7 II. A farmer Rajesh grows a corn cob in his farm. Corn
22 cob contains valuable Vitamin B, antioxidants,

= 32 ×
7 Carotenoids, lutein and Zeaxanthin which are
useful for body growth.
= 100.57 m2 ½
Q. 2. How much is the volume of a hemisphere if the
radius of the base is 3.5 m?

(A) 87.53 m3 (B) 90.22 m3
20 cm
(C) 75.34 m 3
(D) 89.83 m3
Ans. Option (D) is correct.
Explanation: Volume of the hemisphere
2
= pr 3 2.1 cm
3 A corn cob (above figure). Shaped some what likes
2 22 a cone, has the radius of its broadest end as 2.1 cm
= × × ( 3.5)3

3 7 and length as 20 cm.

= 89.83 m3 1 Then, answer the following questions.
Q. 1. Write the formula to find the curved surface area
Case based Subjective of cone?
B Questions Ans. Curved surface of cone is prl. 1
Q. 2. Slant height of the conical corn cob will be?
Read the following passage and answer the
following questions: Ans. Slant height (l) = h2 + r 2
I. Mr. Kumar a mathematics teacher brings some
l= ( 2.1)2 + ( 20 )2
green clay in the classroom to teach the topic
mensuration. = 4.41 + 400
First he forms a cone of radius 6 cm and height 3 cm
with the clay. Then he mould that clay into a sphere = 404.41
similarly, he mould the sphere into hemisphere. l = 20.11 cm 1
Q. 1. When clay changes into on shape to other what Q. 3. Curved surface area of the corn cob in cm2 is?
remains same. Ans. CSA = prl
Ans. Whenever a shape changes to other 'volume' 22
= ´ 2.1 ´ 20.11
remains the same. 1 7
Q. 2. What is the volume of hemisphere?
= 132.726
2
Ans. Volume of hemisphere = pr 2 1 = 132.73 cm2 2
3
Q. 3. What is the radius of sphere.
Ans. Volume of cone = Volume of sphere
SURFACE AREAS AND VOLUMES 167

Artificial Intelligence
AI
PARAMETERS DESCRIPTION CONCEPTS
INTEGRATED

Chapter Covered Chapter 13: Surface Areas and Volumes


Name of the book Mathematics, Class 9 NCERT

Subject and Artificial Understanding the concept of surface area of solids: cube, cuboid,
Intelligence Integrated right circular cylinder and right circular cone using AI.

Objectives ● To understand the concept of surface area.


● To derive formulas for calculating surface area of given solids
using their nets.
● To calculate the surface area of cube and cuboid using their
formula.
● To calculate the curved surface area of cylinder and cone using
their formula.
● To calculate the total surface area of cylinder and cone using
their formula.
● To estimate the surface area of different prisms.

Time Required 4 sessions of 40 minutes each

Classroom Arrangement Flexible

Material Required Pen, Paper, White Board, Markers, Laptop, Internet Connection.

Pre-Preparation Activity Students will be asked to recall all 2D shapes using Autodraw and
their area and perimeter. Making of 3D shapes with the nets.

Autodraw
Previous knowledge Questioning will be used to check students’ previous knowledge
in the form of a quiz.

Methodology Activity-1:
Identifying Solids from Their Nets and Finding Their Surface Area:
Students will be divided into groups of 4 or 5 and each student
will make cube, cuboid, cylinder and cone using paper folding and
cutting. They will then open and see the nets of each and find the
area of the all 2D shapes obtained. Adding the area of all shapes
of a given net of solid, they will arrive at the formula of surface
area of that solid under the guidance of their teacher.
Exploring Nets of Solids:

Cube Cone Cylinder


Activity-2:
Students will be working in pairs with their partners in computer
lab and individually at their home to understand the formulae
of surface area of solids by using Geogebra tool wherein they
can change measurements of the dimensions and explore
the corresponding change in their SA. This will help them to
understand change in SA in problems related to increase and
decrease of dimensions.
Activity-3:
Model making: Students will be divided into groups of 4 or 5 and
they have to create a model which includes all solids discussed
USING 3D MODELING APP. It can be a classroom scene, factory
model, colony/society model, temple, galaxy model....etc. and
will have to explain the need and use of the solids used to create
it.
168 Oswaal CBSE Question Bank Chapterwise & Topicwise, MATHEMATICS, Class-IX

Activity-4:
Students will be using the formula derived for the SA of solids to
find the surface area of objects in the real life problems of NCERT
exercises and the assignments given.
Activity-5:
Research on shapes of beaker, test Tube, conical flask & gas
cylinders: Students will do research in the use of various laboratory
equipment and their shapes. They will identify the combination of
solids used and will research on why beakers and test tubes are
generally cylindrical but flasks are conical at the bottom and has
cylindrical neck? Also, students will also do research on why gas
cylinders and boilers are cylindrical in shape?

Discussion on the Text Open discussion on all new terms related to Surface Area: Lateral/
Curved Surface Area, Total Surface Area, Cuboid, Cube, right
circular cylinder, Right circular Cone, Right circular Cone, Slant
Height.

Learning Outcomes Students will be able to


● Calculate the surface area of given solids.
● Solve real life word problems involving finding surface area of
the solids done.
● Estimate the change in surface area due to change in their
dimensions.

Self-Evaluation and Follow Teacher will observe students work and give individual feedback.
up Also, models made and research done will be assessed.
Peer assessment: Asking questions to each other in pairs and peer
tutoring wherever required.
Flip teaching
Worksheets/Assignments

  
SELF ASSESSMENT PAPER - 05
Time: 1 hour MM: 30

UNIT-V
I. Multiple Choice Questions
 [1 × 6 = 6]
1. Volume of cone is given by the formula
4 2 1 2
(A) pr2h (B) πr h (C) πr h (D) 2prh
3 3
2. Complete the curved surface area of a hemisphere whose diameter is 42 cm.
(A) 308 cm2 (B) 1232 cm2 (C) 716 cm2 (D) 2772 cm2
3. Find the volume of a cone whose height is 3 cm and radius is 4 cm.
(A) 52.6 cm3 (B) 50.3 cm3 (C) 51.2 cm2 (D) 50 cm3
4. Find the volume of a shot put where mass is 3.85 kg and density is 7.8 g/cm2.
(A) 408 cm2 (B) 490 cm3 (C) 493 cm3 (D) 500 cm3
II. Assertion and Reason Based MCQs

Directions: In the following questions, A statement of Assertion (A) is followed by a statement of Reason (R).
Mark the correct choice as.
(A) Both A and R are true and R is the correct explanation of A.
(B) Both A and R are true but R is NOT the correct explanation of A.
(C) A is true but R is false.
(D) A is false and R is True.
1. Assertion (A): The height of a cone is 15 cm. If its volume is 500p cm3, then the radius of its base is 10 cm.
2
Reason (R): Volume of hemisphere is pr 3
3
2. Assertion (A): Radius 6 cm, slant height 7 cm then the curved surface of right circular cone is 132 cm2
Reason (R): Curved surface of cone is prl.
III. Very Short Answer Type questions [1 × 5 = 5]
1. If area of sphere is equal to volume of the sphere. Then find the diameter of the sphere.
2. Find the amount of water displaced by a solid spherical ball of radius 1.4 cm.
3. Find the volume of right circular cone with radius 7 cm and height 6 cm.
4. How many feed balls each of radius 1 cm can be made from a sphere where radius is 8 cm.
1
5. The volume of a sphere is 113 cm3. Find its diameter.
7

IV. Short Answer Type questions-I [2 × 2 = 4]


1. A spherical ball has 21 cm as diameter, calculate its surface area.
2. Find the area of curved surface of the cone whose slant height is 8 cm and base diameter is 12 cm.
V. Short Answer Type questions -II [3 × 2 = 6]
1. Find the height of a cone whose base radius is 5 cm and volume 50 pcm . 3

2. The volume of a sphere is 4851 cm3. find its surface Area.


VI. Long Answer Type questions [5 × 1 = 5]
1. The radius of a spherical balloon increase from 8 cm to 16 cm as air is pumped into it find the ratio of surface areas
of the balloon in two cases.
2. The cost of polishing the dome at the cost of `270 cm2 of radius is 4.2 cm is.
170 Oswaal CBSE Question Bank Chapterwise & Topicwise, MATHEMATICS, Class-IX

VI. Case Study Based Questions [1 × 4 = 4]



A school took for an education trip to see Museum. The model of Gol gumbaz movement was displayed. The
leader told that the movement is a large dome and the dome is the shape of half of a sphere.
1. How much cloth will be required to cover 2 small domes of radius 4.2 m each?

(A) 52.08 cm2 (B) 52.8 m2 (C) 52 m2 (D) None of these
2. What is the formula of volume of hemispherical dome?
2 3 4 3

(A) πr (B) 2pr2h (C) πr (D) pr2h
3 3
3. What is the volume of hemispherical dome of radius 7 m?

(A) 718.66 cm3 (B) 152.8 m3 (C) 718.66 m3 (D) 56 cm3
4. What is the surface area of hemispherical dome of radius 14 m?

(A) 1234 m2 (B) 1312 m2 (C) 1232 m2 (D) 1223 m2

qq
UNIT-VI Statistics & Probability Study Time:
Maximum time: 2:30 Hrs
Maximum questions: 21
CHAPTER

12 STATISTICS

Syllabus Bar graphs, histograms (with varying base lengths), frequency polygons.

Revision Notes
 Graphical Representation of Data can be represented graphically in following ways :
(a) Bar Graph (b) Histogram (c) Frequency polygon.
 Bar Graph: A bar graph is a pictorial representation of data in which rectangular bars of uniform width are drawn
with equal spacing between them on one axis, usually the x-axis. The value of the variable is shown on the other
axis that is the y-axis.
Following Bar graph depicts number of books sold per month.

Bar Graph Scan to know


500 more about
466 this topic

415
402
400
371
Introduction to
305 305 310 Statistics
302 293
300

200
200

100

0
Jan Feb Mar Apr May June July Aug Sep Oct
Month vs Number of books sold

 Bar charts are used for comparing two or more values.


 Histogram: A histogram is one of the most commonly used graphs. A histogram is a vertical bar-graph with no spacing
between the bars. The histogram is constructed by the following steps :
1. The values of the observations are taken on the x-axis with the class-limits clearly marked.
2. The frequencies are taken along the y-axis.
172 Oswaal CBSE Question Bank Chapterwise & Topicwise, MATHEMATICS, Class-IX
STATISTICS 173

 A histogram is a set of adjacent rectangles whose areas are proportional to the frequencies of a given continuous
frequency distribution. The height of rectangles corresponds to the numerical value of the data and base corre-
sponds to a particular class.
 The histogram is drawn only for exclusive/continuous frequency distributions.
 If classes are not of equal width, then the height of the rectangle is calculated by the ratio of the frequency of that
class, to the width of that class.
 A histogram is different from a bar chart, as in the former case it is the area of the bar that denotes the value, not
the height.
 When the scale on the x-axis starts at a higher value and not from the origin, a kink is indicated near the origin to
signify that the graph is drawn to a scale beginning at a higher value and not at the origin.

 ‘Kinks’ are a tool used to express areas in a graph. In this case, the kink tells us that there is no observation which
takes the value less than 200.
 Frequency Polygon: The frequency polygon of a frequency distribution is a line-graph drawn by plotting the class
marks on the x-axis against the frequencies on the y-axis.
In case of grouped data, where the classes are of equal width, the frequency polygon is obtained by joining the
mid-points of the top edges of the rectangles in the histogram. Two extra lines are drawn by introducing two extra
classes (or values).
 One class is introduced before the first class and the other is introduced after the last class. These classes have zero
frequencies.
 Frequency polygons are used for understanding the shape of distributions.
 If both a histogram and a frequency polygon are to be drawn on the same graph, then first draw the histogram
and then join the mid-points of the tops of the adjacent rectangles in the histogram with line-segments to get the
frequency polygon.
 The cumulative frequency of a class-interval is the sum of frequencies of that class and the classes which preceed
(come before) it.
Range
 Class size =
Number of classes
174 Oswaal CBSE Question Bank Chapterwise & Topicwise, MATHEMATICS, Class-IX

 Class size = Upper limit – Lower Limit


D
20
18
16
14
12
C
10
E
8
B F
6
4
G
2
A H
0 135 145 155 165 175 185 195 205 215

Example 1
The following data on the number of girls (to (iii) What step should be taken to improve the
the nearest ten) per thousand boys in different situation ?
sections of Indian Society is given below : Solution:
Number of girls per Step I : Choose the appropriate data for
Sections of Society horizontal axis (i.e., x-axis) and vertical axis (i.e.,
thousand boys
Schedule Caste (SC) 940 y-axis).
Here, we represent the sections on horizontal
Schedule Tribe (ST) 970
axis choosing any scale, since width of bar is not
Non–SC/ST 920 important but for clarity, we take equal widths
Backward districts 950 for all bars and maintain equal gap between
Non-backward districts 920 them. Let one section be represented by one
unit.
Rural 930
We represent the number of girls per thousand
Urban 910 boys on vertical axis. Here, we can choose the scale
(i) Represent the information above by a bar as 1 unit = 10.
graph. Step II : Draw the graph as per given information.
(ii) In the classroom discuss what conclusions (i) Now, the graph is as shown below according
can be arrived at from the graph? to the given data.

Step III : Draw the conclusion for part (ii). number of girls per thousand boys in urban area.

(ii) From the graph, we observe that in Scheduled Step IV : Suggest one positive step to improve

Tribe (ST), there is maximum number of girls per the situation.
thousand boys among different sections of Indian (iii) Prenatal sex determination should be strictly
Society, i.e., 970 whereas there are minimum banned in urban areas.
STATISTICS 175

SUBJECTIVE TYPE QUESTIONS


Construct a grouped frequency distribution table
Very Short Answer Type with classes 84-88, 88-92 etc.
Questions (1 mark each) R [Board Term II, 2012]
Sol.
Q. 1. In a histogram the class intervals or the groups are
taken along which axis? Class interval Tally Marks Frequency
Sol. Class intervals are taken along x-axis. 1 84 - 88 || 2
Q. 2. What are the graphical representation of statistical 88 - 92 |||| 4
data.
92 - 96 |||| |||| ||| 13
Sol. Histogram, Bar graph and Frequency Polygons are
96 - 100 |||| |||| | 11
all graphical representation of statistical data. 1
Q. 3. What are the facts or information collected with a ½×4=2
definite purpose called? R [Board Term II, 2012]
Sol. Data 1
Short Answer Type
Q. 4. Two consecutive class marks of a distribution are
Questions-II (3 marks each)
52 and 57, Find the class size. Q. 1. A company manufactures car tyres of a particular
R [Board Term II, 2012] type. The lives (in years) of 40 such tyres are as
Sol. 57 – 52 = 5 1 follows :
2.6, 3.0, 3.7, 3.2, 2.2, 4.1, 3.5, 4.5, 3.5, 2.3, 3.2, 3.4, 3.8, 3.2, 4.6,
Short Answer Type 3.7, 2.5, 4.4, 3.4, 3.3, 2.9, 3.0, 4.3, 2.8, 3.5, 3.2, 3.9, 3.2, 3.2, 3.1,
Questions-I (2 marks each) 3.7, 3.4, 4.6, 3.8, 3.2, 2.6, 2.5, 4.2, 2.9, 3.6.
Q. 1. Read the bar graph. Find the percentage of excess Construct a continuous grouped frequency
expenditure on wheat than pulses and ghee taken distribution for the above data of equal class size
together. R and with first class interval as 2-2.5, (2.5 is not
y
Percentage Expenditure

included) R [Board Term II, 2016]


50
Sol.
40
35% Class Number of
Tally Marks
30 Intervals Tyres
20% 20%
20 15% 2.0 - 2.5 || 2
10%
10 2.5 - 3.0 |||| || 7
0 x
Rice Wheat Pulses Ghee Others
3.0 - 3.5 |||| |||| |||| 14
Food Items
Sol. Expenditure on pulses and ghee 3.5 - 4.0 |||| |||| 10
= 10% + 20% = 30% 1
4.0 - 4.5 |||| 4
Expenditure on wheat = 35% ½
4.5 - 5.0 ||| 3
\ Excess expenditure on wheat = 35% – 30% = 5%
Total 40
½
[CBSE Marking Scheme, 2016] 3
Q. 2. The class marks of a distribution are 37, 42, 47, 52,
57. Determine the class size and the class limits of
Q. 2. The blood groups of 30 students of class IX are
one last class mark. U [Board Term II, 2012] recorded as follows :
Sol. Class size = 42 – 37 = 5 1
5 A, B, O, O, AB, O, A, O, B, A, O, B, A, O, O,
Lower limit of last class mark = 57 − = 54.5 ½
2 A, AB, O, A, A, O, O, AB, B, A, O, B, A, B, O.
5 (i) Represent this data in the form of a frequency
Upper limit of last class mark = 57 + = 59.5 ½
2 distribution table.
Q. 3. The relative humidity (in %) of a certain city of (ii) Which is the most common and which is the
month of 30 days was as follows :
rarest blood group among these students ?
98 98 99 90 86 95 92 96 94 95 A [NCERT][Board Term II, 2012]
89 92 97 93 92 95 97 93 95 97
96 92 84 90 95 98 97 96 92 89
176 Oswaal CBSE Question Bank Chapterwise & Topicwise, MATHEMATICS, Class-IX

Sol. (i) Frequency Distribution Table : Q. 5. Two coins were tossed 20 times simultaneously.
Each time the number of "Heads" occurring was
No. of students
Blood group Tally Marks noted down as follows :
(frequency)
0, 1, 1, 2, 0, 1, 2, 0, 0, 1, 2, 2, 0, 2, 1, 0, 1, 1, 0, 2.
A |||| |||| 9
Prepare a frequency distribution table for the
B |||| | 6 data. U [Board Term II, 2017]
O 12 Sol.
|||| |||| ||
AB 3 Observation
||| (Number of Tally marks Frequency
Total 30 2 heads)
(ii) Blood group 'O' is most common as it has 0 |||| || 7
highest frequency i.e., 12. Blood group AB is
rarest. ½+½ 1 |||| || 7
Q. 3. Represent the following frequency distribution by
means of a histogram. R 2 |||| | 6
Marks 10 - 20 20 - 30 30 - 40 40 - 50 50 - 60 60 - 70 Total 20
Number of 3
7 11 9 13 16 4
Students
Sol. Long Answer Type
Questions (5 marks each)
Q. 1. The following table gives the life time of 400 neon
lamps :
Life Time (in hours) Number of Lamps
300 - 400 14
400 - 500 56
500 - 600 60
600 - 700 86
700 - 800 74
800 - 900 62
3 900 - 1000 48
(i) Represent the given information with help of
Q. 4. A family with a monthly income of ` 20,000 had histogram.
planned the following expenditure per month (ii) How many lamps have life time of more than 700
under various heads. R [Board Term II, 2012] hours ? U [NCERT][Board Term II, KVS, 2016]

Expenditure Sol. (i)


Heads
(in thousands rupees)
Grocery 04
Rent 05
Education 05
Medicine 02
Fuel 02
Entertainment 01
Miscellaneous 01
Draw a bar graph for the data above.
y
Sol.
(in thousand rupees)

7
Entertainment

6
Miscellaneous
Expenditure

Medicine

5
Education

4
Fuel
Rent
Grocery

3
2 3½
1 (ii) Lamps having life time for more than 700 hours are
x
0
Heads = 74 + 62 + 48 = 184 1½

3
STATISTICS 177
Q. 2. Draw a histogram to represent the following
grouped frequency.

Age 30- 35-


5-9 10-14 15-19 20-24 25-29
(in yrs) 34 39

No. of
per- 10 28 32 48 50 35 12
sons
Also draw frequency polygon.

U [Board Term-II, 2016]


Sol.
Number Continuous 3
Age Class
of Age [CBSE Marking Scheme, 2016]
(in years) Marks
Persons (in years)
5-9 10 4.5-9.5 7
10-14 28 9.5-14.5 12
15-19 32 14.5-19.5 17
20-24 48 19.5-24.5 22
25-29 50 24.5-29.5 27
30-34 35 29.5-34.5 32
35-39 12 34.5-39.5 37
2

Q. 3. Draw a histogram to represent the following frequency distribution. [KVS 2019]

Marks 0 – 20 20 – 30 30 – 40 40 – 50 50 – 60 60 – 70 70 – 100
Number of students 7 10 10 20 20 15 8
Sol.
20
Marks Number of Frequency density 40 - 50 20 × 10 = 20
10
Students
20
7 50 - 60 20 × 10 = 20
0 - 20 7 × 10 = 3.5 10
20
15
10 60 - 70 15 × 10 = 15
20 - 30 10 × 10 = 10 10
10
10 8
30 - 40 10 × 10 = 10 70 - 100 8 × 10 = 2.66 = 2.6
10 30


Marks → 5
178 Oswaal CBSE Question Bank Chapterwise & Topicwise, MATHEMATICS, Class-IX

OBJECTIVE TYPE QUESTION (1 mark each)

(A) Upper limits of the classes


A Multiple Choice Question (B) Continuous classes
(C) Discontinuous classes
Q. 1. For drawing a frequency polygon of a continuous
frequency distribution, we plot the points whose (D) Maximum frequency
ordinates are the frequencies of the respective Ans. Option (B) is correct.
classes and abscissae are
(A) upper limits of the classes. B Assertion & Reason
(B) lower limits of the classes.
Directions: In the following questions, a statement
(C) class marks of the classes.
of assertion (A) is followed by a statement of
(D) upper limits of preceding classes.
reason (R). Mark the correct choice as:
 [NCERT Exemp.]
(A) Both assertion (A) and reason (R) are true and reason
Ans. Option (C) is correct.
(R) is the correct explanation of assertion (A).
Explanation: We have to take only class marks
(B) Both assertion (A) and reason (R) are true but reason
while drawing the polygons.
(R) is not the correct explanation of assertion (A).
Q. 2. In a bar graph, the width of bars
(C) Assertion (A) is true but reason (R) is false.
(A) Are proportional to the corresponding frequencies
(D) Assertion (A) is false but reason (R) is true.
(B) Have no significance
(C) Are proportional to the space between two Q. 1. Assertion (A): According to statistics more female
consecutive bars. children are born each year than male children in
(D) Are proportional to the corresponding heights. India.
Ans. Option (B) is correct. Reason (R): In India the death rate of a male child is
Explanation: Width of bar's are equal only from higher than that of the female child.
clarity point of view otherwise they have no Ans. Option (C) is correct.
significance. Q. 2. Assertion (A): The class interval needs to be
Q. 3. In a histogram, which of the following is continuous while drawing a Histogram.
proportional to the frequency of the corresponding Reason (R): Histogram is a rectangular diagram
class? using frequency distributions which are joined to
(A) Area of the rectangle one another.
(B) Length of the rectangle Ans. Option (B) is correct.
(C) Width of the rectangle Explanation : In case of Assertion (A):
(D) Perimeter of the rectangle Histograms depicts continuous data which is taken
Ans. Option (A) is correct. in the form of class interval.
Q. 4. Histogram graphically represent the grouped \ Assertion is true, but Reason is not correct
frequency distribution with explanation of Assertion.

COMPETENCY BASED QUESTIONS (4 marks each)

A Case based MCQs


Read the following passage and answer any four questions of the following :
I. Child labour refers to any work or activity that deprives children of their childhood. It is a violation of children's rights.
This can harm them mentally or physically. It also exposes them to hazardous situations or stop them from going to
school. Naman got data on number of child labours (in million) in different country that is given below.
STATISTICS 179

Q. 1. Which country has the highest child labour ?



(A) Peru (B) United States 1200,0
(C) Egypt (D) India 10300
Ans. Option (D) is correct. 1 1000,0
9060 9160
Q. 2. Which country has the lowest child labour ? Rs. (in Million)
800,0
(A) Brazil (B) Kenya
(C) United States (D) India 600,0
Ans. Option (C) is correct. 1
Q. 3. Name the countries having more than 4 million 400,0
child labour.
200,0
(A) India and Bangladesh
5, 4
(B) India and Peru 0
(C) Bangladesh and Brazil 2007-08 2008-09 2009-10 2010-11
Estimated Budget
(D) Bangladesh and Kenya
Ans. Option (A) is correct. 1 Q. 1. Which statement is incorrect about bar graph?
(A) A bar graph is a pictorial representation of
Q. 4. Which country has more than 3 million but less
data.
than 6 million child labour ? (B) A bar graph can be vertical or horizontal.
(A) India (B) Peru (C) Heights of the bars depend on the values of
(C) Egypt (D) Kenya the variable.
Ans. Option (B) is correct. 1 (D) There are no gaps in between consecutive
Q. 5. Name the country which has 1 million child rectangles.
labour. Ans. Option (D) is correct.
(A) Mexico (B) Kenya Explanation: There are no gaps in between
consecutive rectangles.
(C) Peru (D) India
Given statement is incorrect as there is gap in
Ans. Option (B) is correct. 1 between consecutive rectangles in a bar graph. 1
II. Ladli Scheme was launched by the Delhi Q. 2. In which year the budget was minimum?
Government in the year 2008. This scheme helps to (A) 2008 - 09 (B) 2007 - 08
make women strong and will empower a girl child.
(C) 2010 - 11 (D) 2009 - 10
This scheme was started in 2008.
Ans. Option (B) is correct. 1
Q. 3. In which year the budget was maximum?
(A) 2007 - 08 (B) 2008 - 09
(C) 2009 - 10 (D) 2010 - 11
Ans. Option (D) is correct. 1
Q. 4. What was the budget in 2010 - 11?
(A) 9060 Million (B) 10300 Million
(C) 9160 Million (D) 54 Million
Ans. Option (B) is correct. 1
Read the above bar graph and answer the
following questions :
180 Oswaal CBSE Question Bank Chapterwise & Topicwise, MATHEMATICS, Class-IX

Q. 5. What was the difference in budget in the year Q. 2. What does this bar graph show ?
2008 - 09 and 2009 - 10 ? Ans. This bar graph shows decline in number of tigers. 1
(A) 100 Million (B) 1000 Million Q. 3. How much decrease in number of tigers was
(C) 50 Million (D) 150 Million there between 1900 and 1960 ?
Ans. Option (A) is correct. 1
Ans. Number of tigers in 1900 = 40,000
Number of tigers in 1960 = 18000
Case based Subjective
B Questions Decrease in number of tigers = 22000 2
II. Electricity energy consumption is the form of energy
consumption that uses electric energy. Global
Read the following passage and answer the
electricity consumption continues to increase faster
following questions: than world world population, leading to an increase
I. India's national animal Tiger (which has also been in the average amount of electricity consumed per
under the radar of the government as its population person (per capita electricity consumption).
declined in the country) has witnessed an increase A survey is conducted for 56 families of a colony
in its population. A. The following bar graph gives the weekly
consumption of electricity of these families.
A survey was done by the Ministry of Environment,
Forests and Climate change, according to which
tiger population is on rise at the rate of 6 per cent
every year from 2006 and 2018.
Year 1900 1960 2002 2007
No. of Tigers 40,000 18,000 3642 1411

Q. 1. How many families weekly consumption is 50-60


units?
Ans. 0 families have weekly consumption 50-60
(units). 1
Q. 2. What is the weekly consumption of maximum
number of families?
Ans. From bar graph 18 families are maximum number of
families having 20-30 (units) of weekly consumption. 1
Q. 3. What is the difference in the number of families
which consumes (30-40) and (40-50) units?
Also tell whether families are increasing or
decreasing?
Pranav has found a report on status of tigers in India Ans. As we can see from graph
in last 100 years. He has some queries as follow : No. of families consuming (30-40) units = 6
Q. 1. In which year, number of tigers were minimum? No. of families consuming (40-50) units = 4
Ans. According to graph in 2007 number of tigers were
\ Difference = 6 – 4 = 2 families
minimum. 1
As we can see number of families are decreasing.

Artificial Intelligence
AI
PARAMETERS DESCRIPTION CONCEPTS
INTEGRATED

Chapter Covered Chapter 14: Statistics - Understanding Frequency Table

Name of the book Mathematics, Class 9 NCERT

Subject and Artificial Analyzing the frequency tables using AI Tools. (Statistical Data).
Intelligence Integrated
STATISTICS 181

Learning Objectives Students will able to


● Prepare and use the frequency tables.
● Analyze the frequency tables.

Time Required 2 periods of 40 minutes each

Classroom Arrangement Seating arrangement -In pairs for both the sessions.

Material Required Pen, paper, Laptops/ desktops/ Tabs and Internet connection.

Pre-Preparation Activity ● Students will be asked to look at some frequency tables in the
handouts.
● Students will be asked a few questions based on the frequency tables.
Like:
What is the info given in the table?
How many…..
What is the Highest/Lowest?

Previous knowledge Preparing a frequency table and reading it.

Methodology After a preliminary round of pre -knowledge testing, students will be


guided to play the Markov Data Game. Students, Play Rock, Paper,
Scissors with the evil Dr. Markov to save the dog.
A video will be shown for the guidelines.

Introduction to Markov - Part I


Students will play the game:

codap.concord
While playing the games the students will be asked to make a list of their
moves and the Markov’s moves.
After round 1, they will be asked to prepare the frequency tables for their
moves and try to analyze the same.
Video 2 will be shown:

Introduction to Markov - Part II, Advanced Gameplay


Now the students will be in a better position to win the game and save
the dog.

Learning Outcomes Students prepared the tables with the data collected while playing the
game.
They analyzed the data in tables and won the game.

Follow up Activities The following worksheet will be shared.


Markov Student Worksheet
● Students may be asked to search and share more such games either
online or offline.
● Same tables can be later used to draw bar graphs.
● We can collect data of win or loss from all pairs and it can be used to:
- Create discrete or grouped tables.
- Calculate the measures of central tendency: mean/median or mode.

Reflections ● Markov uses a different strategy on each new level, but his strategy
remains consistent throughout each game played on a single level.
● There is increasing variability in Markov’s moves as the student
progresses through the levels, which makes it more challenging to win.
● Discussion of the students’ responses in the worksheet.

qqq
182 Oswaal CBSE Question Bank Chapterwise & Topicwise, MATHEMATICS, Class-IX

AI
PARAMETERS DESCRIPTION CONCEPTS
INTEGRATED
Chapter Covered Chapter 14: Statistics - Understanding the Concept of Data Handling
Name of the book Mathematics, Class 9 NCERT
Subject and Artificial Understanding the concept of data handling using AI Experiential
Intelligence Integrated applications
Learning Objectives To understand the concept of representing data in the form of various
graphs.
To understand the process of Data Handling:
● Sources of data
● Data Acquisition
● Exploration
To understand the process of Data Handling in real life situations using AI
project cycle process of Data Acquisition.
Time Required 2 periods of 40 minutes each
Classroom Arrangement Flexible
Material Required Pen, paper, black board, chalk, Laptop/Desktop, and internet connections.
Pre-Preparation Activities The students are divided into three groups and asked to collect
information on AQI. (pollution level)
Previous Knowledge The students should know the terms like average, range, median, and mode.
Methodology Divide the class into two teams
Activity 1- Different types of graphs
Ask the students to collect data in different types of graphs.
Activity 2- Ask the students to collect and compare data on Fine Particulate
Matter PM2.5 across SAFAR cities
Ask the students to observe and write the activities that pollute the
environment.
Data Acquisition
Activity 3- Ask the students to collect the data on temperature and
humidity and deduce the correlation between them.
Activity 4- To make a chart on the weather forecast using
http://safar.tropmet.res.in/
The students would be able to understand that using AI and past data, the
weather forecasts are done.
Ask the students to draw and compare AQI of various cities of the world.
Activity 5: To check which graph is suited for such type of
data https://datavizcatalogue.com/
Data Exploration
Learning Outcomes The students will
● Understand the importance of data collection in real life
● Appreciate the importance of data analysis and forecasting using AI.
● Develop skills of factual representation of data using AI
● Be able to interpret various graphs
Follow up Activities Ask students to make a presentation
Stating facts and using pictorial representation of Data collected and
analyzed.
Reflections Discussion with Students:
How do you like the site http://safar.tropmet.res.in/?
Try to analyze the data of past 10 years of a place.
Observe the climate change.
How AI and machine learning are transforming weather forecasting.
Do you know of any other tool/app that can help you to access data?

  
SELF ASSESSMENT PAPER - 06
Time: 1 hour MM: 30

UNIT-VI
I. Multiple Choice Questions [1 × 6 = 6]
1. The height of the rectangle in a histogram is
(A) width of the class
(B) upper limit of the class
(C) lower limit of the class
(D) frequency of the class
2. Graphical representation of numerical data by a number of base of uniform width with equal spacing between
then is
(A) Bar graph
(B) Histogram (C) Frequency polygon (D) pie-chart
3. To draw a histogram to represent frequency for the class 24-45. The adjusted frequency is ................... if frequency
of class 25-45 is 8.
Class internal 5-10 10-15 15-25 25-45
Frequency 6 12 10 8
(A) 6
(B) 5 (C) 3 (D) 2
4. A vertical graph with no spacing between then is
(A) Bar graph
(B) frequency polygon (C) Histogram (D) None of these
II. Assertion and Reason Based MCQs [1×6 = 6]

Directions: In the following questions, A statement of Assertion (A) is followed by a statement of Reason (R).
Mark the correct choice as.
(A) Both A and R are true and R is the correct explanation of A.
(B) Both A and R are true but R is NOT the correct explanation of A.
(C) A is true but R is false.
(D) A is false and R is True.
1. Assertion (A): A histogram is a two dimensional graph.
Reason (R): Histogram is a two dimensional graph of frequency distribution that emphasises both lengths and
widths of the rectangle.
2. Assertion (A): A frequency polygon can not be drawn with the help of a Histogram.
Reason (R): A frequency histogram is a type of bar graph that shows the frequency or number of times an outcome
occurs.
III. Very Short Answer Type questions [1 × 2 = 2]
1. Is it correct to say that in a histogram, the area of each rectangle is proportional to the class size of the corresponding
class interval? If not, correct the statement.
2. The class marks of a continuous distribution are : 1.04, 1.14, 1.24, 1.34, 1.44, 1.54 and 1.64.
Is it correct to say that the last interval will be 1.55–1.73? Justify your answer.
IV. Short Answer Type questions–I [2 × 3 = 6]
1. Find the given frequency table construct the bar graph:
Marks 0-20 20-40 40-60 60-100
No. of Students 10 15 20 25
Represent the information above by a bar graph.
2. In the given Figure, there is a histogram depicting daily wages of workers in a factory.
Construct the frequency distribution table.
184 Oswaal CBSE Question Bank Chapterwise & Topicwise, MATHEMATICS, Class-IX

No of Workers 50
40
30
20
10

50 100 150 200 250 300 350 400


Wage in `
3. Draw a frequency polygon graph for the given data :
Weight (kg) 60 61 62 63 64 65
No. of person 15 18 14 16 15 12
V. Long Answer Type questions [5 × 1 = 5]
1. Draw a histogram for the following data :
Age (in years) 0-8 8-16 16-24 24-32 32-40 40-48
Number of person 9 13 15 20 10 5
VI. Case Study Based Questions [1 × 4 = 4]

Locate the bars as shown given below :


see the above bar graph and given the answer of following questions :
1. How many students use bus as the mode of transports ?
(A) 25
(B) 27 (C) 20 (D) 44
2. How many total students use car and scooter as the mode of transport ?
(A) 15
(B) 10 (C) 20 (D) 30
3. How many students use rickshaw as the mode of transport ?
(A) 10
(B) 12 (C) 8 (D) 15
4. How many students use Bicycle as made of transport ?
(A) 15
(B) 20 (C) 30 (D) 25

qq

You might also like